Sie sind auf Seite 1von 281

SOLUTIONS TO CALCULUS VOLUME 1 BY TOM APOSTOL.


ERNEST YEUNG, - PRAHA 10, CESKA REPUBLIKA

S OLUTIONS TO VOLUME 1 One-Variable Calculus, with an Introduction to Linear Algebra

I 2.5 Exercises - Introduction to set theory, Notations for designating sets, Subsets, Unions, intersections, complements.
Exercise 10. Distributive laws

Let X = A ∩ (B ∪ C), Y = (A ∩ B) ∪ (A ∩ C)
Suppose x ∈ X

x ∈ A and x ∈ (B ∪ C) =⇒ x ∈ A and x is in at least B or in C


thenx is in at least either (A ∩ B) or (A ∩ C)
x ∈ Y, X ⊆ Y
Suppose y ∈ Y

y is at least in either (A ∩ B) or A ∩ C
then y ∈ A and either in B or C

y ∈ X, Y ⊆ X
X=Y
Let X = A ∪ (B ∩ C), Y = (A ∪ B) ∩ (A ∪ C)
Suppose x ∈ X

then x is at least either in A or in (B ∩ C)


if x ∈ A, x ∈ Y

if x ∈ (B ∩ C), x ∈ Y x ∈ Y, X ⊆ Y
Suppose y∈Y
then y is at least in A or in B and y is at least in A or in C

if y ∈ A, then y ∈ X

if y ∈ A ∩ B or y ∈ A ∪ C, y ∈ X (various carvings out of A, simply )

y ∈ (B ∩ C), y ∈ X
if y ∈ X, Y ⊆ X
X=Y
Exercise 11. Ifx ∈ A ∪ A, then x is at least in A or in A. Then x ∈ A. So A ∪ A ⊆ A. Of course A ⊆ A ∪ A.
If x ∈ A ∩ A, then x is in A and in A. Then x ∈ A. So A ∩ A ⊆ A. Of course A ⊆ A ∩ A.
Exercise 12. Let x ∈ A. y ∈ A ∪ B if y is at least in A or in B . x is in A so x ∈ A ∪ B . =⇒ A ⊆ A ∪ B .

Suppose ∃b ∈ B and b ∈ / A. b ∈ A ∪ B but b ∈ / A. so A ⊆ A ∪ B .


Exercise 13. Let x ∈ A ∪ ∅, then x is at least in A or in ∅. If x ∈ ∅, then x is a null element (not an element at all). Then

actual elements must be in A. =⇒ A ∪ ∅ ⊆ A.


Let x ∈ A. Then x ∈ A ∪ ∅. A ⊆ A ∪ ∅. =⇒ A = A ∪ ∅.

Exercise 14. From distributivity, A ∪ (A ∩ B) = (A ∪ A) ∩ (A ∪ B) = A ∩ (A ∪ B).

If x ∈ A ∩ (A ∪ B), x ∈ A and x ∈ A ∪ B , i.e. x ∈ A and x is at least in A or in B .


=⇒ x is in A and is in B or is not in B . Then x ∈ A. =⇒ A ∩ (A ∪ B) ⊆ A. Of course, A ⊆ A ∩ (A ∪ B).
=⇒ A ∩ (A ∪ B) = A ∪ (A ∩ B) = A.
Exercise 15. ∀a ∈ A, a ∈ C and ∀b ∈ B, b ∈ C . Consider x ∈ A ∪ B . x is at least in A or in B . In either case, x ∈ C .

=⇒ A ∪ B ⊆ C .
Exercise 16.
if C ⊆ A and C ⊆ B, then C ⊆A∩B
∀c ∈ C, c ∈ A and c ∈ B
x ∈ A ∩ B, x ∈ A and x ∈ B. Then ∀c ∈ C, c ∈ A ∩ B. C ⊆ A ∩ B
1
Exercise 17.

(1)
if A⊂B and B⊂C then

∀a ∈ A, a ∈ B.∀b ∈ B, b ∈ C.
then since a ∈ B, a ∈ C, ∃c ∈ C such that c∈
/ B.
∀a ∈ A, a ∈ B so a 6= c∀a. =⇒ A ⊂ C
(2) If A ⊆ B, B ⊆ C, A ⊆ C since, ∀a ∈ A, a ∈ B, ∀b ∈ B, b ∈ C. Then since a ∈ B, a ∈ C . A ⊆ C
(3) A ⊂ B and B ⊆ C . B ⊂ C or B = C . A ⊂ B only. Then A ⊂ C .
(4) Yes, since ∀a ∈ A, a ∈ B .
(5) No, since x 6= A (sets as elements are different from elements)

Exercise 18. A − (B ∩ C) = (A − B) ∪ (A − C)
Suppose x ∈ A − (B ∩ C)
then x ∈ A and x ∈ / B ∩ C =⇒ x ∈ / B∩C
then x is not in even at least one B or C

=⇒ x ∈ (A − B) ∪ (A − C)
Suppose x ∈ (A − B) ∪ (A − C)
then x is at least in (A − B) or in (A − C) =⇒ x is at least in A and not in B or in A and not in C
then consider when one of the cases is true and when both cases are true =⇒ x ∈ A − (B ∩ C)

Exercise 19.
[ \
Suppose x∈B− A Suppose x∈ (B − A)
A∈F
[ A∈F
then x ∈ B, x ∈
/ A then x ∈ B − A1 and x ∈ B − A2 and ...
A∈F
[ then ∀A ∈ F, x ∈ B, x ∈
/A
x∈
/ A =⇒ x ∈
/ A, ∀A ∈ F then x∈
/ even at least one A∈F
A∈F
\ [
since ∀A ∈ F, x ∈ B, x ∈
/ A, then x∈ (B − A) =⇒ x ∈ B − A
A∈F
A∈F

\
Suppose x∈B− A
A∈F
[
\ Suppose x∈ (B − A)
then x∈
/ A A∈F
A∈F
then x is at least in one B − A
then at most x ∈ A for ∀A ∈ F but one
then for A ∈ F, x ∈ B and x ∈
/A
then x is at least in one B − A
[ Consider ∀A ∈ F
=⇒ x ∈ (B − A) \
A∈F
=⇒ then x∈B− A
A∈F

Exercise 20.

(1) (ii) is correct.

Suppose x ∈ (A − B) − C
Suppose x ∈ A − (B ∪ C)
then x ∈ A − B, x ∈
/C
then x ∈ A, x ∈
/ (B ∪ C)
then x ∈ A and x ∈
/ B and x ∈
/C
then x ∈ A and x ∈
/ B and x ∈
/C
x∈
/B and x∈
/ C =⇒ x ∈
/ even at least B or C
=⇒ x ∈ (A − B) − C
x ∈ A − (B ∪ C)
2
To show that (i) is sometimes wrong,

Suppose y ∈ A − (B − C)
y ∈ A and y ∈
/ B−C
y∈
/ B−C
then y∈
/B or y∈C or y∈
/C
(where does this lead to?)

Consider directly.
Suppose x ∈ (A − B) ∪ C
then x is at least in A − B or in C
then x is at least in A and ∈
/B or in C

Suppose x=c∈C and c∈


/A
(2)
If C ⊆ A,
A − (B − C) = (A − B) ∪ C
I 3.3 Exercises - The eld axioms. The goal seems to be to abstract these so-called real numbers into just x's and y 's that
are purely built upon these axioms.

Exercise 1. Thm. I.5. a(b − c) = ab − ac.


Lety = ab − ac; x = a(b − c)
Want: x = y

ac + y = ab (by Thm. I.2, possibility of subtraction)

Note that by Thm. I.3, a(b − c) = a(b + (−c)) = ab + a(−c) (by distributivity axiom)
ac + x = ac + ab + a(−c) = a(c + (−c)) + ab = a(0 + b) = ab
But there exists exactly one y or x by Thm. I.2. x = y .
Thm. I.6. 0 · a = a · 0 = 0.
0(a) = a(0) (by commutativity axiom)
Given b ∈ R and 0 ∈ R, ∃ exactly one − b s.t. b − a = 0
0(a) = (b + (−b))a = ab − ab = 0 (by Thm. I.5. and Thm. I.2)

Thm. I.7.
ab = ac
By Axiom 4, ∃y ∈ R s.t. ay = 1
since products are uniquely determined, yab = yac =⇒ (ya)b = (ya)c =⇒ 1(b) = 1(c)
=⇒ b = c
Thm. I.8. Possibility of Division.
Given a, b, a 6= 0, choose y such that ay = 1.
Let x = yb.
ax = ayb = 1(b) = b
Therefore, there exists at least one x such that ax = b. But by Thm. I.7, there exists only one x (since if az − b, and so
x = z ).
Thm. I.9. If a 6= 0, then b/a = b(a−1 ).

b
Let x= for ax = b
a
y = a−1 for ay = 1
Want: x = by
Now b(1) = b, so ax = b = b(ay) = a(by)
=⇒ x = by (by Thm. I.7)
−1 −1
a 6= 0, then (a ) = a.
Thm. I.10. If
Now ab = 1 for b = a−1 . But since b ∈ R and b 6= 0 (otherwise 1 = 0, contradiction), then using Thm. I.8 on b,
ab = b(a) = 1; a = b−1 .
3
Thm.I.11. If ab = 0, a = 0 or b = 0.
ab = 0 = a(0) =⇒ b = 0 or ab = ba = b(0) =⇒ a = 0. (we used Thm. I.7, cancellation law for multiplication)
Thm. I.12. Want: x = y if x = (−a)b and y = −(ab).

ab + y = 0
ab + x = ab + (−a)b = b(a + (−a)) = b(a − a) = b(0) = 0
0 is unique, so ab + y = ab + x implies x = y( by Thm. I.1 )
Thm. I.13. Want: x + y = z , if a = bx, c = dy, (ad + bc) = (bd)z .
(bd)(x + y) = bdx + bdy = ad + bc = (bd)z
So using b, d 6= 0, which is given, and Thm. I.7, then x + y = z .
Thm. I.14. Want: xy = z for bx = a, dy = c, ac = (bd)z .
(bd)(xy) = (bx)(dy) = ac = (bd)z
b, d 6= 0, so by Thm. I.7, xy = z .
Thm.I.15. Want: x = yz , if bx = a, dy = c, (bc)z = ad

(bc)z = b(dy)z = d(byz) = da


d 6= 0 so by Thm. I.7, by z = a, byz = abx
b 6= 0 so by Thm. I.7, yz =x

Exercise 2. Consider 0 + z = 0. By Thm. I.2, there exists exactly one z , z = −0. By Axiom 4, z = 0. 0 = −0.
−1
Exercise 3. Consider 1(z)z(1) = 1. Then z = 1 . But by Axiom 4, there exists distinct 1 such that z(1) = 1, so z = 1.

Exercise 4. Suppose there exists x such that 0x = 1, but 0x = 0 and 0 and 1 are distinct, so zero has no reciprocal .

Exercise 5. a + (−a) = 0, 0 + 0 = 0. Then


a + (−a) + b + (−b) = (a + b) + (−a) + (−b) = 0
−(a + b) = −a + (−b) = −a − b

Exercise 6. a + (−a) = 0, b + (−b) = 0, so


a + (−a) + b + (−b) = a + (−b) + (−a) + b = (a − b) + (−a) + b = 0 + 0 = 0
−(a − b) = −a + b.
Exercise 7.

(a − b) + (b − c) = a + (−b) + b + (−c) = a + (b + (−b)) + (−c) = a − c


Exercise 8.
(ab)x = 1 (ab)−1 = x
a(bx) = 1 a−1 = bx
b(ax) = 1 b−1 − ax
a−1 b−1 = (abx)x = 1(x) = (ab)−1
Exercise 9. Want: x = y = z , if
a
z= a = zt b+t=0
−b
(−a)
y= by = u a + u = 0
b³ ´ ³a´
a
x=− +x=v+x=0 vb = a
b b

a + (−a) = vb + by = b(v + y) = 0
if b 6= 0, v + y = 0, but v+x=0
by Thm. I.1 ,x = y
b + t = 0, then z(b + t) = zb + zt = zb + a = z(0) = 0
a + zb = 0 =⇒ −a = zb = by
since b 6= 0, z = y so x = y = z
4
Exercise 10. Since b, d 6= 0, Let
ad − bc
z= (bd)z = ad − bc by previous exercise or Thm. I.8, the possibility of division
bd
a
x= bx = a
b
−c
t= dt = −c (By Thm. I.3, we know that b − a = b + (−a) )
d
dbx + bdt = (bd)(x + y) = ad − bc = (bd)z
b, d 6= 0, so x+y =z
I 3.5 Exercises - The order axioms.

Theorem 1 (I.18). If a < b and c > 0 then ac < bc


Theorem 2 (I.19). If a < b and c > 0, then ac < bc
Theorem 3 (I.20). If a 6= 0, then a2 > 0
Theorem 4 (I.21). 1>0
Theorem 5 (I.22). If a < b and c < 0, then ac > bc.
Theorem 6 (I.23). If a < b and −a > −b. In particular, if a < 0, then −a > 0.
Theorem 7 (I.24). If ab > 0, then both a and b are positive or both are negative.
Theorem 8 (I.25). If a < c and b < d, then a + b < c + d.

Exercise 1.

(1) By Thm. I.19, −c > 0


a(−c) < b(−c) → −ac < −bc
−bc − (−ac) = ac − bc > 0. Then ac > bc (by denition of > )
(2)
a < b → a + 0 < b + 0 → a + b + (−b) < b + a + (−a) → (a + b) − b < (a + b) + (−a)
By Thm.I.18 (a + b) + −(a + b) + (−b) < (a + b) − (a + b) + (−a)
−b < −a
(3)
If a = 0 or b = 0, ab = 0, but 0 ≯ 0
If a > 0, then if b > 0, ab > 0(b) = 0. If b < 0, ab < 0(b) = 0. So if a > 0, then b > 0.
If a < 0, then if b > 0, ab < 0(b) = 0. If b < 0, ab > 0(b) = 0. So if a < 0, then b < 0.
(4)
a < c so a + b < c + b = b + c
b < d so b + c < d + c
By Transitive Law ,a + b < d + c
2
Exercise 2. If x = 0, x = 0. 0 + 1 = 1 6= 0. So x 6= 0.
If x 6= 0, x2 > 0, and by Thm. I.21 ,1 > 0
2 2
x + 1 > 0 + 0 = 0 → x + 1 6= 0
=⇒ @x ∈ R such that x2 + 1 = 0
Exercise 3.

a < 0, b < 0, a + b < 0 + 0 = 0 ( By Thm. I.25)

Exercise 4. Consider ax = 1.
ax = 1 > 0. By Thm. I.24 , a, x are both positive or a, x are both negative
Exercise 5. Dene x, y such that ax = 1, by = 1. We want x > y when b > a.
xb − ax = xb − 1 > 0 =⇒ bx > 1 = by
b > 0 so x > y
5
Exercise 6.
If a = b and b = c, then a=c
If a = b and b < c, then a<c
If a < b and b = c, then a<c
a < b and b < c,
If then a < c (by transitivity of the inequality)
=⇒ a ≤ c
Exercise 7. If a ≤ b and b ≤ c, then a ≤ c. If a = c, then by previous proof, a = b.
Exercise 8. If a ≤ b and b ≤ c, then a ≤ c. If a = c, then by previous proof, a = b.
2 2 2 2
Exercise 8. If a or b is zero, a or b = 0. By Thm. I.20, b ≥ 0 or a ≥ 0, respectively.
2 2
Otherwise, if neither are zero, by transitivity, a + b > 0.

Exercise 9. Suppose a ≥ x. Then a − x ≥ 0.

If a ∈ R so ∃y ∈ R, such that a − y = 0.
Consider y + 1 ∈ R (by closure under addition).

a − (y + 1) = a − y − 1 = 0 − 1 < 0 Contradiction that a ≥ y + 1


Exercise 10.
If x = 0, done.
x
If x > 0, x is a positive real number. Let h= .
2
x
=⇒ > x Contradiction.
2
I 3.12 Exercises - Integers and rational numbers, Geometric interpretation of real numbers as points on a line, Upper
bound of a set, maximum element, least upper bound (supremum), The least-upper-bound axiom (completeness ax-
iom), The Archimedean property of the real-number system, Fundamental properties of the suprenum and inmum.
We use Thm I.30, the Archimedean property of real numbers, alot.

Theorem 9 (I.30). If x > 0 and if y is an arbitrary real number, there exists a positive integer n such that nx > y .
We will use the least upper-bound axiom (completeness axiom) alot for continuity and differentiation theorems later.
Apostol states it as an axiom; in real analysis, the existence of a sup for nonempty, bounded sets can be shown with an
algorithm to zoom into a sup with monotonically increasing and monotonically decreasing sequence of “guesses” and showing
its difference is a Cauchy sequence.

Axiom 1 (Least upper-bound axiom). Every nonempty set S of real numbers which is bounded above has a suprenum; that
is, there's a real number B s.t. B = supS .

Exercise 1. 0 < y − x.
=⇒ n(y − x) > h > 0, n ∈ Z+ , h arbitrary
y − x > h/n =⇒ y > x + h/n > x
so let z = x + h/n Done.
Exercise 2. x ∈ R so ∃n ∈ Z+ such that n > x (Thm. I.29).
Set of negative integers is unbounded below because
If ∀m ∈ Z− , −x > −m, then −x is an upper bound on Z+ . Contradiction of Thm. I.29. =⇒ ∃m ∈ Z such that m < x < n
Exercise 3. Use Archimedian property.

x > 0 so for 1, ∃n ∈ Z+ such that nx > 1, x > n1 .


Exercise 4. x is an arbitrary real number. By Thm. I.29 and well-ordering principle, there exists a smallest n + 1 positive

integer such that x < n + 1 (consider the set of all m + 1 > x and so by well-ordering principle, there must be a smallest
element of this specic set of positive integers).
x = n for some positive integer n, done.
If
x < n, then n + 1 couldn't have been the smallest element such that m > x. x > n.
Otherwise, note that if

Exercise 5. If x = n, done. Otherwise, consider all m > x. By well-ordering principle, there exists a smallest element n such

that n > x.
If x + 1 < n, then x < n − 1, contradicting the fact that n is the smallest element such that x < n. Thus x + 1 > n.

Exercise 6. y − x > 0.

n(y − x) > h, h arbitrary , n ∈ Z+


y > x + h/n = z > x
6
Since h was arbitrary, there are innitely many numbers in between x, y .
Exercise 7. x = ab ∈ Q, y ∈ / Q.
a ± by
x±y =
b µ ¶
a − mb
If a ± by was an integer, say m, then y = ± which is rational. Contradiction.
b
ay ay
xy = =
b1 b
n
If ay was an integer, ay = n, y = , but y is irrational. =⇒ xy is irrational.
a
x
y
y is not an integer
Exercise 8. Proof by counterexamples. We want that the sum or product of 2 irrational numbers is not always irrational. If y
y + 1 is irrational, otherwise, if y + 1 ∈ Q, y ∈ Q by closure under addition.
is irrational,
=⇒ y + 1 − y = 1
1
Likewise, y = 1.
y
Exercise 9.

y − x > 0 =⇒ n(y − x) > k, n ∈ Z+ , k arbitrary. Choose k to be irrational. Then k/n irrational.


k k
y> + x > x. Let z = x + , z irrational .
n n
Exercise 10.

(1) Suppose n = 2m1 and n + 1 = 2m2 .


1
2m1 + 1 = 2m2 2(m1 − m2 ) = 1 m1 − m2 =
. But m1 − m2 can only be an integer.
2
+
(2) By the well-ordering principle, if x ∈ Z is neither even and odd, consider the set of all x. There must exist a
+
smallest element x0 of this set. But since x0 ∈ Z , then there must exist a n < x such that n + 1 = x0 . n is even or
odd since it doesn't belong in the above set. So x0 must be odd or even. Contradiction.
(3)
(2m1 )(2m2 ) = 2(2m1 m2 ) even
2m1 + 2m2 = 2(m1 + m2 ) even
(2m1 + 1) + (2m2 + 1) = 2(m1 + m2 + 1) =⇒ sum of two odd numbers is even

(n1 + 1)(n2 + 1) = n1 n2 + n1 + n + 2 + 1 = 2(2m1 m2 )


2(2m1 m2 ) − (n1 + n2 ) − 1 odd, the product of two odd numbers n1 , n2 is odd
2 2 2 2
(4) If n even, n is even, since for n = 2m, (2m) = 4m = 2(2m ) is even.

a2 = 2b2 . 2(b2 ) even. a2 even, so a even.


If a even a = 2n.a2 = 4n2
If b odd , b2 odd. b has no factors of 2 b2 6= 4n2
Thus b is even.
p
(5) For
q , If p or q or both are odd, then we're done.
l n
Else, when p, q are both even, p = 2 m, q = 2 p, m, p odd.

p 2l m 2l−n m
= n = and at least m or p odd
q 2 p p
a
b can be put into a form such that a or b at least is odd by the previous exercise.
Exercise 11.
2 2 a
However, a = 2b , so a even, b even, by the previous exercise, part (d) or 4th part. Thus
b cannot be rational.
Exercise 12. The set of rational numbers satises the Archimedean property but not the least-upper-bound property.
p
Since
q ∈ Q ⊆ R, n pq11 > p2
q2 since if q1 , q2 > 0,
np1 q2 q1 p2
> np1 q2 > q1 p2
q1 q2 q1 q2
n exists since (p1 q2 ), (q1 p2 ) ∈ R.
7
The set of rational numbers does not satisfy the least-upper-bound property.
r
Consider a nonempty set of rational numbers S bounded above so that ∀x = s ∈ S, x < b.
Suppose x < b1 , x < b2 ∀x ∈ S .
r r
< b2 < nb1 but likewise < b1 < mb2 , n, m ∈ Z+
s s
So it's possible that b1 > b2 , but also b2 > b1 .

I 4.4 Exercises - An example of a proof by mathematical induction, The principle of mathematical induction, The
well-ordering principle. Consider these 2 proofs.

N + N + · · · + N = N2
N
X N
X −1
(N − 1) + (N − 2) + · · · + (N − (N − 1)) + (N − N ) = N 2 − j= j
j=1 j=1
N
X N
X N (N + 1)
N2 + N = 2 j =⇒ j=
j=1 j=1
2
An interesting property is that
n
X n
X
S= j= (n + m − j)
j=m j=m
So that
N
X N
X m
X N
X m(m + 1) N (N + 1)
j= j+ j= j+ =
j=1 j=m j=1 j=m
2 2
N
X N (N + 1) − m(m + 1) (N − m)(N + m + 1)
j= =
j=m
2 2
Another way to show this is the following.

S= 1+ 2+ · · · + (N − 2)+ (N − 1)+ N
but S= N+ N − 1+ · · · + 3+ 2+ 1
N (N + 1)
2S = (N + 1)N S=
2
PN
Telescoping series will let you get j=1 j2 and other powers of j.
N
X N (N + 1)
(2j − 1) = 2 − N = N2
j=1
2
N
X XN XN µ ¶
2 2 2 2 N (N + 1)
(j − (j − 1) ) = (j − (j − 2j + 1)) = (2j − 1) = 2 − N = N2
j=1 j=1 j=1
2
N
X N
X N
X
(j 3 − (j − 1)3 ) = N 3 = (j 3 − (j 3 − 3j 2 + 3j − 1)) = (3j 2 − 3j + 1)
j=1 j=1 j=1
N
X N
N (N + 1) 2N + 2N − 3N − 3N N (N + 1)(2N + 1) X 2
3 2
=⇒ 3 j 2 = −3 + N = N 3 =⇒ = = j
j=1
2 2 6 j=1
N
X N
X N
X
j 4 − (j − 1)4 = N 4 = j 4 − (j 4 − 4j 3 + 6j 2 − 4j + 1) = 4j 3 − 6j 2 + 4j − 1 =
j=1 j=1 j=1
N
X N (N + 1)(2N + 1) N (N + 1)
=4 j3 − 6 +4 − N = N4
j=1
6 2
N
X 1 4 1
=⇒ j3 = (N + N (N + 1)(2N + 1) − 2N (N + 1) + N ) = (N 4 + (2N )N (N + 1) − N (N + 1) + N )
j=1
4 4
1 4 1 1 (N (N + 1))2
= (N + 2N 3 + 2N 2 − N 2 − N + N ) = N 2 (N 2 + 2N + 1) =
4 4 4 2
8
Exercise 1. Induction proof.

N
X +1 n
X
1(1 + 1) n(n + 1) n(n + 1) + 2(n + 1) (n + 2)(n + 1)
j= j+n+1= +n+1= =
2 j=1 j=1
2 2 2

Exercise 6.

(1)
1 1 8k + 8 (2k + 3)2
A(k + 1) = A(k) + k + 1 = (2k + 1)2 + k + 1 = (4k 2 + 4k + 1) + =
8 8 8 8
(2) The n = 1 case isn't true.
(3)
(n + 1)n n2 + n n2 + n + 14
1 + 2 + ··· + n = = <
2 2 2
µ ¶2 2 2
2n + 1 1 (n + 1/2) n + n + 1/4
and = =
2 2 2 2
Exercise 7.
(1 + x)2 > 1 + 2x + 2x2
1 + 2x + x2 > 1 + 2x + 2x2
0 > x2 =⇒ Impossible

(1 + x) = 1 + 3x + 3x2 + x3 > 1 + 3x + 3x2


3

=⇒ x3 > 0
By well-ordering principle, we could argue that n = 3 must be the smallest number such that (1 + x)n > 1 + 2x + 2x2 . Or
we could nd, explicitly
Xn µ ¶ n µ ¶
n j n(n − 1) 2 X n j
(1 + x)n = x = 1 + nx + x + x
j=0
j 2 j=3
j
and
n(n − 1)
>n
2
n2 − n > 2n
n2 > 3n
n>3
Exercise 8.
a2 ≤ ca1 , a3 ≤ ca2 ≤ c2 a1
an+1 ≤ can ≤ ca1 cn−1 = a1 cn
Exercise 9. √
n = 1, 1 = 1
p √ q√ √
12 + 12 = 2 ( 2)2 + 12 = 3
q√ √
( n)2 + 12 = n + 1
Exercise 10.
1 = qb + r
q = 0, b = 1, r = 1
2 = qb + r, q = 0, r = 2, b = 1, 2 or r = 0, q = 2; q = 1, r = 0
Assume n = qb + r; 0 ≤ r < b; b ∈ Z+ , b xed
n + 1 = qb + r + 1 = qb + 1 + r = qb + 1 + b − 1 = (q + 1)b + 0
Exercise 11. For n > 1, n = 2, 3 are prime. n = 4 = 2(2), a product of primes.
Assume the k − 1th case. Consider kj , 1 ≤ j ≤ k .
k +
If
j ∈ Z , only for j = 1, j = k , then k prime.
k + k +
If
j ∈ Z , for some 1 < j < k , j = c ∈ Z . c, j < k .
Thus k = cj . c, j are products of primes or are primes, by induction hypothesis. Thus k is a product of primes.
9
Exercise 12. n = 2. G1 , G2 are blonde. G1 has blue eyes. Consider G2 . G2 may not have blue eyes. Then G1 , G2 are not all
blue-eyed.

I 4.7 Exercises - Proof of the well-ordering principle, The summation notation.


Exercise 1.

n(n+1) P4
(1) = k=1 k = 10
P52 P3
(2) n=2 2n−2 = n=0 2n = 1 + 14 = 15
P3 P3
(3) 2 22r = 2 r=0 4r = 170
P4 r=0j
(4) j = 1 + 4 + 27 + 44 = 288
P5j=1 5(6)
(5) j=0 (2j + 1) = 2 ³2 + 6(1)´= 36
P 1 Pn 1 1 1 n
(6)
k(k+1) = k=1 k − k+1 = 1 − n+1 = n+1

Exercise 2.
Pn Pn Pn
(1) Want: k=1 (ak + bk ) = k=1 ak + k=1 bk (additive property)

a1 + b1 = a1 + b1
(a1 + b1 ) + (a2 + b2 ) = (a1 + a2 ) + (b1 + b2 )
(a1 + b1 ) + (a2 + b2 ) + (a3 + b3 ) = (a1 + a2 + a3 ) + (b1 + b2 + b3 )
n+1
X n
X n
X n
X n+1
X n+1
X
(ak + bk ) = (ak + bk ) + an+1 + bn+1 = ak + an+1 + bk + bn+1 = ak + bk
k=1 k=1 k=1 k=1 k=1 k=1
Pn Pn
(2) Want: k=1 (cak ) =c k=1 ak (homogeneous property).

ca1 = (c)a1
ca1 + ca2 = c(a1 + a2 )
ca1 + ca2 + ca3 = c(a1 + a2 + a3 )
n+1
X n
X n+1
X
(cak ) = c ak + can+1 = c ak
k=1 k=1 k=1
Pn
(3) Want: k=1 (ak − ak−1 ) = an − a0 (telescoping property)

n
X n
X n
X n
X
(ak − ak−1 ) = (ak + (−ak−1 )) = ak + (−1)ak−1 =
k=1 k=1 k=1 k=1
n−1
X n−1
X
= an + ak + (−1) ak − a0 = an − a0
k=1 k=1

Pn P
Exercise 3.
k=1 1= k=1 (k − (k − 1)) = n
Exercise 4.

k 2 − (k − 1)2 = k 2 − (k 2 − 2k + 1) = 2k − 1
n
X n
X
(2k − 1) = k 2 − (k − 1)2 = n2 − 0 = n2
k=1 k=1

Pn n2 +n n(n+1)
Exercise 5.
k=1 k= 2 = 2
Exercise 6.
n n
à n
!
X X X n(n + 1) n
3 3 3 2 2
k − (k − 1) = n = 3k − 3k + 1 = 3 (k ) − +
2 3
k=1 k=1 k=1
n
X 3 2
n n n
=⇒ k2 = + +
3 2 6
k=1
10
Exercise 7.
k 4 − (k − 1)4 = −(−4k 3 + 6k 2 + −4k + 1) = 4k 3 − 6k 2 + 4k − 1
n
X X n µ 3 ¶ µ 2 ¶
4 4 4 3 n n2 n n n
k − (k − 1) = n = 4 k −6 + + +4 + −n
3 2 6 2 2
k=1 k=1
X n
n4 + 2n3 + n2
=⇒ = k3
4
k=1

Exercise 8.

(1)
n
X
− (xk+1 − xk ) = −(xn+1 − x) =
k=1
n
X
= (1 − x) xk = x(1 − xn )
k=1
n
X x(1 − xn ) 1 − x 1 − xn+1
=⇒ xk = + =
1−x 1−x 1−x
k=0
(2) If x = 1, the sum equals (n + 1).
Exercise 9.

n = 1(−1)(3) + 5 = 2 = 2n
n = 2(−1)(3) + 5 + (−1)7 + 9 = 4 = 2n
2n
X
n (−1)k (2k + 1) = 2n
k=1
2(n+1) 2n
X X
n+1 (−1)k (2k + 1) = (−1)k (2k + 1) + (−1)2n+1 (4n + 3) + (−1)2n+2 (4n + 5) =
k=1 k=1
= 2n + 2 = 2(n + 1)
Exercise 10.

(1) am + am+1 + · · · + am+n


(2)

1 1 1 1
n=1 = − =
2 1 2 2
2(n+1) 2n 2n
X 1 X (−1)m+1 1 1 1 X (−1)m+1 1 (−1)2n+1+1
n+1 = − + + = +− +
k m=1 m n + 1 2n + 1 2n + 2 m=1 m 2(n + 1) (2n + 1)
k=n+2
2(n+1)
X (−1)m+1
=
m=1
m

Exercise 13.
√ 1 √
n = 12( 2 − 1) < 1 < 2 since > 2 − 1
2 r
√ √
√ √ √ √ n+1+ n 1 1
n case ( n + 1 − n)( n + 1 + n) = n + 1 − n = 1 < √ = ( 1 + + 1)
2 n 2 n
√ √ √ √
n + 1 case ( n + 2 − n + 1)( n + 2 + n + 1) = n + 2 − (n + 1) = 1
q
√ √ 1 + 1
1 + n+1
n+2+ n+1
√ = >1
2 n+1 2
So then, using the telescoping property,

n−1
X Xm Xm
√ √ √ 1 √ √ √ √
2( n + 1 − n) = 2( m − 1) < √ < 2( n − n − 1) = 2( m − 1) < 2 m − 1
n=1 n=1
n n=1
11
I 4.9 Exercises - Absolute values and the triangle inequality. Exercise 1.

(1) |x| = 0 iff x = 0


If x = 0, x = 0, −x = −0 = 0. If |x| = 0, x = 0, −x = 0.
(2)
( (
−x if −x≥0 x if x≥0
| − x| = =
x if −x≤0 −x if x ≤ 0

(3) |x − y| (
= |y − x| by previous exercise and (−1)(x − y) = y − x (by distributivity)
(x)2 if x ≥ 0
(4) |x|2 = = x2
(−x)2 if x ≤ 0
(
√ x if x ≥ 0
(5) 2
x = = |x|
−x if x ≤ 0
(6) We want to show that |xy| = |x||y|
( (
xy if xy ≥ 0 xy if x, y ≥ 0 or x, y ≤ 0
|xy| = =
−xy if xy ≤ 0 −xy if x, −y ≥ 0 or − x, y ≤ 0

( 
 xy if x, y ≥ 0


x|y| if x ≥ 0 −xy if x, −y ≥ 0
|x||y| = =
−x|y| if x ≤ 0  
 −xy if − x, y ≥ 0

xy if − x, −y ≥ 0

(7) By previous exercise, since


¯ ¯
¯x¯
¯ ¯ = |xy −1 | = |x||y −1 |
¯y¯
¯ ¯ (1 1
(
1 1
¯1¯ if ≥ 0 1 if ≥0
¯ ¯= y y
= y−1 y
¯y ¯ −1 1 |y| 1
y if
y ≤0 y if
y ≤0

(8) We know that |a − b| ≤ |a − c| + |b − c|.


Let c = 0 =⇒ |x − y| ≤ |x| + |y|
(9) x = a − b, b − c = −y .
|x| ≤ |x − y| + | − y| |x| − |y| ≤ |x − y|
(10)
(
|x| − |y| if |x| − |y| ≥ 0
||x| − |y|| =
|y| − |x| if |x| − |y| ≤ 0

|x| ≤ |x − y| + | − y| =⇒ |x| − |y| ≤ |x − y|


|y| ≤ |y − x| + | − x| =⇒ |y| − |x| ≤ |y − x| = |x − y|

Exercise 4.

If ∀k = 1 . . . n; ak x + bk = 0
à n
!2 Ã n !2 Ã n !Ã n ! Ã n !Ã n !
X X X X X X
2 2 2 2 2
ak (−xak ) = x ak = ak (−xak ) = ak bk
k=1 k=1 k=1 k=1 k=1 k=1
bk
⇐ Proving ak x + bk = 0 means x = − , ak 6= 0
ak
n
X n
X n
X n
X
(a1 b1 + a2 b2 + · · · + an bn )2 = a2j b2j + aj ak bj bk == a2j b2j + a2j b2k
j=1 j6=q j=1 j6=k

=⇒ a2j b2k
− aj ak bj bk = aj bk (aj bk − ak bj ) = 0
µ ¶
bj
if aj , bk 6= 0, aj bk − ak bj = 0 =⇒ ak + bk = 0
−aj
12
Exercise 8. The trick of this exercise is the following algebraic trick (“multiplication by conjugate”) and using telescoping

property of products:
j j j
+2j j+1
(1 − x2 )(1 + x2 ) = 1 − x2 = 1 − x2
1
Y Y1 j n
j−1 1 − x2 1 − x2
1 + x2 = =
j=1 j=1
1 − x2j−1 1−x
if x = 1, 2n
Exercise 10.
x>1
x2 > x xn+1 = xn x > x2 > x
x3 > x2 > x
0<x<1
x2 < x xn+1 = xn x < x2 < x =⇒ xn+1 < x
X 3 < x2 < x
S = {n ∈ Z+ |2n < n!}.
Exercise 11. Let

By well-ordering principle, ∃ smallest n0 ∈ S . Now


24 = 16, 4! = 24. So S starts at n = 4.
Exercise 12.

(1)
µ ¶n X n µ ¶ µ ¶j n
X µ ¶k
1 n 1 n! 1
1+ = =
n j=0
k n (n − k)!k! n
k=0


k−1
r´ n−rYµ
k−1
1

n!
¶ µ
1− = = k
r=0
n r=0
n n (n − k)!
µ ¶
1 Y³ r´
Xn k−1
1 n!
1− =
k! r=0 n nk (n − k)!
k=1
(2)

n
à k−1
! n n ¡ 1 ¢n+1 µ ¶n
1 X 1 Y r X 1 X 1 1
− 1
(1 + )n = 1 + (1 − ) < 1 + <1+ =1+ 2 2
1 = 1 + (1 − )
n k! r=0 n k! 2k 2
2
k=1 k=1 k=1
<3
The rst inequality obtained from the fact that if 0 < x < 1, xn < x < 1. The second inequality came from the
1 1
<
previous exercise, that
k! 2k
.

n µ ¶ µ ¶k
X n−1 µ ¶ µ ¶k n−1 µ ¶ µ ¶k µ ¶
1 n 1 1 X n 1 1 X n 1 n 1
(1 + )n = =1+ + =1+ + + > >2
n k n n k n n k n 1 n
k=0 k=1 k=2

Exercise 13.

(1)
p−1 µ ¶k ¡ b ¢p
X b 1− a
S= = b
a 1− a
k=0
p−1 ¡ b ¢p
X 1− bp − ap
bk ap−1−k = ap−1 a
=
k=0
1 − ab b−a
(2)
(3) Given
(n + 1)p+1 − np+1
np < < (n + 1)p
p+1
We want
n−1
X X n
np+1
kp < < kp
p+1
k=1 k=1
13
2p+1
n = 21p < < 1p + 2p
p+1
p=1
1 < 22 /2 = 2.2 < 1 + 2 = 3
p−2
1 < 8/3 < 1 + 4 = 5

I 4.10 Miscellaneous exercises involving induction. Exercise 13.

(1)
(2)
(3) Let n = 2.
2−1
X 2
np+1 2p+1 X p
k p = 1p = 1, = k = 1 + 2p
p+1 p+1
k=1 k=1
What makes this exercise hard is that we have to use induction on p itself. Let p = 1.
1+1
2
1< = 2 < 1 + 21 = 3
1+2
Now assume pth case. Test the p + 1 case.
µ ¶
2p+2 2(p + 1) 2p+1
= >1
p+2 p+2 p+1
since p + 2 < 2p + 2 = 2(p + 1) for p ∈ Z+
For the right-hand inequality, we will use the fact just proven, that 2p − (p) > 0 and pth case rewritten in this manner
2p+1
(1 + 2p ) > =⇒ (1 + 2p )(p + 1) > 2p+1
p+1
So
(p + 2)(1 + 2p+1 ) = (p + 2) + ((p + 1) + 1)2p (2) = (p + 2) + 2(p + 1)2p + 2p (2) >
> (p + 2) + 2(2p+1 − (p + 1)) + 2p (2) = −p + 2p+2 + 2p+1 > 2p+2
So the n = 2 case is true for all p ∈ Z+ .
Assume nth case is true. We now prove the n + 1 case.
n
X n−1
X np+1 np+1 (n + 1)p+1 − np+1 (n + 1)p+1
kp = k p + np < + np < + =
p+1 p+1 p+1 p+1
k=1 k=1
n+1
X n
X np+1 (n + 1)p+1 − np+1 (n + 1)p+1
kp = k p + (n + 1)p > + =
p+1 p+1 p+1
k=1 k=1
(n+1)p+1 −np+1
We had used the inequality proven in part b, np < p+1 < (n + 1)p .
Exercise 14. Use induction to prove a general form of Bernoulli's inequality.

1 + a1 = 1 + a1
(1 + a1 )(1 + a2 ) = 1 + a2 + a1 + a1 a2 ≥ 1 + a1 + a + 2

Test the n + 1 case


(1 + a1 )(1 + a2 ) . . . (1 + an+1 ) ≥ (1 + a1 + a2 + · · · + an )(1 + an+1 ) =
= 1 + a1 + a2 + · · · + an + an+1 + an+1 (a1 + a2 + . . . an ) ≥
≥ 1 + a1 + a2 + · · · + an + an+1
Note that the last step depended upon the given fact that all the numbers were of the same sign.
For a1 = a2 = · · · = an = x, then we have (1 + x)n ≥ 1 + nx.
Xn µ ¶
n n j
(1 + x) = x = 1 + nx
j=0
j

Since x and n are arbitrary, we can compare terms of xj 's. Then x = 0.


2! 1 3! 2
Exercise 15. 2 =
2 2 33 = 9 < 1.
14
n!
¡ 1 ¢k n
So we've shown the n = 2, n = 3 cases. Assume the nth case, that nn ≤ 2 , where k is the greatest integer ≤ 2.

¡ ¢k µ ¶n µ ¶k µ ¶n µ ¶k µ ¶k µ ¶k+1
(n + 1)! (n + 1)nn 12 n 1 1 1 1 1 1
≥ = = 1− < =
(n + 1)n+1 (n + 1)n+1 n+1 2 n+1 2 2 2 2
where in the second to the last step, we had made this important observation:

n 1 n+1 1 1 1
k≤ =⇒ k + ≤ =⇒ ≤ <
2 2 2 n+1 2k + 1 2

Exercise 16.


1+ 5
a1 = 1 <
2
à √ !2 √ √
1+ 5 1+2 5+5 6+2 5
a2 = 2 < = =
2 4 4
à √ !n à √ !n−1 à √ !n µ ¶
1+ 5 1+ 5 1+ 5 2
an+1 = an + an−1 < + = 1+ √ =
2 2 2 1+ 5
à √ !n à √ ! à √ !n+1
1+ 5 2(1 − 5) 4 1+ 5
= + =
2 1−5 4 2

Exercise 17. Use Cauchy-Schwarz, which says

³X ´2 ³X ´ ³X ´
ak bk ≤ a2k b2k

Let ak = xpk and bk = 1. Then Cauchy-Schwarz says

³X ´2 ³X ´ X P 2
( xpk )
xpk ≤ x2p
k n =⇒ (x2p
k ) ≥
n
We dene Mp as follows:

µ Pn ¶1/p
k=1 xpk
Mp =
n
So then
n
X
nMpp = xpk
k=1
ÃP !1/2p
n
k=1 x2p
k
M2p =
n X (nMpp )2
x2p 2p
k = nM2p ≥ = nMp2p
n
X n
2p
nM2p = x2p
k
k=1
2p
M2p ≥ Mp2p =⇒ M2p ≥ Mp

Exercise 18.
µ ¶1/4 µ ¶1/2
a4 + b4 + c4 a2 + b2 + c2 23/2
≥ = since
3 3 31/2
64
a4 + b4 + c4 ≥
3
Pn
Exercise 19. ak = 1, k=1 1=n
15
Now consider the case of when not all ak = 1.
a1 = 1
a1 a2 = 1 and suppose, without loss of generality a1 > 1. Then 1 > a2 .
(a1 − 1)(a2 − 1) < 0
a1 a2 − a1 − a2 + 1 < 0 =⇒ a1 + a2 > 2
(consider n + 1 case ) If a1 a2 . . . an+1 = 1, then suppose a1 > 1, an+1 < 1 without loss of generality

b1 = a1 an+1
b1 a2 . . . an = 1 =⇒ b1 + a2 + · · · + an ≥ n (by the induction hypothesis)
(a1 − 1)(an+1 − 1) = a1 an+1 − a1 − an+1 + 1 < 0, b1 < a1 + an+1 − 1
=⇒ a1 + an+1 − 1 + a2 + · · · + an > b1 + a2 + · · · + an ≥ n
=⇒ a1 + a2 + · · · + an+1 ≥ n + 1
1.7 Exercises - The concept of area as a set function. We will use the following axioms:
Assume a class M of measurable sets (i.e. sets that can be assigned an area), set function a, a : M → R.

Axiom 2 (Nonnegative property).

(1) ∀S ∈ M, a(S) ≥ 0

Axiom 3 (Additive property). If S, T ∈ M, then S ∪ T, S ∩ T ∈ M and
(2) a(S ∪ T ) = a(S) + a(T ) − a(S ∩ T )

Axiom 4 (Difference property). If S, T ∈ M, S ⊆ T then T − S ∈ M and
(3) a(T − S) = a(T ) − a(S)

Axiom 5 (Invariance under congruence). If S ∈ M, T = S , then T ∈ M, a(T ) = a(S)

Axiom 6 (Choice of scale). ∀ rectangle R ∈ M, if R has edge lengths h, k then a(R) = hk

Axiom 7 (Exhaustion property). Let Q such that
(4) S⊆Q⊆T
If ∃ only one c such that a(S) ≤ c ≤ a(T ), ∀S, T such that they satisfy Eqn. (??)
then Q measurable and a(Q) = c
Exercise 1.

(1) We need to say that we consider a line segment or a point to be a special case of a rectangle allowing h or k (or both)
to be zero.
LetTl = { line segment containing x0 }, Q = {x0 }.
Q, only ∅ ⊂ Q
For
By Axiom 3, let T = S .

a(T − S) = a(∅) = a(T ) − a(T ) = 0


∅ ⊂ Q ⊆ Tl =⇒ a(∅) ≤ a(Q) ≤ a(Tl ) =⇒ 0 ≤ a(Q) ≤ 0
=⇒ a(Q) = 0
(2)  
N
[ N
X
a Qj  = a(Qj )
j=1 j=1

if Qj 's disjoint. Let Qj = {xj }.


³S ´
N
Since a(Qj ) = 0. By previous part, a j=1 Qj = 0
16
Exercise 2. Let A, B A, B are measurable. By Axiom 2, A ∩ B measurable.
be rectangles. By Axiom 5,
p 1 p 1
a(A ∩ B) = a2 + b2 d + ab − ( ab + a2 + b2 d) = ab
2 2
Exercise 3. Prove that every trapezoid and every parallelogram is measurable and derive the usual formulas for their areas.

A trapezoid is simply a rectangle with a right triangle attached to each end of it. Tr = R + T1 + T2 . T1 , T2 are right
triangles and so by the previous problem, T1 , T2 are measurable. Then Tr is measurable by the Additive property axiom (note
that the triangles and the rectangle don't overlap).
We can compute the area of a trapezoid:

Tr = R + T1 + T2 =⇒ a(Tr ) = a(R) + a(T1 ) + a(T2 )


1 1 1
a(Tr ) = hb1 + h(b2 − b1 )/2 + h(b2 − b1 )/2 = h(b1 + h2 )
2 2 2
P = R (a parallelogram consists of a right triangle rotated by π and attached to the other side of the same right triangle;
the two triangles do not overlap). Since two right triangles are measurable, the parallelogram, P is measurable.
1
Using the Additive Axiom, a(P ) = 2a(T ) = 2 bh = bh
2
Exercise 4. A point (x, y) in the plane is called a lattice point if both coordinates x and y are integers. Let P be a polygon
1
whose vertices are lattice points. The area of P is I + B − 1, where I denotes the number of lattice points inside the polygon
2
and B denotes the number on the boundary.

(1) Consider one side of the rectangle lying on a coordinate axis with one end on the origin. If the rectangle side has
length l, thenl + 1 lattice points lie on this side (you have to count one more point at the 0 point. Then consider the
same number of lattice points on the opposite side. We have 2(l + 1) lattice points so far, for the boundary.
The other pair of sides will contribute 2(h−1) lattice points, the −1 to avoid double counting. Thus 2(l +h) = B .
I = (h − 1)(l − 1) by simply considering multiplication of (h − 1) rows and (l − 1) columns of lattice points
inside the rectangle.

I + 21 B − 1 = hl − h − l + 1 + (l + h) − 1 = hl = a(R)
(2)
(3)

Exercise 5. Prove that a triangle whose vertices are lattice points cannot be equilateral.

My way: I will take, for granted, that we know an equilateral triangle has angles of π/3 for all its angles.
Even if we place two of the vertices on lattice points, so that its length is 2L, and put the midpoint and an intersecting
perpendicular bisector on a coordinate axis (a picture would help), but the ratio of the perpendicular bisector to the third
vertex to half the length of the triangle iscot π/3 = √13 . Even if we go down by an integer number L, L steps down, we go

“out” to the third vertex by an irrational number 3L. Thus, the third vertex cannot lie on a lattice point.
Exercise 6. Let A = {1, 2, 3, 4, 5} and let M denote the class of all subsets of A. (There are 32 altogether counting A itself
A
and the empty set ∅). (My Note: the set of all subsets, in this case, M, is called a power set and is denoted 2 . This is
A
because the way to get the total number of elements of this power set, |2 |, or the size, think of assigning to each element a
“yes,” if it's in some subset, or “no”, if it's not. This is a great way of accounting for all possible subsets and we correctly
get all possible subsets.) For each set S in M, let n(S) denote the number of distinct elements in S . If S = {1, 2, 3, 4} and
T = {3, 4, 5},
[
n(S T) = 5
\
n(S T) = 2
n(S − T ) = n({1, 2}) = 2
n(T − S) = n({5}) = 1
n S T S T S, T are
satises nonnegative property because by denition, there's no such thing as a negative number of elements. If
A, so are S T , S T since every element in S ST , S T is in S . Thus n could
subsets of
T be assigned to it, so that it's
T
measurable. Since n counts only distinct elements, then n(S T ) = n(S) + a(T ) − a(S T ), where −a(S T ) ensures
there is no double counting of distinct elements. Thus, the Additive Property Axiom is satised.

For S ⊆ T , then ∀ x ∈ T − S , x ∈ T, x ∈
/ S Now S ⊆ T , so ∀ x ∈ S , x ∈ T . Thus T −S is complementary to S “with
respect to” T . n(S) + n(T − S) = n(T ), since n counts up distinct elements.
17
1.11 Exercises - Intervals and ordinate sets, Partitions and step functions, Sum and product of step function. Exercise
4.

(1)
[x + n] = y ≤ x + n, y ∈ Z; y − n ≤ x
[x] + n = z + n ≤ x + n
If y − n < z, then y < z + n ≤ x + n. then y wouldn't be the greatest integer less than x + n

=⇒ y = z + n
(2)
= y2 ≤ x − [x] = −y2 ≥ −x − y2 − 1 ≤ x
−x ≥ y1 = [−x] = −y2 − 1 = −[x] − 1; ( and y1 = −y2 − 1 since −y2 > −x )
Ifx is an integer −[x] = [−x]
(3) Let x = q1 + r1 , y = q2 + r2 ; 0 ≤ r1 , r2 < 1.
(
q1 + q2
= [q1 + q2 + r1 + r2 ] =
q1 + q2 + 1 if r1 + r2 ≥ 1
[x] + [y] = q1 + q2 [x] + [y] + 1 = q1 + q2 + 1
(4)
1
Ifx is an integer , [2x] = 2x = [x] + [x + ] = [x] + [x] = 2x
( 2
1
1 q if r <
2
[x] + [x + ] = q +
2 2q + 1 if r > 12
(
1
2q if r <
2
[2x] = [2(q + r)] = [2q + 2r] =
2q + 1 if r > 12
(5)

( (  r < 13
1 2 q if r< 2
q if r< 1 3q if
3 3 1 2
[x] + [x + ] + [x + ] = q + 2
+ 1
= 3q + 1 if
3 <r < 3
3 3 q + 1 if r > 3 q+1 if r >
3

 2
3q + 2 if r >
3


3q if r < 31
1 2
[3x] = [3(q + r)] = [3q + 3r] = 3q + 1 if
3 <r < 3

 2
3q + 2 if r >
3

Exercise 5. Direct proof.



nq if r < n1
1 2
[nx] = [n(q + r)] = nq + 1 if
n <r < n

 n−1
nq + n − 1 if r >
n
Exercise 6.
1
a(R) = hk = IR + BR − 1
2
b
X
[f (n)] = [f (a)] + [f (a + 1)] + · · · + [f (b)]
n=a
[f (n)] = g ≤ f (n), g ∈ Z, so that if f (n) is an integer,g = f (n), and if f (n) is not an integer, g is the largest integer such
that g < f (n), so that all lattice points included and less than g are included.
Exercise 7.

b + 1 lattice points as the base with b length.


(1) Consider a right triangle with lattice points as vertices. Consider
Start from the vertex and move across the base by increments of 1.
a
The main insight is that the slope of the hypotenuse of the right triangle is
b so as we move 1 along the base, the
a
hypotenuse (or the y -value, if you will) goes up by . Now
b
h na i
(5) = number of interior points at x = n and below the hypotenuse line of the right triangle of sides a, b,
b
including points on the hypotenuse

18
b−1 h
X na i 1 ab
+ ((a + 1) + b) − 1 =
n=1
b 2 2
(a − 1)(b − 1) ab a b 1
Now = − − +
2 2 2 2 2
b−1 h
X i
na (a − 1)(b − 1)
=⇒ =
n=1
b 2
(2) a, b ∈ Z+
b−1 h b−1 · ¸
X na i X a(b − n)
= (reverses order of summation)
b n=1 n=1
b
b−1 h
( P £ ¤ ¡n ¢
X an i
b−1
− n=1 an −a if
an
− a4 is an integer (but a −1 can't be!)
a− = Pb−1 ¡£ban ¤ ¢ b b

n=1
b − n=1 b − a − 1 otherwise

b−1 ³h i ´ b−1 ³h
X an X an i ´
=− −a −1 =− − a − (b − 1) =
n=1
b n=1
b
b−1 h
X an i
=− + a(b − 1) − (b − 1)
n=1
b
b−1 h
X na i (a − 1)(b − 1)
=
n=1
b 2

Exercise 8. Recall that for the step function f = f (x), there's a partition P = {x0 , x1 , . . . , xn } of [a, b] such that f (x) = ck
if x ∈ Ik . (
1 ∀x ∈ S
Given that χs (x) = .
0 ∀x ∈
/S
If x ∈ [a, b], then x must only lie in one open subinterval Ij , since real numbers obey transitivity.
n
X n
X
ck χIk (x) = cj for x ∈ Ij =⇒ ck χIk (x) = f (x)∀x ∈ [a, b]
k=1 k=1

1.15 Exercises - The denition of the integral for step functions, Properties of the integral of a step function, Other
notations for integrals. Exercise 1.
R3
(1) [x]dx = (−1) + 1 + (2) = 2
R13 R 7/2
(2)
−1
[x + 21 ]dx = −1/2 [x]dx = (−1) 12 + (1)(1) + (2)(1) + 12 3 = 4
R3
(3) ([x] + [x + 12 ])dx = 6
R−1
3
(4) 2[x]dx = 4
R−1
3 1
R6
(5) [2x]dx = 2 [x]dx = 12 ((−2)1 + (−1) + (1) + 2 + 3 + 4 + 5) = 6
R−1
3 R−2−3 R1
(6)
−1
[−x]dx = − 1 [x]dx = −3 [x]dx = −3 + −2 + −1 = −6
Exercise 2. (
5/2 if 0<x<2
s=
−1 if 2<x<5
Exercise 3. [x] = y ≤ x so −y ≥ −x.
−y − 1 ≤ −x, otherwise if −y − 1 ≥ −x, y + 1 ≤ x and so y wouldn't be the largest integer ≤ x.
=⇒ [x] + [−x] = y − y − 1 = −1
Or use Exercise 4(c), pp. 64.
Z b Z b Z b
([x] + [−x])dx = [x − x]dx = (−1)dx = a − b
a a a
Exercise 4.
Rn Pn−1 (n−1)(n−1+1) (n−1)n
(1) n ∈ Z+ , 0
[t]dt = t=0 t= 2 = 2
(2)
19
Exercise 5.
R2 2 R2 √ √ √ √ √ √
(1) [t ]dt = 1 [t2 ]dt = 1( 2 − 1) + 2( 3 − 2) + 3(2 − 3) = 5 − 2 − 3
R03 2 R3 2 R0 2 R3 2 R0 2 R3 2
(2)
−3
[t ]dt = 0 [t ]dt + −3 [t ]dt = 0 [t ]dt + − 3 [t ]dt = 2 0 [t ]dt
Z 3 √ √ √ √ √ √ √ √
[t2 ]dt = 4( 5 − 2) + 5( 6 − 5) + 6( 7 − 6) + 7( 8 − 7) + 8(3 − 8)
2
√ √ √ √
16 − 5 − 6 − 7 − 8
Z 2 Z 3 √ √ √ √ √ √
2
[t ]dt + [t2 ]dt = 21 − 3 2 − 3 − 5 − 6 − 7 − 8
0 2
Z 3 √ √ √ √ √
=⇒ [t2 ]dt = 42 − 2(3 2 + 3 + 5 + 6 + 7)
−3

Exercise 6.
Rn Rn Pn−1
(1) [t]2 dt = [t]2 dt = j=1 j 2 = (n−1)n(2n−1)
6
R0x 2 P1
[x−1]
(2)
0
[t] dt = j=1 j 2 + q 2 r where x = q + r, q ∈ Z+ , 0 ≤ r < 1.
Z x
q(q − 1)(2q − 1)
[t]2 dt = + q 2 r = 2(x − 1) = 2(q + r − 1)
0 6
=⇒ q(q − 1)(2q − 1) + 6q 2 r = 12q + 12r − 12
=⇒ x = 1, x = 5/2

Exercise 7.

(1)
Z 9 √ Z 9√
[ t]dt = [ t]dt = 3(1) + 5(2) = 13
0 1
Z 1 √ (4)(3)(17)
6[ t]dt = 3(1) + 5(2) + 7(3) = 34 =
0 6
Z √ n2
n(n − 1)(4n + 1)
Assume [ t]dt =
0 6
Z (n+1)2 Z n2 Z (n+1)2
√ √ √ n(n − 1)(4n + 1)
[ t]dt = [ t]dt + [ t]dt = + n((n + 1)2 − n2 ) =
0 0 n2 6
(n2 − n)(4n + 1) + 6n(2n + 1) 4n3 + n2 − 4n2 − n + 12n2 + 6n 4n3 + 9n2 + 5n
= = =
6 6 6
indeed ,
(n + 1)(n)(4(n + 1) + 1) (n2 + n)(4n + 5) 4n3 + 5n2 + 4n2 + 5n
= =
6 6 6
R b+c R b+c−c Rb
Exercise 8.
a+c
f (x)dx = a+c−c f (x − (−c))dx = a f (x + c)dx
R kb 1
R (kb)/k ³ x ´ Rb
Exercise 9.
ka
f (x)dx = 1 (ka)/k
f 1/k dx = k a f (kx)dx
k
Rp
Exercise 10. Given s(x) = (−1)n n if n ≤ x < n + 1; n = 0, 1, 2, . . . p − 1; s(p) = 0, p ∈ Z+ . f (p) = 0
s(x)dx.
R3
So for f (3) = 0
s(x)dx, we need to consider n = 0, 1, 2.
s(0 ≤ x < 1) = 0
s(1 ≤ x < 2) = (−1)(1)
s(2 ≤ x < 3) = 2;
s(3 ≤ x < 4) = −3
So then
f (3) = (−1)(1) + 2(1) = 1
f (4) = 1 + (−3)(1) = −2
f (f (3)) = f (1) = 0
20
We obtain this formula
(
p p+1
f (p) = 2 (−1) p even
since
p−1 p+1
2 (−1) p even
Z p+1 n
f (p + 1) = f (p) + s(x)dx = p−1
2 (−1)
p+1
p even + (−1)p p
p
( ( (
−p p
2 p even p 2
= p−1 + = −p−1 =
2 p odd −p 2
(
− (p+1)
2 if p + 1 even
= p
2 if p + 1 odd
Thus, p = 14, p = 15.
Exercise 11.

(1)
Z b n
X
s(x)dx = s3k (xk − xk−1 )
a k=1
Z b Z c n1
X n2
X n2
X Z c
s+ s= s2k (xk − xk−1 ) + s3k (xk − xk−1 ) = s3k (xk − xk−1 ) = s(x)dx
a b k=1 k=n1 k=1 a
Rb Pn3 Rb Rb
(2)
a
(s + t) = k=1 (s + t)3k (xk − xk−1 ) 6= a s + a t
Rb Pn Rb
(3)
a
cs = k=1 (cs)3 (xk − xk−1 ) 6= c a s
(4) Consider these facts that are true, that xk−1 < x < xk , s(x) = sk ; x0 = a + c, xn = b + c,
xk−1 − c < x − c < xl − c =⇒ yk−1 < y < yk so then s(y + c) = sk .
n
X k
X
s3k (xk − xk−1 ) = s3k (xk − c − (xk−1 − c)) =
k=1 k=1
Xn Z b
= s3k (yk − yk−1 ) = s(y + c)dy
k=1 a
Rb Pn 3
(5) s < t, a
s= k=1 sk (xk − xk−1 ).
if 0 < s, s3 < s2 t < st2 < t3
if s < 0t, s3 < and t3 > 0
ts > t2
if s < t < 0, s3 < s2 t, s(st) < t(ts) = t2 s
t2 s < t3
s3 < s2 t < t2 s < t3
Rb Rb
Then
a
s< a
t.
Exercise 12.
Rb Rc Pn1 Pn2 Pn3 Rc
(1)
a
s+ b s = k=1 sk (x2k − x2k−1 ) + k=n s (x2k − x2k−1 ) = k=1
1 k
sk (x2k − x2k−1 ) = a s
Rb Pn3 2 2
Pn3 2 2
Pn3 2 2
Pn3 2 2
(2)
a
(s+t) = k=1 (s+t)k (xk −xk−1 ) = k=1 (sk +tk )(xk −xk−1 ) = k=1 sk (xk −xk−1 )+ k=1 tk (xk −xk−1 )

since P3 = {xk } is a ner partition than the partition for s, P1 , t, P2 , then consider
sk (yj2 − yj−1
2
) = sk ((x2k+1 − x2k ) + (x2k − x2k−1 )), so
n3
X n3
X n1
X n2
X
sk (x2k − x − k − 12 ) + tk (x2k − x − k − 12 ) = sj (x2j − x − j − 12 ) + tj (x2j − x − j − 12 ) =
k=1 k=1 j=1 j=1
Z b Z b
= s+ t
a a
Rb Pn Pn Rb
(3)
a
cs = k=1 csk (x2k − x2k−1 ) = c 2
k=1 sk (xk − x2k−1 ) = c a
s
21
R b+c Pn 2
(4)
a+c
s(x)dx = k=1 sk (xk − x2k−1 ) where

s(x) = sk if xk−1 < x < xk


x(y + c) = sk if xk−1 < y + c < xk =⇒ xk−1 − c < y < xk − c =⇒ yk−1 < y < yk

where P 0 = {yk } is a partition on [a, b]

Z b n
X
s(y + c)dy = sk (yk2 − yk−1
2
)=
a k=1
Xn n
X
= sk ((xk − c)2 − (xk−1 − c)2 ) = sk (x2k − 2xk c + c2 − (x2k−1 − 2xk−1 c + c2 )) =
k=1 k=1
Xn n
X
= sk (x2k − x2k−1 − 2c(xk − xk−1 )) 6= sk (x2k − x2k−1 )
k=1 k=1
Pn Rb Pn Rb
(5) Since x2k − x2k−1 > 0,
a
sdx = k=1 sk (x2k − x2k−1 ) < k=1 tk (x2k − x2k−1 ) = a tdx
Note that we had shown previously that the integral doesn't change under ner partition.

Exercise 13.

Z b n
X Z b n2
X
s(x)dx sk (xk − xk−1 ); t(x)dx = tk (yk − yk−1 )
a k=1 a k=1
P = {x0 , x1 , . . . , xn }, Q = {y0 , y1 , . . . , yn }
Note that x0 =S y0 = a; xn = yn2 = b.
Consider P Q = R. R consists of n3 elements, (since n3 ≤ n + n2 some elements of P and Q may be the same. R is
another partition on [a, b] (by partition denition) since xk , yk ∈ R and since real numbers obey transitivity, {xk , yk } can be
arranged such that a < z1 < z2 < · · · < zn3 −2 < b where zk = xk or yk .

(s + t)(x) = s(x) + t(x) = sj + tk if xj−1 < x < xj ; yj−1 < x < yj


If xj−1 ≶ yj−1 , let zl−1 = yj−1 , xj−1 and

If xj ≶ yj , let zl = xj , yj

Let sj = sl ; tk = tl
(s + t)(x) = s(x) + t(x) = sl + tl , if zl−1 < x < zl
Z b Z b n3
X n3
X n3
X
(s(x) + t(x))dx = ((s + t)(x))dx = (sl + t)l)(zl − zl−1 ) = sl (zl − zl−1 ) + tl (zl − zl−1 )
a a l=1 l=1 l=1

In general, it was shown (Apostol I, pp. 66) that any ner partition doesn't change the integral R is a ner partition. So

n3
X n
X n
X n2
X Z b Z b
sl (zl − zl−1 ) + tl (zl − zl−1 ) = sk (xk − xk−1 ) + tk (yk − yk−1 ) = s(x)dx + t(x)dx
l=1 l=1 k=1 k=1 a a

Exercise 14. Prove Theorem 1.4 (the linearity property).

Z b Z b n
X n2
X
c1 s(x)dx + c2 t(x)dx = c1 sk (xk − xk−1 ) + c2 tk (xk − xk−1 ) =
a a k=1 k=1
n3
X n3
X n3
X
= c1 sl (zl − zl−1 ) + c2 tl (zl − zl−1 ) = (c1 sl + c2 tl )(zl − zl−1 ) =
l=1 l=1 l=1
Z b
= (c1 s + c2 t)(x)dx
a

We relied on the fact that we could dene a ner partition from two partitions of the same interval.

Exercise 15. Prove Theorem 1.5 (the comparison theorem).

22
s(x) < t(x) ∀x ∈ [a, b]; s(x)(zl − zl−1 ) < t(x)(zl − zl−1 ) (zl − zl−1 > 0)
Z b n
X n3
X n3
X n2
X
s(x)dx = sk (xk − xk−1 ) = sl (zl − zl−1 ) < tl (zl − zl−1 ) = tk (yl − yk−1 ) =
a k=1 l=1 l=1 k=1
Z b
= t(x)dx
a
Z b Z b
=⇒ s(x)dx < t(x)dx
a a
Exercise 16. Prove Theorem 1.6 (additivity with respect to the interval).

Use the hint: P1 is a partition of [a, c], P2 is a partition of [c, b], then the points of P1 along with those of P2 form a
partition of [a, b].
Z c Z b n1
X n2
X n3
X Z b
s(x)dx + s(x)dx = sl (xk − xk−1 ) + sk (xk − xk−1 ) = sk (xk − xk−1 ) = s(x)dx
a a k=1 k=1 k=1 a

Exercise 17. Prove Theorem 1.7 (invariance under translation).

P 0 = {y0 , y1 , . . . , yn }; yk = xk + c;
=⇒xk−1 + c < y < xk + c
xk−1 < y − c < xk
yk − yk−1 = xk + c − (xk−1 + c) = xk − xk−1

s(y − c) = sk xk−1 < y − c < xk , k = 1, 2, . . . n


if
Z b n
X n
X Z yn Z b+c
s(x)dx = sk (xk − xk−1 ) = sk (yk = yk−1 ) = s(y − c)dy = s(x − c)dx
a k=1 k=1 y0 a+c

1.26 Exercises - The integral of more general functions, Upper and lower integrals, The area of an ordinate set ex-
pressed as an integral, Informal remarks on the theory and technique of integration, Monotonic and piecewise mono-
tonic functions. Denitions and examples, Integrability of bounded monotonic functions, Calculation of the integral
Rb
of a bounded monotonic function, Calculation of the integral
0
xp dx when p is a positive integer, The basic properties
of the integral, Integration of polynomials.

R2
Exercise 16.
0
|(x − 1)(3x − 1)|dx =
Z 2 Z 2 ¯2
(x − 1)(3x − 1)dx = (3x2 − 4x + 1)dx = (x3 − 2x2 + x)¯1 = 2
1 1
Z 1 ¯1 4
(1 − x)(3x − 1)dx = − (x3 − 2x2 + x)¯1/3 =
1/3 27
Z 1/3
4
(x − 1)(3x − 1)dx =
0 27
So the nal answer for the integral is 62/27.
R3 R3 R 3−5/2 ¯1/2
Exercise 17.
0
(2x − 5)3 dx = 8 0 (x − 52 )3 dx = 8 −5/2 x3 dx = 8 14 x4 ¯−5/2 = 39
2
R3 2 R3 2 Rx 2 R3 2 R3 2
Exercise 18.
−3
3 3 3 2
(x − 3) dx = 0 (x − 3) + −3 (x − 3) = 0 (x − 3) + − 0 (x − 3)3 = 0

2.4 Exercises - Introduction, The area of a region between two graphs expressed as an integral, Worked examples.
Exercise 15. f = x2 , g = cx3 , c > 0
1
For 0<x< c, cx < 1 (since c > 0). So cx3 < x2 (since x2 > 0).
Z Z µ ¶¯1/c
1 3 c 4 ¯¯ 1
f − g = x2 − cx3 = x − x ¯ = 3
3 4 0 12c
Z
2 1 1
f −g = = 3
; c= √
3 12c 2 2
23
Exercise 16. f = x(1 − x), g = ax.
Z Z 1−a µ ¶¯1−a
2 1 2 1 3 ¯¯ 1
f −g = x − x − ax = (1 − a) x − x ¯ = (1 − a)3 = 9/2 =⇒ a = −2
0 2 3 0 6
R1 √
Exercise 17. π=2 −1
1 − x2 dx
(1)
Z Z r ³ x ´2 Z
3 p 3 p1

9− x2 dx =3 1− = 3(3) 1 − x2 =
−3 −3 3 −1 2
Now
Z kb ³x´ Z b
f dx = k f dx
ka k a
(2)
Z r Z
2
1 1 p 2π π
1 − x2 dx = 2 1 − x2 dx = =
0 4 0 4 2
R2 √
(3)
−2
(x − 3) 4 − x2 dx
Z 2 p Z −2 p Z 2 p
x 4 − x2 dx = (−1) −x 4 − x2 =⇒ 2 x 4 − x2 = 0
−2 2 −2
Z 2 r ³ x ´2 Z 1p
−3 2 1− dx = (−6)(2) 1 − x2 = −6π
−2 2 −1

Exercise 18. Consider a circle of radius 1 and a twelve-sided dodecagon inscribed in it. Divide the dodecagon by isosceles

triangle pie slices. The interior angle that is the vertex angle of these triangles is 360/12 = 30 degrees.
Then the length of the bottom side of each triangle is given by the law of cosines:

√ ! s √ Ã
2 ◦ 3 √ 3
c = 1 + 1 − 2(1)(1) cos 30 = 2 1 − =⇒ c = 2 1 −
2 2

The height is given also by the law of cosines


s
r √
3
1+ cos 30◦ 1+
h = 1 cos 15◦ = = 2
2 2
The area of the dodecagon is given by adding up twelve of those isosceles triangles
s  s 
√ µ ¶ √
1 3 1  √ 3
(12) 1+ √ 2 1− =3
2 2 2 2

So 3 < π.
Now consider a dodecagon that's circumscribing the circle of radius 1.
 v  Ã
u √ √ !
1 u 1− 3
3
(12) 2t √2  (1) = 12 2 − >π
2 1+ 3 2
2

Exercise 19.

(1) (x, y) ∈ E if x = ax1 , y = by1 such that x21 + y12 ≤ 1


¡ ¢2 ¡ ¢2
=⇒ xa + yb = 1
(2)
³ x ´2 r
1− y=b
a
Z a r ³ x ´2 Z 1p
π
2 b 1− = 2ba 1 − x2 = ba (2) = πba
−a a −1 2
24
Exercise 20. Let f be nonnegative and integrable on [a, b] and let S be its ordinate set.

Suppose x and y coordinates of³ S were


´ expanded in different ways x = k1 x1 , y = k2 y1 .
x
If f (x1 ) = y1 , g(x) = k2 f
k1 = k2 y1 = y .

integrating g on [k1 a, k1 b],


Z k1 b Z k1 b µ ¶ Z b
x
g(x)dx = k2 f dx = k2 k1 f (x)dx = k2 k1 A
k1 a k1 a k1 a

2.8 Exercises - The trigonometric functions, Integration formulas for the sine and cosine, A geometric description of
the sine and cosine functions. Exercise 1.

(1) sin π = sin 0 = 0. sine is periodic by 2π , so by induction, sin nπ = 0.


sin 2(n + 1)π = sin 2πn + 2π = sin 2πn = 0
sin (2(n + 1) + 1)π = sin (2n + 3)π = sin ((2n + 1)π + 2π) = sin (2n + 1)π = 0
(2) cos π/2 = cos −π/2 = 0
by induction, cos π/2 + 2πj = cos π/2(1 + 4j)
cos −π/2 + 2πj = cos (4j − 1)π/2, j ∈ Z+
Exercise 2.

(1) sin π/2 = 1, sin π/2(1 + 4j) = 1, j ∈ Z+ .


(2) cos x = 1, cos 0 = 1, cos 2πj = 1
Exercise 3.
sin x + π = − sin x + π/2 + π/2 = cos x + π/2 = − sin x
cos x + π = cos x + π/2 + π/2 = − sin x + π/2 = − cos x
Exercise 4.

sin 3x = sin 2x cos x + sin x cos 2x = 2 sin x cos2 x + sin x(cos2 x − sin2 x) = 3 cos2 x sin x − sin3 x =
= 3(1 − sin2 x) sin x − sin3 x = 3 sin x − 4 sin3 x

cos 3x = cos 2x cos x − sin 2x sin x = (cos2 x − sin2 x) cos x − (2 sin x cos x) sin x = cos x − 4 sin2 x cos x
cos 3x = −3 cos x + 4 cos3 x
Exercise 5.

(1) This is the most direct solution. Using results from Exercise 4 (and it really helps to choose the cosine relationship,
not the sine relationship),

cos 3x = 4 cos3 x − 3 cos x


x = π/6
cos 3π/6 = 0 = 4 cos3 π/6 − 3 cos π/6 = cos π/6(4 cos2 π/6 − 3) = 0

=⇒ cos π/6 = 3/2, sin π/6 = 1/2( by Pythagorean theorem )

(2) sin 2π/6 = 2 cos π/6 sin π/6 = 32, cos π/3√= 1/2 (by Pythagorean theorem)
(3) cos2π/4 = 0 = 2 cos π/4 − 1, cos π/4 = 1/ 2 = sin π/4
3 2
Note that the most general way to solve a cubic is to use this formula. For x + bx + cx + d = 0,

9bc − 27d − 2b3 p


R= S = (R + Q3 + R2 )1/3
54 p
3c − b2 T = (R − Q3 + R2 )1/3
Q=
9
x1 = S + T − b/3

x2 = −1/2(S + T ) − b/3 + 1/2 −3(S − T )

x3 = −1/2(S + T ) − b/3 − 1/2 −3(S − T )
Exercise 6. Ã 1
!
sin x − y sin x cos y − sin y cos x cos x cos y tan x − tan y
tan x − y = = 1 =
cos x − y cos x cos y + sin x sin y cos x cos y
1 + tan x tan y
25
if tan x tan y 6= −1
Similarly,
sin x + y sin x cos y + sin y cos x tan x + tan y
tan x + y = = = , tan x tan y 6= 1
cos x + y cos x cos y − sin x sin y 1 − tan x tan y
cos x + y cos x cos y − sin x sin y cot x cot y − 1
cot x + y = = =
sin x + y sin x cos y + sin y cos x cot y + cot x
√ √
3 3 3
Exercise 7. 3 sin x + π/3 = A sin x + B cos x = 3(sin x 12 + 2 cos x) = 3
2 sin x + 2 cos x
Exercise 8.
C sin x + α = C(sin x cos α + cos x sin α) = C cos α sin x + C sin α cos x
A = C cos α, B = C sin α
Exercise 9. If A = 0, B cos x = B sin π/2 + x = C sin x + α so C = B, α = π/2 if A = 0.
If A 6= 0,
B
A sin x + B cos x = A(sin x + cos x) == A(sin x + tan α cos x)
A
A A
= (cos α sin x + sin α cos x) = (sin x + α)
cos α cos α
A
where −π/2 < α < π/4, B/A = tan α, C =
cos α
Exercise 10. C sin x + α = C sin x cos α + C cos x sin α.

C cos α = −2, C sin α = −2, C = −2 2, α = π/4
Exercise 11. If A = 0, C = B, α = 0. If B = 0, A = −C, α = π/2. Otherwise,

A B
A sin x + B cos x = B(cos x + sin x) = (cos x cos β + sin β sin x) = C cos x + α
B cos β
A B
where
B = tan β , α = −β , C = cos β .
Exercise 12. p √ π
1 − cos2 x =⇒ cos x = 1/ 2 =⇒ x =
sin x = cos x =
√ 4
Try 5π/4. sin 5π/4 = cos 3π/4 = − sin π/4 = −1/ 2.

cos 5π/4 = − sin 3π/4 = − cos π/4 = −1/ 2. So sin 5π/4 = cos 5π/4. x = 5π/4 must be the other root.
So θ = π/4 + πn (by periodicity of sine and cosine).

Exercise 13. p
sin x − cos x = 1 = 1 − cos2 x = 1 + cos x
=⇒ 1 − cos2 x = 1 + 2 cos x + cos2 x =⇒ 0 = 2 cos x(1 + cos x)
cos x = −1, x = π/2 + 2πn
Exercise 14.

cos x − y + cos x + y = cos x cos y + sin x sin y + cos x cos y − sin x sin y = 2 cos x cos y
cos x − y − cos x + y = sin x cos y − sin y cos x + sin x cos y + sin y cos x = 2 sin x cos y
sin x − y + sin x + y = sin x cos y − sin y cos x + sin x cos y + sin y cos x = 2 sin x cos y
Exercise 15.

sin x + h − sin x sin (x + h/2) cos h/2 + cos (x + h/2) sin h/2 − sin (x + h) cos h/2 − cos x + h/2 sin h/2
=
h h
sin h/2
= cos (x + h/2)
h/2

cos x + h − cos x cos (x + h/2) cos h/2 − sin (x + h/2) sin h/2 − (cos (x + h/2) cos h/2 + sin (x + h/2) sin h/2)
=
h h
sin h/2
=− sin (x + h/2)
h/2
Exercise 16.

(1)
sin 2x = 2 sin x cos x
if sin 2x = 2 sin x and x 6= 0, x 6= πn, cos x = 1 but x 6= πn =⇒ x = 2πn
26
(2) cos x + y = cos x cos y − sin x sin y = cos x + cos y .
p
cos x cos y − cos x − cos y = sin y 1 − cos2 x
Letting A = cos x, B = cos y,

A2 B 2 + A2 + B 2 − 2A2 B − 2AB 2 + 2AB = 1 − A2 − B 2 + A2 B 2


A2 + B 2 − A2 B − AB 2 + AB = 1/2
B 2 (1 − A) + B(A − A2 ) + A2 − 1/2 = 0
p
A(1 − A) ± A2 (1 − A)2 − 4(1 − A)(A2 − 1/2) 1
B= =A± √ (A2 (1 − A) − 4(A2 − 1/2))1/2 =
1−A 1−A
1
=A± √ (−3A2 − A3 + 2)1/2
1−A
Note that −1 ≤ B ≤ 1, but for |A| ≤ 1. √ √
−3A2 − A3 + 2, A0 = −1, −1 + 3, −1 − 3. So suppose cos x = 9/10.
Solve for the roots of Then there is
no real number for y such that cos y would be real and satisfy the above equation.
(3) sin x + y = sin x cos y + sin y cos x = sin x + sin y

=⇒ sin y(1 − cos x) + sin y + − cos x sin y = 0, =⇒ y = 2πn

Checking our result, we nd that sin (2πn + y) = sin 2πn + sin y(1)
(4)
Z y
y
sin xdx = − cos x|0 = −(cos y − 1) = 1 − cos y = sin y
0
p
=⇒ 1 − cos y = 1 − cos2 y
2(j + 1)π
1 − 2 cos y + cos2 y = 1 − cos2 y =⇒ cos y(cos y − 1) = 0; y = , 2πn
2
Rb b
Exercise 17.
a
sin xdx = − cos x|a = − cos b + cos a

(1) − 23 + 1

(2) − 22 + 1
1
(3)
2
(4) 1
(5) 2
(6) 0 We were integrating over one period, over one positive semicircle and over one negative semicircle.
(7) 0 We had integrated over two equal parts, though it only shaded in up to x = 1.
√ √
2 3
(8) −
2 + 2
Rπ ¯π 2 2
Exercise 18.
0
(x + sin x)dx = ( 21 x2 − cos x)¯0 = π2 − (−1 − 1) = π2 + 2
R π/2 2 ¯π/2
Exercise 19.
0
(x + cos x)dx = ( 31 x3 + sin x)¯0 = 13 (π/2)3 + 1
R π/2 π/2
Exercise 20.
0
(sin x − cos x)dx = (− cos x − sin x)|0 = −1 − (−1) = 0
R π/2 R π/4 π/4 √
Exercise 21.
0
| sin x − cos x|dx = ( by symmetry )2 0 (cos x − sin x)dx = 2(sin x + cos x)|0 = 2( 2 − 1)
Rπ 1 ¯π
Exercise 22. ( + cos t)dt = ( 12 t + sin t)¯0 = π2
0 2
Exercise 23.
Z 2π/3 Z π ¯2π/3 ¯2π/3
1 1 t ¯ t ¯
( + cos t)dt + ¯
−( + cos t)dt = ( + sin t)¯ + ( + sin t)¯¯
0 2 2π/3 2 2 0 2 π

π 3 π π √
= 2( + )− = + 3
3 2 2 6
Exercise 24. If −π < x ≤ − 2π
3 ,
Z x Z −π µ ¶¯−π
1 1 t ¯ π x
−( + cos t)dt = ( + cos t)dt = + sin t ¯¯ = − − − sin x
−π 2 x 2 2 x 2 2
27
If −2π/3 ≤ x ≤ 2π/3,
Z −2π/3 Z x
1 1 −π/6 √ x
−( + cos t)dt + ( + cos t)dt = 3/2 + (t/2 + sin t)|−2π/3
−π 2 −2π/3 2 +
√ √ x
= x/2 + sin x − π/3 − 3/2 + 3/2 − π/6 = + sin x − π/3
2
If 2π/3 ≤ x ≤ π ,

√ Z x √ √
2π/3
3/2 + −(1/2 + cos t)dt = 3/2 + (t/2 + sin t)|x = π/3 + 3 − x/2 − sin x
2π/3
Rx 2 6 3
Exercise 25. (t2 + sin t)dt = ( 13 t3 + − cos t) = x −x
x 3 + cos x − cos x2
R π/2 ³ ´¯
¯π/2
Exercise 26.
0
sin 2xdx = − cos2 (2x) ¯ = (−1/2)(−1 − 1) = 1
0
R π/3 π/3
Exercise 27.
0
cos x/2dx = 2 sin x/2|0 = 2 12 = 1
Exercise 28.
Z x Z x ³ cos a ´¯x
¯
cos (a + bt)dt = (cos a cos bt − sin a sin bt)dt = sin bt − sin a(− cos bt/b) ¯ =
0 0 b 0
cos a sin a 1
= sin bx + (cos bx − 1) = sin a + bx − sin a/b
b b bµ ¶¯x
Z x Z x
sin a cos a ¯
sin (a + bt)dt = (sin a cos bt + sin bt cos a)dt = sin bt − cos bt ¯¯ =
0 0 b b 0
1
= (cos bx + a + cos a)
b
Exercise 29.

(1)
Z x Z x µ ¶¯x
3 sin t − sin 3t 3 ¯ cos 3x − 1
3
sin tdt = dt = − cos t + cos 3t/12 ¯¯ = −3/4(cos x − 1) + =
0 0 4 4 0 12
1 3 1
= − cos x + (cos 2x cos x − sin 2x sin x) = 2/3 − 1/3 cos x(2 + sin2 x)
3 4 12
(2)
Z x Z x ¶¯x µ
1 1 sin 3t 3 ¯
3
cos tdt = + sin t ¯¯ =
(cos 3t + 3 cos t)dt =
0 0 4 4 3 4 0
1 3 1
= (sin 2x cos x + sin x cos 2x) + sin x = (2 sin x cos x + sin x(2 cos2 x − 1)) =
12 4 12
sin x cos2 x + 2 sin x
=
3
Exercise 30. Now using the denition of a periodic function,

f (x) = f (x + p); f (x + (n + 1)p) = f (x + np + p) = f (x + np) = f (x)


and knowing that we could write any real number in the following form,

a = np + r; 0 ≤< p, r ∈ R; n ∈ Z
then
Z a+p Z r+p Z r+p Z p Z r+p
f (x)dx = f (x + np)dx = f (x)dx = f+ f (x)dx =
a r r r p
Z p Z r Z p Z r Z p
= f+ f (x − p)dx = f+ f= f
r 0 r 0 0
Exercise 31.

(1)
Z 2π Z 2πn¯2πn
1 1 ¯ 1
sin nxdx = sin xdx = (− cos x)¯¯ = − (1 − 1) = 0
0 0 n n 0 n
Z 2π Z 2πn ¯2πn
1 1 ¯
cos nxdx = cos xdx = sin x¯¯ =0
0 n 0 n 0
28
(2)
Z 2π Z 2π
1
sin nx cos mxdx = (sin (n + m)x + sin (n − m)x)dx = 0 + 0 = 0
0 0 2
Z 2π Z 2π
1
sin nx sin mxdx = (cos (n − m)x + cos (n + m)x)dx = 0 + 0 = 0
0 0 2
Z 2π Z 2π
1
cos nx cos mxdx = (cos (n − m)x + cos (n + m)x)dx = 0 + 0 = 0
0 0 2
While
Z 2π Z 2π
2 1 − cos 2nx
sin nxdx = dx = π
0 0 2
Z 2π Z 2π
1 + cos 2nx
cos2 nxdx = dx = π
0 0 2

Exercise 32. Given that x 6= 2πn; sin x/2 6= 0,


n
X n
X n
X x x x
2 sin x/2 cos kx = 2 sin x/2 cos kx = sin (2k + 1) − sin (2k − 1) = sin (2n + 1) − sin x/2
2 2 2
k=1 k=1 k=1
= sin nx cos x/2 + sin x/2 cos nx − sin x/2 =
= 2 sin nx/2 cos nx/2 cos x/2 + sin x/2(1 − 2 sin2 nx/2) − sin x/2 =
= 2(sin nx/2)(cos (n + 1)x/2)

Exercise 33. Recall that

cos (2k + 1)x/2 − cos (2k − 1)x/2 = cos kx + x/2 − cos kx − x/2 =
= cos kx cos x/2 − sin kx sin x/2 − (cos kx cos x/2 + sin kx sin x/2) =
= −2 sin kx sin x/2

n
X n
X
−2 sin x/2 sin kx = (cos (2k + 1)x/2 − cos (2k − 1)x/2) = cos (2n + 1)x/2 − cos x/2 =
k=1 k=1
= cos nx + x/2 − cos x/2
Now

sin nx/2 sin nx/2 + x/2 = sin nx/2(sin nx/2 cos x/2 + sin x/2 cos nx/2) =
= sin2 nx/2 cos x/2 + sin x/2 cos nx/2 sin nx/2 =
µ ¶
1 − cos nx sin nx
= cos x/2 + sin x/2 =
2 2
1 1
= (cos x/2 − cos x/2 cos nx + sin nx sin x/2) = (cos x/2 − cos (nxx /2)
2 2
Then
n
X 1
−2 sin x/2 sin kx = −2 sin nx/2 sin (n + 1)x
2
k=1
n
X sin nx/2 sin 21 (n + 1)x
sin kx =
sin x/2
k=1

Exercise 34. Using triangle OAP, not the right triangle, if 0 < x < π/2
1 1 x
cos x sin x < sin x <
2 2 2
=⇒ sin x < x

Now if 0 > x > −π/2, sin x < 0,


| sin x| = − sin x = sin −x = sin |x| < |x|
29
2.11 Exercises - Polar coordinates, The integral for area in polar coordinates.
Exercise 1.

(x − 1)2 + y 2 = 1
2 2
=⇒ r2 = 2r cos θ or r = 2 cos θ
=⇒ x − 2x + 1 + y = 1

Exercise 2.
p
x2 + y 2 − x = x2 + y 2
=⇒ r2 = r(cos θ + 1) or r = 1 + cos θ

Exercise 3.

(x2 + y 2 )2 = x2 − y 2 ; y 2 ≤ x2 r2 = cos 2θ
=⇒ √
r4 = r2 (cos2 θ − sin2 θ) r= cos 2θ for cos 2θ > 0

Exercise 4.

r2 = | cos 2θ|
(x2 + y 2 )2 = r4 = |x2 − y 2 | = |r2 cos 2θ| =⇒ p
r = | cos 2θ|

Recall that undened the measure of an angle to be 2 times the area of the sector divided by the radius squared (we chose

this because it doesn't change with a change in circle size). Then if r2 (θ) is integrable, then A = θab 21 θr2 dθ.

See sketches for the following exercises. Otherwise, we compute the area of the sector.

Exercise 5. Spiral of Archimedes: f (θ) = θ; 0 ≤ θ ≤ 2π


Z 2π
1 2 1 4π 3
A= θ dθ = (2π)3 =
0 2 6 3

Exercise 6. Circle tangent to y -axis: f (θ) = 2 cos θ; −π/2 ≤ θ ≤ π/2


Z Z π/2
1 π/2 2 (1 + cos 2θ)
A= 4 cos θdθ = 2 dθ = π
2 −π/2 −π/2 2

Exercise 7. Two circles tangent to y -axis: f (θ) = 2| cos θ|; 0 ≤ θ ≤ 2π


Z Z 2π
1 2π 1 + cos 2θ
4 cos2 θdθ = 2 = 2π
2 0 0 2

Exercise 8. Circle tangent to x-axis: f (θ) = 4 sin θ ; 0 ≤ θ ≤ π


Z Z πµ ¶
1 π 1 − cos 2θ
16 sin2 θdθ = 8 dθ = 4π
2 0 0 2

Exercise 9. Two circles tangent to x-axis: f (θ) = 4| sin θ|; 0 ≤ θ ≤ 2π


Z 2π Z 2π
1 1 − cos 2θ
16 sin2 θdθ = 8 dθ = 8π
2 0 0 2

Exercise 10. Rosepetal: f (θ) = sin 2θ; 0 ≤ θ ≤ π/2


Z π/2 Z π/2 µ ¶
1 1 1 − cos 4θ π
sin2 2θdθ = dθ =
2 0 2 0 2 8

Exercise 11. Four-leaved rose f (θ) = | sin 2θ|; 0 ≤ θ ≤ 2π


Z 2π Z 2π µ ¶
1 1 1 − cos 4θ π
sin2 2θdθ = dθ =
2 0 2 0 2 2
30
p
Exercise 13. Four-leaf clover: f (θ) = | cos 2θ|;
0 ≤ θ ≤ 2π
Z
1 2π
| cos 2θ|dθ =
2 0
ÃZ Z 3π/4 Z 5π/4 Z 7π/4 Z 2π !
π/4
1
= c(2θ) − c(2θ) + c(2θ) − c(2θ) + c(2θ) =
2 0 π/4 3π/4 5π/4 7π/4
à ¯π/4 ¯3π/4 ¯5π/4 ¯7π/4 ¯2π !
1 s(2θ) ¯¯ s(2θ) ¯¯ s(2θ) ¯¯ s(2θ) ¯¯ s(2θ) ¯¯
= − + − + =
2 2 ¯0 2 ¯π/4 2 ¯3π/4 2 ¯5π/4 2 ¯7π/4
µ µ ¶ µ ¶ µ ¶ µ ¶¶
1 1 −1 − 1 1 − (−1) −1 − 1 −(−1)
= − + − + = 2
2 2 2 2 2 2

Exercise 14. Cardiod: f (θ) = 1 + cos θ; 0 ≤ θ ≤ 2π


Z 2π
1 1 1 3π
1 + 2 cos θ + cos2 θ = (2π + (2π)) =
2 0 2 2 2
Exercise 15. Limacon: f (θ) = 2 + cos θ; 0 ≤ θ ≤ 2π
Z
1 2π 1 1 9π
4 + 4 cos θ + cos2 θ = (4(2π) + (2π)) =
2 0 2 2 2

1
R
2.17 Exercises - Average value of a function. Exercise 1.
b−a x2 dx = 13 (b2 + ab + a2 )
1
R 7
Exercise 2.
1−0 x2 + x3 = 12
1
R 4
Exercise 3.
4−0 x1/2 = 3
1
R 45
Exercise 4.
8−1 x1/3 = 28
1
R π/2
Exercise 5.
π/2−0 0 sin x = π2
1
R
Exercise 6.
π/2−−π/2 cos x = 2/π
1
R
Exercise 7.
π/2−0 sin 2x = −1/π(−1 − 1) = 2/π
1
R
Exercise 8.
π/4−0 sin x cos x = π1
R ¯π
Exercise 9.
1
π/2−0 sin2 x = π1 (x − sin 2x/2)¯0 = 12
1
R
Exercise 10.
π−0 cos2 x = 21
Exercise 11.
1
R √
(1) x2 = a2 /3 = c2 =⇒ c = a/ 3
a−0
R n ¯a
1 ¯ an
(2)
a−0 x = a1 n+1
1
xn+1 ¯ = n+1 = cn =⇒ c = a
(n+1)1/n
0
Exercise 12.
Z Z Z Z
A = wf / w wx2 =k x
Z
1 1 1
x3 = x4 = k x2 ; k = ,w = x
4 2 2
Z
1 1 3
x4 = x5 = k x3 ; k = , w = x2
5 3 5
Z
1 6 1 2
5
x = x = k x4 ; k = , w = x3
6 4 3
Exercise 13.
Z Z Z
1 1 1
A(f + g) = f +g = f+ g = A(f ) + A(g)
b−a b−a b−a
Z µ ¶Z
1 1
A(cf ) = cf = c f
b−a b−a
Z Z
1 1
A(f ) = f≤ g = A(g)
b−a b−a
31
Exercise 14.
R R R
w(c1 f + c2 g) c1 wf c2 wg
A(c1 f + c2 g) = R = R + R
w w w
= c1 A(f ) + c2 A(g)
f ≤ g w > 0( nonnegative ), =⇒ wf ≤ wg

Exercise 15.

Z ÃZ Z ! µ ¶Ã Rc ! Rb
b c b
1 1 c−a f b − a − (c − a) a f
Aba (f ) = f= f+ f = a
+
b−a a b−a a c b−a c−a b−a b−c
a<c<b
c−a
c−a Let t =
0< <1 b−a
b−a
=⇒ Aba (f ) = tAca (f ) + (1 − t)Abc (f )
Rb Rc Rc ÃR b Rc ! Rb
b a
wf a
w a wf a
w− a w c
wf
Aa (f ) = R b = Rb Rc + Rb Rb
w w a w w w
a a a c
Rc Rc
w w
0 < Rab < 1 since w is a nonnegative function. Let t = Rab
a
w a
w
=⇒ Aba (f ) = tAca (f ) + (1 − t)Abc (f )
R R
Exercise 16. Recall that xcm = R xρ or rcm =
rdm
ρ M .
RL
x L
xcm = R0L =
1 2
Z0 Z
2
Icm = r dm = x2 (1) = L3 /3
Icm L
r2 = R L = L2 /3 =⇒ r = √
0
1 3

Exercise 17.
R l/2 RL
0
x+ L/2
2xdx yL2
xcm = L =
2 + 2(L − L/2) 12
Z L/2 Z L √
2 5L3 /8 5L2 5L
Icm = x + 2x2 = 5L3 /8 r = 2
= =⇒ r = √
0 L/2 3L/2 12 2 3

Exercise 18. ρ(x) = x for 0 ≤ x ≤ L


R ¯
1 3 ¯L
xxdx 3 x 0 2
xcm = R = ¯ = L
xdx 1 2 ¯L 3
2x 0
Z
Icm = x2 xdx = L4 /4

L4 /4 L
r2 = = L2 /2 r= √
L2 /2 2
Exercise 19.
RR ¯
1 3 ¯L/2
¯L
xxdx + x L2 dx 3x 0 + L2 (x2 /2)¯L/2
xcm = R R = ¯ = 11L/18
xdx + L/2 1 2 ¯L/2
2x 0 + L2 (L − L/2)
Z Z
Icm = x xdx + x2 L/2dx = L4 31/192
2


2 2 2 31L
r = Icm /(L 3/8) = L 31/72 r = √
6 2
32
Exercise 20. ρ(x) = x2 for 0≤x≤L R
xx2 dx
xcm = R = 3L/4
x2
Z
Icm = x2 x2 dx = L5 /5
r
2 Icm 3 2 3
r = 1 3 = L r= L
3L
5 5
Exercise 21.
R L/2 RL 2

0
xx2 dx + L/2
x L4 dx
xcm = R L/2 R L L2 = 21L/32
0
x2 dx + L/2 4
dx
Z L
L/2 2 2 L2
Icm = int0 x x dx + x2
dx = 19L5 /240
L/2 4
Icm √ √
r2 = 2
= 19L /40 =⇒ r = 19L/2 10
L3 /6
Exercise 22. Be exible about how you can choose a convenient origin to evaluate the center-of-mass from
Let ρ = cxn
Z L
1
c xn dx =
Ln+1 c = M
0 n + 1
(n + 1)M
=⇒ c =
Ln+1
Z L
1 n+1 3M L
c xxn dx = c Ln+2 = ML =
0 n+2 n+2 4
R Z
xρ 3L n+1 3
xcm = = =⇒ xρ = = =⇒ n = 2
M 4 n+2 4
3M 2
ρ= x
L3
Exercise 23.

(1) Z
1 6
3 sin 2t =
π/2 − 0 π
(2) Z
1 √
9 sin2 2t = 9/2 =⇒ vrms = 3 2/2
π/2 − 0
Exercise 24. T = 2π (just look at the functions themselves)
Z 2π √
1
160 sin t2 sin (t − π/6) = 80 3
2π 0
Rx
2.19 Exercises - The integral as a function of the upper limit. Indenite integrals. Exercise 1.
0
(1 + t + t2 )dt =
1 2 1 3
x+ 2x + 3x
2 3
Exercise 2. 2y + 2y + 8y /3
2 3 2 3
Exercise 3. 2x + 2x + 8x /3 − (−1 + 1/2 + −1/3) = 2(x + x + 4x /3) + 5/6
R 1−x ¯ 1−x
Exercise 4.
1
(1 − 2t + 3t2 )dt = (t − t2 + t3 )¯1 = −2x + 2x2 − x3
Rx 4 ¯x 5 3
Exercise 5.
−2
t + t2 = 51 t5 + 13 t3 ¯−2 = x5 + x3 + 40 3
R x2 4 ³ 5 ´¯x2
¯
Exercise 6.
x
t + 2t2 + 1 = t5 + 32 t3 + t ¯ = 15 (x10 − x5 ) + 23 (x6 − x3 ) + x2 − x
x
¡ 2 3/2 ¢¯x
Exercise 7. t + t ¯ = 2 (x3/2 − 1) + (x − 1)
3 1 3
¡ 2 3/2 4 5/4 ¢¯x2
¯ = 23 (x3 − x3/2 ) + 54 (x5/2 − x5/4 )
Exercise 8.
3t + 5t x
x
Exercise 9. sin t|iπ = sin x
¡t ¢¯x2 x2
¯ + sin x2
Exercise 10.
2 + sin t 0 = 2
33
¡1 ¢¯x2 x2 −x
Exercise 11.
2t + cos t ¯x = 2 + cos x2 − cos x
¡1 ¢¯x x3
Exercise 12.
3u
3
+ − 13 cos 3u ¯0 = 3 + − 13 (cos 3x − 1)
³ ´¯x2
1 3 cos 3v ¯ x6 −x3 −1 2
Exercise 13.
3v + −3 ¯ = 3 + 3 (cos 3x − cos 3x)
x

R ¡1 ¢¯y y sin 2y y 2
Exercise 14.
1−cos 2x
2 +x= 2x −
sin 2x
4 + 12 x2 ¯0 = − +
2 4 2

¡ − cos 2w ¢¯x (cos 2x − 1) x


Exercise 15. + 2 sin w2 ¯0 = −
2 + 2 sin
2 2
Rx 1 R x ¡ ¢¯x
Exercise 16. ( +cos t)2 dt = −π 14 +cos t+cos2 t = 14 (x+π)+sin x+ 12 t + sin22t ¯−π = 34 (x+π)+sin x+ 14 sin 2x
−π 2
Rx 3 Rx
Exercise 17.
0
(t − t)dt = 13 √2 (t − t3 )dt
3 3 3

Note that t − t < 0 for 0 < t ≤ 1 and t − t > 0 for t > 1. t − t < 0 for t > 2.

µ ¶¯x
1 4 1 2 1 1 2 1 4 ¯¯ 1 1
x − x = t − t ¯√ = x2 − x4
4 2 3 2 4 2 6 12
1 4 2 2 √
=⇒ x − x = 0 =⇒ x = 0, x = 2
3 3
R1 R √2
0
(t3 − t)dt + 1
(t3 − t)dt “cancel” each other out.
Exercise 18.f (x) = x − [x] − 21 if x is not an integer; f (x) = 0 if x ∈ Z.
For any real number, x = q + r, 0 ≤ r < 1, q ∈ Z. So then

x − [x] = r
1
f (x) = r −
2

(1) To show the periodicity, consider

1 1
f (x + 1) = x + 1 − [x + 1] − = r − = f (x) sincex + 1 = q + 1 + r, [x + 1] = q + 1
2 2
1
x + 1 − [x + 1] = r −
2
Rx Rx
(2) P (x) = 0
f (t)dt = 0 (t − 12 ) = 12 x2 − 12 x because given 0 < x ≤ 1, then q = 0 for x, so we can use r = t.
To show periodicity,

Z x+1 Z 1 Z x+1 Z x Z x
P (x + 1) = f (t)dt = f (t)dt + f (t)dt = 0 + f (t + 1)dt = f (t)dt = P (x)
0 0 1 0 0
Z 1 ¯1
1 2 ¯
since f (t)dt = (x − x)¯¯ = 0
0 2 0

(3) Since P itself is periodic by 1, then we can consider 0 ≤ x < 1 only. Now x − [x] = r and P (x) = 12 (r2 − r). So
P (x) = 12 ((x − [x])2 − (x − [x])).
(4)

Z 1 Z 1
(P (t) + c)dt = 0 =⇒ P (t)dt = −c
0 0
1
0 ≤ t ≤ 1 so P (t) = (t2 − t)
2
Z 1 µ ¶¯1
1 1 3 1 2 ¯ ¯ 1 −1 1
=⇒ P (t)dt = t − t ¯ = =⇒ c =
0 2 3 2 0 2 6 12
34
Rx
(5) Q(x) = 0
(P (t) + c)dt
Z x+1 Z 1 Z x+1
Q(x + 1) = (P (t) + c)dt =
(P (t) + c)dt + (P (t) + c)dt =
0 0 1
Z x Z x
=0+ (P (t + 1) + c)dt = (P (t) + c)dt = Q(x)
0 0
so without loss of generality, consider 0≤x<1
Z x
1 2 1 1 1 x
=⇒ Q(x) (t − t) + = x3 − x2 +
0 2 12 6 4 12
R 2π
Exercise 19. g(2n) = 0
f (t)dt
Consider
Z 1 Z 1 Z 0 Z 1 Z 0
1
f (t)dt = f (t)dt + f (t)dt = f (t)dt + f (−1t)dt =
−1 0 −1 0 −1 1
Z 1 Z 0
= f+ f (t)dt = 0
0 1
R3 R1 R1
Consider that
1
f (t)dt = −1 f (t + 2)dt = −1 f (t)dt = 0. Then, by induction,
Z 2n+1 Z 2n−1 Z 2n+1 Z 1 Z 1
f= f+ f (t)dt = 0 + f (t + 2n)dt = f (t)dt = 0
1 1 2n−1 −1 −1

(1)
Z 1 Z 2n−1 Z 2n Z 1 Z 0 Z 1 Z 0
g(2n) = f+ f+ f= f+ f (t)dt = f +− f (−t)dt
0 1 2n−1 0 −1 0 1
Z 1 Z 0
= f+ f =0
0 1
(2)
Z −x Z x Z x
g(−x) = f =− f (−t)dt = f (t)dt = g(x)
0 0 0
Z x+2 Z 2 Z x+2 Z x Z x
g(x + 2) = f (t)dt = f+ f= f (t + 2)dt = f (t)dt = g(x)
0 0 2 0 0

Exercise 20.

(1) g is odd since


Z −x Z x Z x
g(−x) = f (t)dt = − f (−t)dt = − f (t)dt = −g(x)
0 0 0
Now
Z x+2 Z 2 Z x+2 Z x Z x
g(x + 2) = f= f+ f = g(2) + f (t + 2)dt = g(2) + f (t)dt = g(2) + g(x)
0 0 2 0 0
=⇒ g(x + 2) − g(x) = g(2)
(2)
Z 2 Z 2 Z 1 Z 2 Z 0 Z 0
g(2) = f= f+ f= f +A= f (t + 2)dt + A = f (t)dt + A =
0 1 0 1 −1 −1
Z 0
− f (−t)dt + A = 2A
1
g(5) − g(3) = g(2)
Z 3 Z 1
g(3) = g(2) + f (t)dt = 2A + f (t + 2)dt = 2A + A = 3A
2 0
=⇒ g(5) = 3A + 2A = 5A
(3) The key observation is to see that g must repeat itself by a change of 2 in the argument. To make g(1) = g(3) = g(5),
they're different, unless A = 0!
35
Exercise 21. From the given, we can derive
Z x
g(x) = f (x + 5), f (x) = g(t)dt
0
Z 5
=⇒ f (5) = g(t)dt = g(0) = 7
0
(1) The key insight I uncovered was, when stuck, one of the things you can do, is to think geometrically and draw
a picture.
g(−x) = f (−x + 5) = g(x) = −f (x − 5)
=⇒ −g(x) = f (x − 5)
(2)
Z 5 Z 0 Z 0 Z −5 Z 5 Z 5
f (t)dt = f (t + 5)dt = g(t)dt = − g(t)dt = g(−t)dt = g(t)dt = f (5) = 7
0 −5 −5 0 0 0
(3)
Z x Z x−5 Z x−5 Z x−5 Z 0 Z −5
f (t)dt = f (t + 5)dt = g(t)dt = g+ g = f (x − 5) + − g(t)dt =
0 −5 −5 0 −5 0
Z 5
f (x − 5) + g(−t)dt = f (x − 5) + f (5) = −g(x) + g(0)
0
where we've used f (x − 5) = −g(x) in the second and third to the last step.
3.6 Exercises - Informal description of continuity, The denition of the limit of a function, The denition of continuity
of a function, The basic limit theorems. More examples of continuous functions, Proofs of the basic limit theorems.
Polynomials are continuous.
1 1 1
Exercise 1. limx→2 x2 = limx→2 x2 = 4
limx→0 (25x3 +2)
Exercise 2.
limx→0 (75x7 −2) = −1
(x−2)(x+2)
Exercise 3. limx→2 =4
(x−2)
(2x−1)(x−1)
Exercise 4. limx→1 =1
x−1
2 2 2
t +2th+h −t
Exercise 5. limh→0 = 2t
h
(x−a)(x+a)
Exercise 6. limx→0 = −1
(x+a)2
(x−a)(x+a)
Exercise 7. lima→0 =1
(x+a)2
(x−a)(x+a)
Exercise 8. limx→a =0
(x+a)2
limt→0 sin t 0
9. limt→0 tan t =
Exercise
limx→0 cos t = 1 = 0
2 2
Exercise 10. limt→0 (sin 2t + t cos 5t) = limt→0 sin 2t + limt→0 t limt→0 cos 5t = 0 + 0 = 0
|x|
11. limx→0+
Exercise
x =1
|x|
12. limx→0−
Exercise
x = −1

x2
13. limx→0+
Exercise
x = +1

x2
14. limx→0−
Exercise
x = −1
2 sin x cos x
Exercise 15. limx→0 =2
x
2 sin x cos x
16. limx→0
Exercise
cos 2x sin x = 2
sin x cos 4x+sin 4x cos x
¡ ¢
Exercise 17. limx→0
sin x = 1 + limx→0 2 sin sin
2x cos 2x
x = 1 + 2 limx→0 2 sin x cos x cos 2x
sin x =5
5 sin 5x
Exercise 18. limx→0
5x − limx→0 3 sin 3x
3x
=5−3=2
Exercise 19.
¡ ¡ x−a ¢¢
¡ x+a x−a
¢
− sin x+a
sin 2 − 2
2 + 2
lim =
x→0 x−a
à ¡ ¢!
sin x+a x−a x−a x+a x+a x−a x−a x+a
2 cos 2 + sin 2 cos 2 − sin 2 cos 2 − sin 2 cos 2
= lim =
x→0 x−a
2 sin x−a x+a
2 cos 2
= lim = cos a
x→a x−a
36
³ ´
2 sin2 x/2 1 sin x/2 1
Exercise 20. limx→0 4(x/2)2 = 2 limx→0 x/2 = 2
√ ³ √ ´ 1
1− 1−x2 1+√1−x2 1−(1−x2 )
Exercise 21. limx→0 x2 1+ 1−x2
= limx→0 √
x2 (1+ 1−x2 )
=
2
Exercise 22. b, c are given.
sin c = ac + b, a = sin cc−b , c 6= 0.
if c = 0, then b = 0, a ∈ R.

Exercise 23. b, c are given.

2 cos c = ac2 + b, a = 2 cosc2c−b , c 6= 0.


If c = 0, then b = 2, a ∈ R.

Exercise 24.
tangent is continuous for x∈
/ (2n + 1)π/2
cotangent is continuous for x∈
/ 2nπ
Exercise 25. limx→0 f (x) = ∞. No f (0) cannot be dened.
Exercise 26.

(1) | sin x − 0| = | sin x| < |x|. Choose δ = ² for a given ².


Then ∀² > 0, ∃δ > 0 such that | sin x − 0| < ² when |x| < δ .
(2)
x |x|2
| cos x − 1| = | − 2 sin2 x/2| = 2| sin x/2|2 < 2| |2 = < 2²/2 = ²
√ √ 2 2
If we had chosen δ0 = 2² for a given ². |x − 0| < δ = 2².
(3)
² ²
| sin x(cos h − 1) + cos x sin h| ≤ | sin x|| cos h − 1| + | cos h|| sin h| < + =²
2 2
| cos x + h − cos x| = | cos x cos h − sin x sin h − cos x| = | cos x(cos h − 1) − sin x sin h| ≤
² ²
≤ | cos x|| cos h − 1| + | sin x|| sin h| < + = ²
2 2
since ∀² > 0∃δ1 , δ2 > 0 such that | cos h − 1| < ²0 ; | sin h| < ² whenever |h| < min (δ1 , δ2 )
² ²
Choose δ3 such that if |h| < δ3 ; | cos h − 1| < ; | sin h| <
2 2
Exercise 27. f (x) − A = sin x1 − A.
1
Let x= nπ .
|f (x) − A| = | sin nπ − A| > || sin nπ| − |A|| > |1 − |A||
1 |1−|A||
Consider |x − 0| = |x| = nπ ≤ δ(n). Consider ²0 = 2 . Then suppose a δ(n) ≥ |x − 0| but |f (x) − A| > ²0 . Thus,
contradiction.
1
Exercise 28. Consider x≤ n, n ∈ Z+ , n > M (n) (n is a given constant)
· ¸
1 1
f (x) = = [n] = n, for m > M (n), x = f (x) > M (n)
x m
so∀² > 0, we cannot nd δ = n1 such that |f (x) − A| < ² for x < δ .
+
So f (x) → ∞ as x → 0 .
1 −
Consider
n ≥ x > 0, n ∈ Z ; −n > M (n).
· ¸
1
f (x) = = [n] = n < −M (n)
x
Since integers are unbounded, we can consider n < A, so that
|f (x) − A| > ||f | − |A|| = −n − |A| > M (n) − |A|. Choose n such that M (n) − |A| > 0
Exercise 29.

|f − A| = |(−1)[1/x] − A| ≥ ||(−1)[1/x] | − |A|| = |1 − |A||


Choose ² < |1 − |A||. Then ∀δ > 0 ( such that |x| < δ ), |f − A| > ². Thus there's no value for f (0) we could choose to
make this function continuous at 0.

Exercise 30. Since

|f (x)| = |x||(−1)[1/x] | = |x|


So ∀², let δ = ².
37
Exercise 31. f continuous atx0 .
Choose some ²0 , 0 < ²0 < min (b − x0 , x0 − a). Then ∃δ0 = δ(x0 , ²0 ).

Consider ²1= ²20 and δ1 = δ(x0 , ²1 )


Consider x1 ∈ (x0 − δ1 , x0 + δ1 ), so that |f (x1 ) − f (x0 )| < ²1 .
Proceed to construct a δ for x1 , some δ(x1 ; ²0 )

|x − x1 | = |x − x0 + x0 − x1 | < |x − x0 | + |x0 − x1 |
δ1
Without loss of generality, we can specify x1 such that |x0 − x1 | < 2 . Also, “pick” only the x's such that

δ1
|x − x0 | < < δ1
2
δ1 δ1
=⇒ |x − x1 | < + = δ1
2 2
Thus, “for these x's”

|f (x) − f (x1 )| = |f (x) − f (x0 ) + f (x0 ) − f (x1 )| < |f (x) − f (x0 )| + |f (x1 ) − f (x0 )| < ²1 + ²1 = ²0

So ∀²0 , ∃δ1 for x1 . f is continuous at x1 ∈ (a, b). Thus, there must be innitely many points that are continuous in (a, b),
and at the very least, some or all are “clustered” around some neighborhood about the one point given to make f continuous.

Exercise 32. Given ² = n1 , |f (x)| = |x sin x1 | = |x|| sin 1/x| < |x|(1).
1 1
δ = δ(n) =
Let
n , so that |x| < n .
=⇒ |f (x)| < n1
Exercise 33.

(1) Consider x0 ∈ [a, b].

Choose some ²0 , 0 < ²0 < min (b − x0 , x0 − a) 6= 0 , ( x0 could be a or b )


Consider, without loss of generality, only “x's” such that x ∈ [a, b].

|f (x) − f (x0 )| ≤ |x − x0 |

Let δ0 = δ(²0 , x0 ) = ²0 =⇒ |f (x) − f (x0 )| < ²0 .

Since we didn't specify x0 , ∀x0 ∈ [a, b], f is continuous at x0 .


(2)
¯Z ¯ ¯Z ¯ Z
¯ b ¯ ¯ b ¯ b
¯ ¯ ¯ ¯
¯ f (x)dx − (b − a)f (a)¯ = ¯ (f (x) − f (a))dx¯ ≤ |f (x) − f (a)|dx ≤
¯ a ¯ ¯ a ¯ a
Z b ¯b
1 ¯ 1 (b − a)2
≤ |x − a|dx = ( x2 − ax)¯¯ = (b − a)(b + a) − a(b − a) =
a 2 a 2 2
(3)
¯Z ¯ ¯Z ¯ Z Z b
¯ b ¯ ¯ b ¯ b
¯ ¯ ¯ ¯
¯ f (x)dx − (b − a)f (c)¯ = ¯ (f (x) − f (c))dx¯ ≤ |f (x) − f (c)|dx ≤ |x − c|dx =
¯ a ¯ ¯ a ¯ a a
Z c Z b
1 1
= (c − x)dx + (x − c)dx = c(c − a) − (c − a)(c + a) + (b − c)(b + c) − c(b − c) =
a c 2 2
1
= ((c − a)2 + (b − c)2 )
2
Draw a gure for clear, geometric reasoning.
Consider a square of length (b − a) and a 45 − 45 right triangle inside. From the gure, it's obvious that right
triangles of c − a length and (b − c) length lie within the (b − a) right triangle.

Compare the trapezoid of c − a, b − a bases with the b − a right triangle.


1 1 1
(b − c)(b − a + c − a) = (b − c)(b − c + 2(c − a)) > (b − c)2
2 2 2
38
Indeed, the trapezoid andc − a right triangle equals the b − a trapezoid since
1 1 1 1
(b − c)(b − a + c − a) + (c − a)2 = (b2 − c2 − 2ab + 2ac + c2 − 2ca + a2 ) = (b − a)2
2 2 2 2
1 2 1 2 1 2
=⇒ (b − a) > (b − c) + (c − a)
2¯ 2 2¯
¯Z b ¯ (b − a)2
¯ ¯
so then ¯ f (x)dx − (b − a)f (c)¯ ≤
¯ a ¯ 2
3.8 Exercises - Composite functions and continuity.
Exercise 1. f (x) = x2 − 2x; g(x) = x + 1
f (g(x)) = (x + 1)2 − 2(x + 1) = x2 − 1
h:R→R
Exercise 2. f (x) = x + 1; g(x) = x2 − 2x
h = x2 − 2x + 1
h:R→R

Exercise 3. f (x) = x if x ≥ 0, g(x) = x2
h(x) = |x|
h : R → R+

Exercise 4. f (x) = x if x ≥ 0 g(x) = −x2
h undened. D(h) = ∅
2

Exercise 5. f (x) = x ; g(x) = x if x ≥ 0
h(x) = x if x ≥ 0
+ +
h:R →R
2

Exercise 6. f (x) = −x ; g(x) = x if x ≥ 0
h = −x if x ≥ 0
h : R+ → R−

Exercise 7. f (x) = sin x; g(x) = x if x ≥ 0

h(x) = sin x if x ≥ 0
+
h : R → [−1, 1]

f (x) = x if x ≥ 0, g(x) = sin x
Exercise 8.

h(x) = sin x if 2πj ≤ x ≤ 2πj + π; j ∈ Z+
h : {x|2πj ≤ x ≤ 2πj + π; j ∈ Z+ } → [0, 1]
√ √
Exercise 9. f (x) = x if x ≥ 0, g(x) = x + x if x > 0
q

h(x) = x+ x
h : {x > 0} → {x > 0}
p √ √
Exercise 10. f (x) = x + x if x > 0; g(x) = x + x if x > 0
r q
√ √
h(x) = x + x + x + x
h : {x > 0} → {x > 0}
x3 +8 (x+2)(x2 −2x+4)
Exercise 11. limx→−2 x2 −4 = limx→−2 (x+2)(x−1) = −3
f A
(used limf g = AB and lim =g B)
p √ √
Exercise 12. limx→4 1 + x = 3. Used continuity of composite function theorem.
sin tan t sin (tan t)
1
¡ ¢ sin tan t 1
Exercise 13. limt→0 = limt→0
sin t cos t = limt→0 tan t limt→0 cos t = 1.
tan t
sin (tan t)
( we used limf g = AB in the last step; limt→0 is from continuity of composite functionsl this had been a composite
tan t
function f ◦ g of f = sin x and g = tan x).
sin cos x
Exercise 14. limx→π/2
cos x = 1 (by continuity of composite functions; this was a composite function f ◦ g of f = sin x
and g = cos x).
sin (t−π)
Exercise 15. limt→π = 1 (by continuity of composite functions; this was a composite function f ◦ g of f = sin x
t−π
and g = t − π ).
39
sin (x2 −1) sin (x2 −1)
Exercise 16. limx→1 x−1 = limx→1 x2 −1 (x + 1) = 2 (we used continuity of composite functions and product of
limits.

Exercise 17. Since 0 < x sin x1 < x; by squeeze principle, limx→0 x sin x1 = 0
2
1−cos 2x
Exercise 18. limx→0x2 = limx→0 2 sin
x2
x
=2
√ √ ³√ √ ´
1+x− 1−x √1+x+√1−x
Exercise 19. limx→0 = 1
x 1+x+ 1−x
√ ³ √ ´
1− 1−4x2 1+√1−4x2
Exercise 20. limx→0 = 2
x2 1+ 1−4x2
(
x+|x| x for x<0
Exercise 21. f (x) = 2 g(x) =
x2 for x≥0
(
x2 x≥0
h(x) =
0 x<0
h(x) continuous everywhere.
( (
1 if |x| ≤ 1 2 − x2 if |x| ≤ 2
Exercise 22. f (x) = g(x) =
0 if |x| > 1 2 if |x| > 2
( √
0 if |x| ≤ 1, 3 ≤ |x|
h(x) = √
1 if 1 ≤ |x| ≤ 3
√ √
h cont. |x| < 1, 3 < |x|, 1 < |x| < 3
for

Exercise 23. h(x) = g(f (x))


(
0 for x<0
h(x) =
x2 for x≥0
h(x) continuous everywhere.
3.11 Exercises - Bolzano's theorem for continuous functions, The intermediate-value theorem for continuous functions.
These theorems form the foundation for continuity and will be valuable for differentiation later.

Theorem 10 (Bolzano's Theorem).


Let f ∀x ∈ [a, b].
be cont. at
Assume f (a), f (b) have opposite signs.
Then ∃ at least one c ∈ (a, b) s.t. f (c) = 0.

Proof. Let f (a) < 0, f (b) > 0.


Want: Fine one value c ∈ (a, b) s.t. f (c) = 0
Strategy: nd the largest c.
Let S = { all x ∈ [a, b] s.t. f (x) ≤ 0 }.
S is nonempty since f (a) < 0. S is bounded since all S ⊆ [a, b].
=⇒ S has a suprenum.
Let c = supS .
If f (c) > 0, ∃(c − δ, c + δ) s.t. f > 0
c − δ is an upper bound on S
but c is a least upper bound on S . Contradiction.
If f (c) < 0, ∃(c − δ, c + δ) s.t. f < 0
c + δ is an upper bound on S
but c is an upper bound on S . Contradiction. ¤
Theorem 11 (Sign-preserving Property of Continuous functions).
Let f be cont. at c and suppose that f (c) 6= 0.
then ∃(c − δ, c + δ) s.t. f be on (c − δ, c + δ) has the same sign as f (c).
Proof. Suppose f (c) > 0.
∀² > 0, ∃δ > 0 s.t. f (c) − ² < f (x) < f (c) + ² if c − δ < x < c + δ (by continuity).
f (c)
Choose δ for ² =
2 . Then
f (c) 3f (c)
< f (x) < ∀x ∈ (c − δ, c + δ)
2 2
Then f has the same sign as f (c). ¤
40
Theorem 12 (Intermediate value theorem).
Let f [a, b].
be cont. at each pt. on
Choose any x1 , x2 ∈ [a, b] s.t. x1 < x2 . s.t. f (x1 ) 6= f (x2 ).
Then f takes on every value between f (x1 ) and f (x2 ) somewhere in (x1 , x2 ).

Proof. Suppose f (x1 ) < f (x2 )


k be any value between f (x1 ) and f (x2 )
Let
Let g = f − k
g(x1 ) = f (x1 ) − k < 0
g(x2 ) = f (x2 ) − k > 0
By Bolzano, ∃c ∈ (x1 , x2 ) s.t. g(c) = 0 =⇒ f (c) = k ¤

Exercise 1. f (0) = c0 . f (0) ≷ 0.


ck x k ck Pn−1
Since limx→∞ ck−1 xk−1
= limx→∞ ck−1 x = ∞ ∃M > 0 such that |cn M n | > | k=0 ck M k . So then

n−1
X
f (M ) = cn M n + ck M k ≶ cn
k=0

By Bolzano's theorem ∃b ∈ (0, M ) such that f (b) = 0.


Exercise 2. Try alot of values systematically. I also cheated by taking the derivatives and feeling out where the function
changed direction.

(1) If P (x) = 3x4 − 2x3 − 36x2 + 36x − 8, P (−4) = 168, P (−3) = −143, P (0) = −8, P ( 21 ) = 15 16 , P (1) = −7,
P (−3) = −35, P (4) = 200
4 2 1 1 3 11
If P (x) = 2x − 14x + 14x − 1, P (−4) = 231, P (−3) = −7, P (0) = −1, P ( ) = , P ( ) = −
(2)
2 8 2 8 , P (2) = 2
5 3 1 22 1 3
(3)
4 3 2
If P (x) = x + 4x + x − 6x + 2, P (−3) = 2, P (− ) = −
2 16 , P (−2) = 2 , P( ) =
3 81 , P ( 2 ) = − 16 , P ( 23 ) =
14
− 81 , P (1) = 2.
Exercise 3.. Consider f (x) = x2j+1 − a. f (0) = −a > 0.
Sincea is a constant, choose M < 0 such that M 2j+1 − a < 0. f (M ) < 0.
2j+1
By Bolzano's theorem, there is at least one b ∈ (M, 0) such that f (b) = b − a = 0.
Since x2j+1 − a is monotonically increasing, there is exactly one b.
Exercise 4. tan x is not continuous at x = π/2.

Exercise 5. Consider g(x) = f (x) − x. Then g(x) is continuous on [0, 1] since f is.

Since 0 ≤ f (x) ≤ 1 for each x ∈ [0, 1], consider g(1) = f (1) − 1, so that −1 ≤ g(1) ≤ 0. Likewise 0 ≤ g(0) ≤ 1.
If g(1) = 0 or g(0) = 0, we're done (g(0) = f (0) − 0 = 0. f (0) = 0. Or g(1) = f (1) − 1 = 0, f (1) = 1 ).

Otherwise, if−1 ≤ g(1) < 0 and 0 < g(0) ≤ 1, then by Bolzano's theorem, ∃ at least one c such that
g(c) = 0 (g(c) = f (c) − c = 0. f (c) = c).

Exercise 6. Given f (a) ≤ a, f (b) ≥ b,


Consider g(x) = f (x) − x ≤ 0. Then g(a) = f (a) − a ≤ 0, g(b) = f (b) − b ≥ 0.
Since f is continuous on [a, b] (so is g ) and since g(a), g(b) are of opposite signs, by Bolzano's theorem, ∃ at least one c
such that g(c) = 0, so that f (c) = c.

3.15 Exercises - The process of inversion, Properties of functions preserved by inversion, Inverses of piecewise mono-
tonic functions. Exercise 1. D = R, g(y) = y − 1
Exercise 2. D = R, g(y) = 12 (y − 5)
Exercise 3. D = R, g(y) = 1 − y
Exercise 4. D = R, g(y) = y 1/3
Exercise 5. D = R, 

y if y<1

g(y) = y if 1 ≤ y ≤ 16

¡ y ¢2
if y > 16
8
−1
¡ 1 Pn ¢ 1
Pn
Exercise 6. f (Mf ) = f (f i=1 f (ai ) ) = i=1 f (ai )
Pn n n
Exercise 7. f (a1 ) ≶ n1 i=1 f (ai ) ≶ f (an ). Since f is strictly monotonic.
41
g preserves monotonicity.
=⇒ a1 ≶ Mf ≶ an

h(x) = af (x) + b, a 6= 0
Exercise 8.
à n ! à n ! à n
!
1X 1X 1X
Mh = H h(ai ) = H (af (ai ) + b) = H a f (ai ) + b
n i=1 n i=1 n i=1
¡ h−b ¢
The inverse for h is g
a = H(h) = h−1 . So then
à n !
1X
Mh = g f (ai ) = Mf
n i=1

The average is invariant under translation and expansion in ordinate values.

3.20 Exercises - The extreme-value theorem for continuous functions, The small-span theorem for continuous func-
tions (uniform continuity), The integrability theorem for continuous functions.

Since for c ∈ [a, b], m = minx∈[a,b] f ≤ f (c) ≤ maxx∈[a,b] f = M


Rb
f (x)g(x)dx
and aR b
g(x)dx
= f (c)
a

Exercise 1.
1 1
g = x9 > 0 for x ∈ [0, 1]; f = √ m = √ ,M = 1
1+x 2
Z 1 ¯1
¯
1 10 ¯ 1
x9 = x ¯ =
0 10 0 10
Z 1
1 x9 1
√ ≤ √ dx ≤
10 2 0 1+x 10
Exercise 2.
p 1 − x2 1 2
1 − x2 = √ .f = √ g = (1 − x2 ) M = √ , m = 1
1−x 2 1−x 2 3
Z 1/2 ¯1/2
1 ¯ 11
(1 − x2 )dx = (x − x3 )¯¯ =
0 3 0 24
Z 1/2 p r
11 11 4
≤ 1 − x2 dx ≤
24 0 24 3
Exercise 3.
Z a µ ¶¯a
1 1 3 1 5 ¯¯ a3 a5
f= g = 1 − x2 + x4 2
1−x +x = x− x + x ¯ =a− 4
+
1 + x6 0 3 5 0 3 5
1
m= M =1
1 + a6
µ ¶ Z a µ ¶
1 a3 a5 1 a3 a5
a − + ≤ dx ≤ a − +
1 + a6 3 5 0 1+x
2 3 5
1
So if a= 10 , (a − a3 /3 + a5 /5) = a − 0.333 . . . a3 + 0.2a5 = 0.099669
Exercise 4. (b) is wrong, since it had chosen g = sin t, but g needed to be nonnegative.
Exercise 5. At worst, we could have utilized the fundamental theorem of calculus.
Z Z µ ¶
1 1 ¯√(n+1)π
2
sin t dt = (2t sin t2 )dt = (− cos t2 )¯√nπ =
2t 2c
−1 1
= ((−1)n+1 − (−1)n ) = (−1)n
2c c
Rb Rb Rb
f
Exercise 6.
a
(f )(1) = f (c) a
1 = f (c)(b − a). Then f (c) = a
b−a =0 for some c ∈ [a, b] by Mean-value theorem for
integrals.
42
1
Exercise 7. f nonnegative. Consider f at a point of continuity c, and suppose f (c) > 0. Then
2 f (c) > 0.
|f (x) − f (c)| < ² =⇒ f (c) − ² < f (x) < f (c) + ²
1 1
Let ² = f (c) ∃δ > 0 for ² = f (c)
2 2
Z c+δ
1
f (x)dx > f (c)(2δ) = f (c)δ > 0
c−δ 2
Rb
But
a
f (x)dx = 0 and f is nonnegative. f (c) = 0.
Exercise 8. Z Z Z Z Z
m g≤ fg ≤ M
g =⇒ m g ≤ 0 ≤ M g ∀g
Z
m ≤ 0 ≤ M for g = 1 but also
Z
−m ≤ 0 ≤ −M =⇒ m ≥ 0 M ≤ 0 for g = −1
So because of this contradiction, m = M = 0. By intermediate value theorem, f = 0, ∀x ∈ [a, b].
4.6 Exercises - Historical introduction, A problem involving velocity, The derivative of a function, Examples of deriva-
tives, The algebra of derivatives.
Exercise 1. f 0 = 1 − 2x, f 0 (0) = 1, f 0 (1/2) = 0, f 0 (1) = −1, f 0 (10) = −19
Exercise 2. f 0 = x2 + x − 2
(1) f 0 = 0, x = 1, −2
(2) f 0 (x) = −2, x = 0, −1
(3) f 0 = 10, x = −4, 3
Exercise 3. f0 = 2x + 3
Exercise 4. f0 = 4x3 + cos x
Exercise 5. f0 = 4x3 sin x + x4 cos x
−1
Exercise 6. f0 = (x+1)2

−1
Exercise 7. f0 = (x2 +1)2 (2x) + 5x
4
cos x + x5 (− sin x)
x−1−(x) −1
Exercise 8. f0 = (x−1)2 = (x−1)2
−1 sin x
Exercise 9. f0 = (2+cos x)2 (− sin x) = (2+cos x)2
Exercise 10.

(2x + 3)(x4 + x2 + 1) − (4x3 + 2x)(x2 + 3x + 2) −2x5 − 9x4 + 12x3 − 3x2 − 2x + 3


=
(x4 + x2 + 1)2 (x4 + x2 + 1)2
Exercise 11.
(− cos x)(2 − cos x) − (sin x)(2 − sin x) −2 cos x − 2 sin x + 1
f0 = =
(2 − cos x)2 (2 − cos x)2
Exercise 12.
(sin x + x cos x)(1 + x2 ) − 2x(x sin x) sin x + x cos x + x3 cos x − x2 sin x
f0 = =
(1 + x2 )2 (1 + x2 )2
Exercise 13.

(1)
f (t + h) − f (t) v0 h − 32th − 16h2
= = v0 + 32t − 16h
h h
0
f (t) = v0 − 32t
v0
(2) t = 32
(3) −v0
v0 v0
(4) T = 16 , v0 = 16 for 1sec. v0 = 160 for 10sec. 16 for T sec.
00
(5) f = −32
(6) h = −20t2
dV
Exercise 14. V = s3 , dS = 3s2
Exercise 15.
dA
(1)
dr = 2πr = C
43
dV
(2)
dr = 4πr2 = A
1
Exercise 16. f0 = √
2 x
³ ´
−1 1
Exercise 17. f0 = √
(1+ x)2

2 x

Exercise 18. f 0 = 32 x1/2


−3 −5/2
Exercise 19.
2 x
Exercise 20. f 0 = 12 x−1/2 + 13 x−2/3 + 41 x−3/4 x > 0
Exercise 21. f 0 = − 12 x−3/2 + − 31 x−4/3 − 14 x−5/4
1 −1/2 √
x (1+x)− x 1
Exercise 22. f0 = 2 (1+x)2 = 2√x(1+x) 2
√ √
(1+ x)−x 12 √1x 1
1+ 2 x
Exercise 23. f0 = √
(1+ x)2
= (1+√ x)2
Exercise 24.
g = f1 f2
g0 f0 f0
g 0 = f10 f2 + f1 f20 = 1+ 2
g f1 f2
g = f1 f2 . . . fn fn+1
g 0 = (f1 f2 . . . fn )0 fn+1 + (f1 f2 . . . fn )fn+1
0
;
g0 (f1 f2 . . . fn )0 f0
= + n+1
g f1 f2 . . . fn fn+1
f10 f20 f0 f0
= + + · · · + n + n+1
f1 f2 fn fn+1
Exercise 25.
³ cos x ´0 cos2 x − (− sin x) sin x
(tan x)0 = = = sec2 x
sin x cos2 x
³ cos x ´0 − sin x sin x − cos x cos x
(cot x)0 = = = − csc2 x
sin x sin2 x
−1
(sec x)0 = (− sin x) = tan x sec x
cos2 x
−1
(csc x)0 = cos x = − cot x csc x
sin2 x
Exercise 35.
(2ax + b)(sin x + cos x) − (cos x − sin x)(ax2 + bx + c)
f0 = =
(sin x + cos x)2
(2ax + b)(sin x + cos x) − (cos x − sin x)(ax2 + bx + c)
=
(sin x + cos x)2
Exercise 36.

f 0 = a sin x + (ax + b) cos x + c cos x + (cx + d)(− sin x) = ax cos x + (b + c) cos x + (a − d) sin x − cx sin x
So then a = 1, d = 1, b = d, c = 0.
Exercise 37.

g 0 = (2ax + b) sin x + (ax2 + bx + c) cos x + (2dx + e) cos x + (dx2 + ex + f )(− sin x) =


= ax2 cos x − dx2 sin x + (2a − e)x sin x + (b + 2d)x cos x + (b − f ) sin x + (c + e) cos x
g = x2 sin x. So d = −1, b = 2, f = 2, a = 0, e = 0, c = 0.
xn+1 −1
Exercise 38. 1 + x + x2 + · · · + xn = x−1
(1)
(n + 1)xn (x − 1) − (1)(xn+1 − 1)
(1 + x + x2 + · · · + xn )0 = 1 + 2x + · · · + nxn−1 =
(x − 1)2
n+1 n n+1
(n + 1)(x −x )−x +1 nxn+1 − (n1 )xn + 1
= =
(x − 1)2 (x − 1)2
n+2
nx − (n + 1)xn+1 + x
x(1 + 2x + · · · + nxn−1 ) = x + 2x2 + · · · + nxn =
(x − 1)2
44
(2)

(x + 2x2 + · · · + nxn )0 = (1 + 22 x1 + · · · + n2 xn−1 ) =


(n(n + 2)xn+1 − (n + 1)2 xn + 1)(x − 1)2 − 2(x − 1)(nxn+2 − (n + 1)xn+1 + x)
=
(x − 1)4
(n(n + 2)xn+2 − (n + 1)2 xn+1 + x)(x − 1) − 2(nxn+3 − (n + 1)xn+2 + x2 )
x + 22 x2 + · · · + n2 xn = =
(x − 1)3
n2 xn+3 + (−2n2 − 2n + 1)xn+2 + (n + 1)2 xn+1 − x2 − x
=
(x − 1)3
Exercise 39.
f (x + h) − f (x) (x + h)n − xn
=
h h
Xn µ ¶
n
(x + h)n = xn−j hj
j=0
j
Pn ¡n¢ n−j j n µ ¶
(x + h) − xn
n
j=1 j x h X
n−j j−1 n
= = x h
h h j=1
j
µ ¶
(x + h)n − xn n n−1
lim = x = nxn−1
h→0 h 1

4.9 Exercises - Geometric interpretation of the derivative as a slope, Other notations for derivatives.
Exercise 6.

(1)

f = x2 + ax + b f (x2 ) − f (x1 ) x2 − x21 + a(x2 − x1 )


= 2
f (x1 ) = x21 + ax1 + b x2 − x1 x2 − x1
f (x2 ) = x22 + ax2 + b = x2 + x1 + a
(2)
f 0 = 2x + a
x2 + x1
m = x2 + x1 + a = 2x + a x =
2
Exercise 7. The line y = −x as slope −1.
y = x3 − 6x2 + 8x y 0 = 3x2 − 12x + 9
3x2 − 12x + 8 = −1 =⇒ x = 3, 1
The line and the curve meet under the condition

−x = x3 − 6x2 + 8x =⇒ x = 3; f (3) = −3
At x = 0, the line and the curve also meet.
Exercise 8. f = x(1 − x2 ). f 0 = 1 − 3x2 .
f 0 (−1) = −2 =⇒ y = −2x − 2
For the other line,
f 0 (a) = 1 − 3a2
=⇒ y(−1) = 0 = (1 − 3a2 )(−1) + b =⇒ b = 1 − 3a2
Now f (a) = a(1 − a2 ) = a − a3 at this point. The line and the curve must meet at this point.

y(a) = (1 − 3a2 )a + (1 − 3a2 ) =


= a − 3a3 + 1 − 3a2 = a − a3
1 3
=⇒ −2a3 + 1 − 3a2 = 0 = a3 − + a2
2 2
The answer could probably be guessed at, but let's review some tricks for solving cubics.
45
First, do a translation in the x direction to center the origin on the point of inection. Find the point of inection by taking
the second derivative.

1
f 00 = 6a + 3 =⇒ a = −
2
So

1
a=x−
2
1 3 3 1 2 1 3 1
=⇒ (x − ) + (x − ) − = x3 = x − = 0
2 2 2 2 4 4

Then recall this neat trigonometric fact:

cos 3x = cos 2x cos x − sin 2x sin x = 4 cos3 x − 3 cos x


3 cos 3x
=⇒ cos3 x = cos x − =0
4 4

Particularly for this problem, we have x = 0, 2π/3, 4π/3. cos x = 1, − 21 .


cos 3x = 1. So Plugging cos x → x back into
1
what we have for a, a = −1, which we already have in the previous part, and a = . So
2

µ ¶
1 3
f =
2 8
µ ¶
1 1
y(x) = x +
4 4

Exercise 9.

(
x2 if x ≤ c
f (x) =
ax + b if x > c
(
2x if x ≤ c
f 0 (x) =
a if x > c

a = 2c; b = −c2

Exercise 10.

(
1
|x| if |x| > c
f (x) =
a + bx2 if |x| ≤ c

Note that c ≮ 0 since |x| ≤ c, for the second condition.


 1
− x2 if x>c
0 1
f (x) = if x < c
 x2

2bx if |x| ≤ c

1 3
So b=− ,a= .
2c3 2c
Exercise 11.

(
0 cos x if x≤c
f =
a if a > c
46
³ √ ´
1−√2 1−A
Exercise 12. f (x) = 1+ 2
= 1+A

√ 1 −1/2 1 1 1
A= x A0 = a =
x = ; A00 = − x−3/2 = − 3
2 2A 4 4A
−A0 (1 + A) − A0 (1 − A) −2A0 −1
f0 = 2
= 2
=√ √
(1 + A) (1 + A) x(1 + x)2

1 3 x+1
f 00 = 0 2 0
(A (1 + A) + A(2)(1 + A)A ) = 3/2 √
(A(1 + A2 ))2 2x (1 + x)3
µ 1 ¶
1 −1 0 2 3 0 3 2
A2 A (A (1 + A) ) − (2AA (1 + A) + 3A (1 + A) A )(3 +
2 0
A)
f 000 =
2 (A2 (1 + A)3 )2
µ1 ¶ √
−3 A + 4 + 5A 3 (1 + 4 x + 5x)
= = − √ √
4 A4 (1 + A4 ) 4 x(x + x)4

Exercise 13.

P = ax3 + bx2 + cx + d
P 00 (0) = 2b = 10 =⇒ b = 5
P 0 = 3ax2 + 2bx + c
P 0 (0) = c = −1 P (0) = d = −2
P 00 = 6ax + 2b
P (1) = a + 5 + −1 + −2 = a + 2 = −2 =⇒ a = −4

Exercise 14.

f0 f0 g0 1
f g = 2, = 2 = 4 = 2f = , g = 4
g0 g g 2
(1)

f 0 g − g0 f f0 g0 f 1 15
h0 = = − = 4 − 2( ) =
g2 g g g 8 4
(2)

k 0 = f 0 g + f g 0 = 4g 2 + f 2g = 64 + 4 = 68
(3)

g 0 (x) limx→0 g 0 (x) 1


lim = =
x→0 f 0 (x) 0
limx→0 f (x) 2

Exercise 15.

(1) True, by denition of f 0 (a).


(2)

f (a) − f (a − h) f (a − h) − f (a) f (a − h) − f (a)


lim = − lim = lim = f 0 (a)
h→0 h h→0 h −h→0 −h
True, by denition of f 0 (a).
(3)

f (a + 2t) − f (a) f (a + 2t) − f (a)


lim = 2 lim = 2f 0 (a)
t→0 t 2t→0 2t
False.
(4)

f (a + 2t) − f (a) + f (a) − f (a + t)


lim =
t→0 2t
f (a + 2t) − f (a) 1 f (a + t) − f (a)
lim + − lim =
2t→0 2t 2 t→0 t
1 1
f 0 (a) − f 0 (a) = f 0 (a)
2 2
False.

Exercise 16.

47
(1)
(f (x + h) + g(x + h))2 − (f (x) + g(x))2 (F + G)2 − (f + g)2
D∗ (f + g) = lim = lim =
h→0 h h→0 h
2F G − 2f g
= D∗ f + D∗ g + lim
h→0 h
2F G − 2f g (2(F G) − 2f G)(F + f ) (2f G − 2f g)(G + g)
lim = lim + =
h→0 h h→0 (F + f )h (g + G)h
2g F2 − f2 2f G2 − g 2
= lim lim + lim lim =
h→0 F + h h→0 h h→0 G + g h→0 h
g f
= D∗ f + D∗ g
f g
(f (x + h) − g(x + h))2 − (f (x) − g(x))2
D∗ (f − g) = lim =
h→0 h
(F − G)2 − (f − g)2
= lim =
h→0 h
2F G − 2f g
= D∗ f + D∗ g + − lim
h→0 h
g f
= D∗ f + D∗ g − D∗ f + D∗ g
f g

((f g)(x + h))2 − ((f g)(x))2


D∗ (f g) = lim =
h→0 h
(f 2 (x + h))(g 2 (x + h)) − f 2 (x)g 2 (x + h) + (g 2 (x + h) − g 2 (x))f 2 (x)
= lim =
h→0 h
= g 2 D∗ f + f 2 D∗ g
f 2 (x+h) f 2 (x) f 2 (x+h)−f 2 (x) f 2 (x) f 2 (x)
∗ g 2 (x+h) − g 2 (x) g 2 (x+h) + g 2 (x+h) − g 2 (x)
D (f /g) = lim = lim =
h→0 h h→0 h

D f f2
= + (−D∗ g) when g(x) 6= 0
g2 g4
(2)
(3)

4.12 Exercises - The chain rule for differentiating composite functions, Applications of the chain rule. Related rates
and implicit differentiation. Exercise 1. −2 sin 2x − 2 cos x
x
Exercise 2. √
1+x2
Exercise 3. −2x cos x2 + 2x(x2 − 2) sin x2 + 2 sin x3 + 6x3 cos x3
Exercise 4.
f 0 = cos (cos2 x)(−2 cos x sin x) cos (sin2 x) + sin (cos2 x) sin (sin2 x)(2 sin x cos x) =
= − sin 2x(cos (cos 2x))
Exercise 5.

f 0 = n sinn−1 x cos x cos nx + −n sin nx sinn x


Exercise 6.

f 0 = cos (sin (sin x))(cos (sin x))(cos x)


Exercise 7.
2 sin x cos x sin x2 − 2x cos x2 sin2 x sin 2x sin x2 − 2x sin2 x cos x2
f0 = 2 2 =
sin x sin2 x2
1 x 1 x
Exercise 8. f0 = 2 sec2 2 + 2 csc2 2
Exercise 9.f 0 = 2 sec2 x tan x + −2 csc2 x cot x
0
√ x2 1+2x2
Exercise 10. f = 1 + x2 + √1+x 2
=√ 1+x2
4
Exercise 11. f0 = (4−x2 )3/2
48
Exercise 12.
µ ¶−2/3 µ ¶ µ ¶−2/3
1 1 + x3 3x2 (2) 2x2 1 + x3
f0 = =
3 1 − x3 (1 − x3 )2 (1 − x3 )2 1 − x3
Exercise 13. This exercise is important. It shows a neat integration trick.
à √ !
1 1 x − 1 + x2
f (x) = √ √ =√ √ √ =
2
1 + x (x + 1 + x ) 2 1 + x (x + 1 + x2 )
2 x − 1 + x2

x − 1 + x2 x
= √ =1− √
− 1+x 2 1 + x2
√ x2
1 + x2 − √1+x 2 1
0
f = =
1 + x2 (1 + x2 )3/2
Exercise 14.
q µ ¶
1 √ 1 √ 1
(x + x + x)−1/2 (1 + (x + x)−1/2 1 + √ )
2 2 2 x
Exercise 15.

f 0 = (2 + x2 )1/2 (3 + x3 )1/3 + (1 + x)x(2 + x2 )−1/2 (3 + x2 )1/3 + (1 + x)(2 + x2 )1/2 (3 + x3 )−2/3 x2

Exercise 16.
µ ¶ µ ¶
−1 −1 1 −1 −1 f0
f0 = ¡ ¢ = g0 = ³ ´2 f0 =
1 2 x2 (x + 1)2 f2 (f + 1)2
1+ x 1 + f1
(x + 1)−2 1
g0 = ³ ´2 =
x (2x + 1)2
x+1 + 1

Exercise 17. h0 = f 0 g 0
x h h0 k k0
0 f (2) = 0 2(−5) = −10 g(1) = 0 1(5) = 5
1 f (0) = 1 5(1) = 5 g(3) = 1 −6(−2) = 12
2 f (3) = 2 4(1) = 4 g(0) = 2 −5(2) = −10
3 f (1) = 3 −2(−6) = 12 g(2) = 3 1(4) = 4
Exercise 18.
g(x) = xf (x2 )
g 0 (x) = f (x2 ) + x(2x)f 0 (x2 ) = f (x2 ) + 2x2 f 0 (x2 )
g 00 (x) = 2xf 0 (x2 ) + 4xf 0 (x2 ) + 2x2 (2x)f 00 (x2 ) = 6xf 0 (x2 ) + 4x3 f 00 (x2 )
x g(x) g 0 (x) g 00 (x)
0 0 0 0
1 1 3 10
2 12 6 + 8(3) = 30 12(3) + 32(0) = 36
Exercise 19.

(1)
df (x2 )
g0 = 2x = 2xf 0
dx2
(2)

g 0 = 2 sin x cos xf 0 − 2 cos x sin xf 0 = (sin 2x)(f 0 (sin2 x) − f 0 (cos2 x))


(3)
df (f (x)) 0
g0 = f
d(f (x))
(4)
df (f (f (x))) d(f (f (x))) df
g0 =
d(f (f (x))) d(f (x)) dx
49
Exercise 20. V = s3 , s = s(t) dV 2 ds
dt = 3s dt .
s = 5cm 75cm3 /sec
s = 10cm 300cm3 /sec
s = xcm 3x2 cm3 /sec

Exercise 21.
p dl 1 dx
x2 + h2
l= = x
dt l dt µ ¶
dx l dl 10mi 20 mi 3600sec
= = √ (−4mi/sec) =
dt x dt − 102 − 82 3 sec 1hr
Exercise 22.

l 2 = x 2 + s2
dl dx dl x dx
2l = 2x =
dt dt dt l dt
dl ³ s´ √
x= = 20 5
dt 2
dl √
(x = s) = 50 2
dt
Exercise 23.

dl x dx 3 36
= = 12 = mi/hr
dt l dt 5 5

Exercise 24. Given the preliminary information

r 2 1 1
= = α, V = πr2 h = πα2 h3
h 5 3 3
(1)

πr2 2 1
V = (h y − hy 2 + y 3 )
h2 3
dV πr2 2 dy
= 2 (h − 2hy + y 2 )
µ dt ¶h µ dt ¶
dy h2 1 dV 102 1 5
= 2 2 2
= 2 2
5=
dt πr h − 2hy + y dt π4 10 − 2(10)5 + 25 4π
(2)
dV dh dh 1 dV 5
= πα2 h2 , = =
dt dt dt πα2 h2 dt 4π
Exercise 25.
r 3
α= =
h 2
dV 2 2 dh
= πα h
dt dt
9
c − 1π (22 )4 = 36π =⇒ c = 36π + 1
4
Exercise 26. The constraint equation, using Pythagorean theorem on the geometry of a bottom hemisphere, is

r2 = R2 − (R − h)2 = 2Rh − h2
So then
dr dh
r = (R − h)
dt dt
Z
dV
V = πr2 dh =⇒ = πr2 = π(2Rh − h2 )
dh
dV
=⇒ = π(2(10(5) − 25)) = 50π
dh
50
µ ¶
dV dV dh dh dV 1
= , =⇒ =
dt dh dt dt dt π(2Rh − h2 )
µ ¶ µ ¶
r dr dV 1
=
R − h dt dt π(2Rh − h2 )
µ ¶
dr dV R−h
= =
dt dt rπ(2Rh − h2 )
=⇒ µ ¶
√ 10 − 5 1
= (5 3) 3/2
=
π(2(10)5 − 25) 15π
Exercise 27. I suppose the area of the triangle is 0 at t = 0. ¡ ¢
Now the point on vertex B moves up along the y axis according to y = 1 + 2t. y 72 = 8.
r
1 36
A= (y − 1) y
2 7
Ãr r !
dA 1 36 1 1 36 dy
= √ y + (y − 1) =
dt 2 y 2 y−1 7 dt
µ ¶
1 6 66
= 8 + 6 (2) =
2 2(7) 7
πR2
Exercise 28. From the given information, h = 3r + 3. The volume formula is V = 3 H . So then

V = π/3r2 (3r + 3) = πr3 + πr2


dV dr
= πr(3r + 2)
dr dt
With the given information, we get
dr 1
=
dt π(6)(20)
Using this, we can plug this back in for the different case:

dV
= n = π(36)(110)/(120π) = 33
dt
Exercise 29.
dy
(1) = 2x dx
dt dt ; when x = 12 , y = 41 , dy
dt =
dx
dt
π
(2) t=
6
Exercise 30.

(1) 3x2 + 3y 2 y 0 = 0 =⇒ x2 + y 2 y 0 = 0
(2)
2x + 2yy 02 + y 2 y 00 = 0 =⇒ y 2 y 00 = −2(x + yy 02 )
µ 4 ¶
00 xy + yx4
=⇒ y = −2 = −2xy −5
y6
Exercise 31. √
1 1 1 − y
√ + √ y0 = 0 y0 = √ < 0
2 x 2 y x
Exercise 32. r
12 − 3x2
±
4
−3x
6x + 8yy 0 = 0 =⇒ y 0 =
4y
3 + 4(y 02 + yy 00 ) = 0
µ ¶
3 1 −9
y 00 = − − y 02 = 3
4 y 4y
Exercise 33.
sin xy + x cos2 xy(y + xy 0 ) + 4x = 0
y 0 x2 cos xy + xy cos xy + sin xy + 4x = 0
51
Exercise 34. y = x4 . y n = xm .
mxm−1 m xm−1
y n = xm , y 0 ny n−1 = mxm−1 ; y 0 = n−1
= =
ny n xm(1−1/n)
m m/n−1
y0 = x
n

4.15 Exercises - Applications of differentiation to extreme values of functions, The mean-value theorem for derivatives.
Let's recap what was shown in the past two sections:

Theorem 13 (Theorem 4.3).


Let f I.
be dened on
Assume f has a rel. extrema at an int. pt. c ∈ I .
0 0
If ∃ f (c), f (c) = 0; the converse is not true.

Proof. Q(x) = f (x)−fx−c


(c)
if x 6= c, Q(c) = f 0 (c)
0
∃f (c), so Q(x) → Q(c) as x → c so Q is continuous at c.
f (x)−f (c)
If Q(c) > 0, > 0. For x − c ≷ 0, f (x) ≷ f (c), thus contradicting the rel. max or rel. min. (no neighborhood
x−c
about c exists for one!)
f (x)−f (c)
If Q(c) < 0, < 0. For x − c ≷ 0, f (x) ≶ f (c), thus contradicting the rel. max or rel. min. (no neighborhood
x−c
about c exists for one!)
Converse is not true: e.g. saddle points. ¤

Theorem 14 (Rolle's Theorem).


Let f be cont. on [a, b], ∃f 0 (x) ∀x ∈ (a, b) and let

f (a) = f (b)

then ∃ at least one c ∈ (a, b), such that f 0 (c) = 0.

Proof. Suppose f 0 (x) 6= 0 ∀x ∈ (a, b).


By extreme value theorem, ∃ abs. max (min) M, m somewhere on [a, b].
M, m on endpoints a, b (Thm 4.3).
F (a) = f (b), so m = M . f constant on [a, b]. Contradict f 0 (x) 6= 0 ¤

Theorem 15 (Mean-value theorem for Derivatives). Assume f is cont. everywhere on [a, b], ∃f 0 (x) ∀x ∈ (a, b).
∃ at least one c ∈ (a, b) such that

(6) f (b) − f (a) = f 0 (c)(b − a)

Proof.

h(x) = f (x)(b − a) − x(f (b) − f (a))


h(a) = f (a)b − f (a)a − af (b) + af (a)
h(b) = f (b)(b − a) − b(f (b) − f (a)) = bf (a) − af (b) = h(a)
=⇒ ∃c ∈ (a, b), such that h0 (c) = 0 = f 0 (c)(b − a) − (f (b) − f (a))
¤

Theorem 16 (Cauchy's Mean-Value Formula). Let f, g cont. on [a, b], ∃f 0 , g 0 ∀x ∈ (a, b)


Then ∃ c ∈ (a, b). x

(7) f 0 (c)(g(b) − g(a)) = g 0 (c)(f (b) − f (a)) (note how it's symmetrical)

Proof.

h(x) = f (x)(g(b) − g(a)) − g(x)(f (b) − f (a))


h(a) = f (a)(g(b) − g(a)) − g(a)(f (b) − f (a)) = f (a)g(b) − g(a)f (b)
h(b) = f (b)(g(b) − g(a)) − g(b)(f (b) − f (a))
=⇒ h0 (c) = f 0 (c)(g(b) − g(a)) − g 0 (c)(f (b) − f (a)) = 0 (by Rolle's Thm.)

¤
52
Exercise 1. For any quadratic polynomial y = y(x) = Ax2 + Bx + C ,
y(a) = Aa2 + Ba + C
y(b) = Ab2 + Bb + C
y(b) − y(a) A(b − a)(b + a) + B(b − a)
= = A(b + a) + B
b−a b−a
y 0 = 2Ax + B
µ ¶
a+b
y0 = A(a + b) + B
2
Thus the chord joining a and b has the same slope as the tangent line at the midpt.
Exercise 2. The contrapositive of a theorem is always true. So the contrapositive of Rolle's Theorem is
If @ at least one c ∈ (a, b) s.t. f 0 (c) = 0,
then f (a) 6= f (b).

g 0 = 3x2 − 3 = 3(x2 − 1) =⇒ g 0 (±1) = 0


Suppose g(B) = 0, B ∈ (−1, 1)
then ∀ x ∈ (−1, 1), x 6= B, g(x) 6= g(B), so g(x) 6= 0 for x 6= B
so only at most one B ∈ (−1, 1) s.t. g(B) = 0
3−x2 1
Exercise 3. f (x) = 2 if x ≤ 1, f (x) = x if x ≥ 1.
(1) See sketch.
(2)
(
3−x2
2 if x≤1
f (x) = f (1) = 1 = f (1) = 1/1
1/x if x ≥ 1
(
−x; f 0 (1) = −1 for x ≤ 1
f 0 (x) =
−1/x2 ; f 0 (1) = −1 for x > 1
Then f (x) is cont. and diff. on [0, 2].

For 0≤a<b≤1 ³ ´
3−b2 3−a2
2 − −(a + b)
2
= = −c
b−a 2
0
Note that −1 ≤ f ≤ 0 for 0 ≤ x ≤ 1

For 1≤a<b≤2
1
− a1 −1 −1 √
b
= = 2 =⇒ c = ab
b−a ab c
0
Note that −1 ≤ f ≤ −1/4

For 0≤a≤1, 1≤b≤2


³ ´
1 3−a2
b − 2 − (3 − a2 )b
2 −1
== −c or 2
b−a 2b(b − a) c
depending upon if 0 ≤ c ≤ 1 or 1 ≤ c ≤ 2, respectively

f (b)−f (a) √
For instance, for a = 0, b = 2, then = −1/2, so c = 1/2 or c = 2
b−a
Exercise 4.
f (1) = 1 − 12/3 = 0 = f (−1) = 1 − ((−1)2 )3 = 0
−2 −1/3
f0 = x 6= 0 for |x| ≤ 1
3
This is possible since f is not differentiable at x = 0.
2 2 0
Exercise 5. x = x sin x + cos x. g = xS + C − x . g = S + xC − S − 2x = xC − 2x = x(C − 2). Since |C| ≤ 1 then
0
(C − 2) is negative for all x. Then for x ≷ 0, g ≶ 0. Since g(0) = 1 and for x → ±∞, g → ∓∞, then we could conclude
that g must become zero between 0 and ∞ and −∞ and 0.
53
Exercise 6.
f (b) − f (a)
= f 0 (c)
b−a
b=x+h
b−a=h
=⇒ f (x + h) − f (x) = hf 0 (x + θh)
a=x
x < x + θh < x + h
2 0
(1) f (x) = x , f = 2x.
(x + h)2 − x2 = 2xh + h2 = h(2(x + θh))
2x + h 1 1
− x = θh =⇒ θ = so then lim θ =
2 2 h→0 2
(2) f (x) = x3 , f 0 = 3x2 .
! Ãr
3 3 2 2 3 3x2 + 3xh + h2 2
(x + h) − x = 3x h + 3xh + h = h3(x + θh) =⇒ − x /h = θ
3
q q 
r 2 2
3x2 + 3xh + h2 x x2 + xh + h3 − x x2 + xh + h3 + x
θ= − = q =
3h2 h h 2
x2 + xh + h3 + x
h
x+ 3
= q
h2
x+ x2 + hx + 3

1
=⇒ lim θ =
h→0 2
Notice the trick of multiplying by the conjugate on top and bottom to get a way to evaluate the limit.

Exercise 7. f (x) = (x − a1 )(x − a2 ) . . . (x − ar )g(x).

(1) a1 < a2 < · · · < ar .


Since f (a1 ) = f (a2 ) = 0. f 0 (c) = 0 for c1 ∈ (a1 , a2 ).
0
Consider that f (a2 ) = f (a3 ) = 0 as well as f (c2 ) = 0 for c ∈ (a2 , a3 ).
0
Indeed, since f (aj ) = f (aj+1 ) = 0, f (c) = 0 for c ∈ (aj , aj+1 ).
Thus, ∃r − 1 zero's.
f (k) has r − k zeros in [a, b].
f (k) = (x − a1 )(x − a2 ) . . . (x − ar−k )gk (x)
Since f (a1 ) = f (a2 ) = 0, f (k+1) (c1 ) = 0 for c1 ∈ (a1 , a2 ).
f (k) (aj ) = f (k) (aj+1 ) = 0, f (k+1) (cj ) = 0 for cj ∈ (aj , aj+1 )
=⇒ f (k) (x) has at least r − k zeros in [a, b]
We had shown the above by induction.
(2) We can conclude that there's at most r + k zeros for f (since f (k) has exactly r zeros, the intervals containing the r
zeros are denite).

Exercise 8. Using the mean value theorem

(1)
¯ ¯
sin x − sin y ¯ sin x − sin y ¯
= cos c =⇒ ¯ ¯ ¯ = | cos c| ≤ 1
x−y x−y ¯
=⇒ | sin x − sin y| ≤ |x − y|
(2) x ≥ y > 0.
f (z) = z n is monotonically increasing for n ∈ Z.
By mean-value theorem,
xn − y n
= ncn−1 for y<c<x
x−y
xn −y n
Since 0 < y < c < x; ny n−1 ≤ x−y ≤ nxn−1 .
54
³ ´ ³ ´
f (b)−f (a) bf (a)−af (b)
Exercise 9. Let g(x) = b−a x+ b−a .

h(a) = h(c)
f −g =h
h(c) = h(b)
so ∃c1 ∈ (a, c), c2 ∈ (c, b) s. t. h (c1 ) = h0 (c2 ) = 0 by Rolle's Thm.
0

Let h0 = H
since H(c1 ) = H(c2 ) = 0 and H is cont. diff. on (c1 , c2 ). then
0 00
∃c3 ∈ (c1 , c2 ) s.t. H (c3 ) = h (c3 ) = 0
Now h00 = (f − g)00 = f 00 so f 00 (c3 ) = 0
We've shown one exists; that's enough.

Exercise 10. Assume f has a derivative everywhere on an open interval I.


f (x) − f (a)
g(x) = if x 6= a; g(a) = f 0 (a)
x−a
³ ´
1 1
(1) g= x−a f − x−a f (a). f is cont. on (a, b] since ∃ f 0 ∀x ∈ (a, b).
1
x−a is cont. on (a, b]. Then g is cont. on (a, b] (remember, you can add, subtract, multiply, and divide cont. functions
to get cont. functions because the rules for taking limits allow so).
g is cont. at a since limx→a g = limx→a f (x)−f x−a
(a)
= f 0 (a).

By mean value theorem,


µ ¶
f (x) − f (a)
= f 0 (c) = g(x) ∀c ∈ (a, x) ∀x ∈ (a, b]
x−a
Then ∀c ∈ [a, b], f 0 (c) ranges from f 0 (a) to g(b) since f 0 (c) = g(x) so whatever g(x) ranges from and to, so does
0
f (c).
(2) Let h(x) = f (x)−f
x−b
(b)
if x 6= b; h(b) = f 0 (b).
1
h is cont. on [a, b) since x−b is cont., f (x) is cont.
f (x)−f (b)
limx→b h = limx→b x−b = f 0 (b) so h is cont. at b.

h is cont. on [a, b] → h takes all values from h(a) to f 0 (b) on [a, b] (by intermediate value theorem).

By mean value theorem,

f (b) − f (x)
h(x) = = f 0 (c2 ) for c2 ∈ (x, b) ∀ x ∈ [a, b]
b−x
0 0
So then f ranges from h(a) to f (b) just like h.

h(a) = g(b). So then f 0 must range from f 0 (a) to f 0 (b)


4.19 Exercises - Applications of the mean-value theorem to geometric properties of functions, Second-derivative test
for extrema, Curve sketching.

Exercise 1. f (x) = x2 − 3x + 2
0 3
(1) f (x) = 2x − 3 x0 = 2.
(2) f 0 (x) ≷ 0 for x ≷ 32
(3) f 00 = 2 > 0 for ∀ x ∈ R
(4) See sketch.

Exercise 2. f (x) = x3 − 4x
(1) f = 3x2 − 4
0
xc = ± √23
(2) f 0 ≷ 0 when |x| ≷ √23
(3) f 00 = 6x f 00 ≷ 0 when x ≷ 0
(4) See sketch.

Exercise 3. f (x) = (x − 1)2 (x + 2)


0
(1) f = 3(x − 1)(x + 1) f 0 (x) = 0 when x = ±1
55
(2) f 0 ≷ when |x| ≷ 1
(3) f 00 = 3(2x) = 6x f 00 ≷ 0 when x ≷ 0
(4) See sketch.

Exercise 4. f (x) = x3 − 6x2 + 9x + 5


(1) f 0 = 3x2 − 12x + 9 = 3(x − 3)(x − 1) f 0 (x) = 0 when x = 3, 1
(2)

f 0 (x) > 0 when x < 1, x > 3


f 0 (x) < 0 when 1 < x < 3
(3) f 00 = 6x − 12 = 6(x − 2) f 00 ≷ 0 when x ≷ 2
(4) See sketch.

Exercise 5. f (x) = 2 + (x − 1)4


(1) f 0 (x) = 4(x − 1)3 . f 0 (0) = 0 when x = 1
(2) f 0 (x) ≷ 0 when |x| ≷ 1
(3) f 00 (x) = 12(x − 1)2 > 0 ∀ x 6= 1
(4) See sketch.

Exercise 6. f (x) = 1/x2


(1) f 0 = −2 0
x3 f (x) = 0 for no x
0
(2) f ≷ 0 when x ≶ 0
(3) f 00 = x64 > 0 ∀x 6= 0
(4) See sketch.

Exercise 7. f (x) = x + 1/x2


−2 2
(1) f0 = 1 + x3 f 0 (x) = 0 = 1 − x3 =⇒ xc = 21/3
(2)

f 0 (x) > 0 when x < 0, 0 < x < 21/3


f 0 (x) < 0 when x > 21/3
6
(3) f 00 =
x4 > 0 ∀x 6= 0
(4) See sketch.

1
Exercise 8. f (x) = (x−1)(x−3)
(−2)(x−2)
(1) f 0 = (x−1)−1
2 (x−3)2 ((x − 3) + x − 1) = (x−1)2 (x−3)2
0
f (x) = 0 when x = 2
(2) f 0 ≷ 0 when x ≶ 2
(3)
µ ¶
(x − 1)2 (x − 3)2 − (x − 2)(2(x − 1)(x − 3)2 + 2(x − 3)(x − 1)2 )
f 00 = (−2) =
(x − 1)4 (x − 3)4
µ 2 ¶
x − 4x + 13 3
= (6)
(x − 1)3 (x − 3)3
13
x2 − 4x + > 0 since 144 − 4(−3)(−13) = 144 + 12(−13) < 0 so
3
f 00 > 0 if x > 3, x < 1
f 00 < 0 if 1 < x < 3
(4) See sketch.

Exercise 9. f (x) = x/(1 + x2 )


(1)

(1 + x2 ) − x(2x) 1 − x2
f0 = 2 2
=
(1 + x ) (1 + x2 )2
f 0 (x) = 0 when x = ±1
(2) f 0 ≷ 0 when |x| ≶ 1
56
(3)
−2x(1 + x2 )2 − 2(1 + x2 )(2x)(1 − x2 ) 2x(x2 − 3)
f 00 = =
(1 + x2 )4 (1 + x2 )3
00

f > 0 when x > 3

f 00 < 0 when 0 < x < 3

f 00 > 0 when − 3 < x < 0

f 00 < 0 when x < − 3
(4) See sketch.

Exercise 10. f (x) = (x2 − 4)/(x2 − 9)


(1)
2x(x2 − 9) − (x2 − 4)(2x) −10x
f0 = = 2
(x2 − 9)2 (x − 9)2
f 0 (0) = 0
(2) f 0 ≷ 0 when x ≶ 0, x 6= ±3
(3)
µ ¶
(x2 − 9)2 − 2(x2 − 9)(2x)x (x2 + 3)
f 00 = (−10) = (30)
(x2 − 9)2 (x2 − 9)3
f 00 ≷ 0 when |x| ≷ 3
(4) See the sketch.

Exercise 11.f (x) = sin2 x


π
(1) f = sin 2x So then f 0 = 0 when x =
0
2n
(2)
π
0<x<
f 0 > 0 when 2
π
πn < x < + πn
2
π
f 0 < 0 when + πn < x < π(n + 1)
2
(3)
−π π
f 00 > 0 when + πn < x < + πn
f 00 = 2 cos 2x 4 4
π 3π
f 00 < 0 when + πn < x < + πn
4 4
(4) See sketch.

Exercise 12. f (x) = x − sin x


(1) f 0 = 1 − cos x f 0 = 0 when x = 2πn
0
(2) f > 0 if x 6= 2πn
(3)
f 00 > 0 when 2πn < x < 2πn + π
f 00 = sin x
f 00 < 0 when 2πn + π < x < 2π(n + 1)
(4) See sketch.

Exercise 13. f (x) = x + cos x


0 π
(1) f = 1 − sin x x= 2 + 2πn f 0 (x) = 0
(2) f 0 > 0 if x 6= π2 + 2πn
(3)
−π π
f 00 > 0 when + 2πn < x < + 2πn
f 00 = − cos x 2 2
π 3π
f 00 < 0 when + 2πn < x < + 2πn
2 2
(4) See sketch.

Exercise 14. f (x) = 16 x2 + 1


12 cos 2x
0 1 − sin 2x
(1) f = 3x + 6 f 0 (0) = 0
(2) f0 ≷ 0 when x ≷ 0
57
(3) f 00 = 13 − cos32x = 1−cos
3
2x

x = πn for f = 0. Otherwise f 00 > 0 for x 6= πn


00

(4) See sketch.

4.21 Exercises - Worked examples of extremum problems.


Exercise 1.

A = xy
A
P = 2(x + y) = 2(x + )
x
A
P 0 = 2(1 −
2
)=0
x√
x= A
4A √
P 00 = 3 > 0 for x > 0 so x = A minimizes P
x
Exercise 2.
A = xy L = 2x + y
dA L L
A = x(L − 2x) = Lx − 2x2 =⇒ = L − 4x = 0 when x = y=
dx 4 2
L
A00 = −4 so x = maximizes A
4
Exercise 3.

A dL −A A √ √
A = xy L = 2x + y = 2x + = 2 + 2 = 0 when x = √ y= 2 A
x dx x 2
r
2A A
L00 = 3 > 0 for x = so x minimizes L
x 2
Exercise 4.f = x2 + y 2 = x2 + (S − x)2
0 S
f = 2x + 2(S − x)(−1) = −2S + 4x =⇒ x = 2
f 00 = 4 > 0 so x = S2 minimizes f
Exercise 5. x2 + y 2 = R > 0
f =x+y
−x
f 0 = 1 + y0 = 0 = 1 + =0 =⇒ y = x
y
−1 − y 02
f 00 = y 00 = 00
for y > 0, f < 0 so that f is max. when y = x
y
2x + 2yy 0 = 0
−x
Note that y0 = 1 + y 02 + yy 00 = 0 =⇒ yy 00 = −1 − y 02
y
x + yy 0 = 0
Exercise 6.
l2 = (L − x)2 + x2 = L2 − 2Lx + 2x2 = A
dA L
= −2L + 4x = 0 =⇒ x =
dx 2
d2 A
= 4 > 0 =⇒ A minimized
dx2 √
L L 2
l(x = ) =
2 2
Exercise 7.
p
L2 − x2 )2 = A
(x +
p −x L
A0 = 2(x + L2 − x2 )(1 + √ ) = 0 when L2 − x2 = x2 or x= √
L2 − x2 2
r
L L2 2L
so then the side of the circumscribing and area-maximized square is √ + L2 − =√
2 2 2
58
Exercise 8.
p p
R2 − x2 ) = 4x R2 − x2
A = (2x)(2
p µ 2 ¶
0 2 2
−x2 R − 2x2 R
A = 4( R − x + √ )=4 √ =⇒ x = √
2
R −x 2 2
R −x 2 2
0 R R
since A ≷ 0 when x ≶ √ , so A is maximized at x = √
2 2
2R
√ 2R
2x = √
2
; 2 R 2 − x2 = √
2
so then the rectangle that has maximum size is a square.

Exercise 9. Prove that among all rectangles of a given area, the square has the smallest circumscribed circle.
√ √
A0 = (2x)(2 r2 − x2 ) = 4x r2 − x2 (x the area to be A0 )
¡ A ¢2 A2
0
4x = r2 − x2 =⇒ x4 − x2 r2 + 160 = 0

dr
=⇒ 0 = 2xr2 + x2 2r − 4x3
dx
dr r p r
= 0 (for extrema) =⇒ x = √ and r2 − x2 = √
dx 2 2
We could argue that we had found a minimum because at the “innity” boundaries, the circumscribing circle would be
innitely large.

Exercise 10. Given a sphere of radius R, nd the radius r and altitude h of the right circular cylinder with the largest lateral
surface area 2πrh that can be inscribed in the sphere.
µ ¶2
h
R2 = + r2
2
p p
A = 2πrh = 2πr4(R2 − r2 ) = 4πr R2 − r2
µp ¶ µ 2 ¶
dA −r2 R − 2r2 R
= 4π R2 − r 2 + √ = 4π √ =⇒ r = √
dr R2 − r 2 R2 − r 2 2

=⇒ h = 2R

Exercise 11. Among all right circular cylinders of given lateral surface area, prove that the smallest circumscribed sphere has

radius 2 times that of the cylinder.

A0 = 2πRH (A0 is the total lateral area of the cylinder)


µ ¶2 µ ¶2
H A0 A20
r 2 = R2 + = R2 + = R2 +
2 4πR 16π 2 R2
µ ¶ √
dr A20 −2 dr A0 H
2r = 2R + =⇒ = 0 =⇒ R = √ =⇒ =R
dR 16π 2 R3 dR 2 π 2

r2 = R2 + R2 = 2R2 =⇒ r = 2R

Exercise 12. Given a right circular cone with radius R and altitude H . Find the radius and altitude of the right circular cylinder
of largest lateral surface area that can be inscribed in the cone.
h H
R−r = R = α is the constraint (look, directly at the side, at the similar triangles formed)

A = 2πrh = 2πrα(R − r) = 2πα(rR − r2 )


dA R H
= 2πα(R − 2r) = 0 =⇒ r = ; h=
dr 2 2
A00 = 2πα(R − 2r)
dA R R
since ≷ 0 when r ≶ , r= maximizes lateral surface area
dr 2 2

Exercise 13. Find the dimensions of the right circular cylinder of maximum volume that can be inscribed in a right circular

cone of radius R and altitude H .


59
h H
Constraint:
R−r = R =α

V = πr2 h = πr2 α(R − r) = πr2 α(R − r) = πα(Rr2 − r3 )


dV 2R
= πα(2Rr − 3r2 ) = rπα(2R − 3r) r=
dr 3
dV 2R 2R
since ≷ 0 when r ≶ , r= maximizes volume
dr 3 3
1
h= H
3
Exercise 14. Given a sphere of radius R. Compute, in terms of R, the radius r and the altitude h of the right circular cone of
maximum volume that can be inscribed in this sphere.

πr2 ³ p ´
R + R2 − r 2
V =
3 √
µ p 2

dV π r (−r) π (2R R2 − r2 + 2R2 − 3r2 )
= 2rR + 2r R2 − r2 + √ = r √ =0
dr 3 R2 − r 2 3 R2 − r 2

2 2R 4R
=⇒ r = ; h=
3 3

2 2R
Considering the geometric or physical constraints, since limV →∞ V = limh→∞ V = 0, so then r = 3 must maximize
V.
Exercise 15. Find the rectangle of largest area that can be inscribed in a semicircle, the lower base being on the diameter.

p
A= R2 − x2 x
p −x2 R R
A0 = R2 − x2 + √ = 0 =⇒ x = √ ; h= √
2
R −x 2 2 2
Exercise 16. Find the trapezoid of largest area that can be inscribed in a semicircle, the lower base being on the diameter.

1 p
h(2 R2 − h2 + 2R)
A=
2 µ ¶
dA p 2 2
−h
= R −h +R+h √
dh R 2 − h2

dA 3R
= 0 =⇒ h =
dh 2
√ r
5 3R2 p 3 R
=⇒ A = R 2 − h2 = 2 R 2 − R 2 = 2 = R
8 4 2
Exercise 17. An open box is made from a rectangular piece of material by removing equal squares at each corner and turning

up the sides. Find the dimensions of the box of largest volume that can be made in this manner if the material has sides (a)
10 and 10; (b) 12 and 18

(1)
(x − 2r)(Y − 2r)r = (xy − 2rx − 2ry + 4r2 )r = xyr − 2r2 x − 2r2 y + 4r3 = V
dV
= xy − 4rx − 4ry + 12r2 = 0
pdr p
4(x + y) ± 16(x + y)2 − 4(12)xy (x + y) ± x2 + y 2 − xy
=⇒ r = =
2(12) 6
2
d V
= −4x − 4y + 24r = −4(x + y) + 24r
dr2
We can plug in our expression for r into the second derivative of V , the volume of the box, to nd out that we want
to pick the “negative” root from r , in order to maximize the box volume.
5 5 20 20
Then for x = 10; y = 10, we have r = , so that the box dimensions are
3 3 × 3 × 3 .
(2) 12 and 18
√ √ √
=⇒ 5 − 7×2+2 7×8+2 7
60
Exercise 18. If a and b are the legs of a right triangle whose hypotenuse is 1, nd the largest value of 2a + b.

p ³ a ´2
L = 2a + b = 2a + 1 − a2 2
L0 = 0 =⇒ = 1 − a2 =⇒ a = √
−a 2 5
L0 = 2 + √
1−a 2
Ã√ −a
! µ ¶
1 − a2 − √1−a 2
a 1 2
00
L = (−1) = (−1) < 0 (so a = √ maximizes L )
1 − a2 2
(1 − a ) 3/2
5
2
x l
2 + 600
Exercise 19. gallons per hour. l0 = 300 mi x = constant speed. 0 = time spent. K = gas cost = 0.30.
¡ 2K Kx D
¢ x
C = gas cost + driver labor cost = l0 x + 600 + x
µ ¶ r
dC −2K K D dC
dx =0 2K + D √
= l0 2
+ − 2 = 0 −−−−→ x = 10 6
dx x 600 x K
µ µ ¶¶ µ ¶
d2 C −2 2(2K + D)
= l 0 (−2K − D) = l0 >0
dx2 x3 x3
r
2K + D √
Thus, C is minimized if x = 10 6
K
Ã√ √ √ √ √ !
2K + D K K 2K + D10 6 √ √
=⇒ Cmin = (300) √ + = 3 2 6 + 10D
10 6 600

Remember that there is a speed limit of 60 mi/hr.


√ √
(1) D = 0, x = 20√3 C = 6 √3 ≈ 10.39
(2) D = 1, x = 40 2 C = 12 2 ≈ 16.97 ³ ´
2K K(60) D
(3) D = 2, x = 60 (because of the speed limit) C = 300 60 + 600 + 60 = 5(2.4 + D) = 22.00

(4) D = 3, x = 60 C = 27.00
(5) D = 4, x = 60 C = 32.00
x
Exercise 20. y= x2 +1 Suppose the rectangle starts at x0 on the x axis. Then its y coordinate intersecting the curve, and thus
x
the height of rectangle, must be y0 = 2 0
x0 +1
s
1 1
=⇒ x0 = ± −1
2y0 (2y0 )2
s
1
x2 − x1 = −4
y02
where x2 − x1 is going to be the base of the rectangle. The volume of the cylinder, V , which is obtained from revolving the
rectangle about the x axis, is going to be
! Ãs q
1
V = πy02 (x2
− x1 ) = −πy02
4 = πy 0 1 − 4y02
y02
Ãq ! Ã !
dV 2 y0 1 − 8y02 1
=π 1 − 4y0 + p (−8y0 ) = π p =⇒ y0 = √
dy0 2 1 − 4y02 1 − 4y02 2 2
We could argue that V is maximized, since the “innite” boundaries would yield a volume of 0 (imagine stretching and
squeezing the rectangle inside the curve).
π
Then Vmax = π 81 2 =
4
Exercise 21. Draw a good diagram. Note how the right triangle that you folded is now reected backwards , so that this
triangle's right angle is on the left-hand side now.
The constraint is that the crease touches the left edge.

w0 = l sin α + l sin α cos (2α) = l sin α(1 + cos (2α)) =


= 2l sin α cos2 α
61
Note that we will obtain a minimum crease because by considering the “physical innite” boundary, we could make a big
crease along the vertical half of the paper or the horizontal half of the paper.
So, isolating l, the length of the crease, and then taking the derivative,

w0 w0
l= = csc (α) sec2 (α)
2 sin (α) cos2 α 2
dl w0 ¡ ¢
= − cot α csc α sec2 α + csc α2 sec α sec α tan α =
dα 2 µ µ ¶µ ¶ µ ¶ µ ¶¶ µ ¶
w0 −C 1 1 1 1 S w0 −1 2
= + 2 = +
2 S S C2 S C2 C 2 S2C C3
dl
dα =0 1 1
−−−−→ sin α = √ or tan α = √
3 2

where α is the angle of the crease. The corresponding minimum length of the crease will be


w0 1 9 3
l= =
2 √1 2 2
33

Exercise 22.

(1) Consider the center of the circle O, the apex of the isosceles triangle that makes an angle 2α, A, and one of its other
vertices, B. Draw a line segment from O to B and simply consider the two triangles making up one half of the
isosceles triangle. Find all the angles.
Angle AOB is π − 2α by the geometry or i.e. inspection of the gure. The complement of that angle is 2α.
Beforehand, we can get the length of the isosceles triangle leg from the law of cosines.

cos (π − 2α) = − cos (2α)


s = R + R − 2R cos (π − 2α) + 2R2 (1 + cos (2α)) = 2R2 (2 cos2 α) = 4R2 cos2 α
2 2 2 2

s = 2R cos α
The constraint equation is

(8) P = 4R cos α + 2R sin (2α)


So then
P 0 = 4R(− sin α) + 4R cos (2α) = 0 =⇒ cos 2α = sin α
q
− 21 ± 14 − 4(1)(− 12 ) 1
sin α = = >0
2(1) 2

=⇒ P = 3 3R

P = 3 3 is a max because
Look at the “boundary conditions” imposed on P by the physical-geometry. α = 0, triangle is completely attened,
α = π , triangle “completely disappears.”
(2) I had originally thought to Reuse the constraint equation, Eqn. ( ??). This is wrong!

Think about the problem directly and for what it actually is; less wishful thinking.
Consider a xed perimeter L and imagine L to be a string that can be stretched into an isoceles triangle. A “trivial”
isoceles triangle is a collapsed triangle with two sides of length L/2 only. Then the radius of the disk needs to be L/4.

Consider a general isosceles triangle with 2α as the vertex angle and isosceles sides of h. The perimeter for this
triangle, P , is then
P = 2h + 2h sin α = 2h(1 + sin α)
P
=⇒ h =
2(1 + sin α)
h
= R cos α
2
62
We could try to extremize this equation.

dR
(4 cos α + 2 sin (2α)) + R(−4 sin α + 4 cos (2α)) = 0
dα √
dR 1 2
= 0 =⇒ cos (2α) = sin α =⇒ sin α = √ cos α = √
dα 3 3
3P
R= √ √
4 2( 3 + 1)
However, this is the minimized R, minimized radius for the smallest circle tting a particular isosceles triangle of a
L
xed perimeter. We want to smallest circle with a radius big enough to t all the possible triangles. Thus R= 4
Exercise 23. The constraint equation on perimeter is
µ ¶
W π
P = 2h + W + π = 2h + W (1 + )
2 2
Then intensity function, “normalized” is given by
µ ¶2 µ ¶
π W 1 Wp W2 3π 2
I = Wh + = − − W
2 2 2 2 2 16
So then
dI P 3π P
= −W − W = 0 =⇒ W =
dW 2 8 2 + 3π
4

The height of the rectangle is


P− P
(1 + π2 ) µ ¶
2+ 3π 4+π
h= 4
=P
2 16 + 6π
Exercise 24. A log 12 feet long has the shape of a frustum of a right circular cone with diameters 4 feet and (4 + h) feet at its
ends, where h ≥ 0. Determine, as a function of h, the volume of the largest right circular cylinder that can be cut from the
log, if its axis coincides with that of the log.
Remember to label your diagram carefully.
³ ´
xh
y −yh
h/2 h/2 − 2l0
2
= = =⇒
x l0 − x l0 l0 − x
xh 2
V = π(H + y)2 (l0 − x) = π(H + ) (l0 − x)
2l0
2
Note that V (x = 0) = πH l0 = π4(12)

dV hx h xh 2
= π(2(H + ) (l0 − x) + (H + ) (−1)) =
dx 2l0 2l0 2l0 dV
dx =0 (h − H)2l0
−−−−→ x =
hx 3xh 3h
= π(H + )(h − H − )
2l0 2l0
µ ¶µ ¶2
(h − H)2l0 (h − H)2l0 h (h − H)2l0 (h + 2H)3
V (x = ) = π l0 − H+ = πl0
3h 3h 2l0 3h 27h
where H = 2, l0 = 12
Exercise 25.
n
X X n
dS
S= (x − ak )2 =⇒
= 2(x − ak ) = 0
dx
k=1 k=1
n
X Pn
ak
=⇒ nx = ak =⇒ x = k=1
n
k=1
Pn
ak
Since limx→±∞ S = +∞, x = k=1
n minimizes S.
Exercise 26. Hint: draw a picture . Then observe that for f (x) ≥ 24, A must be greater than 0 (we'll show that explicitly
soon) and that f must have a minimum somewhere.
5x7 +A −A1/7
If A < 0, then consider f (x) = x5 . Consider x= 61/7
> 0.
−A1/7 A
f (x = = 5 <0
61/7 6x
63
Thus, A > 0.
µ ¶
df 2x7 − A
= 10x − 5Ax−6 = 5 =0
dx x6
µ ¶1/7
A
x=
2
µ ¶
d2 f 2
= 10 + 30Ax−7 = 10 + 3 − A = 70 > 0
dx2 A
¡ A ¢1/7
Thus x= 2 minimizes f for A > 0.
µ ¶1/7 ¡ ¢
A 5 A2 + A
f (x = ) = ¡ ¢5/7 = 24
2 A
2
µ ¶7/2
24
=⇒ A = 2
7

3
Exercise 27. Consider f (x) = − x3 + t2 x over 0 ≤ x ≤ 1.
1 1
f (0) = 0, f (1) = − + t2 =⇒ f (1) ≷ 0 if t2 ≷
3 3
f 0 (x) = −x2 + t2 = 0
=⇒ x2 = t2 but x ≥ 0, so x = |t|
1 2
f (x2 = t2 ) = − t2 (x) + t2 x = t2 x > 0 for 1 ≥ x ≥ 0
3 3
So the minimum isn't in the interior of [0, 1]. It's on the end points.

1 −1 1
m(t) = 0 for |t| > m(t) = + t2 for |t| <
3 3 3

Exercise 28.

(1)
|t − x|
E(x, t) =
x
|t − x|
M (t) = max as x = a → x = b
x
(
t−x
|t − x| if t ≥ x
= x−t x
x x if t < x
µ ¶ ( t
d |t − x| − 2 if t ≥ x
= tx
dt x x2 if t < x

Now t, x ≥ a > 0 (this is an important, given, fact ). So x = t should be a relative minimum.


So the maximum occurs at either endpoints

t−a b−t
= E(a, t), = E(b, t)
a b
|t−x|
By monotonicity on [a, t) , (t, b], and having shown the relative minimum of x at x = t, the maximum occurs
at x = a or x = b, depending upon the relationship E(a, t) ≷ E(b, t).
(2)
( ¡ b+a ¢ (1 ab
t−a
if
t−a
> b−t
t > 2 dM if t > 2³a+b
M (t) = a a b i.e. ab =
a ´
b−t b−t t−a −1 ab
b if
b > a
dt b if t< a+b 2

dM 2ab 2ab
Since
dt ≷ 0 when t ≷ a+b , M is minimized for t=
a+b
64
4.23 Exercises - Partial Derivatives.

x
Exercise 8. f (x, y) = √ .
x2 +y 2

1 −x2 y2 −3y 2 x
fx = p + = fxx =
x2 + y 2 (x2 + y 2 )3/2 (x2 + y 2 )3/2 (x2 + y 2 )5/2
µ 2 ¶
−xy x − 2y 2
fy = 2 fyy = (−x)
(x + y 2 )3/2 (x2 + y 2 )5/2
à !
(x2 + y 2 )3/2 − x 23 (x2 + y 2 )1/2 (2x)
fxy = (−y) =
(x2 + y 2 )3 2y −3y 2 y (−y)(−2x2 + y 2 )
µ ¶ fyx = + =
−2x2 + y 2 (x2 + y 2 )3/2 (x2 + y 2 )5/2 (x2 + y 2 )5/2
= (−y)
(x2 + y 2 )5/2
Exercise 9.

(1)
z = (x − 2y)2
√ √
zx = 2(x − 2y) = 2 z x(2z) − 4zy = (x − 2y)2 z = 2z

zy = 2(x − 2y)(−2) = −4 z
(2)
z = (x4 + y 4 )1/2
1 √
zx = 2x3 x(2z) − 4zy = (x − 2y)2 z = 2z
z
2y 3
zy =
z
Exercise 10.
xy y −4x2 y y 3 − 3x2 y
f= , f x = + =
(x2 + y 2 )2 (x2 + y 2 )2 (x2 + y 2 )3 (x2 + y 2 )3
So
−6xy(x2 + y 2 )3 − 3(x2 + y 2 )2 (2x)(y 3 − 3x2 y)
fxx = =
(x2 + y 2 )6
12xy(x2 − y 2 )
=
(x2 + y 2 )4
By label symmetry,
12xy(x2 − y 2 ) 12yx(y 2 − x2 )
fxx + fyy = + =0
(x2 + y 2 )4 (x2 + y 2 )4
5.5 Exercises - The derivative of an indenite integral. The rst fundamental theorem of calculus, The zero-derivative
theorem, Primitive functions and the second fundamental theorem of calculus, Properties of a function deduced from
properties of its derivatve.
Review the fundamental theorems of calculus, Thm. 5.1 and Thm. 5.3. Note the differences between the two.

Theorem 17 (First fundamental theorem of calculus).


Let f be integrable on [a, x] ∀x ∈ [a, b]
Let c ∈ [a, b] and
Z x
(9) A(x) = f (t)dt if a≤x≤b
c
Then ∃ A0 (x) ∀x ∈ (a, b) where f is continuous at x and
(10) A0 (x) = f (x)
Theorem 18 (Second fundamental theorem of calculus).
Assume f continuous on open interval I
Let P be any primitive of f on I , i.e. P 0 = f ∀x ∈ I
Then ∀c, x ∈ I
Z x
(11) P (x) = P (c) + f (t)dt
c
65
√ q √
Exercise 6. 2 23 x3/2 + 1 2 3/2
2 3x = 2x3/2
Rb √
a
f= 2(b3/2 − a3/2 )
Exercise 7. f = x3/2 − 3x1/2 + 27 x−1/2 ;
P = 25 x5/2 − 2x3/2 + 7x1/2
Rb
a
f = 25 (b5/2 − a5/2 ) − 2(b3/2 − a3/2 ) + 7(b1/2 − a1/2 )
Exercise 8. P = 23 x4/3 − 32 x2/3 ; x>0
6
x
Exercise 9. P = −3 cos x + 3
3 7/3
Exercise 10. P =
7x − 5 sin x
0 1
Exercise 11. f (x) =
x

X
f= ak xk
k=−∞
X∞
1
f0 = kak xk−1 =
x
k=−∞
Comparing terms, only k = 0 would work, but the coefcient is unequivocally 0
Exercise 12.
Z (R x (
x 1 2
tdt if x≥0 2x if x≥0 1
|t|dt = R0x = −1 2
= x|x|
0 0
−tdt if x < 0
2 x if x<0 2
Exercise 13.
Z (R x (
x 4 3
2 0
(2t)2 dt if x≥0 3x if x≥0 2x2
(t + |t|) dt = = = (x + |x|)
0 0 if x < 0 0 if x < 0 3
Exercise 14. Using 1st. fund. thm. of calc.
Z x
f (t)dt = A(x) − A(0)
0 f 0 = 2 + 4 cos 2x + −4x sin 2x − 2 cos 2x
0
A (x) = f (x) = 2 + 2 cos 2x − 4x sin 2x
=⇒ 2x + sin 2x + 2x cos 2x + − sin 2x
³π ´ π
f =

³π 2
f0 =2−π
4
Rx
Exercise 15. f (t)dt = cos x − 12 f (x) = − sin x c = − π6 .
c
Exercise 16. Suppose f (x) = sin x − 1 and c = 0.
Z x µ ¶¯x
1 2 ¯¯ 1
t sin t − t = −t cos t + sin t − t ¯ = sin x − x cos x − x2
0 2 0 2
So c = 0.
Rx R1 16 18
Exercise 17. For f (x) = −x2 f (x) + 2x1 5 + 2x1 7 (found by taking the derivative of 0 f = x t2 f + x8 + x9 + C ,)
15
Suppose that f = 2x .
x16 x18 1 x16 x18
=⇒ =− +− + + +C
8 9 9 8 9
1
=⇒ C =
9
R x 1+sin t
Exercise 18. By plugging in x = 0 into the dened f (x), f (x) = 3 +
0 2+t2
dt, we get for p(x) = a + bx + cx2 ,
a=3
Continuing on,
1 + sin x (cos x)(2 + x2 ) − 2x(1 + sin x)
f0 = ; f 00 =
2 + x2 (2 + x2 )2
1 1 1
f 0 (0) = = b f 00 (0) = + 2c; c =
2 2 4
66
Exercise 19. Z Z
1 x 2 1 x 2
f (x) = (x − t) g(t)dt = (x − 2xt + t2 )g(t)dt =
2 0 2 0
µ Z x Z x Z x ¶
1 2 2
= x g − 2x tg + t g
2 0 0 0
Z x Z x
x2 1
f0 = x g + g(x) − tg − x(xg(x)) + x2 g(x) =
2 2
Z0 x Z x 0

=x g− tg
Z x0 0
Z x
f 00 = g + xg − xg = g f 00 (1) = 2
0 0

f 000 = g f 000 (1) = 5


Exercise 20.
Rx
(1) ( (1 + t2 )−3 dt)0 = (1 + x2 )−3
R0x2 2x
(2) ( 0 (1 + t2 )−3 dt)0 = (1 + x4 )−3 (2x) = (1+x 4 )3
R x2 2x 3x2
(3) ( x3 (1 + t ) dt) = (1 + x ) (2x) − (1 + x6 )−3 (3x2 ) =
2 −3 0 4 −3
(1+x4 )3 − (1+x6 )3
Exercise 21.
ÃZ !0 µ ¶ µ ¶
x2
0 t6 x12 x1 8
f (x) = dt = (2x) − 3x2
x3 1 + t4 1 + x8 1 + x12
Exercise 22.

(1)
f (x) = 2x(1 + x) + x2 = 2x + 3x2
f (2) = 16
³R ´
d b(x)
(2)
dx a(x)
f (t)dt = f (b)b − f (a)a0
0

2x + 3x2 = f (x2 )(2x)


3x
f (x2 ) = (1 + )
√ 2
3 2
f (2) = 1 +
2
R f (x)
(3)
0
t2 dt = x2 (1 + x)
2x + 3x2 = (f (x))2 f 0 (x)
=⇒ f 3 (2) = 3(4)(3) = 9(4) = 36
f (2) = 361/3
(4)
ÃZ !
x2 (1+x)
d
f (t)dt = 1 = f (x2 (1 + x))(2x(1 + x) + x2 )
dx 0
1
x=1 (f (2))(5) = 1 =⇒ f (2) =
5
Exercise 23. Z a
3 2
a − 2a cos a + (2 − a ) sin a = f 2 (t)dt
0
3x2 − 2 cos x + 2x sin x + −2x sin x + (2 − x2 ) cos x = 3x2 − x2 cos x = f 2 (x)

f (x) = x 3 − cos x

f (a) = a 3 − cos a
t2
Exercise 24. f (t) = 2 + 2t sin t
(1)
f 0 = 2t + 2 sin t + 2t cos t f 0 (π) = 2π − 2π = 0
67
(2)
f 00 = 2 + 2 cos t + 2 cos t + −2t sin t = 2 + 4 cos t − 2t sin t
³π ´
f 00 =2−π
¡ ¢ 2
(3) f 00 3π =0
¡ 5π2¢ 25π2
(4) f 2 = 8 + 5π
2
(5) f (π) = π2
Exercise 25.

(1)
df 1 + 2 sin πt cos πt
= = v(t)
dt 1 + t2
2π(cos (2πt))(1 + t2 ) − 2t(sin (2πt))
a(t) =
(1 + t2 )2

a(t = 2) = a(t = 1) = =π
4
(2) v(t = 1) 12
(3) v(t) = π(t − 1) + 12 ; t > 1
(4)
Z t Z t µ ¶¯t
1 πt2 1 ¯ πt2 t π 1
f (t) − f (1) = v(t)dt = π(t − 1) + = − πt + t ¯¯ = + −πt + + −
1 1 2 2 2 1 2 2 2 2
Exercise 26.

(1)
f 00 (x) > 0 ∀x f 0 (0) = 1; f 0 (1) = 0
Z 1
f 00 (t)dt = f 0 (1) − f 0 (0) = 0 − 1 < 0
0
R1
Thus, it's impossible, since f 00 (x) > 0, so 0
f 00 (t)dt > 0
(2)
Z ³
1
π πx ´ ³ πx ´¯¯1
3−
sin dx = 3x + cos ¯ =3−1=2
0 2 2 2 0
3x2 2 πx
f (x) = + sin +C
2 π 2
(3) f 00 (0) > 0 ∀x f 0 (0) = 1; f (x) ≤ 100 ∀x > 0
Z b Z k
f 00 (t)dt = f 0 (b) − f 0 (a); f 0 (t)dt = f (k) − f (c)
a c
Z b
f 00 = f 0 (b) − f 0 (0) = f 0 (b) − 1 ≷ 0 if b≷0
0
Z k
(f 0 (b) − 1)db = f (k) − f (c) − (k − c) > 0 if k>c>0
c
f (k) − f (c) > k − c f (x) ≤ 100 is untrue for all x > 0
f (k) − f (0) > k − 0
f (100) − f (0) > 100
00 x
(4) f (x) = e > 0 f (x) = ex ;
0
f 0 (0) = 1 f (x) = ex ∀x < 0, ex < 1
Exercise 27. f 00 (t) ≥ 6. b − a = 12 . f 0 (0) = 0
Z b
1
f 00 = f 0 (b) − f 0 (a) ≥ 6(b − a) = 3 since b − a =
a 2
Z a
f 00 = f 0 (a) − f 0 (0) = f 0 (a) ≥ 6(a − 0) = 6a
0
1 0
If a= , f (1/2) ≥ 3
2
68
Then by intermediate value theorem, with f f 0 (0) = 0, f 0 (1/2) ≥ 3, f 0
being continuous and must take on the value of 3
0
somewhere between 0 and 3. Thus there is an interval [a, b] of length 1/2 where f ≥ 3.

5.8 Exercises - The Leibniz notation for primitives, Integration by substitution.


R√
Exercise 1. 2x + 1dx = 13 (2x + 1)3/2 .
R √
Exercise 2. x 1 + 3x = 2x
9 (1 + 3x)
3/2 4
+ − 135 (1 + 3x)5/2
Exercise 3.
Z
√ 2x2 (x + 1)3/2 8x(x + 1)5/2 16(x + 1)7/2
x2 x + 1 = − +
3 15 105
since
µ ¶0
2x2 (x + 1)3/2 4x(x + 1)3/2
= x2 (x + 1)1/2 +
3 3
µ ¶0
8x(x + 1)5/2 4 8(x + 1)5/2
= x(x + 1)3/2 +
15 3 15
µ ¶
7/2 0 5/2
16(x + 1) 8(x + 1)
=
105 15
Exercise 4.
Z
xdx 2x(2 − 3x)1/2 4(2 − 3x)3/2
√ = +
2 − 3x −3 −27
Z 1/3
xdx
√ = −2/9 − 4/27 − (8/9 − 32/27) = −2/27
−2/3 2 − 3x
Exercise 5.
Z
(x + 1)dx ((x + 1)2 + 1)−2
2 3
=
((x + 1) + 1) −4
Exercise 6. Z Z
1
sin3 x = sin x(1 − cos2 x) = − cos x + cos3 x
3
Exercise 7. Z
3 4/3 3 7/3 3 3
x1/3 (1 + x) = x + x = (z − 1)4/3 + (z − 1)7/3
4 7 4 7
sin−2 x
Exercise 8.
−2
¯π/4
(4−sin 2x)3/2 ¯ 33/2 −8
Exercise 9.
−3 ¯ = −3
0
−1
Exercise 10. (3 + cos x)
Exercise 11. 2 cos−1/2 x
√ ¯8
Exercise 12. 2 cos x + 1¯3 = 2(cos 3 − cos 2)
n
Exercise 13. − cosnx
(1−x6 )1/2
Exercise 14.
−3
Exercise 15. Z Z
1/4 4 9/4 4 5/4 4 4
t(1 + t) dt = (x − 1)x1/4 dx = x − x = (1 + t)9/4 − (1 + t)5/4
9 5 9 5
R
Exercise 16. (x2 + 1)−3/2 dx =?
µ ¶0 √ √
x x2 + 1 − x2 / x2 + 1 1
√ = = 2
x2 + 1 x2 + 1 (x + 1)3/2
Exercise 17.
µ ¶µ ¶
3 1 1
(8x3 + 27)5/3 = (8x3 + 27)5/3
5 24 40
3
Exercise 18.
2 (sin x − cos x)2/3
69
Exercise 19.
Z Z 1
xdx 2 du
p = √
=
1 + x + (1 + x2 )3/2
2 u + u3/2
Z
1 du
= √ √ = 2(1 + u1/2 )1/2 =
2 u 1 + u1/2
p
= 2(1 + 1 + x2 )1/2 + C
Exercise 20.
Z Z Z
(x2 − 2x + 1)1/5 dx −(x − 1)2/5
= dx = − (x − 1)−3/5 dx = −5/2(x − 1)2/5
1−x x−1
Rb R b+c
Exercise 21. Thm. 1.18. invariance under translation.
a
f (x)dx = a+c f (x − c)dx.

Thm. 1.19. expansion or contraction of the interval of integration.


Z b Z kb ³x´
1
f (x)dx = f dx
a k ka k

yx + c Z b Z b+c
f (x)dx = f (y − c)dy
dy = dx a a+c

Z Z ³y´
y = kx b
1 kb
f (x)dx = f dy
dy = kdx a k ka k
Exercise 22.

Z t
³x ´ x/a
up u=
F ,1 = du a
a 0 (u2 + 12 )q dt
du =
a
³x ´ 1Z x (t/a)p dt
F ,1 = ³¡ ¢ ´q =
a a 0 t 2
+ 12 a
Z x
tp
= a−p−1+2q dt = a−p−1+2q F (x, a)
0 (t2 + 1 2 )q
Exercise 23.
1
Z 1
u=
dt t
= F (1) − F (x) −1 −1
1 + t2 du = 2 dt, 2 du = dt
Z xx t u
dt Z Z 1/x
= F (x) − F (1) x
dt −du
1 1 + t2 = ¡ ¢=
Z 1/x µ ¶ 1 1 + t2 1 u 1 + u12
2
dt 1 Z 1/x Z 1
=F − F (1) du dt
1 1 + t2 x =− =
u 2+1 t 2+1
1 1/x

Exercise 24.
Z Z Z
1 0 1 u=1−x
xm (1 − x)n dx = − (1 − u)m (un )du = (1 − x)m xn dx using
0 1 0 x=1−u
Exercise 25.
¶m µ
sin 2x m
cos x sin x = m
= 2−m sinm 2x
2
Z π Z π/2 Z π Z π
2 1
cosm x sinm xdx = 2−m sinm 2xdx = 2−m sinm xdx = 2−m−1 sinm xdx =
0 0 0 2 0
Z −π/2 ³π ´ Z π/2 Z π/2
= −2−m−1 sinm − x dx = 2−m−1 cosm xdx = 2−m cosm xdx
π/2 2 −π/2 0

Exercise 26.

70
(1)
Z π Z 0 Z π

u=π−x xf (sin x)dx = (π − u)f (sin (π − u))(−du) = (π − u)f (sin u)du =


0 π 0
Z π Z π
x=π−u
=π f (sin x)dx − xf (sin x)dx
0 0
Z π Z
π π
=⇒ xf (sin x)dx = f (sin x)dx
0 2 0
(2)
Z π Z π Z Z
x sin x x sin x π π sin x π π sin x
dx = 2 = 2 dx = dx =
u = cos x 0 1 + cos2 x 0 2 − sin x 2 0 2 − sin x 2 0 1 + cos2 x
Z Z Z 1
du = − sin xdx π −1 du π 1 du dx
=− = =π
2 1 1 + u2 2 −1 1 + u2 −1 1 + x2
Exercise 27.
x = sin u Z 1 Z π/2 Z π/2
1 1
(1 − x2 )n− 2 dx = (cos2 u)n− 2 cos udu = cos2n udu
dx = cos u 0 0 0

5.10 Exercises - Integration by Parts.


R
Exercise 1. x sin x = −x cos x + sin x
R 2
Exercise 2. x sin x = −x2 cos x + 2x sin x + 2 cos x
R 3
Exercise 3. x cos x = x3 sin x + 3x2 cos x − 6x sin x + −6 cos x
R 3
Exercise 4. x sin x = −x3 cos x + 3x2 sin x + 6x cos x − 6 sin x
R
Exercise 5. sin x cos x = − 14 cos 2x = − 14 (cos2 x − sin2 x)
R R
Exercise 6. x sin x cos xdx = x2 sin 2x = − x cos 4
2x
+ sin82x
R R R
Exercise 7.
R sin2 x = sin x sin x = − sin x cos x + cos2 x
2
sin xdx = −1 4 sin 2x + 2
x

Exercise 8.
Z Z
u = sinn−1 x
sinn xdx = − cos x sinn−1 x + (n − 1) sinn−2 x cos2 x
dv = sin xdx
Z Z
sinn x = − cos x sinn−1 x + (n − 1) sinn−2 x(1 − sin2 x) =
Z Z
n−1 n−2
= − cos x sin x + (n − 1) sin x − (n − 1) sinn x
Z Z
n −1 n−1 (n − 1)
sin x = sin x cos x + sinn−2 x
n n
Exercise 9.

(1) Z Z
−1 1 −1 1
sin2 x =
sin x cos x + 1= sin x cos x + x
2 2 2 2
Z π/2
π
sin2 xdx =
0 4
R π/2 4 ¯ π/2 R π/2
sin x = −1 3 ¯ + 34 0 sin2 x = 3π
(2)
0 4 sin x cos x 0 16
R π/2 6 R π/2 5π
(3)
0
sin x = 65 0 sin4 x =
32
Exercise 10.

(1)
Z Z
−1 2 1 2 −3 1
sin3 xdx = sin2 x cos x + sin x = − sin 2x cos x − cos x = cos x + cos 3x since
3 3 6 3 4 12
−3 1 −3 1
cos x + cos 3x = cos x + (cos x cos 2x − sin 2x sin x) =
4 12 4 12
3 1 −2 1
= − cos x + (cos x(1 − 2 sin2 x) + −2 sin2 x cos x) = cos x − sin2 x cos x
4 12 3 3
71
(2)

Z Z
−1 3 −1 3 x sin 2x −1 3x 3 sin 2x
sin4 xdx = sin3 x cos x + sin2 x = sin3 x cos x + ( − )= sin3 x cos x + −
4 4 4 4 2 4 4 8 16
1 1 1 2 sin 2x 1 3
Now sin 4x = (2 sin 2x cos 2x) = (sin x cos x(1 − 2 sin x) = − sin x cos x
32 32 8 16 4
−1 3 3x 3 sin 2x 3x 1 1
=⇒ sin x cos x + − = − sin 2x + sin 4x
4 8 16 8 4 32
(3)

Z Z Z
sin5 xdx = sin4 x sin xdx = − cos x sin4 x + cos2 x4 sin3 x =
Z Z Z Z
= − cos x sin x + 4( sin x − sin x) = − cos x sin x + 4 sin x − 4 sin5 x
4 3 5 4 3

Z Z
5 sin5 dx = − cos x sin4 x + 4 sin3 x
Z
−3 1
5 sin5 dx = − cos x(1 − cos2 x)2 + 4( cos x + cos 3x)
4 12
1
= − cos x(1 − 2 cos2 x + cos4 x) + −3 cos x + cos 3x
3
3 5 1
= − cos x + 2 cos x − cos x − 3 cos x + (cos x cos 2x − sin x sin 2x) =
3
1 10 cos3 x
= −4 cos x + 2 cos3 x − cos5 x + (4 cos3 x − 3 cos x) = −5 cos x + − cos5 x
Z 3 3
2 cos 3x 1
sin5 xdx = − cos x + − cos5 x
3 5

My solution to the last part of this exercise conicts with what's stated in the book.

Exercise 11.

(1)
Z Z Z µ ¶
x2 x sin 2x x2 cos 2x
x sin2 xdx = ( sin2 x)x − (sin2 t) = − − + =
2 4 4 8
x2 x sin 2x cos 2x
= − −
8 4 Z8
x sin 2x
we had used sin2 x = −
2 4
(2)
Z Z
−3x x 3 1
x sin3 x = cos x + cos 3x − − cos x + cos 3x =
4 12 4 12
−3x x 3 − sin 3x
= cos x + cos 3x + sin x +
4Z 12 4 36
−3 1
sin3 x = cos x + cos 3x
4 12
(3)

Z ¶ Zµ µ ¶ Z
2 2 x sin 2x 2 x sin 2x x3 x2 sin 2x 1 3 x sin 2x
x sin xdx = x − − 2x − = − − x + =
2 4 2 4 2 4 3 2
µ ¶
x3 x2 sin 2x 1 −x cos 2x sin 2x
= − + + =
6 4 2 2 4
x3 x2 sin 2x x cos 2x sin 2x
= − − +
6 4 4 8
72
Exercise 12.

Z Z Z
cosn xdx = cosn−1 x cos xdx = cosn−1 x sin x + (n − 1) cosn−2 x sin2 x =
Z Z
n−1 n−2
= cos x sin x + (n − 1) cos x − (n − 1) cosn x
Z µ ¶Z
n cosn−1 x sin x n−1
=⇒ cos x = + cosn−2 x
n n

Exercise 13.

R sin 2x
(1) cos2 x = 5 + 12 x
R cos2 x sin x
(2) cos3 x = 3 + 23 sin x = 3
4 sin x + 1
12 sin 3x since

1 1 1 1
sin 3x = (sin 2x cos x + sin x cos 2x) = sin x cos2 x + sin x(2 cos2 x − 1) =
12 12 6 12
1 1
= sin x cos2 x − sin x
3 12

(3)

Z µ ¶
cos3 x sin x 3 1 1 3 3 cos3 x sin x
cos4 xdx = + x + sin 2x = x + sin 2x +
4 4 2 4 8 16 4
sin 4x = 2 sin 2x cos 2x = 4 sin x cos x(2 cos2 x − 1) = 8 sin x cos3 x − 2 sin 2x then
Z
3 1 1
cos4 xdx = x + sin 2x + sin 4x
8 4 32

Exercise 14.

Z p p Z
x2
1− x2 dx
=x 1−x +2 √ dx
1 − x2
Z Z Z p
x2 x2 − +1 1
√ dx = √ =− 1 − x2 + √
1−x 2 2
1−x +1−1 1 − x2
2 2
x =x −1+1
Z p Z
2
1 p 2
1 1
=⇒ 1 − x dx = x 1 − x + √
2 2 1 − x2

Exercise 15.

(1)

Z Z Z
2 2 n 2 2 n 2 2 n−1
(a − x ) dx = x(a − x ) − n(a − x ) (−2x)xdx = x(a − x ) + 2n x2 (a2 − x2 )n−1 dx
2 2 n

Z Z Z
x2 (a2 − x2 )n−1 dx = ((x2 − a2 ) + a2 )(a2 − x2 )n−1 dx = −(a2 − x2 )n + a2 (a2 − x2 )n−1 dx
Z Z
x(a2 − x2 )n 2a2 n
=⇒ (a2 − x2 )n dx = + (a2 − x2 )n−1 dx
2n + 1 2n + 1
73
(2)

Z
x(a2 − x2 ) 2a2 −x3
(a2 − x2 )dx =+ x= + a2 x
3 3 3
Z Z
x(a2 − x2 )5/2 a2 5
(a2 − x2 )5/2 dx = + (a2 − x2 )3/2 dx
6 6
Z Z
x(a2 − x2 )3/2 3a2
(a2 − x2 )3/2 dx = + (a2 − x2 )1/2 dx
4 4
Z Z r ³ x ´2 Z
(a2 − x2 )1/2 =a 1− dx = a 2
cos2 θdθ = x
a sin θ =
à r ! a
Z µ ¶ ³ x ´2 dx
2 1 + cos 2θ 2 θ sin 2θ 2 x 1x cos θdθ =
=a =a + = a arcsin + 1− a
2 2 4 a 2a a
Z ap ³π ´ πa2
a2 − x2 = a2 −0 =
0 2 2
Z a 2
µ 2
¶ 4
3a πa 3πa
(a2 − x2 )3/2 dx = =
0 4 2 8
Z a µ ¶
5a2 3πa4 5
(a2 − x2 )5/2 dx = = πa6
0 6 8 16

Rx
Exercise 16. In (x) = 0
tn (t2 + a2 )−1/2 dt
(1)

Z Z
2 2 1/2 n−1 n−2 2 2 1/2 n−1 2 2 1/2 tn−2 (t2 + a2 )
In (x) = (t + a ) t − (n − 1)t (t + a ) =t (t + a ) − (n − 1)
(t2 + a2 )1/2
Z p
n−1 2 2 1/2 2 tn−2 n−1
(n)In = x (x + a ) − a (n − 1) = x x2 + a2 − (n − 1)a2 In−2
(t2 + a2 )1/2

(2) n = 5; x = 2; a = 5.
Z 2 ¯2 √
¯
I1 (2) = x(x2 + 5)−1/2 dx = (x2 + 5)1/2 ¯ = 3 − 5
0 0
Z 2
5I5 (2) = t5 (t2 + 5)−1/2 dt = 25−1 (4 + 5)1/2 − 5(5 − 1)I3 (2) = 48 − 20I3 (2)
0
√ √
3I3 (2) = 22 4 + 5 − 5(3 − 1)I1 (2) = 12 − 10(3 − 5)
√ √
1 10 5 168 40 5
I5 (2) = (48 − 20(−6 + )) = −
5 3 5 3

Exercise 17.

Z Z
3 t2(c + t3 )1/2
3 −1/2 2(c + t3 )1/2
t (c + t ) dt = −
3 3
Z 3 ¯ 3 Z √
3 3 −1/2 t2(4 + t3 )1/2 ¯¯ 2 3 3 1/2
√ 2 3 2
t (4 + t ) dt = ¯ −3 (4 + t ) = 2 31 + − (11.35)
−1 3 −1 −1 3 3

Exercise 18.
Z Z µ ¶ Z
sinn+1 x n sin x sinn x n sinn−1 x
dx = sin x dx = −
cosm+1 x cosn+1 x m cosm x m cosm−1 x
Z n+1 n Z n−1
sin x sin x n sin x
=⇒ m+1
dx = m
− m−1
cos x m cos x m cos x
74
Exercise 19.
Z ¶ Z Z µ
cosm+1 xcos xdx m 1 m cosm−1 x
m
cos x dx = = cos x − =
sinn+1 xsinn+1 x −n sinn x −n sinm x
Z
cosm x −m cosm−1 x
= +
−n sinn x n sinn−1 x
Z Z 1+1 Z
2 cos x −1 cos1 x 1
cot x = = − dx = − cot x − x
sin1+1 x 1 sin1 x 1
Z Z Z
cos3+1 x 1 cos3 x cos3−1 x −1 −1
cot4 xdx = 3+1 = − 3 − 3−1 = cot3 x − (− cot x − x) = cot3 x + cot x + x
sin x 3 sin x sin x 3 3

Exercise 20.

(1)
Z 2 Z 1
tf 00 (t)dt = 2 tf 00 (2t)dt n=2
0 0
(2)
Z 1 Z 2 µ Z 2 ¶
1 1 2 1
xf 00 (2x)dx = tf 00 (t)dt = tf 0 (t)|0 − f 0 (t)dt = (2f 0 (2) − (f (2) − f (0))) = 4
0 2 0 2 0 2

Exercise 21.

(1) Recall Theorem 5.5, the second mean-value theorem for integrals:

Z b Z c Z b
f (x)g(x)dx = f (a) g(x)dx + f (b) g(x)dx
a a c

Z b µ ¶ Z c Z b
φ0 (t) 1 0 1
sin φ(t) dt = φ (t) sin φ(t) + φ0 (t) sin φ(t) =
a φ0 (t) φ0 (a) a φ0 (b) a 1 1
where ≥ 0 ∀t ∈ [a, b]
1 c 1 4 m φ (t)
= 0 cos φ(t)|a + 0 (cos φ(b) − cos φ(a)) ≤
φ (a) φ (b) m
(2) φ(t) = t2 ; φ0 (t) = 2t > 2a if t > a
¯Z x ¯
¯ ¯ 4
¯ sin t2 dt¯¯ ≤ = 2a
¯ 2a
a

5.11 Miscellaneous review exercises.

Exercise 1. g(x) = xn f (x); f (0) = 1

g 0 (x) = nxn−1 f (x) + xn f 0 (x) ; g 0 (0) = 0 especially if n ∈ Z+ (just note that if negative integer values are included,
g 0 (0) easily blows up)

Ph j
¡ ¢
n!
g j (x) = k=0 k (n−k)! x
n−k j−k
f (x)

If j < n , then g j (0), since each term contains some power of x

If j ≥ n,
n µ ¶
X j n!
g j (x) = xn−k f (j−k) (x)
k (n − k)!
k=0
j!
g j (0) = f (j−n) (0)
(j − n)!
If j = n, g n (0) = n!
75
Exercise 2.

5
X
P (x) = aj xj
j=0
5
P (0) = 1 = a0
X
0
P (x) = jaj x j−1
P 0 (0) = 0 = a1 a1 = a2 = 0
j=1
P 00 (0) = 0 = 2(1)a2
5
X
P 00 (x) = j(j − 1)aj xj−2
j=2

P (x) = a5 x5 + a4 x4 + a3 x3 + 1 P (1) = a5 + a4 + a3 + 1 = 2
0
=⇒ P (x) = 5a5 x + 4a4 x + 3a3 x4 3 2
P 0 (1) = 5a5 + 4a4 + 3a3 = 0
P 00 (x) = 20a5 x3 + 12a4 x2 + 6a3 x P 00 (1) = 20a5 + 12a4 + 6a3 = 0
Solve for the undetermined coefcients by Gauss-Jordan elimination process
      ¯   ¯ 
5 4 3 a5 0 5 4 3¯¯ 0 1 0¯¯ −15
20 12 6 a4  = 0 20 12 6¯¯ 0 =  0 1¯¯ 10 
1 1 1 a3 1 1 1 1¯ 1 1 0 0¯ 6
=⇒ a5 = 6 a4 = −15 a3 = 10 P (x) = 6x5 − 15x4 + 10x3 + 1

nπ nπ
Exercise 3. If f (x) = cos x and g(x) = sin x, Prove that f (n) = cos (x + 2 ) and g (n) (x) = sin (x + 2 )

(
(n) nπ sin x(−1)j+1 if n = 2j + 1
f (x) = cos (x + )=
2 cos x(−1)j if n = 2j
(
(n) nπ cos x(−1)j if n = 2j + 1
g (x) = sin (x + )=
2 sin x(−1)j if n = 2j
f (x) = cos x
f (2j) (x) = cos x(−1)j
f 0 (x) = − sin x
f (2(j+1)) (x) = (cos x(−1)j )0 = cos x(−1)j+1
f 00 (x) = − cos x
f (2j+1) (x) = sin x(−1)j+1
f 000 (x) = sin x
f (2j+3) (x) = (sin x(−1)j+1 ) = sin x(−1)j+2
f 0000 (x) = cos x

g(x) = sin x
g (2j) (x) = sin x(−1)j
g 0 (x) = cos x
g (2(j+1)) (x) = (sin x(−1)j )00 = sin x(−1)j+1
g 00 (x) = − sin x
g (2j+1) (x) = cos x(−1)j
g 000 (x) = − cos x
g (2j+3) (x) = (cos x(−1)j+1 )
g 0000 (x) = sin x
76
Exercise 4.

n µ ¶
h0 (x) = f 0 g + f g 0 X n (k) (n−k)
h(n) = f g
h00 (x) = f 00 g + 2f 0 g 0 + f g 00 k=0
k
Xn µ ¶³ ´
n
h(n+1) = f (k+1) g (n−k) + f (k) g (n−k+1) =
k
k=0
X µn¶ ³
n−1 ´
(1) (n) (n+1)
= f g + fg + f (k+1) g (n−k) + f (k) g (n−k+1) + f (n+1) g + f (n) g (1)
k
k=1
n
à n−1
!
X n! X n!
(1) (n) (n+1) (k) (n−k+1) (k) (n−k+1)
= f g + fg + f g + f g +
(n − k + 1)!(k − 1)! (n − k)!k!
k=2 k=1
(n+1) (n) (1)
+f g+f g
µ ¶ µ ¶
n! n! n! n+1
Now + = (k + (n − k + 1)) = so then
(n − k + 1)!(k − 1)! (n − k)!k! (n + 1 − k)!(k)! k
Xn µ ¶ X µn + 1 ¶
n+1
n + 1 (k) (n+1−k)
h(n+1) = f g (n+1) + f g + f (n+1) g = f (k) g (n+1−k)
k k
k=1 k=0

By induction, this formula is true.

Exercise 5.

(1)

f 2 + g 2 = f (−g 0 ) + gf 0
Y = f 2 + g2 Y 0 = 2f f 0 + 2gg 0 = 2(gf + 2g(−f )) = 0 =⇒ Y = C
Y = C = f 2 + g2 f (0) = 0; g(0) = 1 =⇒ C = 1

(2)

h = (F − f )2 + (G − g)2
= f 0 (x) = g(x), g 0 (x) = −f (x); f (0) = 0; g(0) = 1
0 0 0 0 0 0
h = 2(F − f )(F − f ) + 2(G − g)(G − g ); h (0) = 2(0) + 2(0) = 0
= 2(F − f )(G − g) + 2(G − g)(−F + f ) = 0 ∀x
h(x) = C =⇒ h(x) = (F (x) − f (x)) + (G(x) − g(x))2 2

h(0) = 0 so C=0
=⇒ F = f ; G = g

df 3x
Exercise 6. = 3x2
du 2x f 0 (4) = 2 = 3 where we had used the substitution
2
u=x u = 4; x = 2

dg 64
Exercise 7.
du = u3/2 ; g(u) = 25 u5/2 g(4) = 25 25 =
5
Exercise 8.

Z x Z c Z x
sin t 1 1
dt = sin tdt + sin tdt =
0 t+1 0+1 0 x+1 c
1c x −1
= − cos t|0 + − cos t|c = − cos c + 1 + (cos x − cos c) =
x+1 x+1
−x cos c + x − cos c + 1 − cos x + cos c x(1 − cos c) + (1 − cos c)
= = >0
x+1 x+1
77
Exercise 9. y = x2 is the curve for C . y = 12 x2 is the curve for C1 .
Z b
1 1
(x2 − x2 ) = b3 P : (b, b2 )
0 2 6
2
Assume C2 is of the form kx

Z c Z b kc2 = b2
2 2 2 (k − 1) 3
2 1
(kx − x ) + (b − x ) = c + b2 (b − c) + − (b3 − c3 ) b2
0 c 3 3 k=
c2
(b2 − c2 )c b3 c3 2b3 2 2 2
=⇒ + b3 − cb2 − + = − cb = b2 (b − c)
3 3 3 3 3 3
Now A(A) = A(B)

2 1 4
=⇒ b2 (b − c) = b3 =⇒ b = c
3 6 3
2 16 2
so then kx = x
9

Exercise 10.
(
¯ ¯ h2
x is rational
¯ ¯ if
(1) |Q(h) − 0| = ¯ f (h)
h ¯=
|h|
.
0 if x is irrational
For now, consider 0 < h < δ ; let δ(²; h = 0) = ²

¯ ¯ (
¯ f (h) ¯
¯
|Q(h) − 0| = ¯ ¯= h if x is rational

h ¯ 0 if x is irrational

(2)
¯ ¯
¯ f (h) − f (0) ¯
¯ − 0¯¯ = ² =⇒ f 0 (0) = 0
¯ h
R −2
Exercise 11. (2 + 3x) sin bxdx = 5 cos 5x + − 3x
5 cos 5x +
3
25 sin 5x
4 2 3/2
Exercise 12.
3 (1 +x )
¯1
(x 2
−1)10 ¯ −310
Exercise 13.
20 ¯ = 20
−2
Exercise 14.

Z 1 µ ¶¯1 µ µ ¶µ ¶ ¶
1 6x + 7 + 2 1 −(6x + 7)−1 −(6x + 7)−2 ¯¯ 1 1 1 1 −1 1
dx = + ¯ = 3 − 13 + + + =
3 0 (6x + 7)3 3 6 6 0 6 42 (6)132 (6)49
37
=
8281
R (1+x5 )6
Exercise 15. x4 (1 + x5 )5 dx = 30
Exercise 16.
Z 1 u=1−x
x4 (1 − x)20 dx =
0 x=1−u
Z 0 Z 1 Z 1
=− (1 − u)4 u20 du = (1 − u)4 u20 du = (1 + 4(−u) + 6u2 + −4u3 + u4 )u20 du =
1 0 0
121 −412 2 6123 −4124 125
= + + + + =
21 22 23 24 25
1
=
265650
Make sure to check your arithmetic.

R2 ¡ ¢¯2
Exercise 17.
1
x−2 sin x1 dx = cos x1 ¯1 = cos 12 − cos 1
78
R
Exercise 18. sin (x − 1)1/4 dx

u = (x − 1)1/4 Z Z Z
1/4 3
1 1 1 =⇒ sin (x − 1) dx = (sin u)4u du = 4 u3 sin udu
du = (x − 1)−3/4 dx = dx
Z 4 4 u3
u3 sin udu = −u3 cos u + 3u2 sin u + 6u cos u + 06 sin u

= −(x − 1)3/4 cos (x − 1)1/4 + 3(x − 1)1/2 sin (x − 1)1/4 + 6(x − 1)1/4 cos (x − 1)1/4 − 6 sin (x − 1)1/4
sin (x − 1)1/4 dx =

= −4(x − 1)3/4 cos (x − 1)1/4 + 24(x − 1)1/4 cos (x − 1)1/4 + 12(x − 1)1/4 sin (x − 1)1/4 − 24 sin (x − 1)1/4

R
Exercise 19. x sin x2 cos x2 dx = (1/4) sin2 x2 + C
R√ R√ R 2u3/2 −2
Exercise 20. 1 + 3 cos2 x sin 2xdx = 1 + 3 cos2 x2 sin x cos xdx = du
u1/2 −3 = −9 = (1 + 3 cos2 x)3/2 ,
9
where we had used this substitution:

du = −6 cos x sin xdx


u = 1 + 3 cos2 x du
= 2 cos x sin xdx
−3

R2
Exercise 21.
0
375x5 (x2 + 1)−4 dx
Z 2 Z 5 Z µ ¶
375 375 (u2 − 2u + 1) 375 5 1 2 1
u = x2 + 1 du(u − 1)2 u−4 = du = du − + =
0 2 2 1 u4 2 1 u2 u3 u4
du = 2xdx =⇒ µ ¶¯5
375 −1 1 1 ¯
(u − 1)2 = x4 = + 2+ ¯ = 64 = 26
2 u u −3u3 ¯1

R1 3
Exercise 22.
0
(ax + b)(x2 + 3x + 2)−2 dx = 2

³ ´¯1
−1 ¯ −1 1 2
Since
x2 +3x+2 ¯ = 6 + 2 = 3,
0

then ifa = 9/2, b = 272 , we'll obtain 3/2


R1
Exercise 23. In =
0
(1 − x2 )n dx

Z 1 Z 1
¯1
In = (1 − x2 )n dx = x(1 − x2 )n ¯0 − xn(1 − x2 )n−1 (−2x)dx =
0 0 µ ¶
Z Z 2n
1 1 =⇒ In = In−1
=2 x2 n(1 − x2 )n−1 dx = 2n ((x2 − 1) + 1)(1 − x2 )n−1 dx = 2n + 1
0 0
= 2nIn−1 − 2nIn
Z 1 Z 1 µ ¶¯1
2x3 1 ¯ 8
I2 = (1 − x2 )2 dx = dx(1 − 2x2 + x4 ) = x− + x5 ¯¯ =
0 0 3 5 0 15
6 8 16
I3 =
7 15 35
8 128
I4 = I3 =
9 315
10 256
I5 = I4 =
11 693
79
Rx
Exercise 24. F (m, n) = 0
tm (1 + t)n dt; m > 0, n>0
¯x Z x m+1
tm+1 ¯
n¯ t xm+1 (1 + x)n n
F (m, n) = (1 + t) ¯ − n(1 + t)n−1 dt = − F (m + 1, n − 1)
m+1 0 0 m + 1 m + 1 m +1
(m + 1)F (m, n) + nF (m + 1, n − 1) = xm+1 (1 + x)n
Z x Z x µ 12 ¶¯x
t t13 ¯¯ x12 x13
F (11, 1) = t11 (1 + t)1 dt = t1 1 + t12 = + ¯ = +
0 0 12 13 0 12 13
µ 12 13

x x
11F (10, 2) + 2 + = x11 (1 + x)2
12 13
x13 x12 x11
F (10, 2) = + +
13 6 11
R π/4
Exercise 25. f (n) = 0
tann xdx
(1)
Z b Z b Z c
Use this extremely important fact: f g = f (b) g + f (a) g
a c a
Z π/4 Z π/4 Z π/4
n n
f (n + 1) = tan x tan x = tan x < tann x = f (n)
0 c 0

(2)
Z π/4 Z π/4 Z π/4
f (n + 2) + f (n) = tann x tan2 x + tann x = tann x(sec2 x) =
0 0 0
¯π/4
tann+1 x ¯¯ 1
= ¯ =
n+1 0 n+1
(3)
1
f (n + 2) + f (n) = < f (n + 1) + f (n) < 2f (n)
n+1
1
= f (n − 2) + f (n) > f (n − 1) + f (n) > 2f (n)
n−1
1 1
=⇒ < 2f (n) <
n+1 n−1

Exercise 26. f (0), f (π) = 2 0
(f (x) + f 00 (x)) sin xdx = 5
Z π Z Z Z
f 00 sin x = f 0 sin x − f 0 cos x = − f 0 cos x = −f cos x − f sin x
0
Z π Z Z
(f + f 00 ) sin xdx = f sin x + −(f (π)(−1) − f (0)) − f sin x = 2 + f (0) = 5 =⇒ f (0) = 3
0

Exercise 27.
Z π/2 Z π/2 Z µ Z ¶
sin x cos x sin 2x 1 π sin x 1 − cos x cos x
dx = dx = dx = − =
0 x+1 0 2x + 2 2 0 x+2 2 x+2 (x + 2)2
µ ¶
1 1 1 4+π A
= + −A = −
2 π+2 2 4(π + 2) 2
Exercise 28.
Z √ Z √
dx 2 a + bx 2 a + bx
√ = +
x a + bx bx b x2
Z √ 2 Z 2 ³a ´ Z µ ¶
a + bx (a + bx)3/2 3b (a + bx)
3/2
2√ 2 √ a b
= 3b + = a + bx + b + a + bx +
x x x2 3b x 3b x2 x
Z √ Z
a + bx dx √
=⇒ =a √ + 2 a + bx
x x a + bx
80
Exercise 29.
Z p Z Z
2xn (ax + b)3/2 nxn−1 2(ax + b)3/2 2xn (ax + b)3/2 2n √
xn (ax + b)dx = − = − xn−1 (ax + b) ax + b =
3a 3a 3a 3a
√ Z Z
n
2x (ax + b) ax + b 2n √ 2nb √
= − xn ax + b − xn−1 ax + b
3a 3 3a
Z µ Z ¶
√ 2 √ −3
xn ax + bdx = xn (ax + b)3/2 − nb xn−1 ax + b + C n 6=
(2n + 3)a 2

Exercise 30.
Z Z
xm 2xm (a + bx)1/2 mxm−1 2(a + bx)1/2
√ dx = −
a + bx b b
Z m−1 Z Z
2 2m x (a + bx) 2 xm 2ma xm−1
= xm (a + bx)1/2 − √ = xm (a + bx)1/2 − 2m √ − √
b b a + bx b a + bx b a + bx
Z Z
xm 1 2 m 2ma xm−1
√ dx = x (a + bx)1/2 − √
a + bx 2m + 1 b b(2m + 1) a + bx
Exercise 31.
Z √ Z √ r √ Z
dx ax + b n2 ax + b ax + b 2 ax + b 2n ax + b
√ = 2 n
+ n+1
= n
+ √ =
n
x ax + b ax ax ax + b ax a x n+1 ax + b
√ Z Z
2 ax + b 1 2nb 1
= n
+ 2n √ + √
ax n
x ax + b a x n+1 ax + b
Z √ Z ¡ ¢
dx 2 ax + b b 2n
a
=⇒ (1 − 2n) √ − = √
xn ax + b axn xn+1 ax + b
Z √ Z
1 − ax + b (2n − 3)a 1
√ = − √
xn ax + b (n − 1)bxn−1 (2n − 2)b xn−1 ax + b

Exercise 32. I derived the formulas for this and Exercise 33 by doing the following trick.

(C m+1 S 1−n )0 = (m + 1)C m (−S 2−n ) + C m+2 (1 − n)S −n = −(m + 1)C m S 2−n + (1 − n)C m S −n (1 − S 2 ) =
= −(m + 1 + 1 − n)C m S 2−n + (1 − n)C m S −n = −(m − n + 2)C m S 2−n + (1 − n)C m S −n
Z Z
−(C m+1 S 1−n ) (m − n + 2)
=⇒ C m S −n = − C m S 2−n
n−1 n−1
Exercise 33.
(C m−1 S 1−n )0 = (m − 1)C m−2 (−S 2−n ) + (1 − n)C m S −n
= −(m − 1)C m−2 (S −n )(1 − C 2 ) + (1 − n)C m S −n =
= −(m − 1)C m−2 S −n + (m − 1)C m S −n + (1 − n)C m S −n
Z Z
m−1 1−n m−2 −n
C S = −(m − 1) C S + (m − n) C m S −n
Z Z
m−1 m−2 −n C m−1 S 1−n
C S + = C m S −n
m−n m−n
Exercise 34.

(1) P 0 (x) − 3P (x) = 4 − 5x + 3x2


n
X
P = aj xj
n−1
X
j=0
n n−1
=⇒ (aj+1 (j + 1) − 3aj )xj = 4 − 5x + 3x2
X X j=0
P0 = aj jxj−1 = aj+1 (j + 1)xj
j=1 j=0

Generally, we can say


3
aj+1 = aj if j≥3
j+1
81
We also have

a1 (1) − 3a0 = 4
a2 (2) − 3a1 = −5
a3 (3) − 3a2 = 3 =⇒ a3 − a2 = 1

Then let a2 = −1 and a3 = 0. So we have a1 = 1 and a0 = −1. P (x) = −1 + x + −x2 is one possible polynomial
and we were only asked for one.
Suppose Q s.t. Q0 − 3Q = 4 − 5x + 3x2 (another solution). Then

(P − Q)0 − 3(P − Q) = 0 ∀x

X
(P − Q)0 (3x)j
=⇒ =3 =⇒ ln (P − Q) = 3x =⇒ ke3x = P − Q = k
P −Q j=0
j!

X (3x)j
Q = −k +P
j=0
j!

Since we didn't specify what Q has to be, we nd that, in general, any Q is P plus some “amount” of the homoge-
neous solution, ke3x .
(2) If Q(x) P is a polynomial solution to P 0 (x) − 3P (x) = Q(x). Suppose R is
is a given polynomial, and suppose
0 3x
another polynomial solution such that R (x) − 3R(x) = Q(x). Then just like above, P − R = ke . If we wanted
polynomial answers of nite terms, then k must be zero. Thus, there's at most only one polynomial solution P .

Exercise 35. Bernoulli Polynomials.


R1
(1) P1 (x) = 1; Pn0 (x) = nPn−1 (x); 0
Pn (x)dx = 0, if n≥1

Z 1 ¯1
1 2 ¯ 1
n=1 (1)(1) = P10 (x + c) = ( x + Cx)¯¯ = + C = 0 C = −1/2
0 2 0 2
P1 = x − 1/2
Z 1 µ ¶¯1
1 3 1 2 ¯ −1
n=2 2(x − 1/2) = P20 2
(x − x + C) = x − x + Cx ¯¯ = +C =0 C = 1/6
0 3 2 0 6
P2 = x2 − x + 1/6
Z 1
1 3x2 x 1 13 1
n=3 3(x2 − x + ) = P30 (x3 − + + C) = − + +C =0
6 0 2 2 4 2 4
2
3x x
P3 = x3 − +
2 2
Z 1
3 3 2 x 0 1 14 1 −1
n=4 4(x − x + ) = P4 (x4 − 2x3 + x2 + C) = − + 13 + C = 0 C =
2 2 0 5 2 3 30
−1
P4 = x4 − 2x3 + x2 +
30
Z 1
1 5x4 5x3 x 1 (1)5 5(1)4 −(1)2
n=5 5(x4 − 2x3 + x2 − ) = P50 (x5 − + − + C) = − + + + C = 0; C=0
30 0 2 3 6 6 2 12 12
5x4 5x3 x
P5 = x5 − + −
2 3 6

(2) The rst, second, and up to fth case has already been proven.
Pn−1
Assume the nth case, that Pn (t) = tn + j=0 a j tj (the general form of a polynomial of degree n).

n−1
X n−1
X
0 n j n+1 aj tj+1
Pn+1 = (n + 1)(t + aj t ) =⇒ Pn+1 = t + (n + 1) +C
j=0 j=0
j+1
82
(3) The rst, second, and up to fth case has already been proven.
Assume the nth case, that Pn (0) = Pn (1).
Z
0
Pn+1 = Pn+1 (1) − Pn+1 (0) (by the second fundamental theorem of calculus)

0
Pn+1 = (n + 1)Pn
Z 1
(n + 1)Pn (t) = 0 (by the given properties of Bernoulli polynomials)
0
=⇒ Pn+1 (1) = Pn+1 (0)

(4) Pn (x + 1)− = Pn (x) = nxn−1 n = 1, 2, by quick inspection (and doing some algebra mentally).
is true for
Z Z
0 0
Pn+1 = (n + 1)Pn =⇒ Pn+1 = (n + 1) Pn
Z x+1 Z x
Pn+1 (x + 1) − Pn+1 (x) = Pn+1 (a1 ) + (n + 1) Pn (t) − (Pn+1 (a2 ) + (n + 1) Pn (t))
a1 a2
a1 = 1; a2 = 0; Pn+1 (1) − Pn+1 (0) = 0 (from previous problems)
so
Z x+1 Z x
Pn+1 (x + 1) − Pn+1 (x) = (n + 1)( Pn (t) − Pn (t)) =
1 0
=⇒ Z x Z x
= (n + 1) Pn (t + 1) − Pn (t) = (n + 1) ntn−1 = (n + 1)xn
0 0

(5)
Z k Z 0 k
Pn+1 Pn+1 (k) − Pn+1 (0)
Pn = =
0 0 n + 1 n+1
Pn (x + 1) − Pn (x) = nxn−1
Pn (x + 1) − Pn (x)
= xn−1
n
k−1
X Pn+1 (x + 1) − Pn+1 (x) k−1
X k−1
X Pn+1 (r + 1) − Pn+1 (r)
= xn = =
x=1
n+1 x=1 r=1
n+1
=⇒
k−1
X Pn+1 (k) − Pn+1 (0)
= rn = (telescoping series and Pn+1 (1) = Pn+1 (0) )
r=1
n+1

(6) This part was fairly tricky. A horrible clue was that this part will rely directly on the last part (because of the way
Rx Px−1
this question is asked), which gave us
n
j=1 j = 0 Pn (t)dt = Pn+1 (x)−P
n+1
n+1 (0)
.
Use induction. It can be easily veried, plugging in, that Pn (1 − x) = (−1)n Pn (x) is true for n = 0 . . . 5.
Assume the nth case is true.
Z x Z 1−x Z 1−x
Pn (t)dt = −Pn (1 − u)du = − Pn (u)(−1)n du
0 1 1
Pn (1 − x) = (−1)n Pn (x), assumed nth case is true)
(since
u=1−t Z 1−x
du = −dt = (−1)n+1 Pn (t)dt =
1
Z 1 Z 1−x
= (−1)n+1
Pn (t)dt (since Pn = 0 )
0 0
µ ¶
Pn+1 (1 − x) − Pn+1 (0) Pn+1 (x) − Pn+1 (0)
=⇒= (−1)n+1 =
n+1 n+1
=⇒ Pn+1 (1 − x) = (−1)n+1 Pn+1 (x)
In the second to last and last step, we had used (−1)n+1 Pn+1 (0) = Pn+1 (0). For n + 1 even, this is denitely true.
If n + 1 was odd,
Doing some algebra for the rst ve cases, we can show that P2j−1 (0) = 0 for j = 2, 3. Assume the j th case is
P2j−1
true. Since P2j−1 is a polynomial and P2j−1 (0) = 0, then the form of P2j−1 must be P2j−1 = k=1 ak xj . Using
83
0
Pn+1 = (n + 1)Pn ,
Z x XZ x
2j−1 2j−1
X ¯x
k 1 k+1 ¯¯
P2j (x) − P2j (0) = 2j P2j−1 = 2j ak t = 2j ak t ¯ =
0 0 k+1 0
k=1 k=1
2j−1
X 2j
X
ak k+1 ak−1 xk
= 2j x = 2j + P2j (0)
k+1 k
k=1 k=2
Z Ã ! 2j+1
x X ak−1 k X 2jak−2 xk
P2j+1 (x) − P2j+1 (0) = (2j + 1) (2j) t + P2j (0) = (2j + 1) + (2j + 1)P2j (0)x
0 k k(k − 1)
k=2 k=3

0 to 1, then we nd that P2j+1 (0) = 0


If we take the integral from
n
(7) Using Pn (1 − x) = (−1) Pn (x), derived above,

P2j+1 (0) = (−1)2j+1 P2j+1 (1) = (−1)P2j+1 (0)


=⇒ P2j+1 (0) = 0
1 1
P2j−1 (1 − ) = (−1)2j−1 (P2j−1 ( ))
2 2
1
=⇒ P2j−1 ( ) = 0
2
Exercise 36. There's a maximum at c for f , so f 0 (c) = 0
Z x
f 00 (t)dt = f 0 (x) − f 0 (a)
a Z 0 Z c
Z c
|f 0 (0)| = | f 00 (t)dt| ≤ |f 00 |dt ≤ mc
f 00 (t)dt = f 0 (c) − f 0 (0) = −f 0 (0) c 0
Z 0a |f 0 (a)| ≤ m(a − c)
00 0 0 0
f (t)dt = f (a) − f (c) = f (a)
c

|f 0 (0)| + |f 0 (a)| ≤ ma

6.9 Exercises - Introduction, Motivation for the denition of the natural logarithm as an integral, The denition
of the logarithm. Basic properties; The graph of the natural logarithm; Consequences of the functional equation
L(ab) = L(a) + L(b); Logarithm referred to any positive base b 6= 1; Differentiation and integration formulas involv-
ing logarithms;Logarithmic differentiation.

Exercise 1.

(1)
µ ¶
x x
log x = c + (ln |t|)|e = c + ln |x| − 1 =⇒ ln =c−1
|x|
c=1
(2)
µ ¶ µ ¶
1+a 1+b (1 + a)(1 + b) 1+x
f (x) = ln + ln = ln = ln
1−a 1−b (1 − a)(1 − b) 1−x
1+x (1 + a)(1 + b) 1 + a + b + ab
=⇒ = =
1−x (1 − a)(1 − b) 1 − b − a + ab
a+b
x=
1 + ab
Exercise 2.

(1) log (1 + x) = log (1 − x)


=⇒ x = 0
(2) log (1 + x) = 1 + log (1 − x)
ln (1 + x) = 1 + ln (1 − x) = ln (e) + ln (1 − x) ln e(1 − x)
e−1
=⇒ 1 + x = e − ex =⇒ x =
1+e
84
(3) 2 log x = x log 2

ln x2 + ln 2−x = 0 = ln 1 = ln x2 2−x =⇒ x = 2
√ √
(4) log ( x + x + 1) = 1
√ √ √
x+1=e− x =⇒ x + 1 = e2 − 2 xe + x
µ 2 ¶2
√ e −1
2 xe = e2 − 1 =⇒ x =
2e
Exercise 3.

ln x
f=
x
0 1 − ln x f 0 < 0 when x > e f 0 > 0 when 0 < x < e
f =
x2 for x3 > 0, 2 ln x − 3 > 0 =⇒ f 00 (x) < 0 (concave), when 0 < x < e3/2 ;
µ1 2 ¶
00 −2 x x − 2x ln x 2 ln x − 3
f 00 (x) > 0 (convex), when x > e3/2
f = 3 − 4
=
x x x3
Exercise 4. f (x) = log (1 + x2 )
2x
f0 =
1 + x2

Exercise 5. f (x) = log 1 + x2
x
f0 =
1 + x2

Exercise 6. f (x) = log 4 − x2
1 −2x −x
f0 = 2
=
24−x 4 − x2
Exercise 7. f (x) = log (log x) µ ¶
1 1 1
f0 = =
ln x x x ln x
Exercise 8. f (x) = log x2 log x
0 2 1
f 0 = (2 log x + log log x) = +
x x log x
1 x2 −1
Exercise 9. f (x) = 4 log x2 +1
µ ¶ µ ¶
1¡ 0
¢0 1 2x 2x 1
f = log x2 − 1 − log x2 + 1 = − =x
4 4 x2 − 1 x2 + 1 x4 − 1

Exercise 10. f (x) = (x + 1 + x2 )n
p
ln f = n ln (x + 1 + x2 )
µ ¶µ ¶
f0 1 x n
=n √ 1+ √ =√
f x+ 1+x 2 1+x 2 1 + x2
p n
f 0 = (x + 1 + x2 )n √
1 + x2
√ √
Exercise 11. f (x) = x + 1 − log (1 + x + 1)
µ ¶
1 1 1 1
f0 = √ − √ √ = √
2 x+1 1+ x+1 2 x+1 2(1 + x + 1)
√ √
Exercise 12. f (x) = x log (x + 1 + x2 ) − 1 + x2
p µ ¶
0 2
x x x
f = log (x + 1 + x ) + √ 1+ √ −√
x+ 1+x 2 1+x 2 1 + x2
Note to self: Notice how this had made some of the square root terms disappear.
√ √
√1 a+x√b
Exercise 13. f (x) = 2 ab
log √
a−x b
85
1 √ √ √ √
f = √ (ln ( a + x b) − ln ( a − x b))
2 ab
µ ¶ √
0
√ 1 √ 1 √ −x b
f = f rac12 ab √ √ b− √ √ (− b) = √
a+x b a−x b a(a − bx2 )
Exercise 14. f (x) = x(sin (log x) − cos log x)
f 0 = sin (ln x) − cos (ln x) + (cos (ln x) + sin (ln x)) = 2 sin (ln x)
Exercise 15. f (x) = log−1 x
−1
f0 =
x(ln x)2
R dx 1
Exercise 16.
2+3x = 3 ln (2 + 3x)
R 2
Exercise 17. log xdx
(x ln2 x)0 = ln2 x + 2 ln x
=⇒ xln2 x − 2(x ln 2 − x) = x ln2 x − 2x ln x + 2x
(x ln x − x)0 = ln x + 1 − 1 = ln x
R
Exercise 18. x log xdx
µ ¶0
x2 ln x x
= x ln x +
2 2
Z
x2 ln x x2
x ln x = −
2 4
R
Exercise 19. x log2 xdx
µ ¶0 µ ¶
x2 ln2 x 2 2 1
= x ln x + x ln x
2 x
Z 2
x ln x x ln x x2
2 2
=⇒ x log2 x = − −
2 2 4
R e2 −1 dt
Exercise 20.
0 1+t
Z e3 −1
dt e3 −1
= ln (1 + t)|0 =3
0 1+t
R
Exercise 21. cot xdx Z
cos x
dx = ln | sin x|
sin x
R
Exercise 22. xn log (ax)dx Solve the problem directly.
Z Z Z Z
n n n xn+1
x log ax = x log a + x log x = log a + xn log x
n+1
Z Z n+1
xn+1 x 1 xn+1 xn+1
xn ln x = ln x − = log x −
n+1 n+1x n+1 (n + 1)2
Z
xn+1 xn+1 xn+1
=⇒ xn log ax = log a + log x −
n+1 n+1 (n + 1)2
R
Exercise 23. x2 log2 xdx
Z Z µ ¶
1 x2 x3 ln2 x 2 x3 ln x x3 x3 ln2 x 2x3 ln x 2x3
x log x = x3 ln2 x −
2 2
2 ln x = − − = − +
3 3 3 3 3 9 3 9 27
R dx
Exercise 24.
x log x Z
dx
= ln (ln x) + C
x ln x
R 1−e−2 log (1−t)
Exercise 25.
1 1−t dt
Z 1−e2 ¯1−e−2
ln (1 − t) 1 ¯

dt = − (ln (1 − t)) ¯ = −2
0 1−t 2 0
86
R
Exercise 26. √log |x| dx
x 1+log |x|
Z Z Z
log x 2(1 + log x)1/2
√ = (2(1 + log x)1/2 )0 log x = 2(1 + log x)1/2 log x − =
x 1 + log x x
4
= 2 log x(1 + log x)1/2 − (1 + log x)3/2
3
Exercise 27. Derive Z Z
xm+1 n
n n
m
x log xdx = ln x − xm lnn−1 x
m+1 m+1
By inspection, we just needed integration by parts.
Z Z µ Z ¶
x4 3 3 x4 3 3 x4 2 2 x4 ln3 x 3x4 ln2 x 3x4 ln x 3x4
x3 ln3 x = ln x− x3 ln2 x = ln x− ln x − x3 ln x = − + −
4 4 4 4 4 4 4 16 32 128
1
Exercise 28. Given x > 0, f (x) = x − 1 − ln x; g(x) = ln x − 1 + x
(1)
1
f0 = 1 −
x xg 0 = f 0
1 1 1
g = − 2 = f0
0
x x x
0 0
so then if f > 0, g > 0; f 0 < 0, g0 < 0
For f0 < 0 0<x<1 f0 > 0 x>1 f 0 (1) = g 0 (1) = 0
f (1) = g(1) = 0
x − 1 − ln x > 0 since f (1) = 0 is a rel. min.
1
0 < ln x − 1 + since g(0) is a rel. min.
x
(2) See sketch.

Exercise 29. limx→0 log (1+x) =1


Rx 1 x 1
(1) L(x) = 1 t dt ; L0 (x) = x; L0 (1) = 1
(2) Use this theorem.

Theorem 19 (Theorem I.31).


If 3 real numbers a, x, and y satisfy the inequalities
y
a≤x≤a+
n
∀ n ≥ 1, n ∈ Z, then x=a
1 1
1− < ln x < x − 1 =⇒ 1 − < ln x + 1 < x
x x+1
x ln x + 1 ln (1 + x)
1− < <1 =⇒ =1
1+x x x
Exercise 30. Using f (xy) = f (x) + f (y),
p p
r= =⇒ f (ap/q ) = f ((a1/q )p ) = pf (a1/q ) = f (a)
q q
since

f (a) = f (a); f (a2 ) = f (aa) = 2f (a) If f (a) = f ((a1/q )q) = qf (a1/q )


1
f (ap+1 ) = f (ap ) + f (a) = (p + 1)f (a) then f (a
1/q
) = f (a)
q
R |x|
Exercise 31. ln x = 1 1t dt
R |x|
(1) ln x = 1 1t dt From this denition, then for n partitions, a0 = 1, a1 = 1 + b−1
n , . . . , an = b = x
b−a b−1
¡ b−1
¢
n = n ; so if ak = 1 + k
n
Xn µ ¶ Xn µ ¶
ak − ak−1 ak − ak−1
< log x <
ak ak−1
k=1 k=1
87
1
(2) log x is greater than the step function integral consisting of rectangular strips within x and less than rectangular strips
1
covering over
x
ak −ak−1 1
(3) ak = 1 + k =⇒ ak−1 = k

n
X n
X n+1
X1 n
X
1 1 1
< ln (n + 1) < =⇒ < ln (n + 1) <
1+k k k k
k=1 k=1 k=2 k=1
n
X n−1
X1
1
=⇒ < ln (n) <
k k
k=2 k=1

Exercise 32.
1
(1) L(x) = ln b ln x = logb x
loga x = c logb x =⇒ loga a = 1 (There must be a unique real number s.t. L(a) = 1 )
1
loga x = logb x =⇒ logb x = logb a loga x
logb a
loga x
(2) Changing labels for a, b: logb x = loga b

Exercise 33. loge 10 = 2.302585

loge e 1 1
log10 e = =⇒ loge 10 = = ' 0.43429
loge 10 log10 e 2.302585
R xy Rx
Exercise 34. Given
x
f (t)dt = B(y) f (2) = 2, , We Want A(x) =
1
f (t)dt
Z xy
f (t)dt = B(xy) − B(x)
x
Z xy
d d dB(xy) dB(x)
f (t)dt = (B(xy) − B(x)) = y− = f (xy)y − f (x) = 0
dx x dx d(xy) dx
f (x)
=⇒ f (xy) =
y
Z x Z x µ ¶ Z x
t f (2)
A(x) = f (t)dt = f ((2) )dt = dt = 4 ln x
1 1 2 1 (t/2)
R xy Rx Ry
Exercise 35. Given
1
f (t)dt = y 1 f (t)dt + x 1 f (t)dt, and letting F be the antiderivative of f ,
F (xy) − F (1) = y(F (x) − F (1)) + x(F (y) − F (1))
Z y
d/dx d/dx
−−−→ f (xy)(y) = y(f (x)) + f (t)dt −−−→ f 0 (xy)y 2 = y(f 0 (x))
0
R
x=1 (f 0 (1)) y=1
−−→ f 0 (y) = −
→ f (y) = k ln y + C −−→ f (1) = 0 + C = 3
y
R xy Rx Ry
1
(k ln t + 3) = y 1 (k ln t + 3) + x 1 (k ln t + 3)

k ((xy) ln xy − (xy) + 1) + 3(xy − 1) = y (k ((x) ln x − x + 1) + 3(x − 1)) + x (k (y ln y − y + 1) + 3(y − 1))


=⇒ k − 3 = ky − 3y − kxy + kx + 3xy − 3x
y(3 − k) + xy(k − 3) + k − 3 + (3 − k)x = 0 =⇒ k = 3
=⇒ f (x) = 3 ln x + 3
Exercise 36.

6.11 Exercises - Polynomial approximations to the logarithm.

Exercise 1.

Theorem 20 (Theorem 6.5). If 0 < x < 1 and if m ≥ 1,


1+x x3 x2m−1
ln = 2(x + + ··· + ) + Rm (x)
1−x 3 2m − 1
88
where
x2m+1 2 − x x2m+1
< Rm (x) ≤
2m + 1 1 − x 2m + 1
Rm (x) = E2m (x) − E2m (−x)
where E2m (x) is the error term for log 1 − x
1
m = 5, x = 3
µ ¶ Ã ¡ 1 ¢3 ¡ 1 ¢5 ¡ 1 ¢7 ¡ 1 ¢9 !
4/3 1 3 3 3 3
ln = ln 2 ' 2 + + + + ' 0.693146047
2/3 3 3 5 7 9
1
The error for m = 5, x = 3 is
¡ 1 ¢11 ¡ ¢11
3 5 13
≤ R5 (x) ≤
11 2 11
³ ´
1+x
Exercise 2. ln 1−x = ln 32 = ln 3 − ln 2
1
x=
5
m=5
µ ¶ µ ¶
1+x 1 (1/5)3 (1/5)7 (1/5)9
ln =2 + + + = 0.405465104
1−x 5 3 7 9
¡ ¢11
(1/5)11 9 15
' 0.000000002 < R5 (x) ≤ ' 0.000000004
11 4 11
log 3 ' 1.098611
=⇒
1.098611666 < log 3 < 1.098612438
³ ´ ¡ 10 ¢
1 1+ 91
Exercise 3. x= 9 ln 1− 19
= ln 8 = ln 54 = ln 5 − 2 ln 2

For m=2 Ã ¡ 1 ¢5 ¡ 1 ¢5 !
1 9 9
2 + + = 0.2231435
9 3 5
¡ 1 ¢7 17
¡ 1 ¢7
9 9 9
' 0.00000003 < R3 (x) ≤ ' 0.000000063
8
7 7 9
1.609437 < log 5 < 1.609438
³ ´ ¡7¢
1 1+ 61
Exercise 4. x= 6 ln 1− 16
= ln 5 = ln 7 − ln 5.

For m = 3, Ã ¡ 1 ¢3 ¡ 1 ¢5 !
1+x 1 6
ln '2 + + 6 = 0.336471193
1−x 6 3 5
¡ 1 ¢2(3)+1
6
= 0.00000051
2(3) + 1
The error bounds are ¡ ¢
1 2(3)+1 µ ¶
6 2 − 16
= 0.000001123
2(3) + 1 1 − 16
1.945908703 < ln 7 < 1.945910316
Exercise 5.
0.6931460 < ln 2 < 0.6931476
ln 7 = 1.945909
ln 3 = 1.098614
ln 8 = 3 ln 2 = 2.0794404
ln 4 = 1.386293
ln 9 = 2 ln 3 = 2.197228
ln 5 = 1.609436 ln 6 = ln 2 + ln 3 = 1.791700
ln 10 = ln 5 + ln 2 = 1.302577
89
6.17 Exercises - The exponential function, Exponentials expressed as powers of e, The denition of ex for arbitrary
x
real x, The denition of a for a > 0 and x real, Differentiation and integration formulas involving exponentials.
2 2 √
1 −1 −1/x
Exercise 1. f 0 = 3e3x−1 Exercise 2. 8xe4x Exercise 3. −2xe−x Exercise 4. √
2 x
e x Exercise 5. x2 e Exercise 6.

2 2
1 log x
ln 22x Exercise 7. (2x ln 2)2x Exercise 8. cos xesin x Exercise 9. −2 cos x sin xecos x
Exercise 10.
xe Exercise 11.

x ex x R R
ex ee Exercise 12. ee (ex ee ) Exercise 13. xex dx = xex − ex Exercise 14. xe−x dx = −xe−x + −e−x Exercise 15.

R R x2 e−2x xe−2x −e−2x


x2 ex = x2 ex − 2xex + 2ex Exercise 16. x2 e−2x dx = −2 + −2 + 4
R √ √ √ R √ √ √ √ √ √ √
Exercise 17. e x
=e x
(2 x) − √1 e x
x
=e x
(2 x) − 2e x = 2( xe x − e x )
R 2
Exercise 18. x3 e−x .
2 2 2
x2 e−x = −2x3 e−x + 2xe−x
2 2
e−x = −2xe−x

1 ³ ´ 2 2
1 2 −x2 2 2 2 2 x2 e−x + e−x
(x e + e−x )0 = 2xe−x + −2x3 e−x + −2xe−x =
−2 −2 −2
Rb
Exercise 19. ex = b + a et dt = b + ex − ea , ea = b.
Exercise 20.
Z
A= eax cos bxdx
Z
B= eax sin bxdx
Z
eax b sin bxeax eax b
A= cos bx − − = cos bx + B =⇒ aA + bB = eax cos bx + C
a a a a
Z ax
eax e
B= sin bx − b cos bx =⇒ aB + bA = eax sin bx + C
a a
1
A= 2 (aeax cos bx + beax sin bx)
a + b2
−beax cos bx + aeax sin bx
B=
a2 + b2
Exercise 21.
f0
ln f = x ln x; = ln x + 1; f 0 = xx (ln x + 1)
f
Exercise 22.
f0 1 1 x2 2 2
ln = + x 2 (2xe ), f 0 = 1 + ex + 2(1 + x)xex
f 1+x 1+e
ex −e−x
Exercise 23. f= ex +e−x .
ln f = ln (ex − e−x ) − ln (ex + e−x )
f0 1 1
= x (ex + e−x ) − x (ex − e−x )
f e − e−x e + e−x
µ x ¶2
0 e − e−x
f =1−
ex + e−x
a a x
Exercise 24. f 0 = (xa )0 + (ax )0 + (aa )0 .
a a x
f1 = x a f2 = ax f3 = a a
ln f1 = aa ln x ln f2 = xa ln a ln f3 = ax ln a
f10 aa f20 f30
= = axa−1 ln a = (ln a)2 ax
f1 x f2 f3
a a x
f10 = xa −1 a
a f20 = ax +1 a−1
x ln a f30 = (ln a)2 ax+a
90
a a x
=⇒ f 0 = xa −1 a
a + ax +1 a−1
x ln a + (ln a)2 ax+a
Exercise 25.
ef = ln (ln x);
µ ¶
1 1
f 0 ef =
ln x x
µ ¶µ ¶
0 1 1 1
f =
ln (ln x) ln x x

Exercise 26. ef = ex + 1 + e2x
e2x
(ef )f 0 = ex + √
1 + e2x

1 + e2x ex + e2x ex
f0 = √ √ =√
2x x
1 + e (e + 1 + e ) 2x 1 + e2x
Exercise 27. ln f = xx ln x
f0 xx
= (xx )0 ln x + = (xx (ln x + 1)) ln x + xx−1
f x
x xx
f 0 = xx+x (ln x + 1) ln x + xx +x−1

Exercise 28. ln f = x ln (ln x)


f0 1
= ln (ln x) +
f ln x
1
f 0 = (ln x)x (ln (ln x) + )
ln x
Exercise 29. ln f = (ln x) ln x
f0 2 ln x
=
f x
f 0 = 2xlog x−1 ln x

Exercise 30. ln f = x ln (ln x) − ln x ln x = x ln ln x − (ln x)2


f0 1 ln x
= ln (ln x) + −2
f ln x x
x
µ ¶
0 (ln x) 1 2 ln x
f = ln x ln ln x + −
x ln x x
Exercise 31.

ln f1 = cos x ln sin x ln f2 = sin x ln cos x


f10 cos2 x f20 − sin2 x
= − sin x ln sin x + = cos x ln cos x +
f1 sin x f2 cos x
3
cos x ln sin x sin3 x ln cos x
f10 = − sin x cos x(ln sin x)2 + f20 = sin x cos x(ln cos x)2 −
sin x cos x
3 3
cos x ln sin x − sin x ln cos x
=⇒ f = sin x cos x(−(ln sin x)2 + (ln cos x)2 ) + +
sin x cos x
1
Exercise 32. ln f = x ln x
0
f −1 1
f = x2 ln x + x2 =⇒ f 0 = −x1/x−2 ln x + x1/x−2
1 −2
Exercise 33. ln f = 2 ln x + 3 ln (3 − x) − ln (1 − x) + 3 ln (3 + x)
f0 2 −1 −1 2 1
= + − −
f x 3(3 − x) 1 − x 3 3 + x
1/3
µ 2 ¶
x(3 − x) 1 x (3 − x)−2/3 x2 (3 − x)1/3 2 x2 (3 − x)1/3
f0 = 2 2/3
+ − 2/3
+ 2 2/3

(1 − x)(3 + x) 3 (1 − x)(3 + x) (1 − x) (3 + x) 3 (1 − x)(3 + x)5/3
x(18 − 12x + 43 x2 + 23 x3 )
=
(1 − x)2 (3 + x)5/3 (3 − x)2/3
91
Pn
Exercise 34. ln f = i=1 bi ln (x − ai )
n
X bi n
X n
f0 bj Y
= =⇒ f 0 = (x − ai )bi
f i=1
x − ai j=1
x − aj i=1

Exercise 35.

(1) Show that f 0 = rxr−1 for f = xr holds for arbitrary real r.


xr = er ln x
r
(er ln x )0 = er ln x = rxr−1
x
(2) For x ≤ 0, by inspection of xr = er log x , then if xr > 0, then the equality would remain valid. So then xr = |xr | =
r
|x| and so
ln |f (x)| = r ln |x|
0
f (x) 1
= r =⇒ f 0 (x) = rxr−1
f (x) x
Exercise 36. Use the denition ax = ex log a
(1) log ax = x log a

Taking the exponential is a well-dened inverse function to log so taking the log of both sides of the denition, we
getlog ax = x log a
(2) (ab)x = ax bx

(ab)x = ex log ab = ex(log a+log b) = ax bx


(3) ax ay = ax+y
ax+y = e(x+y) log a = ex log a ey log a = ax ay
(4) (ax )y = (ay )x = axy
(ax )y = exy log a = (ay )x = axy
(5) If x = loga y ,
Using the denition
log x
logb x = if b > 0, b 6= 1, x>0
log b
so then
log y log y = x log a
loga y = = x =⇒ x log a
log a e = elog y = y = ax
If y = ax ,

log y
log y = x log a =⇒ x = = loga y
log a
Exercise 37. Let f (x) = 12 (ax + a−x ) if a > 0.
1
f (x + y) = (ax+y + a−(x+y) )
2
1 ³ x+y ´
f (x + y) + f (x − y) = a + a−(x+y) + ax−y + a−(x−y)
2
1 x 1 ³ x+y ´
f (x)f (y) = (a + a )(a + a−y ) =
−x y
a + a−x−y + a−(x−y) + a(x−y)
4 4
Exercise 38.

f (x) = ecx ; f 0 (x) = cecx ; f 0 (0) = c


µ ¶ µ ¶
ecx − 1 ecx − 1 ecx − 1 df (cx) d cx
lim = c lim = c lim =c (0) = (e )(0)
x→0 x x→0 cx cx→0 cx d(cx) dx
ecx − 1
lim = f 0 (0) = c
x→0 x
92
Exercise 39.
g(x) = f (x)e−cx
g 0 (x) = f 0 e−cx + −cg = cg − cg = 0

f = Kekx

Exercise 40. Let f be a function dened everywhere on the real axis. Suppose also that f satises the functional equation

f (x + y) = f (x)f (y) for all x and y


(1)
If f (0) = 0, then we're done
f (0) = f (0)f (0) = f 2 (0)
If f (0) 6= 0 then f (0) = 1 (by dividing both sides by f (0) )
(2) Take the derivative with respect to x on both sides of the functional equation.
df (x + y) d(x + y) df (x) d(f (x + y)) f (x + y)
= f (y) =⇒ = f 0 (x)
d(x + y) dx dx d(x + y) f (x)
Let y = −x + y
df (y) f (y)
= f 0 (x) =⇒ f 0 (x)f (y) = f 0 (y)f (x)
dy f (x)
f 0 (x) f 0 (y)
(3)
f (x) = f (y) ∀ x, y
The only way they could do that for any arbitrary x, for any arbitrary y they one could choose on either side, is for
them to both equal a constant
f 0 (y)
=⇒ f (y) = c
(4) Referring to Exercise 39 of the same section, f = ecx since f 0 (0) = 1
Exercise 41.

(1)
(
f = ex − 1 − x ex > 1 + x for x>0
0 x f (0) = e0 − 1 − 0 = 0
f = e − 1 ≷ 0 if x ≷ 0 e−x > 1 − x for x<0
(2)
Z x
1 1
et = ex − 1 > x + x2 =⇒ ex > 1 + x + x2
0 2 2
2 2
x x
− e−x > −1 + x − =⇒ e−x < 1 − x +
2 2
(3)
Z x
1 1 3 1 1 3
et = ex − 1 > x + x2 + x =⇒ ex > 1 + x + x2 + x
0 2 3∗2 2 3∗2
x2 x3 x2 x3
− e−x > −1 + x − + =⇒ e−x < 1 − x + −
2 3∗2 2 3∗2
(4) Suppose the nth case is true.
( P2m+1
n
X xj
x xj −x > j=0
e > e = P2m xj j!
j=0
j! < j=0 j!

Xn n+1
X xj n+1
X
xj+1 xj
ex > 1 + =1+ =
j=0
(j + 1)! j=1
j! j=0
j!
( P2m+1 j+1 ( P2m+2
x xj
> j=0 (j+1)! < j=0
− e−x + 1 P2m xj+1 = e−x P2m+1 xj!j
< j=0 (j+1)! > j=0 j!

Exercise 42. Using the result from Exercise 41,

³ n µ ¶
x ´n X n n−j ³ x ´j X
n Xn Xn
n! xj n(n − 1) . . . (n − j + 1) xj xj
1+ = 1 = = < < ex
n j=0
j n j=0
(n − j)!j! nj j=0
j! nj j=0
j!
93
If you make this clever observation, the second inequality is easy to derive.
x
x>0 >0
³ n ´ ³
x x n x ´n
e− n >1− =⇒ e−x/n > 1 −
n n
e−x > (1 − x/n)n =⇒ ex < (1 − x/n)−n

Exercise 43. f (x, y) = xy = ey ln x


∂x f = xy y/x
∂y f = xy ln x

6.19 Exercises - The hyperbolic functions.


Exercise 7.

ex − e−x ex + e−x 1
2 sinh x cosh x = 2 = (e2x − e−x ) = sinh 2x
2 2 2
Exercise 8.
µ ¶2 µ ¶2
2 2 ex + e−x ex − e−x 1 2x
cosh x + sinh x = + = (e + 2 + e−2x + e2x − 2 + e−2x ) = cosh 2x
2 2 4
Exercise 9.
ex + e−x ex − e−x
cosh x + sinh x = + = ex
2 2
Exercise 10.
µ ¶
ex + e−x ex − e−x
cosh x − sinh x = − = e−x
2 2
Exercise 11. Use induction.

(cosh x + sinh x)2 = cosh2 x + 2 sinh x cosh x + sinh2 x = cosh 2x + sinh 2x


(cosh x + sinh x)n+1 = (cosh x + sinh x)(cosh nx + sinh nx) =
= cosh nx cosh x + cosh nx sinh x + sinh nx cosh x + sinh x sinh nx =
enx + e−nx ex + e−x enx + e−nx ex − e−x
+ +
= 2 2 2 2
enx − e−nx ex + e−x enx − e−nx ex − e−x
+ +
2 2 2 2
= cosh (n + 1)x + sinh (n + 1)x
Exercise 12.

cosh 2x = cosh2 x + sinh2 x = 1 + 2 sinh2 x

6.22 Exercises - Derivatives of inverse functions, Inverses of the trigonometric functions.


Exercise 1.
p 1
(cos x)0 = − sin x = − 1 − cos2 x D arccos x = √ −1<x<1
− 1 − x2
Exercise 2.
sin2 x + cos2 x
(tan x)0 = sec2 x = = tan2 x + 1
cos2 x
1
D arctan x =
1 + x2
Exercise 3.
(sin2 x + cos2 x) 1
(cot x)0 = − csc2 x = − = −(1 + cot2 x) =⇒ arccotx = −
sin2 x 1 + x2
Exercise 4.
p
(sec y)0 = tan y sec y = sec2 y − 1 sec y; | sec y| > 1 ∀y ∈ R
If we choose to restrict y such that 0 ≤ y ≤ π , then (sec y)0 > 0. Then we must make sec y → | sec y|.
Darcsecx = √1
|x| x2 −1
94
Exercise 5. p
(csc y)0 = − cot x csc x = − csc y( csc2 y − 1)
−π π
Let y such that < y < (csc y) < 0
2 2
1
Darccscx = √
−|x| x2 − 1
Exercise 6.
x
(xarccotx)0 = arccotx −
1 + x2
µ ¶0
1 1x
ln (1 + x2 ) =
2 (1 + x2 )
Z
1
arccotx = xarccotx + ln (1 + x2 ) + C
2
Exercise 7.
x
(xarcsecx)0 = arcsecx + √
|x| x2 − 1
 1+ √ x
 √ x2 −1
µ p ¶0   |x+ x2 −1|
 x>1
x 2
x
ln |x + x − 1| = = √
|x| 
 −1+ √ x |x| x2 − 1

− √ x2 −1 x < −1
|x+ x2 −1|
Z p
x
=⇒ arcsecxdx = xarcsecx − log |x + x2 − 1| + C
|x|
2 2j+1 √
Take a note of this exercise. When dealing with (∓x ± 1) 2 ; j ∈ Z; try x ± x2 ± 1 combinations. It'll work out.

Exercise 8.
x
(xarccscx)0 = arccscx + √
−|x| x2 − 1
 ³ ´
µ p ¶0 
x √1 1 + √ x = √x12 −1 x>1
2 2
ln |x + x2 − 1| = x+−1x −1 x −1
|x| √ 2 x < −1
x −1
Z p
x
=⇒ arccscx = xarccscx + ln |x + x2 − 1|
|x|
Exercise 9.
2x arcsin x
(x(arcsin x)2 )0 = (arcsin x)2 + √
1 − x2
³p ´0 −x
1 − x2 arcsin x = √ arcsin x + 1
1 − x2
Z p
(arcsin x)2 = x(arcsin x)2 + 2 1 − x2 arcsin x − 2x

Exercise 10. µ ¶0
− arcsin x 1 1
= 2
arcsin x − √
x x x 1 − x2
I would note how x is in the denominator of the second term. Again, reiterating,
p −x
( 1 − x2 )0 = √
1 − x2
p p
(y ± ±1 ∓ x2 )(y ∓ ±1 ∓ x2 ) = y 2 − (±1 ∓ x2 )
Multiply by its “conjugate.” As we see, choose y appropriately to get the desired denominator (that's achieved after differen-
tiation). Here, picky = 1.
µ ¶ √ √
p 1 −x −x(1 − 1 − x2 ) −(1 − 1 − x2 )
(ln (1 + 1 − x2 ))0 = √ √ = √ = √
1 + 1 − x2 1 − x2 1 − x2 (x2 ) x 1 − x2
¯ ¯
Z
arcsin x ¯ 1 − √1 − x2 ¯ arcsin x
¯ ¯
=⇒ = ln ¯ ¯− +C
x2 ¯ x ¯ x
Exercise 11.

95
(1)
µ ¶ µ ¶
1 −1 1 −1
D arccotx − arctan = 2 − ¡ ¢ =0
x x + 1 1 + 1 2 x2
x
(2) arccotx − arctan x1 = C
π
Now arccotx = 2 − arctan x.
π 1 π 1
− arctan x − arctan = C =⇒ − C = arctan x + arctan
2 x 2 x
π π
x → ∞ =⇒ − C = + 0 =⇒ C = 0
2 2
but x → −∞
π π
− C = − + 0 =⇒ C = π
2 2
1
There are problems with the choice of brances for arccotx, arctan , even though the derivatives work in all cases.
x
Exercise 12. µ ¶
1 1
f0 = q ¡ x ¢2 2
1− 2
Exercise 13. µ ¶
−1 −1 1
f0 = r ³ ´2 √ =√
2 1 + 2x − x2
1 − 1−x

2

Exercise 14. f = arccos x1 .


µ ¶
0 −1 −1 1
f =q ¡ ¢2 =√
x2 x2 − 1|x|
1 − x1
Exercise 15.
1 cos x
f (x) = arcsin (sin x) = p cos x =
1 − sin x 2 | cos x|
Exercise 16. √
1 1 1 x
− √ =
2 sqrtx x + 1 2 x 2(x + 1)
Exercise 17.
1 x2
+
1 + x2 1 + x6
Exercise 18.
µ ¶ √
1 −2x(2) 1 + x2 −4x −4
r ³q ´2 =p = √
1−x2
(1 + x2 )2 1+ x2 − (1 − x2 ) 2
(1 + x )2
(1 + x2 )3/2 2
1− 1+x2

Exercise 19. f = arctan tan2 x


1 ¡ 2
¢ 2 tan x sec2 x
2 tan x sec x =
1 + tan4 x 1 + tan4 x
Exercise 20. √
0 1 x 1 + x2 + x
f = √ (1 + √ )= √
1 + (x + 2
1+x )2 1+x2 1 + (x + 1 + x2 )2
Exercise 21.
1 cos x + sin x
f0 = q (cos x + sin x) = √
2
1 − (sin x + cos2 x − 2 sin x cos x) 2 sin x cos x

Exercise 22.
p 1 1
f 0 = (arccos 1 − x2 )0 = p =−
− 1 − (1 − x2 ) |x|
Exercise 23.
µ ¶
1 2 2 2 1
f0 = ³ ´2 = = =
1+x (1 − x)2 (1 − x)2 + (1 + x)2 (1 − 2x + x2 + 1 + 2x + x2 ) 1 + x2
1+ 1−x
96
Exercise 24. f = (arccos (x2 ))−2
µ ¶
0 2 −3 −1 4x(arccos x2 )−3
f = −2(arccos x ) √ (2x) = √
1 − x2 1 − x4

! 
Exercise 25.
à µ ¶
0 1  q −1  −1 1
f = = √
arccos √1x 1− 1 2x3/2 2 arccos √1
x
x3 − x2
x

dy x+y
Exercise 26.
dx = x−y .
³ µ ¶ µ ¶0
y ´0 1 y0 x − y 1 2 2 1 1
arctan = ¡ ¢2 = ln (x + y ) = (2x + 2yy 0 )
x 1 + xy x2 2 2 (x2 + y 2 )
x+y
=⇒ y 0 =
x−y
Exercise 27.

1
ln y = ln (arcsin x) − ln (1 − x2 )
µ 2 ¶
y0 1 1 1 1 1 x
= √ − 2
(−2x) = √ +
y arcsin x 1−x 2 21−x arcsin x 1 − x 2 1 − x2
arcsin x
y=√
1 − x2
µ ¶ √
0 1 (arcsin x)x 1 − x2 + x(arcsin x)
y = 2
+ 2 3/2
=
1−x (1 − x ) (1 − x2 )3/2
p 3
ln y 0 = ln ( 1 − x2 + x arcsin x) − ln (1 − x2 )
µ 2 ¶
y 00 1 −x x 3 (−2x)
0
= √ √ + arcsin x + √ −
y 2
1 − x + x arcsin x 1−x 2 1−x 2 2 1 − x2

arcsin x y 0 3x arcsin x ( 1 − x2 + x arcsin x)(3x)
y 00 = y 0 √ + = +
1 − x2 + x arcsin x 1 − x2 (1 − x2 )3/2 (1 − x2 )3/2

Exercise 28.
1 1 − 1 − x2 + x4 x4
f0 = − 1 + x2
= = ≥ 0 ∀x
1 + x2 1 + x2 1 + x2
x3
since f (0) = arctan 0 − 0 + 0 = 0, arctan x > x − , ∀x > 0
3
Exercise 29.
Z
dx x
√ , a 6= 0 =⇒ arcsin
a2 − x2 a
Exercise 30. Z Z
dx dx x+1
p = r ³ ´ = arcsin √
2 − (x + 1)2 √ 2 2
2 1 − x+1

2

Exercise 31. Z
dx 1 x
³ ¡ x ¢2 ´ = a arctan a
a2 1 + a
Exercise 32.

dx 1 bx
³ √ ´2 = √ arctan √
ba a
a(1 + √ba
a
)
Exercise 33.
Z Z ¡ ¢
dx 4 dx 2 2 x − 12
¡ ¢ = ³ ¡ ¢ ´2 = √ arctan √
1 2 7 7 7 7
x− 2 + 4 √2 x − 1
+ 1
7 2
97
Exercise 34.
µ ¶0 µ ¶
x2 arctan x x2 1 1 1
= x arctan x + = x arctan x + 1−
2 2 1 + x2 2 1 + x2
µ ¶0 µ ¶
1 1 1
(x − arctan x) = 1−
2 2 1 + x2
Z
1
x arctan x = x arctan x + − (x − arctan x)
2
Exercise 35.
µ ¶0
x3 x3 −1
arccos x = x2 arccos x + √
3 3 1 − x2
p p x3
(x2 1 − x2 )0 = 2x 1 − x2 + − √
1 − x2
3
((1 − x2 )3/2 )0 = (−2x)(1 − x2 )1/2 = −3x(1 − x2 )1/2
2
Z
x3 1 p 9
x2 arccos x = arccos x − x2 1 − x2 − (1 − x2 )3/2
3 3 2
Exercise 36.
µ ¶0 µ ¶ µ µ ¶¶
x2 (arctan x)2 1 1
= x(arctan x)2 + x2 arctan x = x(arctan x)2
+ 1 − arctan x
2 1 + x2 1 + x2
µ ¶0
(arctan x)2 arctan x
=
2 1 + x2
x
(x arctan x)0 = arctan x +
1 + x2
Z µ ¶
x2 (arctan x)2 ln (1 + x2 ) (arctan x)2
x(arctan x)2 dx = − x arctan x − +
2 2 2
Exercise 37.
¶µ µ¶
√ 1 1
(arctan x)0 = √
1+x 2 x
√ µ ¶
√ 0 √ x √ 1 1 1
(x arctan x) = arctan x + = arctan x + √ −√
2(1 + x) 2 x x(1 + x)
√ √ √
(x arctan x + arctan x + −x1/2 )0 = arctan x + 0
Z
√ √ √
arctan x = x arctan x + arctan x − x1/2

Exercise 38. From the previous exercise,


Z √
arctan x √
√ dx = (arctan x)2
x(1 + x)
Exercise 39. Let x = sin u
Z p Z √
2 u sin 2u arcsin x x 1 − x2
1− x2 dx = cos udu = + = +
2 4 2 4
Exercise 40.
Z
xearctan x
(1 + x2 )3/2
µ ¶0
earctan x −xearctan x earctan x
√ = +
1 + x2 (1 + x2 )3/2 (1 + x2 )3/2
µ arctan x ¶0
xe earctan x x2 earctan x xearctan x earctan x xearctan x
√ =√ +− 2 3/2
+ 2 3/2
= 2 3/2
+
1 + x2 1 + x2 (1 + x ) (1 + x ) (1 + x ) (1 + x2 )3/2
µ arctan x ¶ 0
1 xe earctan x xearctan x
√ −√ =
2 1 + x2 1 + x2 (1 + x2 )3/2
98
Exercise 41. From the previous exercise,
µ ¶
1 xearctan x earctan x
√ +√ +C
2 1 + x2 1 + x2
Exercise 42.
µ ¶0
x(1 + x2 )−1 x2 1
Since − = −
2 2
(1 + x ) 2 2(1 + x2 )
Z 2
x −x 1
2 2
dx = 2
+ arctan x
(1 + x ) 2(1 + x ) 2
Exercise 43. arctan ex .
Exercise 44. Z
arccotex
dx
ex
−ex
(arccotex )0 =
1 + e3x
µ ¶
e−x (−1)ex e2x
− (e−x arccotex )0 = e−x arccotex + = e−x
arccotex
+ − 1 −
1 + e2x 1 + e2x
2e2x
(ln (1 + e2x ))0 =
1 + e2x
1
(−e−x arccotex + x − ln (1 + e2x ))0 = e−x arccotex
2
Exercise 45.
 
Z r Z Z p
a+x a+x q 1a x  x 2 2
dx = √ dx = ¡ x ¢2 + √a2 − x2 dx = a arcsin a + − a − x
a−x a2 − x2 1− a

Exercise 46.
Z Z p
√ √
x − a b − xdx = bx − ab − x2 + axdx =
Z s µ µ ¶¶ µ µ ¶¶
a+b a+b a2 + b2 2ab
= − x− x− + − =
2 2 4 4
sµ v Ã
Z ¶2 µ µ ¶¶2 µ ¶Z u ¡ ¢ !2
a−b a+b a−b u x − a+b
= − x− = t 1− ¡ a−b2¢ dx =
2 2 2 2
µ ¶2 Z p
a−b
= 1 − u2 =
2
³ ´
µ ¶2 arcsin 2x−(a+b)
a−b a−b 2x − (a + b) p
= + (a − b)2 − (2x − (a + b))2
2 2 2(a − b)2
Since, recall,
µ ¶0 √
arcsin x 1 p 2
1 1 1 − x2 1 x(−2x) p
+ x 1−x = √ + + √ = 1 − x2
2 2 2 1 − x2 2 4 1 − x2
Z
dx
Exercise 47. Wow! p
(x − a)(b − x)

x − a = (b − a) sin2 u
dx = (b − a)(2) sin u cos udu

b − x = (a − b) sin2 u + b − a = (b − a)(cos2 u)
Z Z r
dx (b − a)(2) sin u cos udu x−a
p = √ √ = 2u = 2 arcsin
(x − a)(b − x) b − a cos u b − a sin u b−a
99
6.25 Exercises - Integration by partial fractions, Integrals which can be transformed into integrals of rational func-
tions.
R 2x+3
R³ 1
´ ³
1
´
Exercise 1.
(x−2)(x+5) = x−2 + x+5 = ln (x − 2) + ln (x + 5)
R xdx
Exercise 2.
(x+1)(x+2)(x+3)

A B C
+ + = A(x2 + 5x + 6) + B(x2 + 4x + 3) + C(x2 + 3x + 2)
x+1 x+2 x+3 ¯   ¯
      
1 1 1 A 0 1 1 1 ¯¯0 1 0 ¯¯−1/2
=⇒ 5 4 3 B  = 1 =⇒ 5 4 3 ¯¯1 =  1 0 ¯¯ 2 
6 3 2 C 0 6 3 2 ¯0 0 1 ¯−3/2
A = −1/2, B = 2, C = −3/2
−1 −3
=⇒ ln (x + 1) + 2 ln (x + 2) + ln (x + 3)
2 2
R x
R 2 −1
Exercise 3.
(x−2)(x−1) = x−2 + x−1 = 2 ln x − 2 − ln (x − 1)
R x4 +2x−6
Exercise 4.
x3 +x2 −2x dx

x4 + 2x − 6 3(x2 − 2)
3 2
=x−1+ 3 (do long division)
x + x − 2x x + x2 − 2x
Z Z
3(x2 − 2) 1 x2 − 2
x−1+ = x2 − x + 3
x(x + 2)(x − 1) 2 x(x + 2)(x − 1)
Z Z
x2 − 2 1 1/3 −1/3 1 1
= + + = ln x + ln x + 2 − ln x − 1
x(x + 2)(x − 1) x x+2 x−1 3 3
1 2
=⇒ x − x + 3 ln x + ln x + 2 − ln x − 1
2

R 8x3 +7
Exercise 5.
(x+1)(2x+1)3 dx

8x3 + 7 A B C D
3
= 3
+ 2
+ +
(x + 1)(2x + 1) (2x + 1) (2x + 1) (2x + 1) (x + 1)
8x3 + 7 = A(x + 1) + B(2x2 + 3x + 1) + (4x3 + 8x2 + 5x + 1)C + D(8x3 + 12x2 + 6x + 1)
      
0 0 4 8 A 8 1 0 0 0 12
0 2 8 12 B  0  1 0 0 −6
=⇒     
1 3 5 6   C  = 0 =⇒ 
 | 
1 0 0
1 1 1 1 D 7 0 1 1
A = 12, B = −6, C = 0, D = 1
Z
12 −6 1 −6(2x + 1)−2 6(2x + 1)−1
+ + = + + ln (x + 1)
(2x + 1)3 (2x + 1)2 x+1 2 2

R 4x2 +x+1
Exercise 6.
(x−1)(x2 +x+1)

4x2 + x + 1 A Bx + C
= + =⇒ A(x2 + x + 1) + (Bx + C)(x − 1) = 4x2 + x + 1
(x − 1)(x2 + x + 1) x − 1 x2 + x + 1
    
1 1 A 4
=⇒ 1 −1 1  B  = 1 =⇒ A = 2, B = 2, C = 1
1 0 −1 C 1
Z
2 2x + 1
=⇒ + = 2 ln |x − 1| + ln |x2 + x + 1|
x − 1 x2 + x + 1
R x4 dx
Exercise 7.
x4 +5x2 +4
100
³ ´
x4 5x2 +4
Doing the long division,
x4 +5x2 +4 =1+− (x2 +1)(x2 +4)

Ax + B Cx + D 5x2 + 4
+ =
x2 + 1 x2 + 4 (x2 + 1)(x2 + 4)
It could be seen that A + C = 0, 4A + C = 0 so A = C = 0
−1
B+D =5 B=
3
4B + D = 4 16
D=
3
Z Z
5x2 + 4 −1/3 16/3 1
=⇒ 1 − 2 2
=x− 2
+ 2 = x + arctan x + 4/3 arctan x/2 + C
(x + 1)(x + 4) x +1 x +4 3
R x+2
R 1 2
R 1 1
Exercise 8.
x(x+1) dx = x+1 + x(x+1) = ln |x + 1| + 2 x − x+1 = − ln |x + 1| + 2 ln x
R dx
R
Exercise 9.
x(x2 +1)2 = A Bx+C Dx+E
x + (x2 +1) + (x2 +1)2

A(x4 + 2x2 + 1) + x(Bx + C)(x2 + 1) + Dx2 + Ex A(x4 + 2x2 + 1) + Bx4 + Cx3 + Bx2 + Cx + Dx2 + Ex
=
x(x2 + 1)2 x(x2 + 1)2
=⇒ A = 1; B = −1; D = −1; C = 0; E = 0
Z
1 −x −x ln |x2 + 1| (x2 + 1)−1
+ 2 + 2 = ln x + − +
x x + 1 (x + 1)2 2 2
R dx
Exercise 10.
(x+1)(x+2)2 (x+3)3
R x
Exercise 11.
(x+1)2 dx

x A B x = A(x + 1) + B
= + =⇒
(x + 1)2 x + 1 (x + 1)2 A = 1; B = −1
Z µ ¶
1 −1 1
+ 2
dx = ln x + 1 + +C
x + 1 (x + 1) x+1
R dx
R dx
R
Exercise 12.
x(x2 −1) = x(x−1)(x+1) = A B C
x + x−1 x+1

1
A(x2 − 1) + Bx(x + 1) + Cx(x − 1) = Ax2 − A + Bx2 + Bx + Cx2 − Cx =⇒ A = −1, B = = C
Z 2
−1 1/2 1/2 1 1
+ + = − ln x + ln |x − 1| + ln |x + 1|
x x−1 x+1 2 2
R
x2 dx x2 dx
R
Exercise 13.
x2 +x−6 = (x+3)(x−2)
The easiest way to approach this problem is to notice that this is an improper fraction and to do long division:
x2 6−x
x2 +x−6 = 1 + x2 +x−6

−x = (A + B)x − 2A + 3B
6 = A(x − 2) + B(x + 3) 2A = 3B
A B
+ =⇒ −6 6
x+3 x−2 A= ; B= 3B B = −2/5
5 5 A= =⇒
2 A − 3/5
Z
−6/5 6/5 −3/5 −2/5 −9 4
=⇒ 1+ + + + = ln |x + 3| + ln |x − 2| + x + C
x+3 x−2 x+3 x−2 5 5
R R R
Exercise 14.
x+2
(x−2)2 = x−2+4
(x−2)2 = ln |x − 2| + 4
(x−2)2 = ln |x − 2| + −4(x − 2)−1
R dx
Exercise 15.
(x−2)2 (x2 −4x+5)
A B
Consider the denominator with its x2 −4x+5. Usually, we would try a partial fraction form such as x−2 + (x−2)2 + x2Cx+D
−4x+5 ,
but the algebra will get messy. Instead, it helps to be clever here.
101
1 1 1 1
= = −
(x − 2)2 (x2 − 4x + 4 + 1) (x − 2)2 ((x − 2)2 + 1) (x − 2)2 (x − 2)2 + 1
Z Z
dx 1 1
=⇒ = − = −(x − 2)−1 − arctan (x − 2) + C
(x − 2)2 (x2 − 4x + 5) (x − 2)2 (x − 2)2 + 1

R (x−3)dx R (x−3)dx
Exercise 16.
x3 +3x2 +2x = x(x+2)(x+1)

Z Z Z
(x − 3)dx 1 1
= + −3
x(x + 2)(x + 1) (x + 2)(x + 1) x(x + 2)(x + 1)
1 −1 1
= +
(x + 2)(x + 1) x+2 x+1
1 A B C
= + +
x(x + 2)(x + 1) x x+2 x+1

Now to solve for A, B, C in the last expression, it is useful to use Gaussian elimination for this system of three linear
equations:

    
1 1 1 A 0
3 1 2 B  = 0
2 0 0 C 1
 ¯   ¯ 
¯
1 1 1¯ 0 0 1 0¯¯ 1/2
3 1 2¯ 0 = 0 0 1¯ −1 
¯ ¯
2 ¯1 1 0 0¯ 1/2
1 1/2 1/2 −1
=⇒ = + +
x(x + 2)(x + 1) x x+2 x+1
=⇒ − ln |x + 2| + ln |x + 1| + −3/2 ln x + −3/2 ln |x + 2| + 3 ln |x + 1| = −5/2 ln |x + 2| + 4 ln |x + 1| − 3/2 ln x

R 1
Exercise 17. Use partial fraction method to integrate
(x2 −1)2 . Then build the sum.

A B C D
+ + +
(x − 1)2 (x + 1)2 (x − 1) (x + 1)
(x2 − 1)(x − 1) = x3 − x2 − x + 1
Now
(x2 − 1)(x + 1) = x3 + x2 − x − 1
=⇒ (x2 − 1)(x + 1) − (x2 − 1)(x − 1) = 2x2 − 2
x2 + 2x + 1
x2 − 2x + 1
=⇒ (summing the above two expressions we obtain) 2x2 + 2
−1 1/4 1/4 1/4
+ + +
x − 1 x + 1 (x − 1)2 (x + 1)2
Z ¯ ¯ µ ¶
dx 1 ¯¯ x + 1 ¯¯ 1 x
= ln ¯ +−
(x2 − 1)2 4 x − 1¯ 2 x2 − 1
102
Exercise 18. Use the method of partial fractions, where we nd that

Z Z Z
(x + 1) x+1 − 23 x − 13 2
dx = dx = + 3 =
x3 − 1 (x − 1)(x2 + x + 1) 2
x +x+1 x−1
1 2
= − ln |x2 + x + 1| + ln |x − 1|
3 3
where we had used the following partial fraction decomposition for the given integrand

Ax + B C x+1
+ = 3
x2 + x + 1 x − 1 x −1
Ax2 + Bx − Ax − B + Cx2 + Cx + C = x + 1
2Ax + B − A + 2Cx + C = 1 (where we used the trick to take the derivative of the above equation)
=⇒ A = −C B − A − A = 1
2 2 1
−B + C = 1; A = ,C= B=−
−3 3 2

R x4 +1
Exercise 19.
x(x2 +1)2
Again, it helps to be clever here.

Z Z Z
x4 + 1 x4 + 2x2 + 1 − 2x2 (x2 + 1)2 −2x
= = + 2 =
x(x2 + 1)2 x(x2 + 1)2 x(x2 + 1)2 (x + 1)2
= ln x + (x2 + 1)−1 + C

R dx
Exercise 20.
x3 (x−2)
Working out the algebra for the partial fractions method, we obtain

µ ¶
1 −1/2 −1/4 −1/8 1/8
= + + +
x3 (x − 2) x3 x2 x x−2

So then
Z
dx 1 1 −1 1
= 2+ + ln x + ln |x − 2| + C
x3 (x
− 2) 4x 4x 8 8

Exercise 21.
Z Z Z Z
1 − x3 x3 − 1 x2 1
=− =− + =
x(x2 + 1) 2
x(x + 1) 2
x +1 2
x(x + 1)
Z µ ¶ Z
1 1 −x
=− 1− 2 + + 2 =
x +1 x x +1
= −x + arctan x + ln x − ln |x2 + 1| + C

Exercise 22.
Z Z µ ¶µ ¶ Z
dx 1 1 1/2 1/2
= = − =
x4 − 1 x2 + 1 x2 − 1 x2 − 1 x2 + 1
Z µ ¶
1 1/2 1/2 1/2
= − − 2 =
2 x−1 x+1 x +1
1 1 1
= ln (x − 1) − ln (x + 1) − arctan x + C
4 4 2

Z
dx
Exercise 23.
x4 + 1

I had to rely on complex numbers.


103
Notice that with complex numbers, you can split up polynomial power sums

π π π π
x4 + 1 = (x2 + i)(x2 − i) = (x + ie(i ))(x − ie(i ))(x + e(i ))(x − e(i )) =
4 4 4 4
3π 3π π π
= (x + e(i ))(x − e(i ))(x + e(i ))(x − e(i ))
4 4 4 4
A B C D 1
+ + + = 4
(x + e(i 3π 4 )) (x − e(i 3π
4 ))
(x + e(i π4 )) (x − e(i π4 )) x +1
3π 3π π π
A(x2 − i)(x − e(i )) + B(x2 − i)(x + e(i )) + C(x2 + i)(x − e(i )) + D(x2 + i)(x + e(i )) = 1
4 4 4 4
x3 : A + B + C + D = 0
3π 3π π π
x2 : −Ae(− ) + Be(i ) − Ce(i ) + De(i ) = 0
do the algebra
−−−−−−−−→ 4 4 4 4
x1 : −iA − iB + iC + iD = 0
π π 3π 3π
x0 : −e(i )A + Be(i ) + C(−e(i )) + D(e(i )) = 1
 4 4     4
4
1 1 1 1 A 0
−e(i 3π ) e(i 3π ) −e(i π ) e(i π )  B  0
=⇒   −i
4 4 4 4   =  
−i i i   C  0
−e(i π4 ) e(i π4 ) −e(i 3π 3π
4 ) e(i 4 ) D 1

To do the complex algebra for the desired Gaussian elimination procedure, I treated the complex numbers as vectors and
added them and rotated them when multiplied.

 ¯   ¯   ¯   ¯ 1 
1 1 1 1 ¯¯ 0 1 1 0 0 ¯¯ 0 1 1 0 0 ¯¯ 1 ¯ -
¯ 41
& - . %¯¯ 0  2 %¯¯ 0  2 %¯¯   ¯ ↑
 = 2- = 2- = - ¯ 4 =
↓ ↓ ↑ ↑ ¯¯ 0 0 0 1 1 ¯¯ 0 0 0 1 1 ¯¯ 0 0 0 1 0¯¯ 14 %
. % & -¯ 1 0 0 2& 2 -¯ 1 0 4 -¯ 1 0 1¯ 1 .
 ¯ 14 
1 ¯ -
¯ 41
 1 ¯ &

= ¯ 41 
1 ¯ %
¯ 4
1¯ 14 .
1 i3π
A= e( )
4 4
1 i3π Z
B = − e( ) 1/4e( i3π −1/4e(i 3π 1/4e(i π4 ) −1/4e(i π4 )
4 4 4 ) 4 )
=⇒ =⇒ + + π +
1 π 3π
x + e(i 4 ) 3π
x − e(i 4 ) x + e(i 4 ) x − e(i π4 )
C = e(i )
4 4
−1 π
D= e(i )
µ ¶µ 4 4 ¶
1 3π 3π 3π 3π π π π π
e(i ) ln (x + e(i )) − e(i ) ln (x − e(i )) + e(i ) ln (x + e(i )) − e(i ) ln (x − e(i ))
4 4 4 4 4 4 4 4 4

After doing some complex algebra,

à ¯ ¯ ¶!
1 ¯ x2 + √2x + 1 ¯ µ
1
¶ µ
1
¯ ¯
=⇒ √ ln ¯ √ ¯ − 2 arctan √ − 2 arctan √
4 2 ¯ x2 − 2x + 1 ¯ 2x − 1 2x + 1

The computation could be done to do the derivative on this, so to check our answer and reobtain the integrand.
Is there a way to solve this without complex numbers?
104
R x2 dx
Exercise 24.
(x2 +2x+2)2

Z Z µZ ¶
x2 dx x2 + 2x + 2 − 2x − 2 1
= = + (x2 + 2x + 2)−1
(x2 + 2x + 2)2 (x2 + 2x + 2)2 x2 + 2x + 2
Z Z
1 1
= = arctan (x + 1)
x2 + 2x + 2 (x + 1)2 + 1
Z
x2 dx 1
= arctan (x + 1) + 2 +C
(x2 + 2x + 2)2 x + 2x + 2

R 4x5 −1
Exercise 25.
(x5 +x+1)2 dx

(−(x5 + x + 1)−1 )0 = (x5 + x + 1)−2 (5x4 + 1) (doesn't work)


5 −1 0
(−x(x + x + 1) ) = (x + x + 1) (5x + x) + −(x5 + x + 1)−1 = (x5 + x + 1)−2 (5x5 + x − x5 − x − 1)
5 −2 5
Z
4x5 − 1
=⇒ dx = −x(x5 + x + 1)−2
(x5 + x + 1)2

R dx
Exercise 26. (good example of the use of half angle substitution )
2 sin x−cos x+5

Z Z µ 1 ¶ Z
dx dx C2 sec2 x2 dx
= 1 = =
2 sin x + − cos x + 5 4SC + −C 2 + S 2 + 5 C2
4T − 1 + T 2 + 5(1 + T 2 )
x
Z Z Z u = tan
sec2 x2 dx sec2 x2 dx 2du 2
= = = (where x )
6T 2 + 4T + 4 6(T + 13 )2 + 10
6(u + 31 )2 + 10 sec2 2
3 3 du = dx
2
Z µ ¶
3 du 1 3(tan x2 + 13 )
= 9(u+ 31 )2
= √ arctan √
5 +1 5 5
5

x
u = tan
R dx 2
Exercise 27. (0 < a < 1) Again, using the half-angle substitution, x ,
1+a cos x sec2 2
du = dx
2

Z Z Z
1 dx 1 dx 1 sec2 x2 dx
1 = 1 = 1 2 x
=
a a + cos x a a
2
+C −S 2 a a sec 2 + 1 − T
2
Z Z Z
1 sec2 x2 dx 1 2du 2 du
= 1 = ¡ ¢ = ³ q ´2 =
a + 1 + T 2 ( a1 − 1) a 1
a +1+u
2 1 −1 1+a 1−a
a a 1 + u 1+a
q Ãr !
1−a
2 arctan 1+a u 2 1−a x
q ==⇒ √ arctan tan
1+a 1−a 1 − a2 1+a 2
1+a
105
R dx
Exercise 28.
1+a cos x Half-angle substitution.

Z Z Z
dx dx sec2 x2 dx
= = x =
1 + a cos x 1 + a(C 2 − S 2 ) sec2 2 + a(1 − T 2 )
u = tan θ/2 = T Z Z Z
µ ¶ 2du du 2 du
2 1 =⇒ = 2 + a(1 − T 2 )
=2 2 + (1 + a)
= a+1 =
du = sec θ/2 dθ 1 + T (1 − a)T 1 − a u 2 − a−1
2
 
Z
2  1 1  q1
= q − q =
1−a u − a−1 a+1
u + a−1 a+1 a+1
2 a−1
r µ ¶Ã r r !
a−1 1 a+1 a+1
= ln (u − ) − ln (u + ) =
a+1 1−a a−1 a−1
  q 
x a+1
−1   tan 2 − a−1
= √ ln q 
a2 − 1 tan + a+1
x
2 a−1

R sin2 x
Exercise 29.
1+sin2 x
dx

ZZ 2 Z
s2 s +1−1 dx
dx = dx = x + −
1+s 2 1+s 2
1 + sin2 x
Z Z Z Z Z Z
dx dx 2dx 2 dx 2 dx 2 sec2 xdx
2 = ¡ ¢ = = cos 2x = ¡ ¢ = ¡ 2¢ =
1 + 1−cos 2x 2 2
1 + sin x 2
3 − cos 2x 3 1− 3 3 1 − c −s3
3 sec2 x − 1−T3

Z Z Z
u = tan x 2 du 2 du du
=⇒ = ¡ 2¢ = 2 = √ =
3 1 + u2 − 1−u 3 + 43 u2 1 + ( 2u)2
du = sec2 xdx 3 3

1 √
= √ arctan 2 tan x
2
Z √
sin2 x 1
=⇒ 2 dx = x −
√ arctan ( 2 tan x)
1 + sin x 2

It seems like for here, when dealing with squares of trig. functions, “step up” to double angle.
R dx
Exercise 30.
a2 sin2 x+b2 cos2 x
(ab 6= 0) Take note, we need not change the angle to half-angle or double-angle.

1 1 1 1 sec2 sec2
= 2 = 2 = 2 = 2 = 2 =
a2 s2 2
+b c2 2 2
a (1 − c ) + b c2 2 2
a + (b − a )c2 2
a (1 + (kc) ) 2 2
a (sec +k ) a (1 + T 2 + k 2 )
√ µ ¶
u = tan x du 1/a2 du 1 + k2 u
=⇒ = = u2
= arctan √
a2 (1 + u2 + k 2 ) (1 + k 2 )(1 + 1+k 2)
a2 (1 + k 2 ) 1 + k2
du = sec2 xdx
Z µ ¶
dx 1 a tan x
=⇒ 2 = arctan
2 2 2
a sin x + b cos x ab b

R dx
Exercise 31.
(a sin x+b cos x)2 (a 6= 0)

Note it's a good idea to simplify, cleverly, your constants as much as you can.

Z Z
dx 1 dx
= 2
(a sin x + b cos x)2 a (sin x + k cos x)2
106
Thus, only one constant, k , is only worried about.
1 1 1/c2 sec2
2
= 2 2 2
= 2 2
= =
(s + kc) s + 2ksc + k c t + 2kt + k (t + k)2

u = tan x du
=⇒ =
(u + k)2
du = sec2 xdx
Z
1 1 −1
=⇒ = 2
a2 (s + ab c)2 (a tan x + ab)

Again, note, we need not always step up or step down a half angle in the substitution.

Exercise 32. Note that we have a rational expression consisting of single powers of sin and cos. Then use the tan θ2 substitution.
Z Z Z Z
sin x 2CS CS T
= = =
1 + cos x + sin x 1 + 2CS + C 2 − S 2 C(S + C) (T + 1)
u = tan θ/2
C = cos x/2
where 2
S = sin x/2 du = sec θ/2dθ
Z µ ¶ Z 2
u 2du 2udu
=
(u + 1) u2 + 1 (u2 + 1)(u + 1)
A Bu + C
+ 2
u+1 u +1
Au2 + A + Bu2 + Cu + Bu + C = u
1 1 1
A = −B C + B = 1 A + C = 0 =⇒ C = ; B = ; A = −
2 2 2
Z µ 1 ¶ Z
−1/2 (u + 1) −1 u+1
2 + 2 2 du = +
u+1 u +1 u + 1 u2 + 1
1
= − ln |u + 1| + ln |u2 + 1| + arctan u =
2
1 x
= − ln | tan x/2 + 1| + ln | sec2 x/2| +
2 2
1 π 1 π
=⇒ − (ln |2|) + ln |2| + = − ln |2| +
2 4 2 4
R√
Exercise 33. 3 − x2 dx
Z p p Z p Z p Z p Z
x(−x) −x2 + 3 − 3 1
(x)0 3 − x2 dx = x 3 − x2 − √ = x 3 − x2 − √ = x 3 − x2 − 3 − x2 + 3 √
3−x 2 3−x 2 3 − x2
Z p p √ Z
1
=⇒ 2 3 − x2 = x 3 − x2 + 3 r ³ ´2
1 − √x3
Z p
xp 3 x
=⇒ 3 − x2 = 3 − x2 + arcsin √
2 2 3
R
Exercise 34. √ 1 dx = −(3 − x2 )1/2 + C .
3−x2
µ ¶0
x 1 −1 1
arccos √ =√ q = −√
3 3 1− x 2 3 − x2
3
³ p ´0 p −x2
x 3 − x2 = 3 − x2 + √
3 − x2

x 3 − x2 3 x
+ − arccos √
2 2 3
107
R √
3−x2
Rq 3
Exercise 35.
x dx = x2 − 1dx.

3
= sec θ
√ x
3 cos θ = x

dx = − sin θ 3
Z p √ Z √ √ Z
2
sec θ − 1(− sin θ) 3 = tan θ sin θ(− 3) = − 3 (sec θ − cos θ) =
√ √
= − 3 ln | sec θ + tan θ| + 3 sin θ =
¯√ ¯
√ ¯ 3 r3 ¯ √ r x2
¯ ¯
= − 3 ln ¯ + − 1 ¯ + 3 1 −
¯ x x2 ¯ 3

Rq
Exercise 36. 1 + x1 dx
à r !0 r µ ¶ r
1 1 x −1 1 −1/2
= 1+ + q
x 1+ = 1+ + √
x x 2 1+ 1 x x x2 + x
x
µ µ ¶¶0 µ ¶
1 p 1 x + 12 1
ln x + + x2 + x = 1
√ 1 + √ =√
2 x+ 2 + x +x2 2
x +x 2
x +x
Z µ ¶ r µ ¶
1 1 1 1 p
=⇒ 1+ dx = x 1 + + ln x + + x2 + x
x x 2 2

Exercise 37.
p p x2
(x x2 + 6)0 = x2 + 5 + √
x2 + 5
p µ ¶µ ¶
2 0 1 x 1
(ln (x + x + b)) = √ 1+ √ =√
2
x+ x +b 2
x +b 2
x +b
Z p ³ p p ´
1
x2 + 5 = x x2 + 5 + 5 ln (x + x2 + 5)
2
Exercise 38.
µ ¶0 µ ¶
1 p 1 x + 12 1
ln (x + + x2 + x + 1) = √ 1 + √ =√
2 x + 12 + x2 + x + 1 x2 + x + 1 x2 + x + 1
Z Z
x x + 12 − 12 1 1 p
√ = √ = (x2 + x + 1)1/2 − ln (x + + x2 + x + 1)
2
x +x+1 2
x +x+1 2 2
The trick is to note how I formed a “conjugate-able” sum from x2 + x + 1's derivative.
Exercise 39.
Z Z Z
dx dx 2dx
√ = q¡ ¢ q¡ ¡= ¢¢2
x2 + x x + 2 − 14 1 2
2 x + 12 −1
³ ³p ´´0
ln (2(x + 1/2))2 − 1 + 2(x + 1/2) =
à !
1 2(x + 1/2)2
=p 2+ p =
(2(x + 1/2))2 − 1 + 2(x + 1/2) (2(x + 1/2))2 − 1
2
=p
(2(x + 1/2))2 − 1
Z µ µ ¶ ¶
dx 1 p
q¡ = ln 2 x + 2
+ (2(x + 1/2)) − 1 + C
¢2 2
x + 21 − 14

Exercise 40.

108
6.26 Miscellaneous review exercises. Exercise 1.
Z x
log t
f (x) =
1 t+1
µ ¶ Z 1 Z x µ ¶
1 x log t − ln (u) −1
f = dt = 1 du =
x 1 t+1 1 u +1
u2
Z x 1
ln (u) u=
= du t
1 u+u
2 1
du = − 2 dt
t
µ ¶ Z x Z x ¯x
1 t ln t + ln t ln t (ln t)2 ¯¯ (ln x)2
f (x) + f = dt = dt = ¯ =
x 1 t(t + 1) 1 t 2 1 2
µ ¶
1 1
f (2) + f = (ln 2)2
2 2

Exercise 2. Take the derivative of both sides, using the (rst) fundamental theorem of calculus.

sin x sin x
2f f 0 = f (x) ; =⇒ 2f 0 =
2 + cos x 2 + cos x
At this point, it could be very easy to evaluate the integral by guessing at the solution.

sin x ln |2 + cos x|
(− ln 2 + cos x)0 = =⇒ f = − +C
2 + cos x 2
Otherwise, remember that for rational expressions involving single powers of sin and cos, we can make a u = tan θ/2
substitution.
x
u = tan
2 C = cos x/2, S = sin x/2
2 x
2du = sec dx
Z Z 2 Z µ ¶
sin x 2SCdx 2T 2du
dx = = =
2 + cos x 2 + C2 − S2 2 sec2 x/2 + 1 − T 2 sec2 x/2
Z Z
udu T T
=4 2 2
=2 2
− 2
(1 + u )(3 + u ) T +1 T +3
µ ¶ µ µ 2 ¶¶ µ ¶
1 2 1 2 T +1 2
=2 ln T + 1 − ln T + 3 = ln = ln
2 2 T2 + 3 4 + 2 cos x
x sin2 x2 1 − cos x
where tan2 = =
2 cos2 x2 1 + cos x

Exercise 3.
Z Z Z
ex ex (x − 1) ex (x − 1)
dx = ex − xdx = ex − dx . . .
x x2 x
No way.
R π/2 3/2
Exercise 4.
0
ln (ecos x )dx = − cos x|0 = 1 .

Exercise 5.

(1)

f = 4x + 2x(x + 1)(x + 2)
µ ¶
1 4(x + 2) 1
ln f = ln = (ln (4x + 2) − ln x − ln (x + 1) − ln (x + 2))
2 x(x + 1)(x + 2) 2
µµ ¶ ¶
f0 1 4 1 1 1 7
= − − − f 0 (1) = −
f 2 4x + 2 x x+1 x+2 12
s µµ ¶ ¶
1 4x + 2 4 1 1 1
=⇒ f 0 = − − −
2 x(x + 1)(x + 2) 4x + 2 x x+1 x+2
109
(2)
Z 4 Z 4
4x + 2 (2x + 1)
π dx = 2π dx =
1 x(x + 2)(x + 1) 1 x(x + 2)(x + 1)
µ ¶¯4
1 3 ¯ 25
= 2π ln x + − ln |x + 2| + ln |x + 1| ¯¯ = πln
2 2 1 8
since we can nd the antiderivative through partial fractions:

A B C 2x + 1
+ + =
x x+2 x+1 x(x + 2)(x + 1)
A(x2 + 3x + 2) + B(x2 + x) + C(x2 + 2x) = 2x + 1
      ¯ 
1 1 1 A 0 0 1 0¯¯ −3/2
3 1 2 B  = 2 =⇒ 0 0 1¯ 1 
¯
2 0 0 C 1 1 0 0¯ 1/2

Exercise 6.

(1)
Z x Zif x > 0
x
1 et
log x = dt F (x) = dt; t
1 t 1 t e > 1 for t > 0
If 0 < x < 1
Z x Z 1 Z 1 t
1 −1 e
log x = dt = dt > − = F (x)
1 t x t x t

log x ≤ F (x) for x ≥ 1


(2)
Z x+a Z a+1 Z x Z 1
et dt et dt et+a dt et+a dt
F (x + a) − F (1 + a) = − = −
1 t 1 t 1−a t+a 1−a t+a
Z x
et
= ea dt
1 t+a
(3)
Z x Z ax Z ax Z 1
eat et et et
dt = dt = + =
1 t a t 1 t a t
= F (ax) − F (a)
x t Z Z µ x ¶
e 1 t 1 t e
2
=− e − − e = − − e + F (x)
1 t t t x
Z x Z 1/x µ u Z ¶
1/t −eu −e eu
e dt = du = − − − =
1 1 u2 u u
1
u=
= xe 1/x
− e − F (1/x) where we used the substitution
t
1
du = − dt
t2
Exercise 7.

(1)

ex = F (x) − F (0); F (x) = ex + F (0) =⇒ F (0) = 1 + F (0)


0 6= 1. False.
(2)
Z x2
d 2
f (t)dt = f (x2 )(2x) = −(2x) ln 2ex ln 2
f (x) = − ln 2ex ln 2
dx 0
Z x2 ¯x 2 2
− ln 2eln 2 dt = − et ln 2 ¯0 = −ex ln 2 + 1
0
110
(3)
x
f (x) = 2f (x)f 0 (x); =⇒ f (x) = + C
2
Z xµ ¶ µ 2 ¶¯x
1 t ¯ x2
t + c dt = + ct ¯¯ = + cx
0 2 4 0 4
x2
f 2 (x) − 1 = + Cx + C 2 − 1
4
x
=⇒ C = ±1, f (x) = + ±1
2
Exercise 8.

(1)
f (x + h) − f (x) f (x)f (h) − f (x) f (x)(hg(h))
= = = f (x)g(h)
h h h
g(h) → 1 as h → 0 so =⇒ f 0 (x) = f (x)
x x 0
(2) Since for f (x) = e , we dened e such that f = f , if

(ex + g)0 = ex + g 0 = ex + g
=⇒ ex + g = Cex =⇒ g = (C − 1)ex but f 0 (0) = 1 so g = ex
Exercise 9.

(1)
g(2x) = 2ex g(x)
g(3x) = ex g(2x) + e2x g(x) = ex 2ex g + e2x g = 3e2x g
(2)
Assume g(nx) = ne(n−1)x g
g((n + 1)x) = ex g(nx) + e(n+1)x g(x) = nenx g(x) + enx g = (n + 1)enx g
y x
(3) From g(x + y) = e g(x) + e g(y),

g(0) = g(0) + g(0) =⇒ g(0) = 0


µ h ¶
g(x + h) − g(x) eh g(x) + ex g(h) − g(x) e −1 ex g(h)
= = g(x) +
h h h h
g(x + h) − g(x) g(h)
f 0 (0) = 2 = lim = lim h → 0
h→0 h h
0 x
(4) g (x) = g(x) + 2e C = 2
Exercise 10.
∀x ∈ R, f (x + a) = bf (x); f (x + 2a) = bf (x + a) = b2 f (x)
f (x + (n + 1)a) = f (x + na + a) = bf (x + na) = bn+1 f (x)
f (x + na) = bn f (x)

f (x) = bx/a g(x) where g is periodic in a


Exercise 11.
f0 g0
(ln (f g))0 = + =⇒ (f g)0 = f 0 g + f g 0
f g
µ µ ¶¶0 µ ¶0
f f0 g0 f 0 g − g0 f f
ln = − =⇒ =
g f g g2 g
R1 et
Exercise 12. A= 0 t+1
dt
(1)
Z a
e−t
dt
a−1 t − a − 1
Z 0 Z 0 t−a Z 1 t
e−t−a e e
u=t−a =⇒ dt = − dt = −e−a = −e−a A
−1 t−1 1 −t − 1 0 t+1
R1 tet2 R1 1
2 due
u 1
(2)
0 t2 +1
dt = 0 u+1 = A
2
111
R1 ¯1 R1
−e1
et −et ¯ −et
(3)
0 (t+1)2
dt = (t+1) ¯ − +1+A
0 t+1
=
0 2
R1 t R et
(4)
0
e ln (1 + t)dt = et ln (1 + t) − 1+t = e ln 2 − A
Exercise 13.

(1) p(x) = c0 + c1 x + c2 x2 ; f (x) = ex p(x) p0 = c1 + 2c2 x p00 = 2C2


0 x 0
f =f +e p
Xn µ ¶ µ ¶ µ ¶
n x n! n!
f (n) (x) = e (p(x))j = f + ex (c1 + 2c2 x) + + ex (2c2 )
j=0
j (n − 1)! (n − 2)!2!

f (n) (0) = c0 + c1 n + n(n − 1)c2


(2) See generalization below.
(3)
m
X m
X j!
p= aj xj ; p(0) = a0 ; p(k) (x) = aj xj−k = p(k) (0) = ak k!
j=0 j=0
(j − k)!
Xn µ ¶
(n) n x (j)
f (x) = e p (x)
j=0
j

Xn µ ¶ Xm µ ¶ Xm
(n) n (j) n n!
f (0) = p (0) = aj j! = aj
j=0
j j=0
j j=0
(n − j)!

So for m = 3, then f (n) (0) = a0 + na1 + n(n − 1)a2 + n(n − 1)(n − 2)a3
Exercise 14. f (x) = x sin ax ; f (2) = −a2 x sin ax + 2a cos ax
f (2n) (x) = (−1)n (a2n x sin ax − 2na2n−1 cos ax)
f (2n+1) (x) = (−1)n (a2n+1 x cos ax + a2n sin ax + 2na2n sin ax)
f (2n+2) (x) = (−1)n (−a2n+2 x sin ax + a2n+1 cos ax(2n + 2))
Exercise 15.
n
X µ ¶ Xn µ ¶Z 1 Z 1X n µ ¶
nk 1 k n k+m k n
(−1) = (−1) t dt = (−1) tk+m dt =
k k+m+1 k 0 0 k
k=0 k=0 k=0
Z 1 Xn µ ¶ Z 1
n
= tm (−t)k dt = tm (1 − t)n dt =
0 k 0
k=0
Z 0 Z 1
=− (1 − u)m un du = (1 − u)m un du =
1 0
1XZ µ ¶ Xm µ ¶Z 1
m m
= 6m (−u)j un du = (−1)j tj+n dt
0 j=0 j j=0
j 0

u1 − t
du = −dt
Rx
Exercise 16. F (x) = 0
f (t)dt
(1)
Z (R x
x
2 (2t)2 dt = 34 x3 if t, x ≥ 0
F (x) = (t + |t|) = R0x
0 0
0dt = 0 if t, x < 0

(2)
Z (R x
x
(1 − t2 )dt if |t| ≤ 1
F (x) = f (t)dt = R0x =
0 0
(1 − |t|)dt if |t| > 1
¡ ¢¯
1 3 ¯x x3
(t − 3Rt 0 = x − 3
 if |x| ≤ 1
2 x x2 1
= 3 + 1R (1 − t)dt = x − 2 + 6 if x>1

 −2 x x 2
1
if |x| ≥ 1
3 + −1 (1 + t)dt = x + 2 − 6 if x < 1

112
(3) f (t) = e−|t|
Z x Z x
F (x) = f (t)dt = e−|t| dt =
(0R x 0
−t −t 0
e dt = e |x = 1 − e−x if x≥0
= R0x t x
0
e dt = et |0 = ex − 1 if x < 0

(4) f (t) = max. of 1 and t2


R x
Z x 
 0 1dt =x if |x| ≤ 1
Rx
F (x) = f (t)dt = 1 + 1 t2 dt if x>1 =
0 
 Rx
− 0 f if x < −1


x if |x| ≤ 1
¯x 3
= 1 + 13 t3 ¯1 = x3 + 23 if x > 1

 R −1 2 R0 ¯
1 3 ¯x x3 2
− x t + − −1 1 = 3 t −1 −1= 3 + 3 if x > 1

R Ra
Exercise 17. πf 2 = π 0
f 2 = a2 + a.
µ ¶0 r
x2 + x 2x + 1
=
π π
Z a
2x + 1 ¡ ¢¯a
for = x2 + x ¯0 = a2 + a
0 π

Exercise 18. f (x) = e−2x .


Rt ¯
−2x ¯t −2t
(1) A(t) = 0 e−2x dx = e−2 ¯ = e −2−1
Rt 0
π
(2) V (t) = π 0 e−4x dx = −4 (e−4t − 1)
(3)

ln y
y = e−2x =⇒ =x
−2
Z 1 µ ¶2
ln y π ¯1
W (t) = π dy = (y(ln y)2 − (2(y ln y − y)))¯e−2t =
e−2t −2 4
π¡ ¡ −2t 2 ¢¢
= 2 − e 4t − (2(e−2t (−2t) − e−2t )) =
³4 π π ´
= − πte−2t − e−2t
2 2
2 0 2
where the antiderivative used was (y(ln y) ) = (ln y) + 2 ln y

(4)
³ ´
π e−4t −1
4³(1 − e−4t ) π t
´ = e−2t −1
= π
1−e−2t 2 t
2
ecx − 1
where we used the limit lim =c
x→0 x
3
Exercise 19. sinh c = 4
(1)
p
ec = ex + e2x + 1

ec − e−c ex + e2x + 1 − ex +√1e2x +1
= =
2 √ 2
e2x + 2ex e2x + 1 + e2x + 1 − 1 3
= √ = ex =
x 2x
2(e + e + 1) 4
x = ln 3 − 2 ln 2
113
(2)

ec − e−c ex − e2x − 1 − ex −√1e2x −1
= =
2 √ 2
e2x − 2ex e2x − 1 + e2x − 1 − 1
= √
2(ex − e2x − 1)
ex −1 1 3
= √ = ex − c =
+ e − e −1
x 2x e 4
=⇒ x = ln 5 − 2 ln 2
Exercise 20.

(1) True. ln (2log 5 ) = ln 5ln 2 = (ln 2) ln 5.


log2 5
(2) log3 5 = log 3 This is a true fact.
2
log3 5 log2 5
= = log2 5
log2 3 (log2 3)2
=⇒ 1 = log2 3 False
(3) Use induction

n = 1 1−1/2 < 2 1
1 √
n = 21 + √ < 2 2
2
n+1
X Xn
1
k −1/2 = k −1/2 + p <
k=1 k=1
(n + 1)
n + 1 case Ãp !
√ (n + 1) 1
<2 n p +p
(n + 1) (n + 1)
Now (n + 12 )2 = n2 + n + 1
2 > n2 + n, certainly. So then
1 p p
n + > n2 + n =⇒ n + 1 > n2 + n
2
n+1
X √
=⇒ k −1/2 < 2 n + 1
k=1

(4)
ex + e−x − ex + e−x
f = (cosh x − sinh x − 1) = − 1 = e−x − 1 < 0 for x > 0
2
False.
π
Exercise 21. For 0<x< 2,
π
(sin x)0 = cos x > 0 for 0 < x <
2
π
(x − sin x)0 = 1 − cos x ≥ 0 for 0 < x <
2
(x − sin x)(x = 0) = 0 =⇒ sin x < x
Exercise 22. µ ¶
1 1 x+1
< if 0<x<t<x+1
t t t
Z x+1 Z x+1
1 x+1 1 x+1 1
dt = ln (x + 1) − ln x; = So ln <
x t x t x x x
Exercise 23.

(x − sin x)0 = 1 − cos x ≥ 0 ∀x > 0


since (x − sin x)(x = 0) = 0; (x − sin x)0 (x = 0) = 0, then x − sin x > 0 in general for x > 0
µ ¶
x3 0 x2
(sin x − x − ) = cos x − 1 + > 0 ∀x > 0
6 2
x3
=⇒ x − < sin x < x
6
114
Exercise 24. (xb + y b )1/b < (xa + y a )1/a if x > 0, y > 0 and 0 < a < b
¡ ¢n
(xn + y n )1/n = x(1 + xy )1/n . Without loss of generality, assume x < y .
Consider (1 + An )1/n , A constant.
µ ¶ µ µ ¶ ¶
1 −1 1 1
((1 + An )1/n )0 = (exp ln (1 + An ) )0 = (1 + An )1/n ln (1 + A n
) + (ln A)A n
=
n n2 n 1 + An
µ ¶
−(1 + An ) ln (1 + An ) + n(ln A)An
= (1 + An )1/n =
n2 (1 + An )
µ ¶
n 1/n − ln (1 + An ) − An ln (1 + An ) + An ln An
= (1 + A ) =
n2 (1 + An )
 ³ ´ 
n n An µ ¶
(1 + An ) n  − ln (1 + A ) + A ln 1+An 
1−n
An
= < 0 since ln <0
n2 n2 (1 + An ) 1 + An

=⇒ (xb + y b )1/b < (xa + y a )1/a if b>a


Exercise 25.

(1)
Z x ¡ ¢¯x
e−t t = −te−t − e−t ¯0 = −xe−x − e−x + 1
0
(2)
Z t Z Z
¯x
t2 e−t dt = −t2 e−t ¯0 − −e−t (2t)dt = −x2 e−x + 2 te−t dt =
0
= −x2 e−x + −2xe−x − 2e−x + 2
(3)
Z x Z x Z x
¯x
t3 e−t dt = −t3 e−t ¯0 − 3 t2 (−e−t )dt = −x3 e−x + 3 t2 e−t dt =
0 0 0
µ ¶
3 −x −x x x2
= −x e + 3(2)(e ) e − 1 − x −
2!
(4) Assume the induction hypothesis, that
 
Z x n
X xj
tn e−t dt = n!e−x ex − 
0 j=0
j!
 
Z x Z n
X
¯x xj
tn+1 e−t dt = −tn+1 e−t ¯0 − (n + 1)t(−e−t ) = −xn+1 e−x + (n + 1)n!e−x ex − 
0 j=0
j!
 
n+1
X xj 
= (n + 1)!e−x ex −
j=0
j!

Exercise 26. Consider the hint a1 sin x + b cos x = A(a sin x + b cos x) + B(a cos x − b sin x). Solve for A, B in terms of
a1 , b1 , a, b. Matching up term by term the coefcients for sin and cos separately,
Aa2 − abB = aa1 −Aab + Bb2 = −a1 b
Ab2 + Bab = b1 b Aab + Ba2 = ab1
aa1 + bb1 ab1 − a1 b
A= B=
a2 + b2 a2 + b2
So if not both a, b = 0 ,
Z Z
a1 sin x + b1 cos x A(a sin x + b cos x) + B(a cos x − b sin x)
= =
a sin x + b cos x a sin x + b cos x
= Ax + B ln |a sin x + b cos x| + C

Exercise 27.

115
(1)
1
f 0 (x2 ) =
x
df
= u−1/2
du
f (x2 ) = 2x − 1
(2)

1 x2 1
f 0 (sin2 x) = 1 − sin2 x f 0 (u) = 1 − u f = u − u2 + C =⇒ f (x) = x − +
2 2 2
(3)

1 x3 1
f 0 (sin X) = (1 − sin2 x) f (u) = u − u3 + C f (x) = x − +
3 3 3
(4)
( (
0 1 for x≤1 1 0<x≤1
for
f (ln x) = =
x for x > 1 eln x x>1
(
y for y<0
f (y) =
ey − 1 for y>0

Exercise 28.

(1)
x Z
dt
Li(x) = if x ≥ 2
ln t
Z x 2 Z x
x 1 −1 x dt 2
Li(x) = −2 − 2
dt = + 2

ln x ln 2 2 (ln x) ln x 2 (ln x) ln 2
(2)

Z x X2
x 2 2 x −2 −2 2(j − 1)!
Li(x) = − − + − dt C2 = +−
ln x ln x (ln 2) 2 (ln x) 2
a (ln t)
3 ln x j=2
(ln 2)j
n−1
X k!x µ Z x ¶ n
x x −(n + 1)dt 1 X 2(j − 1)!
Li(x) = + + n! − Cn = −2 −
ln x
k=1
lnk+1 x lnn+1 x 2 lnn+2 t ln 2 j=2 (ln 2)j
Xn Z x
x k!x dt 1
n+1
X 2(j − 1)!
Li(x) = + k+1
+ (n + 1)! (n+1)+1 Cn+1 = −2 −
ln x ln x 2 ln t ln 2 j=2 (ln 2)j
k=1

(3)
u = ln t
Z x
dt 1
Li(x) = du = dt
2 ln t t
eu du = dt eu = t
Z ln x
et dt
Li(x) =
ln 2 t
(4)
1
c=1+ ln 2
2
Z x−1 Z x−1 2u
e2(u+1) du e
= e2 du =
c−1 u c−1 u
Z 2(x−1)
t = u + 1 =⇒ 21
3 et
=e t dt =
2 2(c−1)
3 2

= e2 Li(e2(x−1) )
116
(5)

f (x) = e4 Li(e2(x−2) ) −e2 Li(e2(x−1) ) µ ¶


Z x 2t Z x 0 e2x e2x 2x 1
e e2t =⇒ f (x) = t − 2 + − t − 1 = e t2 − 3t + 2
= −
c t−2 c (t − 1)

1
Exercise 29. f (x) = log |x| if x < 0. ∀x < 0 ∃ uniquely ln |x| since f 0 = x < 0 ∀x < 0.

−ey = x(y) = g(y) D=R


Recall Theorem 3.10.

Theorem 21. Assume f is strictly increasing and continuous on an interval [a, b]. Let c = f (a), d = f (b) and let −g be the
inverse of f.
That is ∀y ∈ [c, d], Let g(y) be that x ∈ [a, b], such that y = f (x).

Then

−g is strictly increasing on [c, d]


(1)
−g is continuous on [c, d]
(2)

Rx
Exercise 30. f (x) =
0
(1 + t3 )−1/2 dt if x ≥ 0.
(1)
1
f 0 (x) = √ > 0 for x > 0
1 + x3
(2)
1 p 3x2
g 0 (x) = = 1 + x3 g 00 (x) = √
f 0 (x) 2 1 + x3
7.4 Exercises - Introduction, The Taylor polynomials generated by a function, Calculus of Taylor polynomials. Use
the following theorems for the following exercises.

Theorem 22 (Properties of Taylor polynomials, Apostol Vol. 1. Theorem 7.2.). (1) Linearity Tn (c1 f +c2 g) = c1 Tn (f )+
c2 Tn (g)
(2) Differentiation (Tn f )0 R= Tn−1 (f 0 )
x
(3) Integration. If g(x) = f (t)dt
Rx a
Tn+1 g(x) = a Tn f (t)dt
Theorem 23 ( Substitution Property, Apostol Vol. 1. Theorem 7.3. ). Let g(x) = f (cx), c is a constant.
(12) Tn g(x; a) = Tn f (cx; ca)
This theorem is useful for nding new Taylor polynomials without having to nd the j th derivatives of the desired function.
Theorem 24. Pn is a polynomial of degree n ≥ 1.
Let f, g be 2 functions with derivatives of order n at 0.
(13) f (x) = Pn (x) + xn g(x)
where g(x) 7→ 0 as x 7→ 0.
Then Pn = Tn (f, x = 0).

Exercise 3.
n
X f (j) (a)
Tn f (x) = (x − a)j
j=0
j!
x x ln a
a =e
(a ) = (ax ) ln a
x 0

(ax )(n+1) = (ax (ln a)n )0 = ax (ln a)n+1


n
X
x (ln a)j
Tn (a ) = xj
j=0
j!
117
Exercise 4.
µ ¶0 µ ¶00
1 −1 1 (−1)2 2
= ; =
1+x (1 + x)2 1+x (1 + x)3
µ ¶(n+1) µ ¶0
1 (−1)n n! (−1)n+1 (n + 1)!
= =
1+x (1 + x)n+1 (1 + x)n+2
µ ¶ X n
1
Tn = (−1)j xj
1+x j=0

Exercise 5. Use Theorem 7.4. . Theorem 7.4 says forf (x) = Pn (x) + xn g(x), Pn (x) is the Taylor polynomial.
   
Xn 2 n+1 X n
1 (x ) xn xn+2
2
=  (x2 )j  + 2
= x2j  +
1−x j=0
1−x j=0
1 − x2
 
Xn
x  2j+1  (x2n+3 )
= x +
1 − x2 j=0
1 − x2
µ ¶ n
X
x
T2n+1 = x2j+1
1 − x2 j=0

Exercise 6.
µ ¶ n
X n
X X n
0 1 1 j xj+1
j xj
(ln (1 + x)) = Tn = (−x) Tn (ln 1 + x) = (−1) = (−1)j+1
1+x 1+x j=0 j=0
j + 1 j=1 j

Exercise 7.
à r !0 r r
1+x 1−x 1−x 1 1 1
log = = =
1−x 1 + x 1 + x (1 − x)2 (1 + x)(1 − x) 1 − x2
Z r Z Xn n
à r !
1 1+x X x2j+1 1+x
2j
= log so x = = T2n+1 ln
1 − x2 1−x j=0 j=0
2j + 1 1−x

Exercise 8.
à !  
µ ¶ µ ¶ Xn Xn
1 1/2 1 1 1 1 xj
Tn = Tn = Tn ¡1¢ =  ( x)j  =
2−x 1 − x/2 2 1− 2 x 2 j=0 2 j=0
2j+1

Exercise 9. We can show this in two ways.

We could write out the actual polynomial expansion.

Xn µ ¶ n µ ¶
α α−j j X α j
(1 + x)α = 1 x = x
j=0
j j=0
j

or determine each of the coefcients of the Taylor polynomial.

((1 + x)α )0 = α(1 + x)α−1 ; ((1 + x)α )00 = α(α − 1)(1 + x)α−2
µ ¶0
α! α!
((1 + x)α )(n+1) = (1 + x)α−n = (1 + x)α−(n+1)
(α − n)! (α − (n + 1))!

Exercise 10. Use the substitution theorem, Apostol Vol.1. Thm. 7.3., to treat cos 2x.
n
X Xn
(−1)j x2j (−1)j (2x)2j
T2n (cos x) = ; T2n (cos 2x) =
j=0
(2j)! j=0
(2j)!
 
Xn n
1 1 (−1)j (2x)2j  X (−1)j+1 22j−1 x2j
T2n (sin x2 ) = T2n ( (1 − cos 2x)) = 1 − =
2 2 j=0
(2j)! j=1
(2j)!

118
7.8 Exercises - Taylor's formula with remainder, Estimates for the error in Taylor's formula, Other forms of the
remainder in Taylor's formula. We will use Theorem 7.7, which we learn in the preceding sections, extensively.

Theorem 25. If for j = 1, . . . , n + 1, m ≤ f (j) (t) ≤< ∀t ∈ I, I containing a,


n+1 n+1
(x − a) (x − a)
(14) m ≤ En (x) ≤ M if x>a
(n + 1)! (n + 1)!
n+1
(a − x) (a − x)n+1
(15) m ≤ (−1)n+1 En (x) ≤ M if x<a
(n + 1)! (n + 1)!
Z x
1
(16) En (x) = (x − t)n f (n+1) (t)dt
n! a

Exercise 1. For a = 0, | sin(j) (x)| ≤ 1 for ∀x ∈ R.


(x)2n+1 (−x)2n+1
En (x) ≤ if x > 0; (−1)2n+1 E2n (x) ≤ (+1)
(2n + 1)! (2n + 1)!
|x|2n+1
=⇒ E2n (x)| ≤
(2n + 1)!

Exercise 2.
n
X (−1)k x2k
cos x = + E2n+1 (x) | cos(j) (x)| ≤ 1
(2k)!
k=0
2n+2
x (−x)2n+2
E2n+1 (x) ≤ ; (−1)2n+2 E2n+1 (x) ≤ (1)
(2n + 2)! (2n + 2)
2n+2
|x|
=⇒ |E2n+1 (x)| ≤
(2n + 2)!
Exercise 3.
n−1
X (−1)k x2k+1
arctan x = + E2n (x)
2k + 1
k=0
n−1
X k 2k+1
µ ¶ n−1
X
(−1) x (2k)! (−1)k (2k)!x2k+1
= =⇒ f (2k+1) (0) = (−1)k (2k)!
2k + 1 (2k)! (2k + 1)!
k=0 k=0
f (2n+1) (0)x2n+1 (−1)n (2n)!x2n+1 (−1)n x2n+1 x2n+1
= = ≤
(2n + 1)! (2n + 1)! 2n + 1 2n + 1
Note how j th derivative (arctan x)(j) changes sign with each differentiation for f (2j+1) (0). Then we can always pick a
(2j+1)
small enough closed interval with a = 0 as a left or right end point to make the f (0) value the biggest for f (2j+1) (t).
Exercise 4.

(1)
x3 x3 x³ ³ √ ´´ ³ √ ´
x2 = sin x = x − =⇒ + x2 − x = x − −3 + 15 x − (−3 − 15)
6 6 6

x = 15 − 3
(2)
Z r 3
1 r5 r 3 1
E4 (r; 0) = (r − t)4 cos tdt > 0 sin r − r2 = 0 + E4 (r) ≤ < = = 4
<
4! 0 5! 5! 5(2)(5)(4)(3)(2)1 (5)(4)(2)5 200
Exercise 5.
r3
arctan r − r2 = r − − r2 + E4 (r; 0) = 0 + E4 (r; 0)
3
Z
1 r M (r5 ) r5 7
E4 (r, 0) = (x − t)4 f (5) (t)dt ≤ = 0.065536 <
4! 0 5! 5! 100
E4 (r, 0) < Ej (r, 0); j > 4
f (5) (0) 5 24 5
the 5th degree term is r = r >0
5! 5!
2
so r − arctan r = −E4 (r, 0) < 0
119
Exercise 6. Apply long division on the fraction in the integrand.
Z 1 Z 1 Z 1 µ ¶
1 + x30 x30 − x60 30 1 − x30
dx = 1+ dx = 1 + x dx =
0 1 + x60 0 1 + x60 0 1 + x60
¯1
1 31 ¯¯ c
=1+c x ¯ =1+
31 0 31
R 1/2 1
Exercise 7.
0 1+x4 dx.
X∞ n−1
X
1 4 j
4
= (−x ) = (−x4 )j + En = 1 + −x4 + x8 . . .
1+x j=0 j=0

16 x4n+1 1
≤ En (x; 0) ≤ (x4 )n+1
17 (n + 1)! (n + 1)!
Z 1/2 ¡ 1 ¢4n+5 µ ¶
1
=⇒ En = 2
0 (n + 1)! 4n + 5
Z 1/2 µ ¶5
1 1 −1 1
' +
0 1 + x4 2 5 2
0.493852 < 0.49375 < 0.493858
Exercise 8.

(1)
1 x3
0≤x≤ sin x = x − + E4 (x)
2 3!
¡ ¢5
M |x|5 sin 13 |x|5 1 12
|E4 (x)| ≤ = ≤
5! 5! 5!
(2)
x6
sin x2 = x2 −
+ E4 (x2 )

6
Z √
2 µ ¶¯ 2/2 µ ¶
2
2 1 3 x7 ¯¯ √ 1 1
sin x = x − = 2 −
0 3 42 ¯0 12 42(16)

2
E4 (x2 ) ≤
64(5!)
Z √22 µ ¶
√ 55 1
sin x2 ≤ 2 + = 0.1159
0 672 64(5!)
Exercise 9.
x3 x5 (1)x7
sin x = x −
+ E6 (x; 0) ≤
6 5! 7!
sin x x2 x4 E6 (x; 0) x2 x4 x6
=1− + + ≤1− + +
x 6 5! x 6 5! 7!
Z 1 µ ¶
sin x 1 1 1 1 1
dx = 1 − + + = 0.9461 + = 0.9461 + 0.0000283
0 x 3 6 5(5!) 7(7!) 7(7!)
Exercise 10. α = arctan 15 , β = 4α − π
4.
(1)
(tan A + tan B) 2 tan α 2/5
tan (A + B) = ; A = B = α; tan 2α = = = 5/12
1 − tan A tan B 1 − tan2 α 24/25
2(tan 2α) 2(5/12) 10/12 120 π
tan 4α = = = = A = 4α, B = −
1 − tan2 (2α) 1 − (5/12)2 119/144 119 4
¡ π¢ 120 119
π tan 4α + tan − 4 +− 1
tan (4α − ) = tan β = ¡ ¢ = 119 120 119 =
4 1 − tan 4α tan − π4 1 − 119 (−1) 239
1 π 1
4 arctan = + arctan This is incredible.
5 4 239
120
(2)
6−1
X (−1)k x2k+1
T11 (arctan x) = + E2(6) (x)
2k + 1 x2(6)+1
k=0 ; |E2(6) (x)| ≤
2(6) + 1
−x3 x5 x7 x9 x11
=x+ + − + − ...
3 5 7 9 11
1
=⇒ 3.158328957 < 16 arctan < 3.158328958
5
(3)
1
T3 (arctan x); x =
239
1
−0.016736304 < −4 arctan < −0.016736304
239
(4)
3.141592625
3.158328972 − 0.016736300 = 3.141592672
7.11 Exercises - Further remarks on the error in Taylor's formula. The o-notation; Applications to indeterminate
forms.
Exercise 1.

x2 (ln 2)2
2x = exp x ln 2 = 1 + (x ln 2) + + o(x2 )
2!
Exercise 2.

cos 1(x − 1)3


x(cos x) = ((x − 1) + 1)(cos x) = (x − 1) cos 1 + (− sin 1)(x − 1)2 − + cos 1+
2
cos 1(x − 1)2 sin 1(x − 1)3
+ (− sin 1)(x − 1) − + =
2 µ 3! ¶ µ ¶
cos 1 2 sin 1 − 3 cos 1
= cos 1 + (cos 1 − sin 1)(x − 1) + − sin 1 − (x − 1) + (x − 1)3 + o(x − 1)3
2 3!
Exercise 3. Just treat the argument of sin x − x2 just like u with u → 0.
(x − x2 )3 (x − x2 )5
sin (x − x2 ) = (x − x2 ) − + + o(x − x2 )5 =
3! 5!
1¡ 3 ¢ 1 ¡ 5 ¢
= (x − x2 ) − x − 3x4 + 3x5 − x6 + x − 5x6 + 10x7 − 10x8 + 5x9 + x1 0 =
6 120
1 1 61 5 25 6
= (x − x2 ) − x3 − x4 + x − x
6 2 120 120
Exercise 4.
(x − 1)2 (x − 1)3
log x = log (1 + (x − 1)) = (x − 1) − +
2 3
−1
=⇒ a = 0; b = 1, c =
2
Exercise 5.
1 2
x + o(x3 )
1 − cos x =
1 2
cos x = 1 − x2 + o(x3 ) as x → 0
2 1 − cos x 1 o(x3 )
2
= +
x 2 x2
1 − cos x 1 1 − cos x 1
since
2
= + o(x), 2
→ as x → 0
x 2 x 2
1 x4 2
cos x = 1 − x2 + + o(x5 ) =⇒ cos 2x = 1 − 2x2 + x4 + o(x5 )
2 4! 3
1 − cos 2x − 2x2 − 23 x4 − o(x5 ) −2 2
4
= 4
= − o(x) → − as x → 0
x x 3 3
Exercise 6.
3
sin ax ax − (ax) 4
3! + o(x ) a
lim = lim =
x→0 sin bx x→0 bx + o(x2 ) b
121
Exercise 7.
(2x)3
sin 2x (2x) + 3! ´ + o(x4 ) 2
lim = lim ³ ³ ´=
x→0 cos 2x sin 3x x→0 (2x)2 (2x)4 (3x)3 3
1− 2! + 4! + o(x5 ) (3x) − 3! + o(x4 )

Exercise 8.

sin x − x 1
lim 3
= −
x→0 x 6
Exercise 9.
ln 1 + x x − o(x) 1
lim = lim =
x→0 e2x − 1 x→0 2x + o(x) 2
Exercise 10. Don't do the trig. identity.

³ ´2
x2 2
2
1 − cos x 1 − 1 − 2! + o(x ) 1 − (1 + x2 + o(x2 ))
lim = lim ¡ 3 ¢ = lim =1
x→0 x tan x x→0 x
x x + 6! + o(x3 ) x→0 x2 + o(x2 )

Exercise 11.
3
x + − x6 + o(x4 )
lim x3
=1
x→0 x− 3

Exercise 12.

ex ln a − 1 x ln a + o(x)
lim = lim = ln a/b
x→0 ex ln b − 1 x→0 x ln b + o(x)

Exercise 13.
(x−1)2 3
(x − 1) − 2 + (x−1)
3 + o(x − 1)4 1
lim =
x→1 (x + 2)(x + 1) 3

Exercise 14. 1 .

Exercise 15.

x2
x(ex + 1) − 2(ex − 1) x(2 + x + 2 ) − 2(x + x2 /2 + x3 /6) x3 ( 16 ) 1
lim = lim = lim =
x→0 x3 x→0 x3 x→0 x3 6

Exercise 16.
x2 x3 3
ln (1 + x) − x x− 2 + 3 + o(x ) −x
lim = lim 3
= −1
x→0 1 − cos x x→0 x /2
Exercise 17.

cos x 0 + −1(x − π2 )
lim π = = −1
x→π/2 x − 2 x − π2

Exercise 18. 1/6


Exercise 19.
ex +e−x
cosh x − cos x 2 − cos x
lim = lim =
x→0 x2 x→0 x2 ³ ´
x2 x2
1+x+ 2 −2 1− 2 + o(x3 )
= lim = 2
x→0 x2
Exercise 20.
³ ´
(4x)3 x3
3 tan 3x − 12 tan x −
(4x) + 4 x+ 3 3 + o(x4 )
lim = lim (4x) 3 ¡ ¢ =
x→0 3 sin 4x − 12 sin x x→0 (4x) − x3
+ o(x4 )
3! − 4 x − 3!
43 − 4
= −43 +4
= −2
2
122
Exercise 21.

ax − asinx ex ln a − esin x ln a
lim 3
= lim =
x→0 x x→0 x3 ³ ´
(x ln a)2 (x ln a)3 sin2 x(ln a)2 sin3 x(ln a)3
1 + x ln a + 2! + 6 − 1 + sin x ln a + 2 + 3! + o(x4 )
= lim =
x→0
³ ´ x3
3 2 2 2 3
x (x ln a) (ln a) (lna)
ln a(x − x − 3! )+ 2! − 2 (x2 ) + 6 (x3 − x3 ) + o(x4 ) ln a
= lim =
x→0 x3 6
Exercise 22.
³ ´
sin2 x sin4 x x2 x4
cos sin x − cos x 1− 2! + 4! + o(x5 ) − 1 − 2! + 4!
lim = lim =
x→0 x4 x→0
³
4
´ x ³ 4 4´
4
1
2 x2 − (x2 − 3 ) + x −x
x
4! + o(x5 ) 1
= lim =
x→0 x4 6
Exercise 23.
µ ¶ µ ¶
1 1
ln x ln x (x − 1) + o(x − 1)2
lim x → 1x 1−x = lim x → 1e 1−x = exp lim x → 1 = exp lim x → 1 = e−1
1−x 1−x
Exercise 24.

1
lim (x + e2x )1/x = exp lim ln (x + e2x )
x→0 x→0 x
ln (x + e2x ) ln (1 + x + e2x − 1)
lim = lim =
x→0 x x→0 x
2x 2
x + e − 1 + o(x ) 3x + o(x2 )
= lim = lim =3
x→0 x x→0 x
=⇒ lim (x + e2x )1/x = e3
x→0

Exercise 25.
2
1 x+− x +o(x2 )
2
(1 + x)1/x − e e x ln (1+x) − e e x −e
lim = lim = lim =
x→0 x x→0 x x→0 x
x
e1− 2 +o(x) − e e(1 + − x2 + o(x)) − e −e
= lim = lim =
x→0 x x→0 x 2
Exercise 26.
µ ¶1/x µ µ ¶ ¶1/x 2
(1 + x)1/x 1 x− x +o(x3 )−x
2
lim = lim exp ln (1 + x) − 1 = lim e x2 = e−1/2
x→0 e x→0 x x→0

Exercise 27.

1
(arcsin x)0 = √
1 − x2
x
(arcsin x)00 =
(1 − x2 )3/2
1 3x2
(arcsin x)000 = 2 3/2
+
(1 − x ) (1 − x2 )5/2
µ µ µ ¶¶¶ µ µ ¶¶
1 arcsin x 1 arcsin x
exp lim 2 ln = exp lim 2 ln 1 + −1 =
x→0 x x x→0 x x
µ µ ¶¶ µ µ ¶¶
1 x + x3 /6 + o(x4 ) − x 1 x2
= exp lim 2 ln 1 + = exp lim 2 ln 1 + + o(x3 ) =
x→0 x x x→0 x 6
µ µ ¶¶
1 x2 + o(x3 )
= exp lim 2 = e1/6
x→0 x 6
³ ´ ³ ´ x2 3 1
1 1 ex −1−x 2 +o(x )
Exercise 28. limx→0 x − ex −1 = limx→0 x(ex −1) = limx→0 x2 +o(x3 ) =
2
123
Exercise 29.
µ ¶ µ ¶
1 1 (x − 1) − log x
lim − = lim =
x→1 log x x − 1 x→1 (x − 1) log x
2
(x − 1) − ((x − 1) − (x−1)
2 + o(x − 1)3 ) 1
= lim =
x→1 (x − 1)((x − 1) + o(x − 1)2 ) 2

Exercise 30.
2
x2
eax − ex − x 1 + ax + (ax)2 + o(x3 ) − 1 − x − 2 −x
lim = lim
x→0 x2 x→0 x2
if a = 2, the limit is 2

Exercise 31.
Rx ¡R x ¢
(1) Prove
0
f (t)dt = o 0
g(t)dt as x → 0, given f (x) = o(g(x)).
Rx
f (t)dt
Consider limx→0 R0x g(t)dt .
0
Since f, g have derivatives in some interval containing 0, f, g continuous and differentiable for |t| ≤ x.

³ ´
Rx A(x)−A(0)
0
f (t)dt f (0) limx→0 f (x)
x
lim = lim ³ ´= = =0
x→0 limx→0 g(t)dt x→0 B(x)−B(0) g(0) limx→0 g(x)
x

We can do the second to last step since f, g have derivatives at 0 and thus are continuous about 0.
x
(2) Consider limx→0 ex = 0. However, limx→0 e1x = 1.
Exercise 32.

(1) Use long division to nd that

1 −g 3 (x)
= 1 − g(x) + g 2 (x) + = 1 − g(x) + g 2 (x) + o(g 2 (x))
1 + g(x) 1 + g(x)
(2)

³ ´1/x
f (x)
Exercise 33. Given limx→0 1 + x + x = e3 , use the hint.

lim g(x) = A, then G(x) = A + o(1) as x → 0


x→0

Then
µ ¶1/x
3 f
g(x) = e + o(1) = 1 + x +
x
¡ 3 ¢x 2
x e + o(1) = x + x + f (x) =⇒ f (0) = 0

x exp x ln (e3 + o(1)) = x + x2 + f (x)


µ ¶
3 3 3 x
1 exp x ln (e + o(1)) + x exp x ln (e + o(1)) ln e + o(1) + 3 (o (1)) = 1 + 2x + f 0 (0)
0
e + o(1)
1 + 3(0) + 0 = 1 + 0 + f 0 (0) =⇒ f 0 (0) = 0
We need to assume that in general o(1) = x + kx2 + o(x2 ).
µ ¶
x
2 exp x ln (e + o(1)) ln (e3 + o(1)) +
3 0
(o (1)) +
e3 + o(1)
µ ¶
3 3 x 0
+ x exp x ln (e + o(1)) ln (e + o(1)) + 3 o (1) +
e + o(1)
µ ¶
3 2o0 (1) xo00 (1) x
x exp x ln (e + o(1)) + +− 3 (o (1)) = 2 + f 00 (x)
0 2
(e3 + o(1)) e3 + o(1) (e + o(1))

x→0
−−−→ 2(3) = 2 + f 0 (0) =⇒ f 0 (0) = 4
124
³ ´1/x
f (x)
To evaluate limx→0 1 + x , consider a Taylor series expansion of f.

µ ¶1/x à 2
!1/x
f (x) 0 + 0 + 4 x2 + o(x3 ) 1/x
lim 1 + = lim 1 + = lim (1 + x (2 + o(x))) =
x→0 x x→0 x x→0

1/x
= lim lim (1 + x (2 + o(y))) = lim exp 2 + o(y) = ey
y→0 x→0 y→0

7.13 Exercises - L'Hopital's rule for the indeterminate form 0/0. Exercise 1.

3x2 + 2x − 16 (3x + 8)(x − 2) 14


lim = lim =
x→2 x2 − x − 2 x→2 (x − 2)(x + 1) 3

Exercise 2.

x2 − 4x + 3 (x − 3)(x − 1)
lim 2
= lim = −2
x→3 2x − 13x + 21 x→3 (2x − 1)(x − 3)

Exercise 3.

µ ¶ µ ¶ µ ¶
sinh x − sin x 0 cosh x − cos x 0 sinh x + sin x cosh x + cos x 1
lim = = lim = = lim = lim =
x→0 x3 0 x→0 3x2 0 x→0 6x x→0 6 3

Exercise 4.

(2 − x)ex − x − 2 −ex + (2 − x) − 1 (2 − x)(1 + x + x2 /2 + x3 /6 + o(x3 )) 1


lim 3
= lim 2
= lim = −
x→0 x x→0 3x x→0 x3 6

Exercise 5.

log (cos ax) − cos bx sin axa cos axa2 a2


lim = lim = lim =
x→0 log (cos bx) x→0 − cos ax sin bxb x→0 sin bxb2 b2

Exercise 6. When it doubt, Taylor expand.

2 4
x − sin x 1 − cos x 2 1 − (1 − x2 + x24 + o(x4 ))
lim+ = lim 3 = lim 1 =
x→0 (x sin x)3/2 x→0+ (x sin x)1/2 (sin x + x cos x)
2
3 x→0+ e 2 ln (x sin x) (x − x63 + o(x3 ) + x − x23 + o(x3 ))
 
x2 x4 4
2 − + o(x )
= lim  q 2 24 =
3 x→0+ x3 3 2 3
x(x + 6 + o(x ))(2x − 3 x + o(x )) 3
 
1 x2 2 µ ¶
2 2 − 24 + o(x )  = 2 lim 1/2 = 1
= lim+  q
3 x→0 2
1 + x + o(x2 )(2 + −2 x2 + o(x2 )) 3 x→0+ 2 6
6 3

1
Notice in the third step how in general we deal with powers, (x sin x)1/2 , is to convert it into exponential form, e 2 ln (x sin x) ,
but it wasn't necessary.

Exercise 7. Do L'Hopital's rst.

√ √ √ 1 √1
√ √
x− a+ x−a √
2 x
+ 1
2 x−a x2 − a2 x+a
lim √ = lim+ = lim + =
x→a+ x2 − a2 x→a √ x 2 x→a+ x3/2 x
x2 −a2
√ √ √ √
1 x2 − a2 + x1/2 x + a 1 2a 2
= lim = = √
2 x→a+ x3/2 2 a3/2 2 a

Exercise 8. Do L'Hopital's at the second step.

1
exp (x ln x) − x exp x ln x(ln x + 1) − 1 exp (x ln x)(ln x + 1)2 + x exp (x ln x)
lim+ = lim+ = lim =
x→1 1 − x + ln x x→1 −1 + x1 x→1+ −1/x2
= lim+ x2 exp (x ln x)(ln x + 1)2 + x exp (x ln x) = 1 + 1 = 2
x→1
125
Exercise 9. Keep doing L'Hopital's.

√ 2 3 − 2√ 1 2
arcsin 2x − 2 arcsin x 1−(2x) 1−x
lim 3
= lim 2
=
x→0 x x→0 3x
2 − 1 (1 − (2x)2 )−3/2 (−8x) − (− 12 )(1 − x2 )−3/2 (−2x)
= lim 2 =
3 x→0 6x
1 4(1 − (2x)2 )−3/2 + 4x(− 32 )(1 − (2x)2 )−5/2 (−8x) + −(1 − x2 )−3/2 − x(− 32 )(1 − x2 )−5/2 (−2x)
= lim =
9 x→0 1
14−1 1
= =
9 1 3
Exercise 10.

x cot x − 1 x cos x − sin x cos x − x sin x − cos x − sin x


lim 2
= lim 2
= lim 2
= lim =
x→0 x x→0 x sin x x→0 2x sin x + x cos x x→0 2 sin x + x cos x
cos x cos x 1
= − lim = − lim =
x→0 2 cos x + cos x + −x sin x x→0 3 cos x − x sin x 3
Exercise 11.
Pn Pn n
X
k=1 xk − n k=1 kxk−1 n(n + 1)
lim = lim = k=
x→1 x−1 x→1 1 2
k=1

Exercise 12.
³ ³ ´ ³ ´´
µ √ √ ¶ a 1 1√
− b 1 1

1 x x x
1+ a2 2a x x
1+ b2 2b x
lim √ a arctan − b arctan = lim 3 1/2 =
x→0+ x x a b x→0+
2 x
µ ¶ µ ¶
1 a2 1 b2 1 1 (b2 + x)a2 − b2 (a2 + x)
= lim 2 − 2 = lim =
3 x→0+ a + x x b +x x 3 x→0+ (a2 + x)(x + b2 )x
1 (a2 − b2 ) 1 a2 − b2
= lim =
3 x→0+ (a2 − x)(b2 + x) 3 a2 b2
Exercise 13.

(sin 4x)(sin 3x) (2 sin 2x cos 2x)(sin 3x) 2(−2 sin 2x sin 3x + cos 2x3 cos 3x)
= = = 6 as x → 0 otherwise
x sin 2x x sin 2x 1
2 cos 2x sin 3x 4 π
→ as x →
x π 2
We used L'Hopital's at the second to last step for x → 0.

Exercise 14.

lim (x−3 sin 3x + ax−2 + b) = 0


x→0
sin 3x + ax + bx3 3 cos 3x + a + 3bx2 −9 sin 3x + 6bx −27 cos 3x + 6b −27 + 6b
lim 3
= = = = =0
x→0 x 3x2 6x 6 6
9
So b= , a = −3 .
2
Exercise 15.

Z x √ x2 √
1 t2 dt a+x 2x 2x a + x
lim √ = = √ = 1 =
x→0 bx − sin x b − cos x √1 (1 − cos x) + a + x(sin x)
0 a+t 2 a+x 2 (1 − cos x) + (a + x) sin x
√ √
a + x + 2√xa+x a
= 2 lim sin x =2 =1
x→0
2 + sin x + (a + x) cos x a
=⇒ a = 4
Note that we had dropped the limit notation in some earlier steps and applied L'Hopital's a number of times, and we also
rearranged the denominator and numerator cleverly at each step.

Exercise 16.

126
(1)
x x
angle ABC
, length BC is tan
is
2 2
x x x
2 tan cos = 2 sin is the base length of ABC
2 2 2
x x
tan sin is the height of triangle ABC
2 2
x x 1 − cos2 x2 x x 1
=⇒ T (x) = tan sin2 = sin = tan − sin x
2 2 cos x2 2 2 2
(2)
³x´ 1 x x x sin 2
S(x) = (π(1)) − cos (2 sin ) = −
2π 2 2 2 2 2
(3) Use L'Hopital's theorem.

T (x) tan x2 − 21 sin x dx


d 1
2 sec2 x
2 − 2
cos x
= x−sin x

−→ 1−cos x
S(x) 2 2
x
d
dx
sec2 2 tan x2 + sin x d
dx
sec2 x2 tan2 x2 + 12 sec4 x
2 + cos x
−−→ −−→ →
sin x cos x
x→0 3
−−−→
2
Exercise 17. Use L'Hopital's rule.

E Rt
(1 − e− L )
I(t) =
R
¡ ¢
E(−1)(e−Rt/L ) −t
L Et
lim I(t) = lim =
R→0 R→0 1 L
Exercise 18.
c − k → 0 since c → k
k−c=u
k−u=c
A(sin kt − sin ct) A(sin (kt) − sin (k − u)t)
f (t) = 2 2
= =
c −k −u(2k − u)
A(cos (k − u)t)(t) −At cos kt
= →
−(2k − u) + u 2k
7.17 Exercises - The symbols +∞ and −∞. Extension of L'Hopital's rule; Innite limits; The behavior of log x and ex
for large x.

Exercise 15. Use L'Hopital's at the second to last step.


³ ´
1 ¡ ¢
ln (1 − x) 1−x (−1) x 2 ln x x1
¡ 1 ¢ = lim 2
lim (ln x)(ln (1 − x)) = lim 1 = lim −1 (ln x) = lim = 0
x→1−1 x→1−1
ln x
x→1−1
(ln x)2 x
x→1−1 1−x x→1−1 (−1)

Exercise 16. Persist in using L'Hopital's and trying all possibilities systematically.

x x x ln x ln x
lim xx −1
= lim+ e(x −1) ln x
= lim+ e(e −1) ln x
= exp lim+
=
x→0+ x→0 x→0 (ex ln x − 1)−1 x→0
µ 2x ln x ¶
1/x e − 2ex ln x + 1
= exp lim+ = exp − lim+ =
x→0 (−1)(ex ln x − 1)−2 (ex ln x (ln x + 1)) x→0 ex ln x (x ln x + x)
ex ln x (ln x + 1) + e−x ln x (− ln x − 1) ex ln x − e−x ln x
= exp− lim+ = exp − lim+ 1 = 1
x→0 (ln x + 1 + 1) x→0 1 + ln x+1

Exercise 17.
x
³ x ´ ³ x ln x ´ ³ x ln x
´
lim+ (xx − 1) = lim+ ex ln x − 1 = lim+ ee ln x
− 1 = elimx→0+ e ln x
−1 =
x→0 x→0 x→0
lim x ln x
e x→0+ limx→0+ ln x
=e − 1 = 0 − 1 = −1
127
We used
lim xα log x = 0 ∀α > 0
x→0+
since
− log t
t = x1 , xα log x = tα → 0 as t → ∞.
Exercise 18.
µ ¶ Ã 1
¡ ¢!
sin x ln (1−2x ) ln (1 − ex ln 2 ) 1−ex ln 2
− ln 2ex ln 2
lim e = exp lim− = exp lim− −1 =
x→0− x→0 1/ sin x x→0
sin2 x
cos x
µ ¶ µ ¶
(sin2 x) ln 2ex ln 2 2 sin x cos x
= exp lim− = exp (ln 2) lim = 1
x→0 (1 − ex ln 2 ) cos x x→0− − ln 2ex ln 2

Exercise 19.
1
lim+ e ln x ln x = e
x→0
Exercise 20. At the end, L'Hopital's could be used to verify that indeed sin x ln sin x → 0 as x → 0.
lim esin x ln cot x = elimx→0+ sin x(ln cos x−ln sin x) = elimx→0+ − sin x ln sin x = 1
x→0+

Exercise 21. Rewrite tan into sin and cos and use L'Hopital's.
1
lim (tan x)tan 2x = limπ etan 2x ln tan x = exp limπ (ln sin x − ln cos x) =
x→ π
4 x→ 4 x→ 4 cos 2x
1
cos x − cos1 x (− sin x) 1
= exp limπ sin x
= exp limπ = e−1
x→ 4 −2 sin 2x cos 2x x→ 4 − sin2 2x

Exercise 22.

Exercise 23. Use L'Hopital's theorem, taking derivatives of top and bottom.

e ln x 1/x
lim exp ln x = exp e lim = exp e lim = ee
x→0+ 1 + ln x x→0 1 + ln x x→0 1/x

Exercise 24. Rewrite tan into sin and cos and take out sin since we could do the limit before doing L'Hopital's.
(−1)
tan (πx/2) tan πx
ln (2−x) limx→1
sin πx/2 ln (2−x)
2−x −2
lim (2 − x) = lim e 2 =e cos πx/2 = exp lim = exp
x→1 x→1 x→1 π sin πx/2 π
2

Exercise 26.
³ ´ ³ ´
1 (x−c)−(x+c)
µ µ ¶¶ 1+c/x
ln 1−c/x (x−c)2
x+c 1+c/x
( 1−c/x )
lim exp x ln = exp lim = exp lim −1 =
x→+∞ x−c x→+∞ 1/x x→+∞
x2

= exp (2c) = 4 =⇒ c = ln 2
Exercise 27.

(1 + x)c = exp (c ln (1 + x)) = exp (c(x − o(x))) = 1 + c(x − o(x)) + o(x − o(x)) = 1 + cx + o(x)
µ ¶1/2 µ ¶1/2
2 1 2 2 1
x 1+ 2 −x =x 1+ 2 − x2
x x
2 1
Let x = . So t → 0 as x → +∞.
t

(1 + t)1/2 − 1 1 + 12 t − 1 + o(t) 1
=⇒ = =
t t 2
Exercise 28. µ ¶c
5 4 7 2
c 5
(x + 7x + 2) − x = x 1 + + 5 −x
x x
1 1
Let
t = x and guess that c = 5
µ ¶c
1 7 1 ¡ ¢1/5 1
5
+ 4 +2 − = 1 + (7t + 2t5 ) /t − =
t t t t
1 + 15 (7t + 2t5 ) + o(t) − 1 7
= =
t 5
128
Exercise 29.
Z xµ ¶
1
g(x) = xe x2 f (x) = g(t) t + dt
1 t
2 2 µ ¶
g 0 (x) = ex + 2x2 ex 1
f 0 (x) = g(x) x + − g(1)2;
2 2 2 2 2 x
g 00 (x) = 2xex + 4xex + 4x3 ex = 6xex + 4x3 ex
f 00 (x) = g 0 (x)(x + 1/x) + g(x)(1 − 1/x2 )
2 2 2 2 2 2
1
f 00 (x) (ex + 2x2 ex )(x + 1/x) + xex (1 − x2 ) 2xex + 2x3 ex + 2xex
= = =
g 00 (x) 6xex2 + 4x3 ex2 (6xex2 + 4x3 ex2 )
4x + 2x3 1
= = as x→∞
6x + 4x3 2
Exercise 30. Z x
g(x) = xc e2x f (x) = e2t (3t2 + 1)1/2 dt
0
g 0 (x) = cxc−1 e2x + 2xc e2x
f 0 (x) = e (3x2 + 1)1/2 − 1
2x

Guessing that c=1


√ √
f 0 (x) e2x (3x2 + 1)1/2 − 1 3x(1 + 3x1 2 )1/2 − e−2x 3
0
= = =
g (x) 2xc e2x + cxc−1 e2x 2x + 1 2
So c=1 .

Exercise 31.

Exercise 32.

(1)
³ r´ ³ r ´2 ³ r ´m
P 1+ ,P 1 + ,...P 1 +
m m m
For each year, there are the just previously shown m compoundings, so for n years,
³ r ´mn
P 1+
m
(2)
2 = ert
ln 2
= t = 11.55years
r
(3)
³ r ´mt
2P0 = P0 1 +
m
ln 2 = mt ln (1 + r/m)
ln 2 ln 2
t= = = 11.64years
m ln (1 + r/m) 4 ln (1 + 0.06/4)

7.17 Exercises - The symbols +∞ and −∞. Extension of L'Hopital's rule; Innite limits; The behavior of log x and ex
for large x. Exercise 1.
1
2
u=
x
−1/x2 500
e u 1
lim = lim e−u u5 00 = lim =0 x2 =
x→0 x1000 u→∞ u→∞ eu u
1
x1000 = 5
u 00
Where we had used Theorem 7.11, which are two very useful limits for log and exp.

Theorem 26. If a, b > 0,


(log x)b
(17) lim =0
x→+∞ xa
xb
(18) lim ax = 0
x→+∞ e
129
Proof. Trick - use the denition of the logarithm as an integral.
If c > 0, t ≥ 1, tc ≥ 1 =⇒ tc−1 ≥ t−1 .
Z x Z x
1 1 c xc
0 < ln x = dt ≤ tc−1 dt = (x − 1) <
1 t 1 c c
(ln x)b xcb−a
=⇒ 0 < a
<
x cb
cb−a −a/2
a x x
Choose c = , b = → 0 as x → ∞
2b c cb
(ln x)b
then → 0 as x → 0
xa
xb (ln t)b
For exp, Let t = ex . ln t = x. eax = ta → 0 as t → ∞ as x → ∞. ¤

Exercise 2.
1
¡ ¢
sin x1 x − o x1
lim = lim ¡ ¢= 1
x→0 arctan 1 x→0 1
− o x1
x x

Exercise 3. Use L'Hopital's.

tan 3x cos x − sin x 1


limπ = limπ = limπ = −
x→ 2 tan x x→ 2 cos 3x x→ 2 −3 sin 3x 3

Exercise 4. Use L'Hopital's.

1 x 1
ln (a + bex ) a+bex (be ) ae−x +b 1
lim√ = lim = lim =√
x→∞ a + bx2 x→∞ √ bx x→∞ √ a1 b
a+bx2 x2
+b

1
Exercise 5. Make the substitution x= t.
µ ¶ µ ¶
1 1 1 t2 t4 /4! + o(t4 ) 1 1
lim x4 cos − 1 + 2 = lim 4 cos t − 1 + = lim = =
x→∞ x 2x t→∞ t 2 t→∞ t4 4! 120

Exercise 6.
1
ln | sin x| sin x cos x 1 sin 2x 1 2 cos 2x
lim = lim 2 = − lim = − lim = 1
x→π ln | sin 2x| x→π sin 2x cos 2x 2 x→π sin x 2 x→π cos x

Exercise 7.
³ ´
1
ln (1 − 2x) 1−2x (−2) −2(cos2 πx)
lim− = lim− = lim =
x→ 12 tan πx x→ 12 (sec2 πx)π x→ 12 − (1 − 2x)π

2 2 cos πxπ − sin πx


=− lim =1
π x→ 12 − −2

Exercise 8.

cosh x + 1 ex+1 + e−x−1 1 1 e


lim = lim = lim e1 + 2x+1 =
x→∞ ex x→∞ 2ex 2 x→∞ e 2

Exercise 9.

ax ex ln a
lim = lim → ∞; a > 1
x→∞ xb x→∞ xb
xb
since lim (in this case, ln a > 0 )
x→∞ eax

Exercise 10.

tan x − 5 sec2 x sec x 1 cos x


limπ = limπ = limπ = limπ = 1
x→ 2 sec x + 4 x→ 2 tan x sec x x→ 2 tan x x→ 2 cos x sin x

130
8.5 Exercises - Introduction, Terminology and notation, A rst-order differential equation for the exponential function,
First-order linear differential equations.
The ordinary differential equation theorems we will use are

(19) y 0 + P (x)y = 0
Z x
A(x) = P (t)dt
a
(20) y = be−A(x)
Rx
Consider y 0 + P (x)y = Q(x); A(x) = a
P (t)dt.
A(x)
Let h(x) = g(x)e ; g a solution.

h0 (x) = (g 0 + P g)eA = QeA


Z x
2nd. fund. thm. of calc.
−−−−−−−−−−−−−→ h(x) = h(a) + Q(t)eA(t) dt
a
since h(a) = g(a)
µZ x ¶
(21) y = g(x) = e−A(x) Q(t)eA(t) dt + b
a
We had done some of these problems previously, using an integration constant C, but following Apostol's notation for
y(a) = b for initial conditions is far more advantageous and superior as we seem clearly the dependence upon the initial
conditions - so some of the solutions for the exercises will show corrections to the derived formula using Apostol's notation
for y(a) = b initial conditions.
Rx
Exercise 1. A(x) = 0 (−3)dt = −3x
µZ x ¶
¯x
y = e3x e2t e−3t dt + 0 = e3x (−e−t )¯0 = −e2x + e3x
0
R x ¡ −2 ¢
Exercise 2. y 0 − x2 y = x4 . A(x) = 1 t dt = −2 ln x.
µZ x ¶ R x 4 −2 ln t
µZ x ¶
y = e−A(x) Q(t)eA(t) dt + b = e2 ln x( 1 t e dt+1)
= x2 t2 dt + 1 =
a 1
x2 3 x5 2x2
= −x2 + (x − 1) = +
3 3 3
0
¡ π π¢
Exercise 3. y + y tan x = sin 2x on − 2 , 2 , with y = 2 when x = 0.
Z x Z x
x
A(x) = P (t)dt = tan tdt = − ln | cos t||0 = − ln cos x
0 0
µZ ¶ ÃZ !
x b
y = e−A(x) Q(t)eA(t) dt + b = cos x sin 2te− ln cos t dt + 2 =
a a
µZ x ¶
= cos x 2 sin tdt + 2 = −2 cos2 x + 4 cos x
a
0 3
Exercise 4. y + xy = x . y = 0, x = 0.
1
A(x) = x2
µZ x ¶ 2 ³ t2 ´¯x
− 12 x2
2
3 t2 −x2 t2 ¯
y=e t e dt + 0 = e 2 t2 e 2 − 2e 2 ¯ =
0 0
2
− x2
= x2 − 2 + 2e

Exercise 5. y 0 + y = e2t . y = 1, t = 0.
µZ x ¶ µ ¯x ¶
e3t ¯¯
y(x) = e−x e2t et dt + 1 = e−x + 1
0 3 ¯0
A=x
2x
e 2
= + e−x
3 3
131
Exercise 6. y 0 sin x + y cos x = 1; (0, π). =⇒ y 0 + cot xy = csc x
Z x ¶µ
sin x
A(x) = cot tdt = ln
a sin a
µZ ¶ µ ¶µ ¶
− ln ( sin x
) ln ( sin
sin t
) sin a x−a
y(x) = e sin a (csc t)e a dt + b = +b
sin x sin a
x − a b sin a
indeed y = +
sin x sin x
x − a b sin a x + b sin a − a
x → 0 for y = + =
sin x sin x sin x
b sin a = a for x → 0
a − b sin a = π for x→π

Exercise 7.

2 1 2
x(x + 1)y 0 + y = x(x + 1)2 e−x =⇒ y 0 + y = (x + 1)e−x
x(x + 1)
Z xµ ¶
1 1 x x+1
A(x) = − dt = ln − ln
a t t + 1 a a+1
(a + 1)x
eA(x) =
a(x + 1)
µZ x µ ¶ ¶ µZ x ¶
a(x + 1) 2 (a + 1)t (x + 1) 2 a(x + 1)
y= (t + 1)e−t dt + b = te−t dt + b =
(a + 1)x a a(t + 1) x a (a + 1)x
x + 1 ³ −a2 2
´ a(x + 1)b
= e − e−x +
2x (a + 1)x

It's easy to see that the last equation above goes to 0 as x → −1.

2 2 1 ab 1
y = (e−a − e−x )(1/2)(1 + ) + (1 + ) =
x a+1 x
à 2 2
!
1 e−a − e−x ab
lim y = + a=0
x→0 x 2 a+1

Exercise 8. y 0 + y cot x = 2 cos x on (0, π).


Z x ¶ µ
sin x A(x) sin x
A(x) = cot tdt = ln e =
a sin a sin a
µZ ¶ µ ¯x ¶
sin a sin t sin a cos 2t ¯¯
y= 2 cos t +b = − +b =
sin x sin a sin x 2 sin a ¯a cos (2a) − cos (2x) sin a
y = sin x y = +
cos (2a) + cos 2x sin a 2 sin x sin x
= +
2 sin x sin x

2 x−1
Exercise 9. (x − 2)(x − 3)y 0 + 2y = (x − 1)(x − 2). y 0 + (x−2)(x−3) y = x−3 .

Z x Z x µ ¶ ¯ ¯¯ ¯
2dt 1 1 ¯x − 3¯ ¯a − 2¯
A(x) = =2 − dt = 2 (ln |t − 3| − ln |t −
x
2|)|a ¯
= 2 ln ¯ ¯ ¯ ¯
a (t − 2)(t − 3) a t−3 t−2 a − 3¯ ¯x − 2¯
x − 3 ≶ 03 − x x−3
If (−∞, 2), (3, +∞), =
x − 2 ≶ 02 − x x−2
(2, 3), x − 3 < 0, but x − 2 > 0
If
µ ¶2 µ¯ ¯¶ µ ¶2 µ ¶
x−2 ¯a − 3¯ 2 x−2 1 1
If (−∞, 2), (3, ∞) y = b ¯ ¯
x−3 ¯a − 2¯ + x − 3 x+
x−2
−a−
a−2
132
µ ¶2 µ¯ ¯¶ µ ¶2 ¯ ¯2 Z µ ¶¯ ¯2 µ ¶2
x−2 ¯a − 3¯ 2 x − 2 ¯¯ a − 3 ¯¯ t − 1 ¯¯ a − 2 ¯¯ 3 − t
If (2, 3) y = b ¯ ¯
x−3 ¯a − 2¯ + 3 − x ¯a − 2¯ t − 3 ¯a − 3¯ t − 2
=
µ ¶2 ¯ ¯ 2 µ Z ¶
x − 2 ¯¯ a − 3 ¯¯ x−2 (t − 1)(3 − t)
=b + − =
3 − x ¯a − 2¯ 3−x (t − 2)2
µ ¶2 ¯ ¯2 µ ¶2
x − 2 ¯¯ a − 3 ¯¯ x−2
=b + (x + (x − 2)−1 − a − (a − 2)−1 )
x − 3 ¯a − 2¯ x−3

sin x
Rx
Exercise 10. s(x) = x ;x 6= 0 s(0) = 1, T (x) = 0
s(t)dt f (x) = xT (x)

f 0 = T + x(s(x)) = T + sin x
xf 0 − f = x sin x
y
y 0 − = sin x
Z x x
−1 x x
A(x) = dt = − ln , e−A(x) =
a t a a
µZ x ¶
x a bx
y= sin t + b = xT +
a a t a
1
y(0) = 0 6= 1 since P (x) = is not continuous at x = 0
x

Exercise 11.

Z x
1
f (x) = 1 + f (t)dt
x 1
Z x
1
xf (x) = x + f (t)dt =⇒ f (x) + xf 0 = 1 + f (x) =⇒ f 0 =
1 x
=⇒ f (x) = ln |x| − C
Z x
1
ln |x| − C = 1 + (ln |t| − c) dt =
x 1
f (x) = ln |x| + 1
1 x 1+C
= 1 + (t ln |t| − t − ct)|1 = ln |x| − C + =⇒ C = −1
x x

Exercise 12. Rewrite the second property we want f to have:

Z x µ ¶
1 − f (x) 1 f 0x − f
f (t)dt = =⇒ f (x) = − 2 −
1 x x x2

So then
µ ¶
1
0 1
f + x− f =−
x x
Z Z µ ¶ µ ¶ µ ¶
1 1 2 1 2
A(x) = P (t)dt = t− dt = x − ln x − a − ln a
t 2 2
x2 −a2 a x a2 −x2
eA(x) = e 2 ; e−A(x) = e 2
x a
µZ x ¶
x a −x2
2 −1 t2 −a2
f (x) = e 2
2
ae 2 dt + b
a a t

Exercise 13.

v = yk
v 0 + kP v = kQ = ky k−1 y 0 + kP y k = kQ where k = 1 − n
v 0 = ky k−1 y 0
=⇒ y 0 y −n + P y 1−n = Q =⇒ y 0 + P y = Qy n
133
Exercise 14. y 0 − 4y = 2ex y 1/2
1 1 1 1 1
n= k = 1 − = ; v = y 1/2 ; v 0 + (−4)v = (2ex ) = v 0 − 2v = ex
2 2 2 Z 2 2
x
A(x) = P (t)dt = −2(x − a) = −2x
a
µZ ¶
2x
¡ ¢
v=e e e dt + b = e2x −e−x + b = be2x − ex
t −2t

√ √
=⇒ y = (1 + 2)2 e4x − 2(1 + 2)e3x + e2x
y(x) = b2 − 2b + 1 = 2

b=1+ 2

Exercise 15. y 0 − y = −y 2 (x2 + x + 1), n = 2 k = 1 − n = −1, v = y k v = y −1 .


v 0 + kP v = kQ v 0 + (−1)(−1)v = (−1)(−(x2 + x + 1)) = v 0 + v = x2 + x + 1
Z Z x
A(x) = P (t)dt = 1dt = x
0
µZ ¶
¡ ¢¯x
v = e−x (t2 + t + 1)et dt + b = e−x t2 et − tet + 2et ¯0 + be−x = (x2 − x + 2) − (2e−x ) + be−x
1
y=
x2 − x + 2 − 2e−x
Exercise 16.
1
v 0 + − v = 2x2
µZ x ¶
ln x
v=e 2x2 e− ln x dx + C = x(x2 + C) = x3 + Cx

Then since v = y k , k = 1 − n,
y = (x3 + Cx)2 = x2 (x2 + C)2 ; x 6= 0

y = x2 (x2 − 1)2
Check:
y 0 = 2x(x2 − 1)2 + x3 (4)(x2 − 1)
2x2 + x4 (4)(x2 − 1) − 2x2 (x2 − 1)2 = 4x3
1
Exercise 17. xy 0 + y = y 2 x2 log x on (0, +∞) with y = 2 when x = 12 , x 6= 0.
y
y0 +
= y 2 x log x
x
k = 1 − n = 1 − 2 = −1, v = y k = y −1
µ ¶
1
v 0 + kP v = kQ; v 0 + − v = −1x log x
x
µZ ¶ µZ ¶
−x log x
v=x dt + C = x − log tdt + C = 1 1 1
x y= C = − (1 − log ) + 2
Cx + x − x2 log x
2 2 2
= x(−(x log x − x) + C) = −x2 log x + x2 + Cx
Check:
y 0 = (C + 2x − 2x log x − x)(−y 2 )
y 2 (−C + −2x + 2x log x + x + C + x − x log x)

1 1
y= ; x = 1 y(x = 1) = b = 2
2 2
1
y=
x(x ln x − x + 3)
134
√ √
Exercise 18. 2xyy 0 + (1 + x)y 2 = ex on (0, +∞), y = e when x = 1; y = − e when x = 1.
If x 6= 0, y 6= 0,
(1 + x) e2 −1
y0 + y= y
2x 2x
k = 1 − n = 2; v = y k = y 2
µ ¶
0 0 1+x 2ex 1+x ex
v + 2P v = 2Q; v + 2 v= =⇒ v 0 + v= = v 0 + Pv = Qv
2x 2x x x
Z x Z x ³x´ Rx
1+t x
Av = Pv = dt = ln + (x − a); e a Pv dt = eln x/a+x−a = ex−a
a a t a a
µZ x t ¶ µ ¯x ¶
ae −x+a
e t t−a a 1 2t−a ¯¯
v= e dt + bv = e−x+a e ¯ + bv
x a t a x 2a a
µ 2x−a ¶
a −x+a e ea
= e − + bv
x 2a 2a
s µ ¶
a −x+a e2x−a ea
y=± e − + bv
x 2a 2a
Now yk = v
³ ´
e2−1 1
y 2 = v = e = 11 e−1+1 − e2 + bv = bv = e
2(1)
r ³ ´
2x−1
(1) bv = e =⇒ y = x1 e−x+1 e 2 − 2e + e
r ³ ´
2x−1
(2) bv = e =⇒ y = − x1 e−x+1 e 2 − 2e + e
q q
ex e−x+2a bv ae−x+a 1+x−e2(0) (1−x)+2bv 0e−x+a
If we could take a = 0, then limx→0 y = ±
(3)
2x − 2x + x =± 2x = ±1

If we consider limx→0 y 2 , and let a go to 0, then


1¡ x ¢ sinh x
v= e − e−x =⇒
x x
Exercise 19. The Ricatti equation is y 0 + P (x)y + Q(x)y 2 = R(x).
1
If u is a known solution, y = u + v is also a solution if v satises a rst-order ODE.
−1 0
(u + 1/v)0 = u0 +
v
v2
−v 0 1 2u 1
y 0 + P y + Qy 2 = R =⇒ u0 + 2 + P (u + ) + Q(u2 + + 2) = R
v v v v
=⇒ v 0 − P v = Q(2uv + 1)

Exercise 20. y 0 + y + y 2 = 2, y = 1, −2.


(1) If −2 ≤ b < 1,
y 0 + y + y 2 = 2 P = 1, Q = 1, R = 2
1
v 0 + (−P − 2Qu)v = Q y = u +
v
u=1
v 0 + (−1 − 2(1)(1))v = v 0 − 3v = Q = 1
µZ ¶ µ −3t ¯x ¶
e ¯¯ 1¡ ¢
v = e3x 1e−3t dt + b = e3x ¯ + b = be3x − 1 − e3x−3a
−3 a 3
3
y =1+
3be3x − (1 − e3x−3a )
1
b=1+
b
3 2
y(0) = 1 + =⇒ b − b − 1=0
3b − (1 − e−3a ) √
1± 5
b=
2
135

3 1− 5
y =1+ b=
3be3x − (1 − e3x ) 2
(2)

u = −2
0 0
v + (−1 − 2(1)(−2))v = v + 3v = 1
µZ x ¶ µ 3x ¶
−3x 3t −3x e − e3a 1 − e3a−3x
v=e e dt + b = e + be−3x = + be−3x
a 3 3
3 3 a=0 3 √
y = −2 + ; y(0) = −2 + −−→ y(0) = −2 + =⇒ b = −1 ± 2
1 − e3a−3x + 3be−3x 1 − e3a + 3b 3b
3 √
b ≥ 1 or b < −2, y = −2 + b = −1 ± 2
1 − e−3x + 3be−3x

8.7 Exercises - Some physical problems leading to rst-order linear differential equations.

Exercise 3.

y0
(1) y 0 = −αy(t). y(T ) = y0 e−αT = n. n = eαT so the relationship between T and n doesn't depend upon
1
y0 . ln e = T .
k
(2) f (a) = y0 e−ka ; f (b) = y0 e−kb .
f (a) f (a) 1 f (a)
= e−ka+kb =⇒ ln = −k(a − b); ln = −k
f (b) f (b) a − b f (b)
 
ln ff (a) µ ¶a/b
(b) t f (b) f (a) 1− a
f (t) = y0 exp   ; f (a) = y0 ; =⇒ = y0 b
a−b y0 (f (b)) a/b

µ ¶ b−a
b µ ¶ a−b
t b t
f (a) f (a) (f (a)) b−a f (a)− b−a
f (t) = = a t =
(f (b))a/b f (b) f (b) b−a f (b) a−b
b−t
f (a) b−a b−t t−a
= a−t = f (a) b−a f (b) b−a
(f (b)) b−a

b−t b − a − (b − t) t−a
w(t) = ; 1 − w(t) = =
b−a b−a b−a
Exercise 4. F = mv 0 = w0 − 34 v

w0 = 192
w0 1 1 w0
w0 v0 = − v =⇒ v 0 + v = = 32
=6=m m 8 8 m
g
µZ t ¶ µ ¶ ³ ´
t w0 t −t 8w0 t/8
v(t) = e− 8 e dt + b = e
8 8 (e − 1) + 0 = 256(1 − e−t/8 )
0 m m
v(10) = 256(1 − e−5/4 ) = 256(1 − 37/128) = 182
12 w0
F = mv 0 = w0 − 12v v 0 + v = = v 0 + 2v = 32
m m
µZ t ¶
¡ ¢ ³ ´
v(t) = e−2t e2x 32dx + b = e−2t 16(e2t − e2t0 ) + b = 16(1 − e2(t0 −t) ) + be−2t
t0

v(10) = be−2(10) = 182 =⇒ b = 182e20


v(t) = 16 + 166e20−2t
So then
(
256(1 − e−t/8 ) if t < 10
v(t) =
16 + 166e20−2t if t > 10
Exercise 7.

136
(1)

y 0 (t) = (y − M )k = ky − kM y 0 + −ky = −kM


Z
y = ekt ( −kM e−kt dt + b) = ekt (M (e−kt − 1) + b) M = 60◦

y(0) = b = 200◦
yf = ekT (M (e−kT − 1) + 200) = M + ekT (200 − M )
µ ¶ µ ¶
1 yf − M 1 1 3 1
ln = k = (ln (60) − ln (140)) = ln = (ln 3 − ln 7)
T b−M T T 7 30
ln 3−ln 7
t
y(t) = 60 + 140e 30

(2)

(ln 7 − ln 3)
y(t) = 60 + 140e−kt ; k=
µ ¶ 30
Y − 60 (ln 140 − ln (T − 60))
ln = −kt =⇒ tf = for 60 < T ≤ 200
140 k
(3) tf = ln (140)−ln
k
(30)
= ln30 14
7/3 ln 3 = 54 minutes
1
(4) M = M (t) = M0 − αt α = 10
µZ ¶ µZ t ¶
kt −kt kt −ku
y=e −kM e dt + b = e −k(M0 − αu)e du + b =
0
Z t
= −kekt (M0 e−ku − αueku )du + bekt =
0
à ¯t µ −ku ¶¯t !
M0 e−ku ¯¯ ue −e−ku ¯¯ kt
= −ke −α + ¯ + be =
−k ¯0 −k k2 0
µ µ −kt ¶¶
kt M0 −kt te e−kt 1
= −ke (1 − e ) − α − 2 + 2 + bekt =
k −k k k
y(t) = −M0 ekt + M0 − αt − α/k + αekt /k + bekt = (−M0 + α/k + b)ekt + (M0 − αt − α/k) =
µ ¶
3 t 3
y(t) = (140 + )e−kt + (60 − − )
(ln 3 − ln 7) 10 ln 3/7

Exercise 8. y 0 (t) = −k(y − M0 ); y 0 + ky = kM0 .


µZ tf ¶
−kt
¡ ¢
y=e kM0 e du + b = e−ktf M0 (ektf − ekti ) + b = M0 (1 − e−k(tf −ti ) ) + be−ktf
ku
ti

y(tf ) − M0 = −M0 e−k(tf −ti ) + be−ktf


µ ¶ µ ¶
65 − M0 1 75 − M0
65 − M0 = −M0 e−k(5) + 75e−k(5) = (75 − M0 )e−5k =⇒ ln = −5k; k= ln
=⇒ 75 − M0 5 65 − M0
µ ¶
60 − M0
60 − M0 = −M0 e−k(5) + 65e−k(5) = (65 − M0 )e−5k =⇒ ln = −5k
65 − M0
65 − M0 60 − M0
=⇒ =
75 − M0 65 − M0
M0 = 55

Exercise 9.

Let y(t) = absolute amount of salt.


Water is leaving according to
³ ´³ w(t)´ = w0 + (3 − 2)t = w0 + t.
2 gal y(t) salt
Salt leaving =
min. w(t)
So then
µ ¶
−2y
0 w0 + t
y = =⇒ ln y = −2(ln (w0 + t) − ln w0 ) = −2 ln
w0 + t w0
137
is the equation of motion given by the problem.

“ ¶−2
”−2 µ
ln w0 + t
w0 +t
y(t) = Ce =C w0

w0
µ ¶2 µ ¶2
100 100 25 625
y(t) = 50 y(t = 60 min.) = 50 = 50 = ' 19.53
100 + t 160 64 32

Exercise 10.

Let y be the dissolved salt (total amount of) at t time. The (total) amount of water at any given time in the tank is w = w0 + t.
There is dissolved salt in mixture that is leaving the tank at any minute. There is also salt from undissolved salt in the tank
that is “coming into” the dissolved salt, adding to the amount of dissolved salt in the mixture. Thus

³y´ ³y ´ −1 gal
y 0 (t) = (−2) +α −3 ; α=
w w 3 min
We obtained α easily by considering only the dissolving part and how it dissolves 1 pound of salt per minute if the salt
y
concentration,
w was zero, i.e. water is fresh.

−7 y 7 y
y 0 (t) = + 1; y0 = =1
3 w 3 w0 + t
7/3
P =
w0 + t
Z Z µ ¶
7/3 7 t 7 w0 + t
P = = ln (w0 + t)|0 = ln
w0 + t 3 3 w0
“ ”
w0 +t −7/3
µZ t “ ”
w0 +u 7/3

− ln ln
y=e w0
(1)e w0
du + b =
0
µ ¶7/3 ÃZ t µ ¶7/3 ! µ ¶7/3 à õ ¶10/3 ! !
w0 w0 + u w0 3w0 w0 + t
= du + b = −1 +b =
w0 + t 0 w0 w0 + t 10 w0
µ ¶7/3 à õ ¶10/3 ! !
100 3(100) 100 + 60
y= − 1 + 50 ' 54.78 lbs.
100 + 60 10 100

Exercise 11. LI 0 (t) + RI(t) = V (t) V (t) = E sin ωt.


Z t
V (x) Rx/L
I(t) = I(0)e−Rt/L + e−Rt/L e dx
0 L
Z t
E
I(t) = I(0)e−Rt/L + e−Rt/L sin ωxeRx/L dx
L 0
R
Using eax sin bxdx = aeax sin bx − beax cos bx
à !
R Rt Rt
−Rt Ee−Rt/L Le
L sin ωt − ωe L cos ωt ω
I(t) = I(0)e L + ¡ R ¢2 + ¡ ¢2
L +ω 2 R
+ ω2
L L

I(0) = 0 So

R
E sin ωt − ω cos ωt EωL E(R sin ωt − ωL cos ωt) EωL
I(t) = L
¡ R ¢2 + 2 2
e−Rt/L = 2 + (ωL)2
+ 2 2
e−Rt/L =
L +ω 2 R + (ωL) R R + (ωL)
L
ωL
sin α = p
R + (ωL2 )
2

E sin (ωt − α) EωL


=⇒ I(t) = p + 2 2 L2 )
e−Rt/L
2
R + (ωL) 2 (R + ω
L=0 sin α = 0
138
Exercise 12.
(
E if 0<a≤t<b
E(t) =
0 otherwise
Z t
I(t) = e−Rt/L 0=0 for t<a
0
Z t
E Rx/L E L ³ Rt/L ´ E³ R(a−t)
´
I(t) = e−Rt/L e dx = e−Rt/L e − eRa/L = 1−e L
a L L R R
E
I(b) = (1 − eR(a−b)/L )
R
for t > b, I(t) = Ke−Rt/L
Ee−Rt/L Rb Ra
=⇒ I(t) = (e L = e l ) for I(b) = I(b)
R
dx
Exercise 13. From Eqn. 8.22,
dt = kx(M − x)

dx dx 1/kdx
= kx(M − x) = kM x − kx2 ; =⇒ 2
= dt = = dt
dt kM x − kx x(M − x)
µ ¶
1 1 1 ln x + − ln (M − x)
=⇒ kdt = + dx =
x M −x M M
x M k(t−ti )
M k(t − ti ) = ln ; e (M − x) = x
M −x
M eM k(t−ti ) M
x(t) = =
1 + eM k(t−ti ) 1 + e−M k(t−ti )

Exercise 14. Note that we are given three equally spaced times.

M − x2
M = x2 + x2 e−α(t2 −t0 ) ; = e−α(t2 −t0 )
x2
µ ¶
M − x2 x1
= e−αt2 +αt0 +αt1 −αt0 = e−α(t2 −t1 )
x2 M − x1
µ ¶ µ ¶
M − x3 x2 M − x2 x1
= e−α(t3 −t2 ) =
x3 M − x2 x2 M − x1
(M − x3 )(M − x1 )x22 = x1 x3 (M − x2 )2 = x1 x3 (M 2 − 2M x2 + x22 ) = x22 (M 2 − M (x1 + x3 ) + x1 x3 )
(x22 − x1 x3 )M 2 = M (x22 (x1 + x3 ) + −2x2 x1 x3 ) = (−x1 (x3 − x2 ) + x3 (x2 − x1 ))x2
(x3 (x2 − x1 ) − x1 (x3 − x2 ))
=⇒ M = x2
x22 − x1 x3

Exercise 15.
Z t µ ¶
dx dx x
= k(t)M x − k(t)x2 = k(t)dt =⇒ M k(u)du = ln
dt M x − x2 ti M −x
Rt
x M k(u)du
= e ti
M −x
Rt
M k(u)du
Me ti
M
x= Rt = Rt
M k(u)du −M k(u)du
1+e ti
1+e ti

Exercise 16.

(1) M = 23 92(23−3.9)−3.9(92−23)
232 −3.9(92) = 201
(2)
µ ¶ µ ¶
150(122 − 92) − 92(150 − 122) 150(30) − 92(28)
M = 122 = 122 = 216
(122)2 − 92(150) (122)2 − 92(150)
(3) Reject.

139
8.14 Exercises - Linear equations of second order with constant coefcients, Existence of solutions of the equation
y 00 + by = 0, Reduction of the general equation to the special case y 00 + by = 0, Uniqueness theorem for the equation
y 00 + by = 0, Complete solution of the equation y 00 + by = 0, Complete solution of the equation y 00 + ay 0 + by = 0.

Exercise 1. y 00 − 4y = 0 y = c1 e2x + c2 e−2x .


00
Exercise 2. y + 4y = 0 y = c1 cos (2x) + c2 sin (2x).
Use Theorem 8.7.

Theorem 27. Let d = a2 − 4b be the discrimnant of y 00 + ay 0 + by = 0.


Then ∀ solutions on (−∞, ∞) has the form

(22) y = e−ax/2 (c1 u1 (x) + c2 u2 (x))


where

(1) If a = 0, then u1 (x) = 1 and u2 (x) = x √


kx −kx d
(2) If d > 0, then u1 (x) = e and u2 (x) = e , where k =
2
1

If d < 0, then u1 (x) = cos kx and u2 (x) = sin kx; where k =
(3)
2 −d

Exercise 3. y 00 − 4y 0 = 0; a = −4.

y = e2x (c1 e2x + c2 e−2x ) = c1 e4x + c2

Exercise 4. y 00 + 4y 0 = 0
y = e−2x (c1 e2x + c2 e−2x ) = c1 + c2 e−4x
√ √
Exercise 5. y 00 − 2y 0 + 3y = 0 d = 4 − 4(3) = −8 =⇒ y = e−x (c1 sin 2x + c2 cos 2x)
00 0 x
Exercise 8. y − 2y + 5y = 0 d = −16 y = e (c1 cos 2x + c2 sin 2x)
00 0
Exercise 9. y + 2y + y = 0 d = 4 − 4(1)(1) = 0 y = e−x (1 + x)
Exercise 10. y 00 − 2y 0 + y = 0 d = 4 − 4(1)(1) = 0 y = ex (1 + x)
Exercise 11. y 00 + 32 y 0 = 0 y = 1, y 0 = 1; x = 0 d = 9
4 >0
−3 3x −3x −3x
y=e 4 x
(c1 e + c2 e 4 ) = c1 + c2 e
4 2

−2 5 −2 −3x
c2 = =⇒ y = + e 2
3 3 3
Exercise 12. y 00 + 25y = 0; y = −1, y 0 = 0, x = 3.
y = c1 sin 5x + c2 cos 5x −1 = c1 sin 15 + c2 cos 15
µ ¶ c1 = − sin 15
y 0 = 5c1 cos 5x + −5c2 sin 5x cos2 15 1
−1c1 (sin 15 + ) = c1 c2 = − cos 15
0 = 5c1 cos 15 − 5c2 sin 15c2 sin 15 = c1 cos 15 sin 15 sin 15

y = − sin 15 sin 5x − cos 15 cos 5x

y 00 − 4y 0 − y = 0;
Exercise 13. y = 2; y 0 = −1 when x=1
d = 10 − 4(1)(−1) = 20
√ √
y = c1 e(2+ 5)x
+ c2 e(2− 5)x
√ √
y(x = 1) = c1 e2+ 5 + c2 e2− 5 = 2 √ √ √ √
√ √ √ √ =⇒ (5 + 2 5)c1 e2+ 5 + (5 − 2 5)c2 e2− 5 = 0
y 0 (x = 1) = (2 + 5)c1 e2+ 5 + (2 − 5)c2 e2− 5 = −1

2 5 − 5 −2√5
c1 = √ c2 e
5+2 5
à √ ! √ √ √ √
2 5−5 √
2− 5

2− 5 4 5c2 e2− 5 5 + 2 5 √5−2 2 5 − 5 −2−√5
√ c2 e + c2 e =2= √ =⇒ √ e = c2 c1 = √ e
5+2 5 5+2 5 2 5 2 5
√ √
2 5 − 5 −2−√5 (2+√5)x 5 + 2 5 √5−2 (2−√5)x
y= √ e e + √ e e
2 5 2 5
140
y 00 + 4y 0 + 5y = 0, with y = 2 and y 0 = y 00
Exercise 14. when x=0
16 − 4(1)(5) = −4
y = e−2x (c1 sin 2x + c2 cos 2x) y(x = 0) = c2 = 2
y 0 = −2e−2x (c1 sin 2x + 2 cos 2x) + 2e−2x (c1 cos 2x − 2 sin 2x)
y 00 = 4e−2x (c1 sin 2x + 2 cos 2x) − 8e−2x (c1 cos 2x − 2 sin 2x) + 4e−2x (−c1 sin 2x − 2 cos 2x)
y 0 (0) = −2(2) + 2(c1 ) = −4 + 2c1
4
y 00 (0) = 4(2) + (c1 ) + 4(−2) = −c1 = −4 + 2c1 c1 =
3
4
y = e−2x ( sin 2x + 2 cos 2x)
3

y 00 − 4y 0 + 29y = 0
Exercise 15.

d = 16 − 4(1)(29) = −100 =⇒ u = e2x (c1 sin 5x + c2 cos 5x).


v: y 00 + 4y 0 + 13y = 0
d = 10 − 4(13) = −36 =⇒ v = e−2x (b1 sin 3x + b2 cos 3x)
v(0) = b2

u(0) = 1(0 + c2 ) = c2 = 0
u = e2x c1 sin 5x v = e−2x b1 sin 3x
u0 = 2e2x c1 sin 5x + e2x c1 5 cos 5x v 0 (0) = 3b1
³π ´ 1 1 2x
u0 = 1 = 2eπ c1 (1) c1 = π u= e sin 5x
2 2e 2eπ
1
u0 (0) = π 5 5
2e v = e−2x sin 3x
5 6eπ
u0 (0) = v 0 (0) =⇒ b1 = π
6e
Exercise 16.
3x 5x 5x
y 00 − 3y 0 − 4y = 0 u 9 − 4(1)(−4) = 25 u=e 2 (c1 e 2 + c2 e− 2 )
00 0 −2x 3x −3x
y + 4y − 5y = 0 v 16 − 4(1)(−5) = 36 + b2 ev=e) (b1 e
µ ¶
3x 5x
u = 2e 2 c1 (sinh )
u(0) = c1 + c2 = 0v(0) = b1 + b2 = 0 =⇒ 2
v = 2b1 e−2x (sinh (3x))
µ ¶
3 3x 5x −5x 3x 5 5x
u0 = c1 e 2 (e 2 − e 2 ) + 2e 2 c1 cosh v 0 = −4b1 e−2x sinh (3x) + 6b1 e−2x cosh (3x)
2 2 2
v 0 (0) = 6b1
u0 (0) = 5c1
6b1
c1 =
5
Exercise 17.

y 00 + ky = 0 Assume k>0
√ √
d = −4(k) y = c1 sin kx + c2 cos kx
√ √ y(0) = c2 = 0
d −4k √ √ √
= = −k
2 2 y(1) = c1 sin k1 = 0 =⇒ k = nπ

k < 0; −k = κ > 0

κx

− κx

y = c1 e + c2 e ;y(0) = c1 + c2 = 0 y = c1 sinh κx

y = c1 sinh κ1 = 0 c1 = 0
so if k < 0, there are no nontrivial solutions satisfying fk (0) = fk (1) = 0
141

d 2k
Exercise 18. y 00 + k 2 y = 0 d = −4k 2 < 0 2 = 2 =k>0

y = c1 sin kx + c2 cos kx y(a) = b = c1 sin ka + c2 cos ka


0
y = kc1 cos kx − c2 k sin kx y 0 (a) = m = kc1 cos ka − c2 k sin ka
kb cos ka = kc1 cos ka sin ka + c2 k cos2 ka
m sin ka = kc1 cos ka sin ka − c2 k sin2 ka
kb cos ka − m sin ka
kb cos ka − m sin ka = c2 k =⇒ c2 =
µ ¶k
kb kb cos ka − m sin ka
c1 sin ka = b − c2 cos ka = − cos ka =
k k
kb(1 − cos2 ka) + m sin ka cos ka kb sin2 ka + m sin ka cos ka
= =
k k
kb sin ka + m cos ka
c1 =
µ k ¶ µ ¶
kb sin ka + m cos ka kb cos ka − m sin ka
y= sin kx + cos kx
k k
k = 0 =⇒ y = mx − ma + b

Exercise 19.

(1) y = k1 sin x + k2 cos x

b2 c1 = k1 s2 c1 + k2 c2 c1
b2 = k1 sin (a2 ) + k2 cos (a2 ) = k1 s2 + k2 c2 b2 c1 − b1 c2
− (b1 c2 = k1 s1 c2 + k2 c1 c2 ) =⇒ k1 =
b1 = k1 sin (a1 ) + k2 cos (a1 ) = k1 s1 + k2 c1 s2 c1 − s1 c2
=⇒b2 c1 − b1 c2 = k1 (s2 c1 − s1 c2 )
s1 b2 = k1 s1 s2 + k2 s1 c2 s1 b2 − s2 b1
=⇒ k2 =
−(s2 b1 = k1 s1 s2 + k2 c1 s2 ) s1 c2 − c1 s2
µ ¶ µ ¶
b2 cos a1 − b1 cos a2 b2 sin a1 − b1 sin a2
y= sin x + cos x
sin a2 cos a1 − sin a1 cos a2 sin a1 cos a2 − cos a1 sin a2
b2 cos a1 − b1 cos a2 b2 sin a1 − b1 sin a2
y= sin x + cos x if a2 − a1 6= πn
sin (a2 − a1 ) sin (a1 − a2 )
b2 c1 − b1 c2 = 0
otherwise, if a2 − a1 = πn; b2 c1 = b1 (−1)n c1 ; if c cos (a1 ) 6= 0, b2 = b1 (−1)n
b2 s1 − b1 s2 = 0
π
a1 = a2 =
(2) It's true if
4; b1 = b2 .
00 2
(3) y +k y =0
· ¸· ¸ · ¸
y = A sin kx + B cos kx S1 C1 A b
= 1
S2 C2 B b2
y(a1 ) = A sin ka1 + B cos ka1 = b1 = AS1 + BC1 · ¸ · ¸µ ¶· ¸
A C2 −C1 1 b1
y(a2 ) = A sin ka2 + B cos ka2 = b2 = AS2 + BC2 =
B −S2 S1 S1 C2 − C1 S2 b2
S1 C2 − C1 S2 = sin ka1 cos ka2 − cos ka1 sin ka2 = sin (k(a1 − a2 ))
b1 cos (ka2 ) − b2 cos (ka1 ) −b1 sin (ka2 ) + b2 sin (ka1 )
y= sin ka1 + sin ka2
sin (k(a1 − a2 )) sin (k(a1 − a2 ))
µ ¶
b2 − b1
if k(a1 − a2 ) 6= πn k = 0; y= (x − a1 ) + b1
a2 − a1

Exercise 20.

(1)

u1 (x) = ex ; u2 (x) = e−x u02 = −e−x u002 = e−x =⇒ y 00 − y = 0


142
(2)

u1 = e2x u2 = xe2x
u01 = 2e2x u02 = e2x + 2xe2x
u001 = 4e2x u002 = 2e2x + 2e2x + 4xe2x = 4e2x + 4xe2x = 4e2x (1 + x)
u002 − 4u02 + 4u2 = 0 =⇒ y 00 − 4y 0 + 4y = 0

(3)

u2 (x) = e−x/2 sin x


−x/2
u1 (x) = e cos x; −1 −x/2
u02 = e sin x + e−x/2 cos x
0 1 −x/2 −x/2 2
u1 = − e cos x + −e sin x
2 1
u002 = e−x/2 sin x + −e−x/2 cos x − e−x/2 sin x =
1 4
u001 = e−x/2 cos x + e−x/2 sin x + −e−x/2 cos x
4 −3 −x/2
= e sin x − e−x/2 cos x
−3 −x/2 −x/2 4
= e cos x + e sin x
4 5
u002 + u02 + u2 = 0
4
5
y 00 + y 0 + y = 0
4

(4) u1 (x) = sin (2x + 1); u2 (x) = sin (2x + 2)

u01 = 2 cos (2x + 1) u02 = 2 cos (2x + 2)


u100 = −4 sin (2x + 1) u002 = −4 sin (2x + 2)
y 00 + 4y = 0

(5)

u1 = cosh x
u01 = sinh x y 00 − y = 0
u001 = cosh x

Exercise 21. w = u1 u02 − u2 u01 .


(1) w=0 ∀x ∈ open interval I,
µ ¶0
u2 u02 u1 − u01 u2 u2
= = 0 =⇒ =c
u1 u21 u1
u2
If
u1 is not constant, then w(0) 6= 0 for at least one c in I (otherwise, it'd be constant).
0
(2) w = u1 u002 − u2 u001

Exercise 22.

(1) w0 + aw = u1 u002 − u2 u001 + a(u1 u02 − u2 u01 ) = u1 (−bu2 ) + −u2 (bu1 ) = 0


w(x) = w(0)e−ax if w(0) 6= 0, then w(x) 6= 0 ∀x.
(2) u1 6= 0 If w(0) = 0, w(x) = 0 ∀x, so uu12 constant. If uu21 constant, w(0) = 0 since from the previous part.

Exercise 23. Recall the properties of the Wronskian.

(1) If W (x) = v1 (x)v20 − v2 v10 = v1 v20 − v2 v10 = 0 ∀x ∈ I ,


v
then 2 constant on I
v1
(2) W = v1 v200 − v2 v100
0

(3) W 0 + aW = 0 if v1 , v2 are solutions to y 00 + ay 0 + by = 0


W (x) = W (0)e−ax
So if W (0) 6= 0, W (x) 6= 0 ∀ x
143
Consider adding together the solutions and the solution's derivatives into some function f. By the linearity of the differential
equation, we know that f is also a solution since it is a linear superposition of solutions.

y(x) = Av1 (x) + Bv2 (x) y 0 (x) = Av10 (x) + Bv20 (x)
f (0) = Av1 (x) + Bv2 (0) f 0 (0) = Av10 (0) + Bv20 (0)
· ¸· ¸ · ¸ · 0 ¸· ¸ · ¸
v1 (0) v2 (0) A f (0) 1 v2 (0) −v2 (0) f (0) A
= 0 =⇒ =
v10 (0) v20 (0) B f (0) W (0) −v 1
0
(0) v 1 (0) f 0
(0) B
since W (0) 6= 0, this division is allowed above
µ ¶ µ ¶
v20 (0)f (0) − v2 (0)f 0 (0) v1 (0)f 0 (0) − v10 (0)f (0)
so y(x) = v1 (x) + v2 (x)
W (0) W (0)

f (0), f 0 (0) are initial conditions for y . f (0), f 0 (0) are arbitrary.
But since W (0) 6= 0, W (0) = v1 (0)v20 (0) − v2 (0)v10 (0), we can do things like
v20 (0)f (0) − v2 (0)f 0 (0) v2 (0)f 0 (0) − v10 (0)f (0)
f (0) = 0 0 v1 (0) + v2 (0)
v1 (0)v2 (0) − v2 (0)v1 (0) W (0)

8.17 Exercises - Nonhomogeneous linear equations of second order with constant coefcients, Special methods for
determining a particular solution of the nonhomogeneous equation y 00 + ay 0 + by = R.

y 00 − y = x
Exercise 1. homogeneous solution c1 ex + c2 e−x . yp = −x
y = c1 e + c2 e−x − x
x

Exercise 2. y 00 − y 0 = x2 For the homogeneous solution


x ¡ x x¢
y − y0 = 0
00
d = (−1)2 − 4(1)(0) = 1 yh = e 2 c1 e 2 + c2 e− 2 = c2 ex + c1

yp = Ax3 + Bx2 + Cx + D
−1 3
yp0 = 3Ax2 + 2Bx + C y = c1 + c2 ex + x + x2
3
yp00 = 6Ax + 2B

Exercise 3. y 00 + y 0 = x2 + 2x
x ¡ x x¢
e− 2 c1 e− 2 + c2 e 2 = c1 e−x + c2
P = Ax3 + Bx2 + Cx + D; P 0 = 3Ax2 + 2Bx + C P 00 = 6Ax + 2B
1
3Ax2 + 2Bx + C + 6Ax + 2B = x2 + 2x A= B=0 C=0
3
1
y = c1 e−x + c2 + x3
3
√ √
Exercise 4. y 00 − 2y 0 + 3y = x3 u = ex (c1 sin 2x + c2 cos 2x)
3(Ax3 + Bx2 + Cx + D)
1 2 8 16
2(3Ax2 + 2Bx + C) A= B= C= D=
3 3 9 27
(6Ax + 2B)
√ √ 1 2 8 16
y = C1 ex sin 2x + C2 ex cos 2x + x3 + x2 + x +
3 3 9 27
00 0 2
Exercise 5.
5x
³y 3x− 5y −3x
+ 4y
´ = x − 2x + 1
yh = e 2 e 2 +e 2 = c1 e4x + c2 ex
p
d = 25 − 4(4) = 3
4(Ax2 + Bx + C) 4Ax2 + 4Bx + 4C 1 5
1 1 − + 4C = 1
− 5(2Ax + B) −10Ax − 5B A= B= 2 8
4 8 9
2A 2A C=
32
1 1 9
y = c1 e4x + c2 ex + x2 + x +
4 8 32
144
Exercise 6.

y 00 + y 0 − 6y = 2x3 + 5x2 − 7x + 2
−x −5x 5x
yh = e 2 (e 2 + e 2 ) = e−3x + e2x
p
d = 1 − 4(−6) = 5
yp = Ax3 + Bx2 + Cx + D −6Ax3 − 6Bx2 − 6Cx − 6D
yp0 = 3Ax2 + 2Bx + C =⇒ 3Ax2 + 2Bx + C
yp00 = 6Ax + 2B 6Ax + 2B
−1 1 −7
A= B = −1 C= D=
3 2 12
1 1 7
y = C1 e−3x + C2 e2x − x3 + −x2 + x −
3 2 12

Exercise 7.

y 00 − 4y = e2x v1 = e2x v20 = −2e−2x


yh = c1 e2x + c2 e−2x v10 = 2e2x v2 = e−2x

Use Theorem 8.9.

Theorem 28. Let v1 , v2 be solutions to L(y) = 0 where L(y) = y 00 + ay 0 + by


Let W = − v2 v10 . Then L(y) = R where
v1 v20
yp = t1 v1 + t2 v2
Z Z
R R
(23) t1 = − v2 dx; t2 (x) = v1 dx
W (x) W

Z
e2x 1
t1 = − e−2x = x
−4 4
Z 2x
w = e2x (−2)e−2x − e−2x 2e2x = −4
2x e e4x
t2 = e =
−4 −16
x e2x
yp = e2x +
4 −16
x e2x
y = c1 e2x + c2 e−2x + e2x +
4 −16

Exercise 8.

y 00 + 4y = e−2x
w = sin 2x − sin 2x(2) − 2 cos 2x cos 2x = 2
yh = c1 sin 2x + c2 cos 2x
Z Z
cos 2xe−2x dx −1
t1 = − = e−2x cos 2xdx (e−2x cos 2x)0 = −2e−2x cos 2x + −2e−2x sin 2x
2 2
Z Z
sin 2xe−2x 1 (e−2x sin 2x)0 = −2e−2x sin 2x + 2e−2x cos 2x
t2 = = e−2x sin 2xdx
2 2
µ −2x ¶0
e sin 2x − e−2x cos 2x e−2x cos 2x − e−2x sin 2x
= e−2x cos 2x t1 =
4 8
µ −2x ¶0 −2x
e sin 2x + e−2x
cos 2x e sin 2x + e−2x cos 2x
= e−2x sin 2x t2 =
−4 −8

e−2x sin 2x cos 2x − e−2x sin2 2x e−2x sin 2x cos 2x + e−2x cos2 2x e−2x
yp = + =
8 −8 8
e−2x
y = c1 sin 2x + c2 cos 2x +
8
145
Exercise 9. y 00 + y 0 − 2y = ex d2 = 1 − (4)(1)(−2) = 9
−1x
³ 3x −3x
´
yh = e 2 e 2 + e 2 = ex + e−2x
(xex )0 = ex + xex
1
+(xex )00 = +(2ex + xex ) y = c1 ex + c2 e−2x + xe2
3
=⇒ 3ex + 2xex
x
³ 3x 3x
´
Exercise 10. y 00 + y 0 − 2y = e2x . yh = e− 2 c1 e 2 + c2 e− 2 = c1 ex + c2 e−2x .

W (x) = v1 v20 − v2 v10 = ex (−2)e−2x − e−2x ex = −3e−x


Z Z
v2 R v1 R
t1 = − t2 =
W W
Z −2x 2x Z x 2x
e e 1 x e e −1 4x
t1 = − = e t2 = = e
−3e−x 3 −3e−x 12
1 1 1
y1 = t1 v1 + t2 v2 = ex ex + − e4x e−2x = e2x
3 12 4
x −2x 1 2x
y = c1 e + c2 e + e
4
Exercise 11. y 00 + y 0 − 2y = ex + e2x .

Consider solutions to Exercise 9,10.

L(ya ) = ex ; L(yb ) = e2x ; L(ya + yb ) = ex + e2x


1 1
=⇒ y = c1 ex + c2 e−2x + xex + e2x
3 4
Exercise 12. y 00 − 2y 0 + y = x + 2xex . d = 4 − 4(1) = 0. Recall the denition to be learned for this section of exercises:

Theorem 29. Let d = a − 4b be the discriminant of y + ay 0 + by = 0.


2 00
Then every solution of this equation on (−∞, ∞)
has the form

(24) y = e−ax/2 (c1 u1 (x) + c2 u2 (x))


(1) If d = 0 then u1 = 1, u2 = x √
(2) If d > 0, u1 = ekx ; u2 = e−kx , k = 2d

−d
(3) If d < 0, u1 = cos kx, u2 = sin kx, k =
2

yh = ex (c1 + c2 x) = c1 ex + c2 xex
Z Z x
−xex (2xex ) 2x3 e (2xex )
t1 = = t2 = = x2
e2x −3 e2x
W (x) = ex (ex + xex ) − (xex )(ex ) = e2x
2x3 x x 3 ex
yp = e + x3 ex =
−3 3
x3 ex
y= + c1 ex + c2 xex
3
e−x
Exercise 13. y 00 + 2y 0 + y = x2
yh = e−x (c1 + c2 x) = c! e−x + c2 xe−x
³ ´ ³ ´
Z −xe−x e−x Z e−x e−x
x2 x2 −1
t1 = = − ln x t2 = =
e−2x e−2x x
−x −x −x −x −x −2x
W = e (e + −xe ) − (−e )(xe ) = e
µ ¶
−1
yp = − ln xe−x + xe−x = − ln xe−x − e−x
x
y = c1 e−x + c2 xe−x + (− ln x − 1)e−x
146
Exercise 14. y 00 + y = cot2 x. yh = c1 sin x + c2 cos x.
Z Z Z
− cos x 2 cos x cos2 x cos x(1 − sin2 x) 1
t1 = cot xdx = 2 = 2 =− + − sin x
−1 sin x sin x sin x
Z Z Z
sin x cos2 x 1 − sin2 x
t2 = cot2 x = − =− = ln | csc x + cot x| + − cos x
−1 sin x sin x
yp = −1 − sin2 x + cos x ln | csc x + cot x| − cos2 x = −2 + cos x ln | csc x + cot x|

y = c1 sin x + c2 cos x − 2 + cos x ln | csc x + cot x|


2
Exercise 15. y 00 − y = 1+ex

yh = c1 ex + c2 e−x =⇒ W (x) = −ex e−x − ex e−x = −2


Z −x 2 Z
e 1+ex e−x
t1 = − = =
−2 1 + ex
Z Z Z
1 1 −x 1 −x −e−x
= − = −e − = −e + = −e−x + ln (1 + e−x )
ex 1 + ex 1 + ex e−x + 1
Z x 2
e 1+ex
t2 = = − ln |1 + ex |
−2
y = −1 + ex ln (1 + e−x ) + −e−x ln (1 + ex ) + c1 ex + c2 e−x
ex
Exercise 16. y 00 + y 0 − 2y = 1+ex
√2
−1± 1 −4(−2)
Discriminant:
2 = −2, 1 =⇒ yh = c1 ex + c2 e−2x =⇒ W = ex (−2)e−2x − e−2x ex = −3e−x
³ t ´
Z e−2t e t Z
1+e 1 1 −1
t1 = − −t
= t
= ln (1 + e−x )
−3e 3 1+e 3
³ ´
Z et et t Z Z 2
1+e 1 e3t u=et −1 u du
t2 = = −−−→ =
−3e−t −3 1 + et 3 1+u
Z Z
−1 −u −1 1 −1 1 2
= u+ = u + −1 + = ( u − u + ln u + 1)
3 u+1 3 u+1 3 2
−1 2x ex −1
= e + + ln (ex + 1)
6 3 3
−1 x −1 e−x e−2x
y1 = e ln (1 + e−x ) + + − ln (ex + 1) + c1 ex + c2 e−2x
3 6 3 3

Exercise 17. y 0 + 6y 0 + 9y = f (x); where f (x) = 1 for 1 ≤ x ≤ 2. f (x) = 0 for all other x.
d = 36 − 4(1)(9) = 0
−3x
yh = e (c1 + c2 x) = c1 e−3x + c2 xe−3x
W (x) = e−3x (e−3x − 3xe−3x ) − (xe−3x )(−3e−3x ) = e−6x
 Rx ³ 3t ´¯x
−te e3t ¯ −3xe3x +e3x

a<1<x −te3t
dt = + 9 ¯ = + 29 e3
 1
R2
3
6 6 3
1 6
9
3
t1 (x) = a < 1 < 2 < x 1 −te3t dt = −6e9+e + 2e9 = −5e + 2e9

 R ³ 3t ´¯x 9
1 < a < x < 2 x ¯ 3x 3x 3a
e3a
a
−te3t dt = −te 3 + e3t 9 ¯ = −xe 3 + e9 + ae3 − 9
a
Z −3t Z Z x
e f (t) 3t 1 ¡ 3x ¢
t2 (x) = = e f (t) = e3t = e − e3a
e−6t 3
µa ¶
−3x −3xe3x + e3x x
y1 = e +C +
9 3
1
y = c1 e−3x + c2 xe−3x + when 1 ≤ x ≤ 2; otherwise y = yh
9

Exercise 18. Start from y 00 − k 2 y = R(x). Suppose L(yp ) = yp00 − k 2 yp = R(x).


147
yh = c1 sinh (kx) + c2 cosh (kx); L(yh ) = 0
So consider L(yp + yh ) = R(x); yp + yh = y1 is a nother particular solution.

The key to this problem is to apply the integration directly on the ODE itself, not to go the other way around by differenti-
ating the supposed particular solution.
Rx Z x Z x Z x
dt sinh (k(x−t)) d2 y
−−0−−−−−−−−−−→ dt 2 (t) sinh (k(x − t)) − k 2 dty(t) sinh (k(x − t)) = dtR(t) sinh (k(x − t))dt
dt
Z x 0 Z 0 0

y 00 sinh (κ) = −y 0 (0) sinh (kx) − y 0 cosh (κ)(−k) =


0
Z
= −y 0 (0) sinh (kx) + k(y(x) − y(0) cosh (kx) + k y(t) sinh (κ)) =
Z
= −y 0 (0) sinh (kx) + ky(x) − ky(0) cosh (kx) + k 2 y(t) sinh (κ)
Z
y 0 (0) sinh (kx) 1 x
=⇒ y(x) − − y(0) cosh (kx) = dtR(t) sinh (k(x − t))dt
k k 0
Rx
Now note that L(yh ) = 0, so applying dt sinh (k(x − t)) results in 0 still.
0
R
1 x y 0 (0) sinh (kx)
With yp (x) =
k 0 dtR(t) sinh (k(x − t))dt + k + y(0) cosh (kx), we can add a homogeneous solution of
y 0 (0) sinh (kx)
k + y(0) cosh (kx) to yp (x) to obtain
Z x
1
y1 (x) = dtR(t) sinh (k(x − t))dt
k 0

Now for y 00 − 9y = e3x ,


Z Z µ 3x−3t ¶ Z
1 x 1 x e − e−3x+3t 1 x
y1 (x) = dt(e3t ) sinh (3(x − t))dt = dte3t = dt(e3x − e−3x+6t )
3 0 6 0 2 6 0
µ ¯x ¶
1 3x ¯
−3x 1 6t ¯ 1 e6x − 1 1 e3x e−3x
= e x−e e ¯ = xe3x − e−3x = (xe3x − + )
6 6 0 6 36 6 6 6
1 3 3
y10 = (e3x + 3xe3x − e3x − e−3x )
6µ 2 2 ¶
00 1 3x 3x 3x 9 3x 9 −3x 1 3 3
y1 = 3e + 3e + 9xe − e + e = e3x + e−3x + xe3x
6 2 2 4 4 2
1 1
y100 − 9y1 = e3x + e−3x
2 2
Thus, we need to add homogeneous parts to our particular solution to make it work. So if

xe3x e3x e−3x


yp = − +
6 y 6
then it could be checked easily with some computation, that this satises the ODE.

Exercise 19. Start from y 00 + k 2 y = R(x)


Again, note that if L(yp ) = yp00 + k 2 yp = R(x), L(yp + yh ) = R(x) + 0 = R(x), so y1 = yp + yh is also a particular
solution.

Rx Z x Z x Z x
0
dt sin k(x−t) d2 y 2
−−−−−−−−−−→ + k
dt sin k(x − t) dt sin k(x − t)y = R(t) sin k(x − t)
0 dt2 0 0
Z x Z x
d2 y
dt sin k(x − t) 2 = −y 0 (0) sin (kx) + k y 0 (t) cos (k(x − t))dt =
0 dt 0
Z x
= −y 0 (0) sin (kx) + k(y(x) − y(0) cos (kx) − k y(t) sin (k(x − t))dt) =
Z 0x
= ky(x) − ky(0) cos (kx) − y 0 (0) sin (kx) − k 2 y(t) sin (k(x − t))dt
0
148
Z x
1 y 0 (0) sin (kx)
=⇒ y(x) = dtR(t) sin k(x − t) + y(0) cos (kx) +
k 0 k
−y 0 (0)
We can add yh with c1 = −y(0), c2 =
Z k
x
1
y1 = dtR(t) sin k(x − t)
k 0

Now for y 00 + 9y = sin 3x, then k = 3,


Z x
1
y1 = sin 3t sin 3(x − t)dt
3 0
Z x
sin 3t(sin 3x cos 3t − cos 3x sin 3t)
0
Z x Z x
s(6t) −1
sc = dt = (c(6x) − 1)
0 0 2 12
Z x Z x ¯x
1 − cos (6t) x sin (6t) ¯¯ x sin (6x)
s2 = = − ¯ = 2 − 12
0 0 2 2 12 0
µ µ ¶¶
1 −1 x sin (6x) sin 3x x cos 3x
y1 = sin 3x (cos (6x) − 1) − cos 3x − = −
3 12 2 12 18 6

It could be shown with some computation that this particular solution satises the ODE without having to add or subtract
parts of a homogeneous solution.

Exercise 20. y 00 + y = sin x

yh = c1 sin x + c2 cos x =⇒ W (x) = −s2 − c2 = −1


Z Z Z µ ¶
cs − cos 2x ss 1 − cos 2x x sin 2x
t1 = − = ; t2 = =− =− −
−1 4 −1 2 2 4
µ ¶ 2 3
sin x cos 2x sin 2x − 2x sin x cos x + sin x − 2x cos x
yp = − + cos x =
4 4 4
sin x cos2 x + sin3 x − 2x cos x
y = c1 sin x + c2 cos x +
4

Exercise 21. y 00 + y = cos x

yh = c1 sin x + c2 cos x = c1 S + c2 C W (x) = −1


Z Z Z
−CC1 + cos 2x x + sin22x SC cos 2x
t1 = = = ; t2 = =
−1 2 2 −1 4
x sin x sin 2x sin x cos x cos 2x
yp = + +
2 4 4
x sin x sin 2x sin x cos x cos 2x
=⇒ y = + + + c1 sin x + c2 cos x
2 4 4

Exercise 22. y 00 + 4y = 3x cos x

yh = c1 sin 2x + c2 cos 2x W (x) = − sin2 2x(2) + − cos2 2x(2) = −2


Z Z Z Z Z
− cos (2x)(3x cos x) 3 3 3
t1 = = x cos x cos (2x) = xc(1 − 2s2 ) = xc − 3 xcs2 =
−2 2 2 2
Z µ 3 ¶0 Z Z
3 c 3 3
= (xs + c) − 3 x = (xs + c) − (xs − s ) = (xs + c) − xs + s(1 − c2 ) =
3 3 3
2 3 2 2
3 1 3 c 1
= (xs + c) − xs3 + −c + c3 = xs + − xs3 + c3
2 3 2 2 3
149
Z Z Z Z Z
sin (2x)(3x cos x)
t2 = = −3 xsc2 = x(c3 )0 = xc3 − c3 = xc3 − c(1 − s2 ) =
−2
1
= xc − s + s3
3

µ 3 ¶
3 c 1 3 s3
yp = xs + − xs + c (2sc) + (xc3 − s + )(1 − 2s2 ) = (lots of algebra) =
3
2 2 3 3
2
= xc2 + s
3
2
=⇒ y = c1 sin 2x + c2 cos 2x + x sin x − cos x
3
Remember, persistence is key to work through the algebra, quickly.

Exercise 23. y 00 + 4y = 3x sin x. From the work above, we could guess at the solution.

(xs)0 = s + xc µ ¶00 µ ¶
−2 −2
(xs)00 + 4(xs) = 2c − xs + 4xs = 2c + 3xs (c)00 + 4c = 3c =⇒ c +4 c = −2c
(xs)00 = 2c + −xs 3 3
2
=⇒ yh = xs − c
3
2
y = x sin x − cos x + c1 sin 2x + c2 cos 2x
3

Exercise 24. y 00 − 3y 0 = 2e2x sin x Guessing and stitching together the solution seems easier to me.
(e2x s)0 = 2e2x s + e2x c
(e2x c)0 = 2e2x c + −e2x s
(e2x s)00 = 4e2x s + 4e2x c + −se2x =
(e2x c)00 = 4e2x c + −4e2x s − e2x c = 3e2x c − 4e2x s
= 3e2x s + 4e2x c
(e2x c)00 − 3(e2x c)0 = 3e2x c − 4e2x s − 6e2x c + 3e2x s =
(e2x s)00 − 3(e2x s)0 = 3e2x s + 4e2x c − 6e2x s − 3e2x c =
= −3e2x c − e2x s
= −3e2x s + e2x c
(3e2x s)00 − 3(3e2x s)0 + (e2x c)00 − 3(e2x c)0 = −10e2x s
e2x (3 sin x + cos x) √ √ e2x (3 sin x + cos x)
=⇒ yp = =⇒ y = c1 sin 3x + c2 cos 3x +
5 −5

Exercise 25. y 00 + y = e2x cos 3x.

(e2x c(3x))00 = 4e2x c(3x) + −12e2x s(3x) − 9e2x c(3x) = −5e2x c(3x) − 12e2x s(3x)
(e2x s(3x))00 = 4e2x s(3x) + 12e2x c(3x) + −9e2x s(3x) = −5e2x s(3x) + 12e2x c(3x)
L(e2x c(3x) − 3e2x s(3x)) = −40e2x cos (3x)
e2x cos (3x) − 3e2x sin (3x)
y = c1 sin x + c2 cos x +
−40

8.19 Exercises - Examples of physical problems leading to linear second-order equations with constant coefcients.
In exercises 1-5, a partcile is assumed to be moving in simple harmonic motion, according to the equation y = C sin (kx + α).
The velocity of the particle is dened to be the derivative y0 . The frequency of the motion is the reciprocal of the period.
(Period = 2π/k , frequency = k/2π )
Exercise 1. Find the amplitude C if the frequency is 1/π and if the initial values of y and y0 (when x = 0) are 2 and 4,
respectively.
k 1
frequency = = =⇒ k = 2
2π π
y(x = 0) = C sin α y(x = 0) 1 1
=⇒ 0 = tan α =
y 0 (x = 0) = C cos α y (x = 0) k 2
π √
α = and C = 2 2
4
150
Exercise 2. Find the velocity when y is zero, given that the amplitude is 7 and the frequency is 10.

y = C sin (kx + α) k
0 C=7 = 10 =⇒ k = 20π
y = Ck sin (kx + α) 2π
−α −α
y(x = ) = 0 =⇒ y 0 (x = ) = 140π
k k

Exercise 3.

y = A cos (mx + β)
y = A cos (mx + β) = A cos β cos (mx) − A sin β sin (mx)
=⇒ k = m (since x is arbitrary )
y = C sin kx + α = C cos α sin kx + C sin α cos kx
−A sin β = C cos α ³π ´ π
=⇒ tan α = ± cot β = ∓ tan − β = tan (β − )
A cos β = C sin α 2 2
π
=⇒ α = β − and |C| = |A|
2


Exercise 4.
T = 4π

y = C cos (kx + α) = C cos (kx) = 3 cos (4πx)


Exercise 5. y = C cos (x + α) y0 = C cos (x0 + α)

y 0 = −C sin (x + α) = ±v0
q
v02 + y02 = C 2 sin2 (x + α) + C 2 cos2 (x0 + α) = C 2 =⇒ C = v02 + y02

Exercise 6.

y 0 = −kC sin (kx + α)


y = C cos (kx + α)
y(0) = C cos (α) = 1 y 0 (0) = −kC sin (α) = 2
y 00 (0) −12 √
y 00 (0) = −k 2 C cos (α) = −12 = −k 2 = =⇒ k = 2 3
y(0) 1
y 0 (0) −kC sin (α) 2 −π
= = = 2 = −k tan (α) =⇒ α =
y(0) C cos (α) 1 6


Exercise 7. k= 3 y = −C sin (kx)

2πx
y = −C sin ; C>0
3
Exercise 8. Let's rst solve the homogenous equation.

y 00 + y = 0
Z x
− cos t(1)
yh = C1 sin x + C2 cos x t1 = = sin x
0 −1
Z x
W (x) = −S 2 − C 2 = 1 (sin t)(1)
t2 = = (cos x − 1)
0 −1
for 0 ≤ x ≤ 2π otherwise, for x > 2π, t1 = 0, t2 = 0

y1 = sin2 x + cos2 x + (1 − cos x) y(0) = 0 = c2


0
y (x) = C1 cos x + sin x y 0 (0) = c1 = 1
y = sin x + (1 − cos x)
=⇒ I(t) = sin t + (1 − cos t) 0 ≤ t ≤ 2π

Exercise 9.

151
(1) Consider large t. Then I(t) = F (t) + A sin (ωt + α) → A sin (ωt + α)

I = AS(ωt + α) = A(S(ωt)C(α) + C(ωt)S(α))


I 0 = ωAC(ωt + α) = ωA(C(ωt)C(α) − S(ωt)S(α))
I 00 = −ω 2 AS(ωt + α) = −ω 2 A(S(ωt)C(α) + C(ωt)S(α))
I 00 + RI 0 + I = I 00 + I 0 + I =
= A((−ω 2 C(α) + −ωS(α) + C(α))S(ωt) + (−ω 2 S(α) + ωC(α) + S(α))C(ωt)) = S(ωt)
−ω
=⇒ tan (α) =
1 − ω2

1 − ω2
C=√
With tan α = −ω 2
= sec2 , 1
= sec2 , and S 2 +C 2 = 1, we can get 1 − ω2 + ω4
1−ω 2 and the trig identities t +1 C2 −ω
S=√
1 − ω2 + ω4
Note that the sign of S is xed by tan.

1 D
A(−ω 2 C(α) + −ωS(α) + C(α))S(ωt) = S(ωt) =⇒ A = =
(1 − ω 2 )C(α) − ωS(α) (1 − ω 2 )2
− ω(−ω)
1
=⇒ A = √
ω4 − ω2 + 1

We could immediately see that √1 , f =


ω= 1√
will maximize A.
2 2π 2
(2) We could have, from the beginning, considered the problem with any R, in general.

A((−ω 2 C(α) + −RωS(α) + C(α))S(ωt) + (−ω 2 S(α) + ωRC(α) + S(α))C(ωt)) = S(ωt)


−ωR
S(α) = p
−ωR (ωR) + (1 − ω 2 )2
2
tan α =
1 − ω2 1 − ω2
C(α) = p
(ωR)2 + (1 − ω 2 )2
1 1
=⇒ A = =p
−ω 2 C(α) − ωRS(α) + C(α) (ωR)2 + (1 − ω 2 )2
d
(ωR)2 + (1 − ω 2 )2 = ω 4 + ω 2 (−2 + R2 ) + 1 −−→ 4ω 3 + 2ω(−2 + R2 ) = 2ω(2ω 2 + (−2 + R2 )) = 0

ω=0 to have resonances 1


=⇒ 2 −−−−−−−−−−→ R < √
2
2ω = 2 − R 2

Exercise 10. A spaceship is returning to earth. Assume that the only external force acting on it is the action of gravity, and

k pounds per
that it falls along a straight line toward the center of the earth. The rocket fuel is consumed at a constant rate of
c feet per second relative to the rocket.
second and the exhaust material has a constant speed of
Let M (t) = M be the mass of the rocket + f uel combination at time t. With +y direction being towards earth, then the
2
equation of motion is Fg = +M (t)g , where g = 9.8m/s .

M (t)v(t) = M vR is the momentum of the rocket.


M (t + h) = M (t) − ∆m = M − ∆m is the change in mass of the rocket due to spent fuel.
ve = velocity of the exhaust in the lab frame = c + vR (t)

∆p = ∆m(c + vR ) + (M − ∆m)vR (t + h) − M vR = M (vR (t + h) − vR ) + −∆m(vR (t + h) − vR ) + ∆mc


µ ¶ µ ¶ µ ¶
∆p vR (t + h) − vR vR (t + h) − vR ∆m
=M + −∆m + c = M (t)g
∆t ∆t ∆t ∆t
152
kc 0
M vR +
= M (t)g
g
kt 0 kc/g kc/g
Now M (t) = M0 − =⇒ vR =g− =g−
g M M0 − kc
g
kc g kt kt
vR = gt − ln (M0 − ) = gt + c ln (M0 − )
g −k g g
2
µµ ¶ µ ¶ ¶
gt g k kt kt M0 cg
yR = +c t − M0 ln M0 − − + ln M0
2 k g g g k

Exercise 11.

0 −kc
M vR =
g
à !
0 −kc 1
vR = kt
µµ ¶ µ ¶ ¶
g M0 − g kt kt kt M0 cg ln M0
g =⇒ yR = c − M0 ln M0 − − +
kt k g g g k
vR = c ln (M0 − )
g
µ µ ¶ ¶
g kt − w w − kt kt wc w
M0 g = w =⇒ yR = c ln − + ln
k g g g k g

We could've also solved this problem with an initial velocity of v0 and gravity. Then

kt
vR (t) = gt + c ln (1 − ) + v0
M0 g
µµ ¶ µ ¶ ¶
1 M0 g kt
y(t) = v0 t + gt2 + c t− ln 1 − −t
2 k M0 g

Exercise 12.

M vR = (M − ∆m)(vR (t + h)) + 0
M (vR (t + h) − vR (t)) = (∆m)vR (t + h)
0k v0 k k
M vR =vR =⇒ R = kt
=
g vR g(M0 − g ) M0 g(1 − Mkt0 g )
µ ¶
kt −w kt
ln vR = (k/w) ln (1 − ) = − ln (1 − )
w k w
µ ¶
v0 −w kt −v0 w kt
vR = =⇒ x(t) = v0 ln (1 − ) = ln (1 − )
1 − kt
w
k w k w

8.22 Exercises - Remarks concerning nonlinear differential equations, Integral curves and direction elds.
−2
Exercise 1. 2x + 3y = C =⇒ y 0 = 3

Exercise 2. y = Ce−2x =⇒ y 0 = −2y


Exercise 3. x2 − y 2 = c =⇒ yy 0 = x =⇒ y 0 = xy ; y 6= 0
−y
Exercise 4. xy = c =⇒ y 0 = x ; x 6= 0
y2 y
Exercise 5. y 2 = cx =⇒ x = c =⇒ y 0 = 2x x 6= 0
Exercise 6. x2 + y 2 + 2Cy = 1
x2 1
+ y − = −2C
y y
2xy − y 0 x2 1
+ y0 + 2 y0 = 0
y2 y
−2xy
y0 =
1 + y 2 − x2
153
Exercise 7. y = C(x − 1)ex
y
=C
(x − 1)ex
y 0 (x − 1)ex − (ex + (x − 1)ex )y
=0
(x − 1)2 e2x
xy
y0 =
x−1

Exercise 8. y 4 (x + 2) = C(x − 2)
y 4 (x + 2)
=C
x−2
(4y 3 y 0 (x + 2) + y 4 )(x − 2) − y 4 (x + 2) y 4 (x + 2)
2
= 0 =⇒ 4y 3 y 0 (x + 2) + y 4 =
(x − 2) x−2
y
y0 =
(x − 2)(x + 2)

Exercise 9. y = c cos x =⇒ y 0 = − tan xy


Exercise 10. arctan y + arcsin x = C

1 1 −(1 + y 2 )
2
y0 + √ =0 =⇒ y 0 = √
1+y 1 − x2 1 − x2

Exercise 11. All circles through the points (1, 0) and (−1, 0).
Start with the circle equation: (x − A)2 + (y − B)2 = R2
(1, 0): (1 − A)2 + B 2 = R2 =⇒ −(1 − 2A + A2 + B 2 = R2 )
(−1, 0): (−1 − A)2 + B 2 = 1 + 2A + A2 + B 2 = R2
=⇒ 4A = 0, A = 0 1 + B 2 = R2

p
x2 + (y − ± R2 − 1)2 = R2
x2 + y 2 − 2By + (R2 − 1) = R2 =⇒ x2 + y 2 − 2By = 1
B depends upon R, the radius of the circles, so we could use B as the parameter for the family of circles.

x2 + y 2 − 1 = 2By
x2 1 2xy − y 0 x2 1
+ y − = 2B =⇒ + y0 + 2 y0 = 0
y y y2 y
2xy
y0 =
y2 − x2 + 1

Exercise 12.

(x + A)2 + (y + B)2 = r2
(1 + A)2 + (1 + B)2 = 1 + 2A + A2 + 1 + 2B + B 2 = r2
¡ ¢ ¡ ¢
− (−1 + A)2 + (−1 + B)2 = − 1 − 2A + A2 + 1 − 2B + B 2 = r2
=⇒4A + 4B = 0 =⇒ A = −B
(x + −B)2 + (y + B)2 = r2
B−x
2(x − B) + 2(y + B)y 0 = 0 =⇒ y 0 =
y+B
(y + B)y 0 = B − x
r
2
√ p
2 2 r2
(1 − B) + (1 + B) = r =⇒ 2 (1 + B 2 ) = r or −1=B
2
=⇒ (so B could be treated as a parameter for the family of curves)
154
8.24 Exercises - First-order separate equations.

Exercise 1. y 0 = x3 /y 2
1 3
3y = 14 x4 + C =⇒ y 3 = 34 x4 + C
Exercise 2. tan x cos y = −y 0 tan y
1
ln | cos x| = cos y
Exercise 3. (x + 1)y 0 + y 2 = 0
1
y = ln (x + 1) + c
Exercise 4. y 0 = (y − 1)(y − 2)
µ ¶
1 −1
+ y0 = 1 =⇒ ln (y − 2) − ln (y − 1) = x
y−2 y−1
y−2
= ex
y−1

y 1 − x2 y 0 = x
Exercise 5.

1 2
√ √
2y = − 1 − x
2 =⇒ y 2 = −2 1 − x2
Exercise 6. (x − 1)y 0 = xy
Z
1
ln y = 1+ = x + ln |x − 1|
x−1
y = ex (x − 1) + C

Exercise 7. (1 − x2 )1/2 y 0 + 1 + y 2 = 0
arctan y = arccos x + C
Exercise 8. xy(1 + x2 )y 0 − (1 + y 2 ) = 0
Z µ ¶
1 1 x 1
ln (1 + y 2 ) = − + C = ln x −
2 x 1 + x2 2 ln |1 + x2 |
µ ¶2
x
y2 = k √
1 + x2

Exercise 9. (x2 − 4)y 0 = y

−1 ³x´
ln y =arctanh
Z 2 Z 2
1 1 dx
since dx = ¡ x ¢2 =
x2 − 4 4 −1
2
Z
1 du x
= ( where u = )
2 u2 − 1 2
2 2
cosh u − sinh u
(tanh (u))0 = = 1 − tanh2 u
cosh2 u
1 ³x´
=⇒ y = k exp (− arctanh )
2 2

Exercise 10. xyy 0 = 1 + x2 + y 2 + x2 y 2


2
1
2 ln (1 + y 2 ) = ln x + 12 x2 + C =⇒ y 2 = kx2 ex − 1
Exercise 11. yy 0 = ex+2y sin x
ye−2y e−2y ex sin x − ex cos x
− = +C
−2 4 2
(2y + 1)e−2y = −2ex (sin x − cos x) + C
155
Exercise 12. xdx + ydy = xy(xdy − ydx)
y(1 − x2 )dy = x(−y 2 − 1)dx
ydy xdx 1 1
2
= 2 =⇒ ln |1 + y 2 | = ln |x2 − 1| + C
1+y x −1 2 2
1 + y 2 = (x2 − 1)K =⇒ y 2 = K(x2 − 1) − 1
Rx
Exercise 13. f (x) = 2 + 1
f (t)dt
f 0 (x) = f (x) =⇒ f (x) = Cex
2
=⇒ Cex = 2 + Cex − Ce1 C=
e
2 x
f (x) = e
e

Exercise 14. f (x)f 0 (x) = 5x f (0) = 1


p
f (x)2 = 5x2 + C =⇒ f (x) = ± 5x2 + C
p
f (x) = 5x2 + 1

Exercise 15. f 0 (x) + 2xef (x) = 0 f (0) = 0


e−y y 0 = −2x =⇒ −e−y = −x2 + C

y = ln (x2 + C)−1 =⇒ y = − ln (x2 + 1)

Exercise 16. f 2 (x) + (f 0 (x))2 = 1


f = −1 p
02 2 y0 = ± 1 − y2
y =1−y
± arcsin (y) = x + c =⇒ f (x) = ± sin (x + c)

Exercise 17.
Z x
f (t)dt = K(x − a)
a =⇒ f (x) = k > 0
f >0 ∀x ∈ R
Exercise 18.
Z x d
→ f (x) = kf 0 (x)
dx
f (t)dt = k(f (x) − f (a)) −−
a
1
=⇒ f (x) = Ce k x ; C>0
Rx x
Exercise 19.
a
(f (t))dt = k(f (x) + f (a)) =⇒ f (x) = Ce k
x a x
kCe k − kCe k = kCe k + kCea/k =⇒ 2kCea/k = 0 =⇒ C =
f =0
Rx
Exercise 20.
a
f (t)dt = kf (x)f (a); f (x) = kf 0 (x)f (a)
µ ¶ µ ¶
1 f 0 (x) 1 x
= =⇒ ln f (x) = x + C; =⇒ f (x) = C exp
kf (a) f (x) kf (a) kf (a)
Z x ³ ´¯x
t ¯ x a x a
f (t)dt = kf (a)Ce kf (a) ¯ = kf (a)Ce kf (a) − kf (a)Ce kf (a) = kCe kf (a) Ce kf (a)
a a
³ x a
´ x a
f (a) e kf (a) − e kf (a) = Ce kf (a) e kf (a)
x=a 2a
−−−→ 0 = Ce kf (a) =⇒ C = 0
=⇒ f = 0
156
8.26 Exercises - Homogeneous rst-order equations.
Exercise 1. f (tx, ty) = f (x, y) homogeneity (or homogeneity of zeroth order).

³ x ´0
v − xv 0 x 1
y 0 = f (x, y) = 2
= = f (x, ) = f (1, )
v v v v
Z
1 dv
v − v 2 f (1, ) = xv 0 =⇒ ln x =
v v − v 2 f (1, v1 )

Exercise 2. y0 = −x
y =⇒ 12 y 2 = − 12 x2 + C =⇒ y 2 = −x2 + C
y
Exercise 3. y0 = 1 + x

y
x =v =⇒ y 0 = v + xv 0 = 1 + v =⇒ v = ln x

y = x(ln x + C)

x2 +2y 2
Exercise 4. y0 = xy

x2 + 2y 2 x 2y
y0 = = +
xy y x
y 1 v0 1
v= =⇒ v + xv 0 = + 2v =⇒ 1 =
x v v +v x
1
ln |1 + v 2 | = ln x + C =⇒ y 2 = (Cx2 − 1)x2
2

Exercise 5. (2y 2 − x2 )y 0 + 3xy = 0

3xy
if 2y 2 6= x2 , y 0 =
x2 − 2y 2
y = vx 3vx2 3v
=⇒ y 0 = v 0 x + v = =
y0 = v0 x + v 2 2
x − 2v x 2 1 − 2v 2
1 − 2v 2 0 1
=⇒ v =
2v(1 + v 2 ) x
µ ¶
1 1 −3v 1 1 −3 v y/x
+ v0 = =⇒ ln v + ln (1 + v 2 ) = ln x + C =⇒ = Cx2 = ³ ´3
2 v 1 + v2 x 2 2 (1 + v 2 )3 x2 +y 2
x2

yx3 = C(x2 + y 2 )3

However,

3xy
y0 =
x2 − 2y 2
y
v= 3x2 v 3v
x =⇒ v 0 x + v ==
2
x − 2v x 2 2 1 − 2v 2
y0 = v0 x + v
µ ¶
3v (v − 2v 3 ) 2(v + v 3 ) 1 3v 2 3
v0 x = 2
− 2
= 2
=⇒ + − 2
v 0 = =⇒ ln v + − ln |1 + v 2 | = 2 ln x + C
1 − 2v 1 − 2v 1 − 2v v 1+v x 2
2 4
v y /x
= Cx2 =⇒ 2 = Cx4
(1 + v 2 )3/2 (x + y 2 )3
=⇒ y 2 = C(x2 + y 2 )3
157
p
Exercise 6. xy 0 = y − x2 + y 2
r ³ y ´2
0y
=⇒ y = = 1+
x x

p
v0 x = − 1 + v2
p −v 0 1
y 0 √ =
v= v x+v =v− 1+ v2 =⇒ 1+v 2 x
x p
vx = y 2
=⇒ ln (v + 1 + v ) = ln x + C since
p µ ¶
2 0 1 v 1
(ln (v + 1 + v )) = √ 1+ √ =√
v+ 1+v 2 1+v 2 1 + v2
p
v+ 1 + v 2 = Cx =⇒ 1 + v 2 = C 2 x2 − 2vCx + v 2
Cx 1 Cx2 1
=⇒ v = − =⇒ y = −
2 2Cx 2 2C

Exercise 7. x2 y 0 + xy + 2y 2 = 0

2 0 2 −2y 20 y y v 0 x + v = −2v 2 − v
x y = −2y − xy =⇒ y = − v = =⇒
x2 x x v 0 x = −2(v 2 + v)
Z µ ¶
v0 −2 1 1 v C
= = v0 − = −2 ln x + C =⇒ = 2
v(v + 1) x v v+1 v+1 x
−Cx
y=
C − x2

Exercise 8. y 2 + (x2 − xy + y 2 )y 0 = 0
µ ³ y ´2 ¶
y ³ y ´2 0
+ 1− + y =0
x x x
y −v 2
Let v = =⇒ = v0 x + v
x 1 − v + v2
µ ¶
0 −v 2 −v(1 + v 2 ) v2 − v + 1 0 −1 1 −1 1
vx= −v = =⇒ v = = + v0 = −
1 − v + v2 1 − v + v2 v(1 + v 2 ) x v v2 + 1 x
y
=⇒ ln v − arctan v = − ln x + C =⇒ ln (vx) = arctan x + C ln y = arctan + C
x

y(x2 +xy+y 2 )
Exercise 9. y0 = x(x2 +3xy+y 2 )
à ! µ ¶
y(x2 + xy + y 2 ) ³y´ 1+ y y2
1 + v + v2 −2v 2
x + x2
y
v= x
y0 = = y2
0
−−−→ v x + v = v =v+
x(x2 + 3xy + y 2 ) x 1+ 3y 1 + 3v + v 2 v 2 + 3v + 1
x + x2
3 1 −2 −1
v 0 (1 + + 2 ) = =⇒ v + 3 ln v + = −2 ln x + C
v v x v
y x
+ 3 ln y − = ln x + C
x y

y
Exercise 10. y0 = x + sin xy
y v + v 0 x = v + sin v
=x
x =⇒ v0 1
y0 = v + v0 x =
sin v x
K
− ln csc v + cot v = ln x + C =⇒ csc v + cot v =
x
158
Exercise 11. x(y + 4x)y 0 + y(x + 4y) = 0
−y(x + 4y) − y (1 + 4y
x ) v= x
y
−v(1 + 4v)
y0 = = xy −−−→ v + xv 0 =
x(y + 4x) x + 4 v+4
µ ¶
−5v(1 + v) −5 v+4 0 4 −3
xv 0 = =⇒ = v = + v0
v+4 x v(1 + v) v 1+v
R
→ 4 ln v − 3 ln (1 + v) = −5 ln x + C =⇒ (yx)4 = (x + y)3 C

8.28 Miscellaneous review exercises - Some geometrical and physical problems leading to rst-order equations.
Exercise 1.
2 3 3
2x + 3y = C y0 = − g0 = =⇒ g − x = C
3 2 2
Exercise 2.
d/dx 1 2 1
xy = C −−−→ y + xy 0 = 0 =⇒ y 0 = −y/x x 6= 0 =⇒ g 0 = x/g =⇒ g = x2 + C
2 2
Exercise 3. x2 + y 2 + 2Cy = 1
x + yy 0 + Cy 0 = 0 =⇒ y 0 (y + C) = −x
−x −x −2xy
y0 = = 1−x2 −y 2
= 2
y+C y+ y − x2 + 1
2y
µ ¶ µ ¶
orthogonalcurves 0 y 2 − x2 + 1 1 1 1
−−−−−−−−−−−→ y = = y+ − x y −1
2xy 2x 2 x
Recognize that this is a Ricatti equation and we know how to solve them.

µ ¶
0 −1 −x 1
y + y = y −1 + n = −1
2x 2 2x
k = 1 − n = 1 − (−1) = 2
¶ µ µ ¶
−1 2 1
v = yk = y2 0
v +2 v= −x
2x 2 x
Z x Z x Z x Z x µ ¶
−1 a A 1 a −a
A(x) = P (t)dt = = ln Qe = −t = + 1 − a(x − a)
a a t x a a t t x
x bx
y 2 = v = −1 + − x(x − a) +
a a
Exercise 4. y 2 = Cx.
y2 0
d/dx 2yy x − y
2
= C −−−→ =0
x x2
y 1 −2x
y0 = =⇒ y 0 = ¡ −y ¢ =
2x 2x
y

=⇒ y 2 + 2x2 = C

Exercise 5. x2 y = C .
2y x
2xy + x2 y 0 = 0 y0 = − =⇒ y 0 =
x 2y
1 2 x2
y = +C
2 4
2y 2 − x2 = C

Exercise 6. y = Ce−2x
e2x y = C =⇒ 2e2x y + e2x y 0 = 0
invert 1
y 0 = −2y −−−→ y 0 =
2y
=⇒ y 2 = x + C
159
Exercise 7. x2 − y 2 = C
2x − 2yy 0 = 0
x −y
y 0 = =⇒ y 0 =
y x
C
=⇒ ln y = − ln x + C =⇒ y =
x
Exercise 8. y sec x = C
y 0 sec x + y tan x sec x = 0
(invert) 1
y 0 = −y tan x −−−−→ y 0 =
y tan x
1 2
y = ln | sin x| + C
2
2xy
Exercise 9. All circles through the points (1, 0) and (−1, 0) From Sec. 8.22, Ex.10, we had obtained y0 = y 2 −x2 +1
x2 −y 2 −1 x2 −1 y 1 x2 −1 −1
=⇒ y 0 = 2xy = 2yx − 2x =⇒ y 0 + 2x y = 2x y

Recognize this is a Ricatti equation.

Fory 0 + P y = Qy n , in this case, n = −1, and so k = 1 − n = 1 − (−1) = 2.


Then v = y k and v³0 + kP´v = kQ. In this case,
¡1¢ 2 2
0
v + 2 2x v = 2 x 2x −1
= v 0 + x1 v = x x−1 = x − 1/x.
Z x Z x
1 x
A(x) = P (t)dt = = ln
a a t a
Z µ ¶ Z µ 2 ¶ µ1 3 ¶¯x
1 t t 1 3t t ¯¯
(t − ) exp ln dt = − dt = −
t a a a a a ¯a
x a
e− ln a =
x
1 3
x − x − 13 a3 + a ba
=⇒ y 2 = v = 3 +
x x
Exercise 10. All circles through the points (1, 1) and (−1, −1).
(x + A)2 + (y + B)2 = r2
(1 + A)2 + (1 + B)2 = 1 + 2A + A2 + 1 + 2B + B 2 = r2
¡ ¢ ¡ ¢
− (−1 + A)2 + (−1 + B)2 = − 1 − 2A + A2 + 1 − 2B + B 2 = r2
=⇒4A + 4B = 0 =⇒ A = −B
(x + −B)2 + (y + B)2 = r2
B−x
2(x − B) + 2(y + B)y 0 = 0 =⇒ y 0 =
y+B
(y + B)y 0 = B − x
r
2
√ p 2 2 r2
(1 − B) + (1 + B) = r =⇒ 2 (1 + B 2 ) = r or −1=B
2
=⇒ (so B could be treated as a parameter for the family of curves)
−1 y+B
y0 = ³ ´=
B−x x−B
y+B

y0 1
= =⇒ y = C(x − B) − B
y+B x−B
Exercise 11. With (0, Y ) = Q the point that moves up wards along the positive y -axis and
P = (x, y) being the point P that pursues Q,
Y −y Y −y
y0 = X−x = 0−x is the slope of the tangent line on a point on the trajectory of P .
The condition given, that the distance of P from the y -axis is k the distance of Q from the origin, is
160
kY = x.
¡1¢
x−y 0 k
= f (x, y) y =
x
f (x, y) is homogeneous of zero order =⇒ y = vx (try this substitution)
0
1 v 1
y0 = v0 x + v = − v =⇒ 1 =
k k − 2v x
µ ¶
1 1 x 1
=⇒ − ln − 2v = ln x + C =⇒ y = −
2 k 2k 2C 2 x
(1,0) x 1 1 1
−−−→ y = = k= y =x−
2k 2kx 2 x
Exercise 12.
x 1
y= −
2k 2kx
Exercise 13. y = f (x).
Z x Z x Z x
n f (t)dt = xf (x) − f (t)dt; (n + 1) f (t)dt = xy
0 0 0
ny = xy 0
n y0
=⇒ (n1 )y = y + xy 0 =⇒ =
x y
n ln x = ln y
y = Cxn of y = Cx1/n

Exercise 14.
Z x Z x
2
n πf (t)dt = (π(y(x))2 − π(f (t))2 )dt
0 0
Z x
(n + 1) πf 2 (t)dt = xy 2 (x) = xy 2 ; (n + 1)f 2 (x) = y 2 + 2xyy 0
0
ny n
y0 = =⇒ ln y = ln x + C
2x 2
y = Cxn/2 or y = Cx1/2n

Exercise 15.
Z x
πf 2 − 2xf
π f 2 (t)dt = x2 f (x) πf 2 (x) = 2xf + x2 f 0 =⇒ f 0 =
0 x2
The left hand side of the last expression shown is homogeneous. Thus do the y = vx substitution.

πv 2 x2 − 2x2 v
v0 x + v == πv 2 − 2v
x2 µ ¶
v0 x v0 1 1 1 1
= 1 =⇒ = = − v0
πv 2 − 3v π(v 2 − 3vπ )
x 3 v − π3 v
µ ¶
3 v − 3/π
ln (v − ) − ln v = ln = 3 ln x + C
π v
3
vx − xCvx4
π
3 3x/π
y − x = Cyx3 =⇒ y = 3
π 1 + x2
Ra R1
Exercise 16. A= 0
f; B= a
f
Ra Ra
A−B = 0
f+ 1
f = 2f (a) + 3a + b
d/dx f 0 (a)
−−−→ 2f (a) = 2f 0 (a) + 3 =⇒ 1 = f (a)− 32
161
So then

3 3
a + C = ln (y − ); f (a) = Cea +
2 2
1 3 3
f (1) = 0 = Ce + ; =⇒ C = −
2 2e
−3 x−1 3
f (x) = e +
2 2

To nd b,
µ ¶
−3 a−1 3 3 3 3
2 e + a + e−1 + − = 2f (a) + 3a + b =
2 2 2 2 2
µ ¶
−3 a−1 3
=2 e + + 3a + b
2 2
3 −1
=⇒ b = e −3
2

Exercise 17.

Z µ
x µµ ¶ ¶¶
y(x) − 1
A(x) = f (t) − t+1 dt = x3 =
0 x
Z x µ ¶ Z x µ ¶
y(x) − 1 1 2 y(x) − 1
= f +− x −x= f +− x−x
0 x 2 0 2
d/dx 1 1
−−−→ f (x) + − (y 0 x + y) − = 3x2
2 2
2 1
−2(3x + − y/2) y
y0 = 2
= −6x − x−1 +
x x
y
As a leap of faith, try y = vx substitution to solve y 0 = −6x − x−1 + x.

v 0 x + v = −6x − x−1 + v v 0 = −6 − x−2 ; v = −6x + x−1 + C

y = −6x2 + 5x + 1

Exercise 18. Assuming no friction at the orice and energy conservation.

1
mgh = mv 2
2 f

(imagine how the top layer of water is now at the bottom of the tank (nal “potential energy congurations”))

Vf = 2gh (how fast water is rushing out)

A0 = cross-sectional area of the orice.

dV dh p
=A = −c 2ghA0
dt dt
¯hf p A0
¯
2h1/2 ¯ = −c 2g t
hi A
√ ³ ´
2A p p
=⇒ T = √ hf − hi = 59.6sec
c gA0

Note that we included the discharge coefcient C = 0.6.


dV
√ √
Exercise 19.
dt = −c 2ghA0 + γ0 = A dh
dt = −κh
1/2
+ γ0 . κ = c 2gA0 .
162
Adh dh
= dt = (A/γ0 )
γ0 − κh 1/2 1 − γκ0 h1/2
³ ³ ´´0 µ ¶
1 a 1 κ
ln 1 − ah1/2 = − (where a = )
1 − ah1/2 2 h1/2 γ0
µ ¶ ¡ 1 a 1/2 ¢
1/2 0 1 1 − ah1/2 − h
(h ) = 1/2 1/2
= 1/22 ¡ 2 ¢
2h 1 − ah h 1 − ah1/2
Z µ 2 µ ¶ ¶¯hf
dh 2γ0 κ 1/2 2γ0 h1/2 ¯¯
=⇒ (A/γ0 ) = − (A/γ 0 ) ln 1 − h + ¯ =
1 − γκ0 h1/2 κ2 γ0 κ hi
=T
µ ´ ¶ κ2 κ 1/2
−γ0 2γ0 ³ 1/2 1/2
1− γ0 hf
=⇒ exp t− hf − hi 2 =
A κ 2γ0 κ 1/2
1− γ0 hi

t→∞ γ02 (100in3 /s)2 2


−−−→ hf = = = (25/24)
κ2 c2 (2)(32f t/s2 )(5/3in2 )2 (12in/1f t)
Exercise 20.
1 2
V0 =
πR H0
3 0
µ ¶2
1 R0 1 R2
V (h) = V0 − πh h = V0 − π 02 h3 = V0 − αh3
3 H0 3 H0
1 p
mg(H0 − h) = mvf2 ; 2g(H0 − h) = vf (energy conservation)
s2 µ ¶ r
h h
cA0 vf = cA0 2gH0 1 − =β 1−
H0 H0
r
dV p 2 dh dh β h
= −β 1 − h/H = −3αh =⇒ = 1−
dt dt dt 3αh2 H
Z
h2 /H02 β/H02
q = T =
1− h 3α
H0
Z Z Z
u2 du (1 − y)2 (−dy) 1 − 2y + y 2
= H0 √ = H0 √ = −H0 √ =
1−u y y
µ ¶¯hf
2 2 ¯
= −H0 2y 1/2 − y 3/2 + y 5/2 ¯¯ =
3 5 hi
à µ ¶1/2 µ ¶3/2 µ ¶5/2 !¯¯hf
h 4 h 2 h ¯
= −H0 2 1 − − 1− + 1− ¯ =
H0 3 H0 5 H0 ¯
hi

cA0 2gH0 /H02
= ³ ´ T
R2
3 13 π H02
0

For hi = 0, hf = H ,
√ √
p 16
H0 πR02 2 πR02 H0
H0 (2(1) − 4/3(1) + 2/5) = H0 (16/15) = cA0 2gH0 T /(πR02 ); T = 15
√ =
cA0 2g 9 A0
Exercise 21. m2 x − m + (1 − x) = 0 =⇒ (m2 − 1)x + 1 − m = 0, =⇒ m = 1
3 2 0
Exercise 22. Given x + y + 6xy y = 0

Notice that (2xy 3 )0 = 2(y 3 + 3xy 2 y 0 ) = 2y 3 + 6xy 2 y 0 . So then

x + y + 6xy y = (2xy 3 )0 + −y 3 + x = 0
3 2 0

Let u = 2xy 3 . So then y3 = u


2x . Thus, we have
u
u0 − = −x
2x
163
This equation is linear because it is a linear combination of y0
and y . Now we can use the formula for solving rst-order
0
linear differential equations of the form u + P u = Q.
µZ xf ¶ Z
−A A
u=e Qe dx + u(x0 ) for A = P dt
x0
Z µ ¶
−1 −1 1/2 −2 3/2 3/2 −2 2
=⇒ A = dt = ln t and u = x (x − x0 ) + u(x0 ) = x + Cx1/2
2t 2 3 3
−x C −x
=⇒ y 3 = + 1/2 for x > 0 or y 3 = ∀x
3 x 3

Exercise 23. Given (1 + y 2 e2x )y 0 + y = 0, let y = uemx , u unknown.


y 0 = u0 emx + muemx
(1 + u2 e2mx e2x )(u0 emx + muemx ) + uemx = 0 = u0 emx + u0 (u2 e3mx+2x ) + muemx + mu3 e3mx e2x + uemx

u0 (emx + u2 e(3m+2)x ) + muemx + mu3 e3mx+2x + uemx = 0


−(m + 1)u − mu3 e(2m+2)x
u0 =
1 + u2 e2(m+1)x
Let m = −1 .
µ ¶
u3 1 + u2 1 1
=⇒ u0 = 2
=⇒ du = + du = dx =⇒ u−2 / − 2 + ln u − C = x
1+u u3 u3 u
y = ue−x or u = yex
e−2x
=⇒ ln y = x + +C
2y 2
¡1¢
Exercise 24. Given f s.t. 2f 0 (x) = f x if x > 0, f (1) = 2 and x2 y 00 + axy 0 + by = 0

(1)
µ ¶ µ ¶
00 d 1 0 −1
2f (x) = f = (f )
dx x x
µ ¶
−1 −1 1
=⇒ x2 y 00 = f0 = f
2 4 x
1
a = 0; b=
4
(2)
f (x) = Cxn
−1 n
f 0 = nCxn−1 =⇒ n(n − 1)Cxn = Cx
4
f 00 = n(n − 1)Cxn−2
1 1
n2 − n + =⇒ n =
4 2
Exercise 28. Choose the units for time to be in days rst - we can convert into years later.

If no one died from accidental death, then the population will grow by e. That means, with x = x(t) being the population at
time t
dx
=x =⇒ x = Cet
dt
which makes sense because if C is the original population number, then after 1 year, x = Ce.

1
With t in days, 100 x in population each day due to death. Add up the changes from the decrease
we have a decrease of
due to deaths and the increase due to growth for the DE:

dx 1 1 100 − 365 −265


= x− x= x=
dt 365 100 36500 36500
−265
=⇒ x = Ce 36500 t
164
−265
Change t units to years by multiplying the “time constant” 36500 by 365 days.

x = 365 exp (−2.65t)


To get the total fatalities, simply integrate the deaths during each year.
Z t
365 365
y= (− exp (−2.65t) + 1)
0 100 2.65
Exercise 29. For constant gravity, ∆K = −∆U
=⇒ −(0 − mgh) = 21 mvf2
√ ¡ 1 m ¢ ³ 1 ft ´ mi mi
vf = 2gh = (6.37 × 108 cm) 2.54 cm 12 in = 6.93 sec = 24940 hr
The constant energy formula could also be obtained by considering
r
GMe m d
F = −r 2 = m dt 2 = −∂r U

2y 2 +x
Exercise 30. Let y = f (x) be the solution to y 0 = 3y 2 +5 f (0) = 0
0 2 0 2
(1) y 0 (0) = 0 as easily seen. Now y 00 = (4yy +1)(3y(3y
+5)−(6yy )(2y +x)
2 +5)2 , so then
00 1
y (0) = 5 > 0. It is a minimum.
(2) f 0 (x) ≥ 2/3 ∀ x ≥ 10/3. a = 2/3 since f will be above this tangent line.
0
Suppose, in the “worst case,” f (x) = 0 for 0 ≤ x ≤ 2/3. Then f (x) = 0 for 0 ≤ x ≤ 2/3. Then the tangent line
must be at y = 0 at x = 10/3 to remain below the graph of f (x).

=⇒ 23 x − 20/9 < f (x)


0 2 10
Since f (x) ≥
(3)
3 for each x ≥ 3 , then f → ∞ for x → ∞ (otherwise, f would have to decrease somewhere, which
0
would contradict the given fact about f ). Rewrite the DE for y to be

2y 2 + x 5 x
y0 = 2
=⇒ (3y 2 + 5)y 0 = 2y 2 + x =⇒ (3 + 2 )y 0 = 2 + 2
3y + 5 y y
Consider
x
2y 2 + x 2+ y2
y0 = 2
= 5
3y + 5 3+ y2
x 2
specically,
y 2 . Now y must, at the very least, have some linear increase because we had already shown that y 0 ≥ 3.
2 5 0 x→∞
So y 2 would go to innity faster than linear x. Thus limx→∞ y 0 = 3 . So then (3 + y 2 )y −−−−→ (3 + 0) 23 = 2 =
2 + yx2 .
x
0=
y2
2+ yx2 x→∞ y
2
(4) y0 = 3+ y52
−−−−→ 3. =⇒ y = 23 x or x = 2
3.

Exercise 31. Given a function f which satises the differential equation xf 00 (x) + 3x(f 0 (x))2 = 1 − e−x
(1) c 6= 0 for an extrenum.
1−e−c
cf 00 (c) + 3c(f 0 (c))2 = cf 00 (c) = 1 − e−c =⇒ f 00 (c) = c >0
(2) Cleverly, consider the limit.
µ ¶
1 − e−x x→0
xf 00 (x) + 3x(f 0 (x))2 = 1 − e−x =⇒ f 00 (x) + 3(f 0 (x))2 = −−−→ f 00 (0) + 0 = 1
x
So a critical point at x = 0 would be a minimum.
(3) We'll have to “cheat” a little and use the idea of power series early on here.

1−e−x
f 00 + 3(f 0 )2 = x suggests that we consider the Taylor series of e−x .
P∞ (−x)j ∞
1 − e−x − j=1 j!
X (−1)j xj
= =
x x j=0
(j + 1)!

This further suggests that f itself has a power series representation because its rst and second order derivatives are
simply a combination of innitely many terms containing powers of x.
165
P∞
Then suppose f= j=0 aj xj .

X
f0 = (j + 1)aj+1 xj
−x
1−e j=0
f 00 + 3(f 0 )2 = =⇒ ∞
x X
f 00 = (j + 2)(j + 1)aj+2 xj
j=0

X ∞ X
X ∞ ∞
X (−1)j xj
=⇒ (j + 2)(j + 1)aj+2 xj + 3 (j + 1)(k + 1)aj+1 ak+1 xj+k =
j=0 j=0 k=0 j=0
(j + 1)!
If f (0) = 0 a0 = 0
0
If f (0) = 0 a1 = 0
P∞
Then f = a2 x2 + j=3 aj xj . Consider the x0 terms in the DE. (f 0 )2 doesn't contribute, because f 0 's leading order
1
term is x . So then

1 1 2 X
2(1)a2 + 0 = 1 =⇒ a2 = i.e. f= x + aj xj
2 2 j=3

1
A= in order for f (x) ≤ Ax2
2

9.6 Exercises - Historical introduction, Denitions and eld properties, The complex numbers as an extension of the
real numbers, The imaginary unit i, Geometric interpretation. Modulus and argument.
Exercise 6. Let f be a polynomial with real coefcients.

(1) Since z1 z2

(z1n+1 ) = z1n z1 = z1n z1 = z n+1


1
X X
f (z) = aj z j = aj z j = f (z)

(2) If f (z) = 0, then f (z) = f (z) = 0 as well.

Exercise 7. The three ordering axioms are

Ax. 7 If x, y ∈ R+ , x + y, xy ∈ R+
Ax. 8 ∀ x 6= 0, x ∈ R+ or − x ∈ R+ but not both
Ax. 9 / R+
0∈

x < y means y − x positive.

Suppose i positive: i(i) = −1 but −1 is not positive.


Suppose −i is positive. −i(−i) = −1 but −1 is not positive.
i is neither positive nor negative so Ax. 8 is not satised.

Exercise 8. Ax. 8 , Ax. 9 are satised.

(a + ib)(c + id) = (ac − bd, ad + bc) so Ax. 7 might not be satised.

Exercise 9. Ax. 7, Ax. 8 , Ax. 9 are trivially satised (all are positive).

Exercise 10. x > y x > y is well dened. Ax. 8 is satised.

For Ax. 7, ( 32 , 1), (1, 12 ) contradicts Ax. 7 since we required the product to be positive as well if the factors are positive.
We found this particular counterexample by considering factors (a, b), (c, d), so the product of the two is (ac − bd, ad + bc)
and so we need ac − bd − ad − bc = a(c − d) − b(c + d) < 0
Exercise 11. See sketch.

166
Exercise 12.
az + b
w=
cz + d
(az + b)(cz + d) ac|z|2 + adz + bcz + bd
w= = 2 2
(cz + d)(cz + d) c |z| + cd(z + z) + d2
ac|z|2 + adz + bcz + bd − ac|z|2 − adz − bcz − bd (ad − bc)(z − z)
w + −w = 2
=
|cz + d| |cz + d|2
If ad − bc > 0
ad − bc ad − bc
w − w = 2Imw = 2Imz; >0
|cz + d|2 |az + d|2
So Imw has the same sign as Imz

9.10 Exercises - Complex exponentials, Complex-valued functions, Examples of differentiation and integration formu-
las.
Exercise 7.

(1)
Z 2π ¯2π
ix(n−m) eix(n−m) ¯¯ 1−1
if m 6= n , e dx = ¯ = =0
0 i(n − m) 0 i(n − m)
Z 2π
if m=n, eix(0) dx = 2π
0
(2)
Z 2π Z 2π
einx e−imx dx = (cos nx + i sin nx)(cos mx − i sin mx) =
0 0
Z 2π
= cos nx cos mx + sin nx sin mx + i(sin nx cos mx − sin mx cos nx)
0
Z 2π Z 2π
e−inx e−imx dx = (cos nx − i sin nx)(cos mx − i sin mx) =
0 0
Z 2π
= cos nx cos mx − sin nx sin mx + i(− sin nx cos mx − sin mx cos nx)
0

Summing the two equations above


Z 2π Z 2π
0= 2 cos nx cos mx + 2 i − sin mx cos nx
0 0
Z 2π Z 2π
=⇒ cos nx cos mx = 0, sin mx cos nx = 0
0 0
Z 2π Z 2π
inx −imx
e e dx = (cos nx + i sin nx)(cos mx − i sin mx) =
0 0
Z 2π
= cos nx cos mx + sin nx sin mx + i(sin nx cos mx − sin mx cos nx)
0
Z 2π Z 2π
einx eimx dx = (cos nx + i sin nx)(cos mx + i sin mx) =
0 0
Z 2π
= cos nx cos mx − sin nx sin mx + i(sin nx cos mx + sin mx cos nx)
0

Subtract the two equations above


Z 2π Z 2π
sin nx sin mx − i sin mx cos nx = 0
0 0
Z 2π
=⇒ sin nx sin mx = 0
0
167
Z 2π Z 2π
einx e−inx = 2π = (cos nx + i sin nx)(cos nx − i sin nx) =
0 0
Z 2π
= cos2 nx + sin2 nx
0
Z 2π Z 2π
e inx inx
e =0= cos2 nx − sin2 nx + i(2 cos nx sin nx)
0 0
Z 2π Z 2π
2 2
=⇒ cos nx − sin nx = 0 cos nx sin nx = 0
0 0
Z 2π
2π = 2 cos2 nx
0
Summing the two results above, we obtain Z 2π
=⇒ cos2 nx = π
0
Z 2π
Then also, sin2 nx = π
0
Exercise 8.
z = reiθ = rei(θ+2πm) , m ∈ Z
z 1/n = r1/n ei(θ/n+2πm/n) m = 0, 1, . . . n − 1
=⇒ z 1/n = Reiα ²m = z1 ²m
The roots are spaced equally by an angle 2π/n
iπ/2+i2πn 1/3 iπ/6 i5π/6
i=e =⇒ i =e ,e , ei3π/2
i1/4 = eiπ/8 , e5iπ/8 , e9iπ/8 , e13iπ/8
−i = e−iπ/2+i2πn =⇒ (−i)1/4 = e−iπ/8 , e3iπ/8 , e7iπ/8 , e11iπ/8
Exercise 9.

eiu eiv = ei(u+v) = cos u + v + i sin u + v =


= (cos u + i sin u)(cos v + i sin v) = cos u cos v − sin u sin v + i(cos v sin u + cos u sin v)
=⇒ sin u + v = cos v sin u + cos u sin v
=⇒ cos u + v = cos u cos v − sin u sin v
µ iz ¶2 µ iz ¶2
2 2 e − e−iz e + e−iz
sin z + cos z = + =
2i 2
−(e2iz + e−2iz − 2) + (e2iz + 2 + e−2iz )
= =1
4
eiiy + e−iiy eiiy − e−iiy
cos iy = = sin iy = =
2 2i
e−y + ey e−y − ey
= = cosh y = = i sinh y
2 2i
eiz = ei(x+iy) = eix e−y = (cos x + i sin x)e−y
e−iz = e−i(x+iy) = e−ix ey = (cos x − i sin x)ey
Thus it is clear, by mentally adding and subtracting the above results that

cos z = cos x cosh y − i sin x sinh y


=⇒
sin z = i cos x sinh y + sin x cosh y
Exercise 10.

(1) Log(−1) = iπ log (i) = ln 1 + i π2 = i π2


(2) Log(z1 z2 ) = Log(|z1 ||z2 |ei(θ1 +θ2 ) ) = ln |z1 ||z2 | + i(θ1 + θ2 + 2nπ) = Logz1 + Logz2 + i2πn
(3) Log(z1 /z2 ) = Log(|z1 |/|z2 |ei(θ1 −theta2 ) ) = ln |z 1|
|z2 | + i(θ1 − θ2 + 2nπ) = Logz1 − Logz2 + i2πn
(4) exp (Logz) = exp (ln |z| + iθ + i2πn) = z
Exercise 11.

168
(1)
1i = eiLog1 = ei(i2πn) = e−2πn = 1 if n = 0
π π
ii = eiLogi = ei(i 2 +i2πn) = e− 2 −2πn = e−π/2 if n=0
(−1)i = eiLog−1 = ei(iπ+i2πn) = e−π−2πn = e−π
(2) z a z b = eaLogz ebLogz = eaLogz+bLogz = e(a+b)Logz = z a+b x
(3)
(z1 z2 )w = ewLogz1 z2 = ew(Logz1 +Logz2 +2πmi)
(z1w z2w ) = ewLogz1 ewLogz2 = ew(Logz1 +Logz2 )
m = 0 is the condition required for equality.
Exercise 12.

if L(u) = P, L(v) = Q,
L(u + iv) = (u + iv) + a(u + iv) + b(u + iv) = u00 + au0 + bu + i(v 00 + av 0 + bv) = L(u) + iL(v) = P + iQ = R
00 0

if L(f ) = R
L(u + iv) = L(u) + iL(v) = P + iQ
then, equating real and imaginary parts, L(u) = P, L(v) = Q
Exercise 13.

L(y) = −ω 2 y + aiωy + by = Aeiωx =⇒ (−ω 2 + aiω + b)B = A


2 2
We cannot let (−ω + aiωb) = 0 for a nontrivia solution. Thus b 6= ω or aω 6= 0.

A
B=
−ω 2 + aiω + b
Exercise 14.
c
y ) = ceiωx ; yb = Beiωx =
L(b eiωx
−ω 2 + aiω + b
c aω
=⇒ yb = p ei(ωx−α) where tan α =
2 2
(b − ω ) + (aω) 2 b − ω2
c
=⇒ <by=p cos (ωx − α)
(b − ω 2 )2 + (aω)2
Exercise 15.
c
y) = p
=(b sin (ωx + α)
(b − ω 2 )2 + (aω)2
c aω
=⇒ A = p ; − tan α =
(b − ω 2 )2 + (aω)2 b − ω2

10.4 Exercises - Zeno's paradox, Sequences, Monotonic sequences of real numbers. Exercise 1. Converges to 0.
−2
n n+1 2
n − (n + 2n + 1) 2
−2n − 1 n − n12 n→∞
f (n) = − = = 2 = −−−−→ 0
n+1 n n(n + 1) n +n 1 + n1
Exercise 2. Converges to −1.
n3 − (n3 + n + n2 + 1) n2 − n − 1 −1 − n1 − 1
n2 n→∞
f (n) = = = −−−−→ −1
(n + 1)n n(n + 1) 1 + n1
Exercise 3. Diverges since
nπ ¯¯ nπ ¯¯ ¯
¯¯ ¯
|cos − L| ≥ ¯¯1 cos ¯ − |L|¯ ≥ |1 − |L||
2 2
|1−|L|| nπ
Choosing ²1 = , | cos − L| > ²1 for n = 4m.
2 2
1 3 2 1
Exercise 4. f (n) =
5 + 5n − 5n2 → limn→∞ f (n) = 5
x x xα
Exercise 5. f (x) = x =
2 exp (x ln 2) → 0 since limx→∞ (ex )β
.
n
Exercise 6. f (n) = 1 + (−1) = 0 of 1.

|1−|L||
Thus, choosing ²1 = ;
2
|f (n) − L| ≥ ||f (n)| − |L|| = |1 − |L|| > ²1 for any n = 2m
n
1+(−1)
Exercise 7. f (n) = n .
169
3
Suppose ²= N.

1 1
So for n > N, N > n, n ≥ N = N (²) = 3/².
¯ ¯
¯ 1 + (−1)n ¯
|f (n)| = ¯¯ ¯≤ 2 < 3 < 3 =²
n ¯ n n N
(−1)n 1+(−1)n
Exercise 8. f (n) = n + 2
¯ ¯ ¯¯ ¯ ¯ ¯¯
¯ (−1)n 1 + (−1)n ¯ ¯¯ (−1)n ¯¯ ¯¯ 1 + (−1)n ¯¯¯¯
|f (n) − L| = ¯¯ + ¯ ¯ ¯
− L ¯ ≥ ¯¯L − −
n 2 n ¯ ¯ 2 ¯¯ ≥
¯¯ ¯ ¯ ¯ ¯ ¯ ¯ ¯ ¯ ¯ ¯ ¯¯
¯¯ ¯ (−1)n ¯¯ ¯ 1 + (−1)n ¯¯ ¯¯ 1 ¯¯ ¯¯ 1 + (−1)n ¯¯¯¯
¯ ¯
≥ ¯¯|L| − ¯¯ ¯ ¯ − ¯ ¯ ¯ ¯ ¯
n ¯¯ ¯ 2 ¯¯ = ¯¯|L| − n ¯ = ¯ 2 ¯¯ ≥
¯¯ ¯ ¯
¯¯ 1 ¯ 1¯
≥ ¯¯¯¯|L| − ¯¯ − ¯¯
n 2
Thus, consider
¯¯ ¯ ¯ ¯¯ ¯ ¯
¯¯ ¯ 1 ¯¯ ¯¯¯¯ 1 ¯ ¯
¯¯|L| − 1 ¯ − > − |L| ¯ − 1¯ = ²0 for n>N
¯¯ n¯ 2 ¯ ¯ ¯ N ¯ ¯
¡1 ¢
Exercise 9. f (x) = exp x ln 2 ; limx→∞ f (x) = 0.
Exercise 10.
n n
|f (n) − L| = |n(−1) − L| ≥ ||n(−1) | − |L|| = ||n| − |L|| = |n| − |L| > N − |L|
Thus, for n > N , N (²) = ² + |L|, so then |f (n) − L| > ².
n2/3 sin n!
Exercise 11. f (n) = n+1 .
¯ 2/3 ¯ ¯ ¯ ¯ ¯
¯ n sin (n!) ¯ ¯ sin (n!) ¯ ¯ 1 ¯
|f (n)| = ¯¯ ¯=¯ ¯≤¯ ¯
n + 1 ¯ ¯ n1/3 + n−2/3 ¯ ¯ n1/3 ¯
1
Thus, for n > N, N (²) = ²3 , |f (n)| < ².
Exercise 12. Converges, since
µ ¶
1 3n+1 + 3(−2)n − 3n+1 − (−2)n+1 (−2)n (3 + 2)
f (n) − = = =
3 3(en+1 + (−2)n+1 ) 3(3n+1 + (−2)n+1 )
µ ¶
5 (−2)n
=
3 3n+1 + (−2)n+1
¯ ¯ ¯ ¯ ¯ ¯
¯ ¯ ¯ (−2)n ¯ ¯ −(−2)n+1 ¯
¯f (n) − 1 ¯ = 5 ¯ ¯≤¯ ¯=
¯ 3 ¯ 3 ¯ 3n+1 + (−2)n+1 ¯ ¯ 3n+1 + (−2)n+1 ¯
¯ ¯ ¯ ¯
¯ ¯ ¯ ¯
¯ −1 ¯ ¯ 1 ¯
¯ ¯ ¯ ¯
= ¯ ³ ´n+1 ¯=¯ ³ ´n+1 ¯ <
¯ 3 ¯ ¯ 3 ¯
¯ −2 + 1 ¯ ¯ 1 + −2 ¯
1 1
< ¡ ¢n+1 < ¡ 3 ¢n
3
2 2
¡ 2 ¢−N − ln ²
For n > N , consider ² = 3 , i.e. N= ln 2/3 = N (²). Thus

1
L= ;
3
Exercise 13.
√ √
f (n) = n+1− n
µ√ √ ¶
¡√ √ ¢ n+1+ n n+1−n √ √ √
f (n) = n+1− n √ √ =√ √ = 1 n+1+ n
n+1+ n n+1+ n
¯ ¯
¯ 1 ¯ 1
|f (n)| = ¯¯ √ √ ¯≤ √ ;
n + 1 + n¯ 2 n
So then∀², we have ² = 2√1N and for n > N, 1

2 n
< 1

2 N
= ².
Thus f (n) converges to 0.
170
Exercise 14.
n xa
f (n) = nan = n exp n ln a = →0 since lim =0
exp n ln a1 x→∞ (ex )b

(log x)a
Exercise 15. f (n) = logna n , a > 1. limn→∞ f (n) = 0 since limx→∞ xb
= 0 for a > 0, b > 0
Exercise 16. limn→∞ f (n) = 0
Exercise 17. limn→∞ f (n) = e2 .
Exercise 18. ¯ ¯ ¯¯ ¯ ¯ ¯¯ ¯ ¯
¯ ¯ ¯¯ ¯ ¯ ¯¯ ¯ ¯
¯1 + n cos nπ − L¯ ≥ ¯¯1 + n cos nπ ¯ − |L|¯ = ¯¯1 + −n ¯ − |L|¯ =
¯ n+1 2 ¯ ¯ ¯ n+1 2 ¯ ¯ ¯ ¯ n+1 ¯ ¯
¯ ¯
¯ 1 ¯
= ¯¯ − |L|¯¯ > |L|
n+1
|L|
Choose ²0 = N , for n > N , |f (n) − L| > ²0 .
2 . For any
Exercise 19. √
i 5 iα Ã√ !−n µ ¶n
1+ = e 5 iα 2
2 2 e , √ e−niα
µ ¶2
1 5 2 5
1+ = µ ¶n
2 4 2
1 lim √ e−niα = 0
tan α = n→∞ 5
2
Exercise 20. limn→∞ f (n) = limn→∞ e−πin/2 diverges since
|eiπin/2 − L| ≥ ||e−iπin/2 | − |L|| = |1 − |L|| > |L|
|L|
So for ²0 = n > N , |f (n) − |L|| > ²0
2 , for
1 −iπn/2 1
Exercise 21. f (n) = ne |f (n)| = n.
1
Suppose N (²) = ² ; then for n > N , |f (n)| < ²0
Exercise 22. |f (n) − L| ≥ ||ne−πin/2 | − |L|| = |n − |L||
Consider ²0 = |1 − |L|| for n > N > 1, |f (n) − L| > ²0 .
Exercise 23. an = n1 .
¯ ¯
¯1¯ 1 1
|an | = ¯¯ ¯¯ < ; N (²) =
n N ²
² = 1, 0.1, 0.01, 0.001, 0.0001
N = 1, 10, 100, 1000, 10000
¯ ¯
¯ ¯
Exercise 24. |an − 1| = ¯ n+1−n 1
n+1 ¯ = n+1 < n
1

1
N (²) = ² = 1, 10, 100, 1000, 10000.
1
Exercise 25. |an | = n
N (²) = 1, 10, 100, 1000, 10000.
¯1¯
Exercise 26. |an | = ¯ ¯ 1 1
n! ≤ exp n ln n < exp n
1
For n > N, exp N = ², so that N = ln 1/².
N (²) = 1, 2, 4, 6, 9
¯ ¯
¯ 2 ¯ ¯¯ 2 ¯¯
Exercise 27. an = n22n
+1 ; |an | = ¯ 2
n +1/n ¯ ≤ n2 .

2
N (²) = √ = 1, 4, 14, 44, 141
²
¯ 9 ¯n ¡ 9 ¢n
Exercise 28. |an | = ¯ 10 ¯ =
10 = en ln 9/10
ln ² − ln 1/²
N (²) = ¡9¢= =
ln 10 ln (9/10)
= 1, 21, 43, 65, 87
171
Exercise 30. If ∀² > 0, ∃N ∈ Z+ such that n > N , |an | < ².
|an |2 < |an |² < ²2
|a2n | < ²2

So for ∀²1 > 0, ²1 = ²2 and ∃N = N (²) = N (²1 ), so that |a2n | < ²1 .


Exercise 31.

|an + bn − (A + B)| = ||an − A||bn − B|| ≤ |an − A| + |bn − B| < ² + ² = 2²

∀² > 0, ∃NA , NB ∈ Z+ , |an − A| < ² if n > NA ; |bn − B| < ² if n > NB


Consider max (NA , NB ) = NA+B
|an + bn − (A + B)| < 2²
∀²1 > 0, ²1 = 2², then ∃NA+B = NA+B (²1 ) ∈ Z+ such that
|(an + bn ) − (A + B)| < ²1 if n > NA+B

|can − cA| = c|an − A| < c²


∀²1 > 0, ²1 = c²; then ∃NcA = N (²) = N (²1 ) ∈ Z+ such that
|can − cA| < ²1 for n > N (²1 )

Exercise 32. Given limn→∞ an = A,


lim (an − A)(an + A) = lim (an − A) lim (an + A) = 0(2A) = 0
n→∞ n→∞ n→∞
2
2an bn = (an + bn ) − a2n − b2n
³ ´2
2 lim an bn = lim (an + bn )2 − lim a2n − lim b2n = lim (an + bn ) − A2 − B 2 = 2AB
n→∞ n→∞ n→∞ n→∞ n→∞
=⇒ lim an bn = AB
n→∞

¡α¢ α(α−1)(α−2)...(α−n+1)
Exercise 33.
n = n!
(1)
µ 1¶
−2 −1/2
= µ 1¶ ¡ ¢ ¡ −3 ¢ ¡ −5 ¢ ¡ −7 ¢
1! 1 −2 − 12 22 2 35
µ 1¶ ¡ 1 ¢ ¡ −3 ¢ = =
−2 −2 2 5 4 4! 128
= = µ 1¶ ¡ 1 ¢ ¡ −3 ¢ ¡ −5 ¢ ¡ −7 ¢ ¡ −9 ¢
2! 2! 8 −2 −2 2 2 2 2 −63
µ 1¶ ¡ 1 ¢ ¡ −3 ¢ ¡ −5 ¢ = =
−2 −2 2 2 5 5 5! 256
= =−
3! 3! 16
¡ −1 ¢
(2) an = (−1)n n2 .
1 3
a1 = 2 > 0. a2 = 8 >0
µ ¶ µ ¶µ 1 ¶ ¡ ¢
n+1− 12 n+1 α − 2 − (n + 1) + 1 an (−1) −1
2 −n
an+1 = (−1) = (−1) = =
n+1 n n+1 n+1
an (n + 1/2)
= >0
n+1
µ ¶
n + 1/2
an+1 = an < an
n+1
Exercise 34.

(1)

µ ¶ n−1 µ ¶
à à !!
1X
n
k 1X k 1 X µk¶
n−1
1X
n−1 µ ¶
k
tn − sn = f − f = f (1) + f − f (0) + f =
n n n n n n n n
k=1 k=0 k=1 k=1
1
= (f (1) − f (0))
n
172
¡k¢ ¡ k+1 ¢ k k+1
Since f n ≤ f (t) ≤ f n for
n ≤t≤ n , by f being monotonically increasing.
Z 1
=⇒ sn ≤ f (x)dx ≤ tn (from denition of integral)
0
Z 1
1
0≤ f (x)dx − sn ≤ tn − sn = (f (1) − f (0))
0 n
(2) Use Theorem 1.9.

Theorem 30. Every function f which is bounded on [a, b] has a lower integral I(f ) and an upper integral I(f )
satisfying
Z b Z b
s(x)dx ≤ I(f ) ≤ I(f ) ≤ t(x)dx
a a
for all step functions s and t with s ≤ f ≤ t. The function f is integrable on [a, b] iff its upper and lower integrals
are equal,
Z b
f (x)dx = I(f ) = I(f )
a
R1
Since f (x) is integrable, then limn→∞ sn = limn→∞ tn = 0 f (x)dx
¡ b−a ¢ 1
P n−1 1
P n
(3)
n = ∆ , sn = ∆ k=0 f (a + k∆), tn = ∆ k=1 f (a + k∆)
Rb
So by increasing monotonicity of f , sn ≤ f (x)dx ≤ tn .
a
à n−1 n−1
!
1 X X f (b) − f (a)
tn − sn = f (b) + f (a + k∆) = f (a + k∆) − f (a) =
∆ ∆
k=1 k=1
Z b
f (b) − f (a)
0≤ f (x)dx ≤
a ∆
Exercise 35.
Pn ¡ ¢2 R 1
(1) limn→∞ n1 k=1 nk = 0 t2 dt = 13
Pn 1
Pn R1
(2) limn→∞ k=1 n+k = limn→∞ n1 k=1 1+1 k = 0 x1 dx = ln 2
n

1
Pn 1
R1 1 1 π
(3) limn→∞ n k=1 k 2
= 0 1+x2 dx = arctan x|0 =
1+( n ) 4
Pn Pn R1 1 √ ¯1 √
(4)
1 1
limn→∞ k=1 √n2 +k2 = limn→∞ n k=1 q 1
k 2
= 0 √1+x2 dx = ln (x + 1 + x2 )¯0 = ln (1 + 2)
1+( n )
Pn Pn R1 ¡ − cos πx ¢¯1
limn→∞ k=1 n1 sin kπ 1 kπ ¯ = −(−1−1) = 2
(5)
n = limn→∞ n k=1 sin n = 0 sin πxdx = π 0 π π
Pn R n 1
(6) limn→∞ k=1 n1 sin2 kπ 2
n = 0 sin xπ = 2

10.9 Exercises - Innite series, The linearity property of convergent series, Telescoping series, The geometric series.
P∞ 1
P∞ 1/2 1/2 1
Exercise 1.
n=1 (2n−1)(2n+1) = n=1 2n−1 − 2n+1 = 2
P∞ 2
P∞ 1 1
Exercise 2.
n=1 3n−1 = 2 n=0 3n = 2 1−1/3 = 3
P∞ 1
P∞ 1/2 1/2 P∞ ³ 1/2 1/2
´ ³
1/2 1/2
´
1 1 3
Exercise 3.
n=2 n2 −1 = n=2 n−1 − n+1 = n=2 n−1 − n +− n+1 − n = 2 + 4 = 4
P∞ 2n +3n
P∞ ¡ 1 ¢n P∞ ¡ 1 ¢n 1/3 1/2 1 3
Exercise 4.
n=16n = n=1 3 + n=1 2 = 1−1/3 + 1−1/2 = 2 +1= 2
P∞ √n+1−√n P∞
Exercise 5. √ = √1 − √1 =1
n=1 n2 +n n=1 n n+1
Exercise 6.

X ∞
X ∞
X µ ¶
n 3/2 −1/2 1 1 −1
= + = + =
n=1
(n + 1)(n + 2)(n + 3) n=1 (n + 2)(n + 3) (n + 1)(n + 2) n=1 (n + 2)(n + 3) 2 6

X 1 1 1 1 1 1
= − − = − =
n=1
1 + 2 n + 3 12 3 12 4
P∞ 2n+1
P∞ 1 1
Exercise 7.
n=1 n2 (n+1)2 = n=1 n2 − (n+1)2 = 1
P∞ 2n +n2 +n P∞ 1 1 1
P∞ 1 1 1/4 1 1
Exercise 8.
n=1 2n+1 n(n+1) = 1 = n=1 2n(n+1) + 2n+1 = 2 n=1 n − n+1 + 1−1/2 = 2 + 2 =1
173
P∞ P∞ ³
´
(−1)n−1 (2n+1) n−11 1
Exercise 9. n=1 n(n+1) = n=1 (−1)
+ n+1 .
n
µ ¶ µ ¶ µ ¶
n−1 1 1 n 1 1 n −1 1
(−1) + + (−1) + = (−1) +
n n+1 n+1 n+2 n n+2
X∞ µ ¶ ∞ µ ¶
−1 1 1X −1 1 1 −1
+ = + = = −1
j=1
2j − 1 2j + 1 2 j=1 (j − 1/2) (j + 1/2) 2 1/2

Exercise 10.

∞ ∞ ¡¡ ¢n ¢
X log ((1 + n1 )n (1 + n)) X log n+1 (1 + n)
n
= =
n=2
(log nn )(log (n + 1)n+1 ) n=2 log (n + 1)n+1 log nn
X∞ X∞
log(n + 1)n+1 − log nn 1 1
= n+1 log nn
= n
− =
n=2
log (n + 1) n=2
log n log (n + 1)n+1
1 √
= = log2 e
2 log 2
1

since if = y , then y = log2 2.
2 log 2
P∞ P∞ n ³ ´0
n d x x
Exercise 11. n=1 nx = x dx n=1 x = x 1−x = (1−x)2 .
Exercise 12.

X ∞ µ ¶0
d X n x
n2 x n = x nx = x =
n=1
dx n=1 (1 − x)2
(1 − x)2 + 2(1 − x)x x(1 − x2 ) x(1 + x)
=x 4
= 4
=
(1 − x) (1 − x) (1 − x)3
Exercise 13.


X ∞ µ ¶0
3 nd X 2 n x + x2
n x =x n x =x =
n=1
dx n=1 (1 − x)3
(1 + 2x)(1 − x)3 + 3(1 − x)2 (x)(x + 1) (1 + 2x)(1 − x) + 3x(x + 1) x(x2 + 4x + 1)
=x 6
=x 4
=
(1 − x) (1 − x) (1 − x)4
P∞ P∞ ³ ´0
4 4 d 3 3 x3 +4x2 +x
Exercise 14.
n=1 n x = x dx n=1 n x = x (1−x)4 .

µ ¶
x3 + 4x2 + x
ln = ln (x3 + 4x2 + x) − 4 ln (1 − x)
(1 − x)4
1 3x2 + 8x + 1 1
(ln f )0 = f 0 = 3 +4 ;
f x + 4x2 + x 1−x
(3x2 + 8x + 1)(1 − x) 4(x3 + 4x2 + x)
f0 = + =
(1 − x)5 (1 − x)5
3x2 + 8x + 1 − 3x3 − 8x2 − x + 4x3 + 16x2 + 4x x3 + 11x2 + 11x + 1
= 5
=
(1 − x) (1 − x)5
x4 + 11x3 − 11x2 + x
=⇒
(1 − x)5
P∞ xn
P∞ R x Rx P∞ Rx 1
Exercise 15.
n=1 n =x n=1 0 tn−1 dt = 0
dt n=1 tn−1 = 0 1−t
= − ln (1 − x).
Exercise 16.

X X∞ Z x Z x X ∞
x2n−1 2j−2
= t dt = dt (t2 )j−1 =
n=1
2n − 1 j=1 0 0 j=1
Z x Z x µ ¶ µ ¶
dt 1/2 1/2 1 1+x
= 2
= dt + = ln
0 1−t 0 1−t 1+t 2 1−x
P∞ P∞ d n+1 ³ ´
n d x 1
Exercise 17.
n=0 (n + 1)x = n=0 dx x = dx 1−x = (1−x)2
174
Exercise 18.

X∞ ∞ µ ¶00 µ ¶ µ ¶
(n + 1)(n + 2) n X xn+2 d2 x2 1 d x x2
x = = 2 = +
n=0
2! n=0
2 dx 2 1 − x dx 1 − x 2(1 − x)2
1 x x x3 1 − 2x + x2 + 2x − 2x2 + x2 1
= + 2
+ 2
+ 3
= =
1 − x (1 − x) (1 − x) (1 − x) (1 − x)3 (1 − x)3
Exercise 19.

X ∞ ∞ ∞
(n + 1)(n + 2)(n + 3) n X d3 xn+3 1 d X d2 x(n+1)+2
x = = =
n=0
3! n=0
dx3 3! 3 dx n=0 dx2 2

Ã∞ µ ¶!
1 d X d2 xn+2 1 d X d2 xn+2 d2 x2
= = − 2 =
3 dx n=1 dx2 2 3 dx n=0 dx2 2 dx 2
µ ¶
1 d 1 1 −3(−1) 1
= 3
− 1 = 4
=
3 dx (1 − x) 3 (1 − x) (1 − x)4
P∞ k n Pk (x)
Exercise 20.
n=1 n x = (1−x)k+1

X∞ ∞ µ ¶ µ 0 ¶
k+1 n d X k n d Pn (x) Pk (x)(1 − x)k+1 + (k + 1)(1 − x)k Pk (x)
n x =x n x =x =x =
n=1
dx n=1 dx (1 − x)k+1 (1 − x)2k+2
µ 0 ¶
Pk (x)(1 − x) + (k + 1)Pk (x) (k + 1)xPk (x) + x(1 − x)Pk0 (x)
=x =
(1 − x)k+2 (1 − x)k+2
((k + 1)Pk (x) + (1 − x)Pk0 (x))x has x as its lowest degree term from xPx0 (x) and
(k + 1)xk+1 + −kxk+1 = xk+1 highest degree term is obtained from (k + 1)Pk (x) + −xPk0 (x).
P∞ ¡n+k¢ n 1 dk
P∞ xn+k
Exercise 21.
n=0 k x = (1−x)k+1 = dxk n=0 k! .

X∞ µ ¶ ∞ ∞
n + k + 1 n X dk+1 xn+k+1 1 d X dk x(n+1)+k
x = = =
n=0
k+1 n=0
dxk+1 (k + 1)! (k + 1) dx n=0 dxk k!
∞ µ ¶ ∞
1 d X dk xn+k 1 d X dk xn+k dk xk
= k
= k
− k =
k + 1 dx dx k! k + 1 dx n=0 dx k! dx k!
k=1
µ ¶ µ ¶
1 d 1 1
= k+1
− 1 =
k + 1 dx (1 − x) (1 − x)k+2
Exercise 22.
P∞ n−1
P∞ 1
P∞ 1 P∞ 1 P∞ 1
(1) n=2 n! = n=2 (n−1)! − n=2 n! = n=1 n! − n=2 n! = 1 .
P∞ n P∞ 1 P∞ 1
P∞ 1 P∞ 1 P∞ 1 P∞ 2
(2) n=2 n! + n=2 n! = n=2 (n−1)! + n=0 n! − 1 − 1 = n=1 n! + n=0 n! −2 = n=0 n! − 3 = 2e − 3 .
(3)
X∞ X∞ 2 X∞ X∞ X∞
(n − 1)(n + 1) n −1 n −1
= + = + =
n=2
n! n=2
n! n=2
n! n=2
(n − 1)! n=2
n!
X∞ ∞ ∞ ∞
n+1 X 1 X 1 X 1
= + − = +1= +1= e+1
n=1
n! n=2
n! n=1
(n − 1)! n=0
n!
Exercise 23.
¡ d P∞ xn ¢ ³ P n−1
´ P∞ nxn P∞ n2 xn ¡ d x¢
d d ∞
(1) x dx x dx n=1 n! = x dx x n=1 nxn! d
= x dx n=1 n! =
d
n=1 n! = x dx x dx e = x2 ex + xex
³P ´ P ¡ ¢ ¡ ¢
d ∞ n 2 xn ∞ 3 n
(2) x dx n=1 n! = n=1 n n!x = x dx
d
(x2 + x)ex = x (2x + 1)ex + (x2 + x)ex = (x3 + 3x2 + x)ex

x=1 k=5
Exercise 24.
P∞ n
P∞ n
(1)
P∞ n=2 (−1) = P n=2 (−1) (n − (n − 1)). Identical.

(2) = n=2 (−1)n . Not
n=2 (1 − 1) P P∞identical.
∞ n
(3) Not identical. n=2 (−1) vs. ( ³¡(−1 + 1)) + 1.´
P∞ ¡ 1 ¢n P∞ n=2 ¢
1 n−1
¡ ¢n P∞ ¡ ¢n P∞ ¡ ¢n
(4) Identical. n=0 2 = 1 + n=1 2 − 12 = n=1 12 (2 − 1) = n=0 12
Exercise 25.

175
(1)
1
1 + x2 + x4 + · · · + x2n + · · · = if |x| < 1
1 − x2
1
=⇒ 1 + 0 + x2 + 0 + x4 + · · · = if |x| < 1
P∞ P∞ P∞ 1 − x2
(2) Thm. 10.2. n=1 (αan + βbn ) = α n=1 an + β n=1 bn . So then

X X∞ X∞ X∞
xj + (−x)j xj − (−x)j 1 1 x
xj − = = x2j+1 = − 2
=
j=0 j=0
2 j=0
2 j=0
1−x 1−x 1 − x2
(3)

X ∞
X ∞
X x
(x2 )j + xj = (xj − x2j ) =
j=0 j=0 j=0
1 − x2

10.14 Exercises - Tests for convergence, Comparison tests for series of nonnegative terms, The integral test. We'll be
using the integral test.

Theorem 31 (Integral Test).


Letf be a positive decreasing
Pn function, dened for all real x ≥ 1.
Pn
For∀n ≥ 1, let sn = k=1 f (k) and tn = 1 f (x)dx.
Then both sequences {sn } and {tn } converge or both diverge.

Exercise 1.

3 −1 3(4j − 1) + (−1)(4j − 3) 8j
+ = =
4j − 3 4j − 1 (4j − 3)(4j − 1) (4j − 3)(4j − 1)
X n Xn µ ¶
j 3/8 −1/8
= +
j=1
(4j − 3)(4j − 1) j=1 4j − 3 4j − 1
Z nµ ¶ µ ¶¯n
3/8 −1/8 ln (4x − 3) ln (4x − 1) ¯¯
+ dx = (3/8) + (−1/8) ¯ =
1 4x − 3 4x − 1 4 4 1
¯ n µ ¶
1 (4x − 3)3 ¯¯ 1 3(4n − 3)3
= ln = ln
32 4x − 1 ¯1 32 4n − 1
µ 3
¶ Z n X n
1 3(4n − 3) xdx j
lim ln = lim dx = ∞, so diverges as well
n→∞ 32 4n − 1 n→∞ 1 (4x − 3)(4x − 1) (4j − 3)(4j − 1)
j=1

Exercise 2.
∞ √
X ∞
X
2j − 1 log (4j + 1)
= aj
j=1
j(j + 1) j=1
√ µ ¶
4j + 1 log (4j + 1) 4 log (4j + 1) (4j + 1) log (4j + 1)(4 + 1/j)
aj ≤ = =4 ≤
j(j + 1/4) j(4j + 1)1/2 (4j + 1) (4j + 1)3/2
16 log (4j + 1)
≤ = bj
(4j + 1)3/2
X X
Now use the integral test on bj to determine the convergence of bj .
Z n Z µ ¶0
log (ax + 1) (−2)
3/2
dx = log (ax + 1)dx =
1 (ax + 1) a(ax + 1)1/2
Z µ ¶
−2 −2 a
= log (ax + 1) − =
a(ax + 1)1/2 a(ax + 1)1/2 ax + 1
−2 −4
= 1/2
log (ax + 1) +
a(ax + 1) a(ax + 1)1/2
Z n à õ ¶1/2 !!
log(ax + 1) −2 log (an + 1) 2 log (a + 1) −4 1 1
lim dx = lim + + − =
n→∞ 1 (ax + 1)3/2 n→∞ a(an + 1)1/2 a(a + 1)1/2 a an + 1 (a + 1)1/2
2 log (a + 1) 4
= +
a(a + 1)1/2 a(a + 1)1/2
P P P
Then by integral test, bj converges. Since bj converges, then aj converges by comparison test.
176
P∞ j+1
Exercise 3.
j=1 2j .
Z n Z n µ ¶¯n
x+1 xe−x ln 2 e−x ln 2 −e−x ln 2 ¯
dx = (xe−x ln 2 + e−x ln 2 )dx =
+ + ¯
ex ln 2 − ln 2 − ln 2 (ln 2)2 ¯
1 1 1
µ ¶ µ ¶
ne−n ln 2 e− ln 2 1 1 1 1
= + +− + e−n ln 2 + + e− ln 2
− ln 2 ln 2 (ln 2)2 ln 2 (ln 2)2 ln 2
Z n µ ¶µ ¶
x+1 2 1 1
lim dx = +
n→∞ 1 ex ln 2 ln 2 (ln 2)2 2
P∞ j+1
By integral test, j=1 2j converges.
P∞ j2
Exercise 4.
j=1 2j .
Z n Z n µ ¶¯n
x2 x2 x2 e−x ln 2 2xe−x ln 2 2e−x ln 2 ¯¯
dx = = + +
1 2x 1 e
x ln 2 − ln 2 −(− ln 2)2 (− ln 2)3 ¯1
Z n µ ¶ µ ¶
x2 − ln 2 1 2 2 1 1 2 2
lim dx = e + + = + +
n→∞ 1 2x ln 2 (ln 2)2 (ln 2)3 2 ln 2 (ln 2)2 (ln 2)3
P∞ j2
By integral test, j=1 2j converges.
Exercise 5.

X ∞
X X∞
| sin jx| 1
= aj ≤
j=1
j2 j=1 j=1
j 2

P∞ 1
j=1 j 2 converges since
Z n ¯n µ ¶µ ¶
1 x−s+1 ¯¯ 1 1 1
lim dx = lim = lim − 1 = if s>1
n→∞ 1 xs n→∞ −s + 1 ¯ n→∞ 1 − s n s−1 s − 1
1
P∞ |sinjx|
j=1 converges by comparison test and integral test.
j2
Exercise 6.

X ∞ µ
X ¶
2 + (−1)j 1 3 2(1/4) 3(1/4) 4
= + 2j = + =
j=1
2j j=1
22j−1 2 1 − 1/4 1 − 1/4 3
P∞ j!
Exercise 7.
j=1 (j+2)! .
j! 1 1
aj = = aj = ≤ 2 = bj
(j + 2)! (j + 1)(j + 2) j
P P
Since bj converges, aj converges, by comparison test.
P∞ log j P∞ P∞
Exercise 8.
j=2

j j+1
= j=2 aj ≤ j=2 log j
j 3/2
Z n Z n ¯n Z n 2x−1/2
log x −1/2 0 −1/2 ¯
3/2
dx = (−2x ) log x = (−2x log x)¯ − − dx =
2 x 2 2 2 x
µ ¶ ¯n
log n log 2 ¯
= (−2) 1/2
− 1/2 + −4x−1/2 ¯¯
n 2 2
Z n
log x 4
lim dx = 21/2 log 2 + √
n→∞ 2 x3/2 2
P
So aj converges by comparison test.
P∞ 1
P∞ 1
Exercise 9. j=1
√ = j=1 aj . Let bj = j.
j(j+1)

aj j 1
= lim p = lim p
lim =1
j→∞ bj j→∞ j(j + 1) j→∞ 1 + 1/j
P P
By limit comparison test, since bj diverges, aj diverges.
P∞ √
1+ j P∞
Exercise 10.
j=1 (j+1)3 −1 = j=1 aj

1
bj = j 5/2
µ √ ¶
aj 1+ j 5/2 j 3 + j 5/2 1 + 1/j 1/2
lim = lim j = lim = lim
j→∞ bj j→∞ (j + 1)3 − 1 j→∞ (j + 1)3 − 1 j→∞ (1 + 1/j)3 − 13
j
177
P P
By limit comparison test, since bj converges, aj converges.
P∞ 1
P
Exercise 11. j=2 (log j)s = aj

X
If s ≤ 0, aj diverges since lim aj 6= 0
j→∞
1 1
P∞ 1
P 1
If 0 < s ≤ 1, (log j)s > j s , and since j=2 j s diverges for 0 < s < 1, so does (log j)s

Z Z µ ¶ Z µ ¶0 Z
1 (log x)−s+1 (log x)−s+1 (log x)1−s
(log x)−s = (log x)−s x = x= x−
x (1 − s) (1 − s) 1−s

Thus, if s > 1 has any decimal part, or is an integer, its integral will diverge, so that by integral test, the series diverges.
P∞ |aj |
Exercise 12.
j=1 10j ; |aj | < 10.


X X∞ X∞
|aj | 10 1 1 10
< = = =
j=1
10j j=1
10 j
j=0
10 j 1 − 1/10 9

1
P∞ P∞ 1 1
P∞ 1
Exercise 13.
j=1 1000j+1 < =
P 1 j=1 1000j 1000 j=1 j
The series diverges since
j diverges.

Exercise 14.


X X∞ X∞
j cos2 (jπ/3) j j
j j
= ≤
j ln 2
j=1
2 j=1
2 j=1
e
Z ∞ Z ∞ µ −kx ¶¯∞
x xe e −kx ¯ −k −k
= xe −kx
= − ¯ =e + e
1 ekx 1 −k (−k)2 ¯1 k (−k)2

P∞ 1
Exercise 15.
j=3 j log j(log (log j))s


x))−s+1
Z Z µ −s+1
¶ 0  (ln (ln
 −s+1 if s > 1, s < 1, s 6= 0
1 (ln (ln x))
= = ln (ln (ln x)) if s=1
x log x(log (log x))s −s + 1 

ln (ln x) if s = 0

Converges, by integral test, for s>1


Exercise 16. Converges by integral test since

Z à !¯∞

e−x
2 ¯ e−1
−x2 ¯
xe = ¯ =0+
1 −2 ¯ 2
1

Exercise 17. I drew a picture to help me see what's going on.

√ √
x x
2

1+x 1
Z 1/n ¯1/n µ ¶3/2
√ 2 ¯ 2 1
xdx = x3/2 ¯¯ =
0 3 0 3 n

X2 1 µ ¶ ∞
2X 1
3/2
=
j=2
3 j 3 j=2 j 3/2

P R 1/n √
x
So 1 n 0 1+x2 dx converges by comparison test.
178
Exercise 18.
√ √ −1
(e− x 0
) = e− x

2 x
√ √
√ √
− x 0 −e− 1 x
√ √ √ √ e− x
( xe ) = + √ e− x ( xe− x + e− x )0 =
2 2 x 2
Z n+1 √ ¯ µ√ ¶¯n+1
√ √
¯ n+1 x 1 ¯ ¯
e− x dx = 2 ( x + 1)e− x ¯ =2 √ + √
x x ¯
=
n n e e n
µ√ √ ¶
n+1 1 n 1
=2 √
n+1
+ √n+1 − √n − √n
e e e e
X∞ µ√ √ ¶
j+1 j 1 1 1
2 √ − √j + √j+1 − √j = −
j=1
e j+1 e e e e
Note the use of telescoping sum in the last step. The series converges.
f increasing
Exercise 19. f is nonnegative and increasing. −−−−−−−→ f (k) ≤ f (x) ≤ f (k + 1), k ≤ x ≤ k + 1
Z k+1 Z k+1 Z k+1 Z k+1 P
f (k)dx ≤ f (x)dx ≤ f (k + 1)dx =⇒ f (k) ≤ f (x)dx ≤ f (k + 1) −→
k k k k
P n−1
X X Z k+1
n−1 Z n n−1
X n
X
−→ f (k) ≤ f (x)dx = f (x)dx ≤ f (k + 1) = f (k)
k=1 k=1 k 1 k=1 k=2

Note that for the summation on the integral, we simply relied on the properties of integrals to get the nal integral.

Rn Rn n
1
f (x)dx = 1
log xdx = (x ln x − x)|1 = n ln n − n + 1.
n−1
X Z n n
X n−1
X n
X
ln k ≤ ln x ≤ ln (k) =⇒ lnk ≤ n ln n − n + 1 ≤ ln (k)
k=1 1 k=2 k=1 k=2
Ãn−1 !
X
exp ln k = (n − 1)! ≤ nn e−n+1
k=1
à n
!
X
exp ln k = (n)! ≥ nn e−n+1
k=2
1/n
e (n!)1/n e1/n n1/n
=⇒ < <
e n e
P∞ (j!)2
10.16 Exercises - The root test and the ratio test for series of nonnegative terms. Exercise 1. j=1 (2j)!
µ ¶
((j + 1)!)2 (2j)! (j + 1)2 j 2 + 2j + 1 j→∞ 1
= = 2 −−−→
(2j + 2) (j!)2 (2j + 2)(2j + 1) 4j + 6j + 2 4
Converges by ratio test.
P∞ (j!)2
Exercise 2.
j=1 2j 2 .
2 2
((j + 1)!)2 2j (j + 1)2 2j j 2 + 2j + 1 j→∞
2 = 2 = −−−→ 0
2(j+1) (j!)2 2j +2j+1 2ej ln 2
Converges by ratio test.
P∞ 2j j!
Exercise 3.
j=1 j j
µ ¶j
2j+1 (j + 1)! j j 2(j + 1) 1 j→∞ 2
= −−−→ <1
(j + 1)j+1 2j j! (j + 1) 1 + 1/j e
Converges by ratio test.
P∞ 3j j!
Exercise 4. j=1 j j
µ ¶ µ ¶
3j+1 (j + 1)! jj 1 j→∞ 3
=3 −−−→ >1
(j + 1)j+1 3j j! (1 + 1/j)j e
Diverges by ratio test.
179
P∞ j!
Exercise 5.
j=1 3j .
(j + 1)! 3j j+1
=
3j+1 j! 3
Diverges by ratio test.
P∞ j!
Exercise 6.
j=1 22j
(j + 1)! 22j (j + 1)
2(j+1)
=
2 j! 4
Diverges.
P∞ 1
Exercise 7.
j=2 (log j)1/j

Draw a picture to see what's going on.



X X ∞ µ ¶
1 −1
= exp ln log j
j=2
(log j)1/j j=2
j
ln log j
0< for j>3
j
ln (log j) ln j ln (log j) ln j
< =⇒ lim < lim =0
j j j→∞ j j→∞ j
−1
=⇒ lim ln (log j) = 0 so then
j→∞ j
µ ¶
−1
lim exp ln (log j) = 1
j→∞ j
P∞ 1
j=2 (log j)1/j diverges because the aj term doesn't go to zero.
P∞ 1/j
Exercise 8.
j=1 (j − 1)j
1 j→∞
((j 1/j − 1)j )1/j = (e j ln j − 1) −−−→ 0
Converges by root test.
P∞ 2
Exercise 9.
j=1 e−j
2 j→∞
(e−j )1/j = e−j −−−→ 0
Converges by root test.

Exercise 10. I systematically tried ratio test and then root test. Both were inconclusive.

P1
Consider comparison with
j.
à 2
! 2
ej − j ej − j j j→∞
j= = 1 − j 2 −−−→ 1
jej 2 ej 2 e
P1 P ³1 1
´
By limit comparison test, since
j diverges, so does
j − ej 2
.

P∞ (1000)j
Exercise 11.
j=1 j! = e1000
P∞ j j+1/j
Exercise 12. j=1 (j+1/j)j .
³ ´
1
j 1/j 2 exp j2 ln j
1/j 1/j
aj = 1 lim aj = lim 1 =1
1+ j2
j→∞ j→∞ 1+ j2
Note that root test is inconclusive.

³ ´ ³ ´
exp 1j ln j exp 1j ln j
aj = ³ ´j ≥ ³ ´j 2
1 + 1j 1 + j12
³ ´
exp 1j ln j 1
lim aj ≥ lim ³ ´j 2 = e > 0
j→∞ j→∞
1 + j12
Diverges since limj→∞ aj > 0.
180
P∞ √
j 3 ( 2+(−1)j )j
Exercise 13.
j=1 .
3j
à √ !1/j √ 3 √ √
j 3 ( 2 + (−1)j )j j 3/j ( 2 + (−1)j ) e j ln j ( 2 + (−1)j ) j→∞ ( 2 + (−1)j )
= = −−−→ <1
3j 3 3 3
Converges by root test.
P∞
Exercise 14.
j=1 rj | sin jx|.
If 0 < r < 1.

X ∞
X
rj | sin jx| < rj
j=1 j=1

X
so by comparison test, rj | sin jx| converges for 0 < r < 1
j=1
If r ≥ 1,

X
lim rj | sin jx| 6= 0 so rj | sin jx| diverges, unless jx = πj
j→∞
j=1

Exercise 15.
bj+1 aj+1
(1) cj = bj − aj >0 ∀j ≥ N . Then there must be a positive number r that's in between cj and 0.

aj bj − aj+1 bj+1 ≥ raj


n
X n
X
r aj ≤ (aj bj − aj+1 bj+1 ) = aN bN − an+1 bn+1 ≤ aN bN
j=N j=N
n
X aN bN
=⇒ aj ≤
r
j=N
(2) cn < 0
bj+1 aj+1 X 1
bj − <0 diverges, so
aj bj
bj aj+1 bj
aj bj < bj+1 aj+1 =⇒ < lim ≥ 1 by ratio test
bj+1 aj j→∞ bj+1

j→∞ bj aj+1
−−−→ 1 ≤ <
bj+1 aj
P
So by ratio test, aj diverges.
Exercise 16. bn+1 = n; bn = n − 1.
nan+1
cn = n − 1 − ≥ r =⇒ aan+1
an n
≤1− 1
n − r
n.
P
Using Exercise 15,
P anPconverges.
1 1
bn diverges since bn is a harmonic series of s = 1.

nan+1 1 an+1
n−1− ≤ 0 =⇒ 1 − ≤
an n an
Exercise 17. For some N ≥ 1, s > 1, M > 0, and given that
à !
an+1 A f (n) A − nfs−1
(n)
=1− + s =1−
an n n n
f (n)
Consider A− ns−1 .

|f (n)| < M , f (n) is nite, so consider s larger than 1 and n going to innity so that nfs−1
Since
(n)
→ 0.
P f (n)
Using Exercise 16, for aj to converge, A − ns−1 = 1 + r where r > 0, for all n ≥ N , where N is some positive number.
M
Let r =
N s−1 so that
f (n)
A = 1 + r + s−1 > 1
P n
If A > 1, then an converges.
181
If A = 1, then consider using Exercise 15 and bn = n log n.
µ ¶
an+1 an+1
cn = bn − bn+1 = (n − 1) log (n − 1) − n log n
an an
µ ¶ µ ¶ µ ¶
1 f (n) (n − 1) f (n)
= (n − 1) log (n − 1) − n log n 1 − + s = (n − 1) log − n log n =
n n n ns
µ ¶ µ ¶
n f (n)
= −(n − 1) log − n log n
(n − 1) ns
log n n→∞
since −−−−→ 0,
µ ¶ ns−1µ ¶
n f (n)
−(n − 1) log − n log n < 0 for n large enough
(n − 1) ns
X
since cn < 0 for n ≥ N for some N > 0, then by Exercise 15, an is divergent.

A < 1, then for A − nfs−1


Given that
(n) M f (n) M
, choose N > 0 so that s−1 < ² < 1 and that A − s−1 ≤ A + s−1 = A + ² ≤ 1.
n n n
We can always choose ² small enough because there's always a real number in between A and 1 (Axiom of Archimedes).
µ ¶
f (n) f (n)
A − s−1 ≤ 1 =⇒ − A − s−1 ≥ −1
n n
an+1 A + nfs−1
(n)
1
=⇒ using Exercise 16, =1− ≥1− for all n≥N
an n n
Exercise 18.
µ ¶k µ ¶k
1 · 3 · 5 . . . (2n + 1) 2 · 4 · 6 . . . (2n) 2n + 1 n→∞
· = −−−−→ 1
2 · 4 · 6 . . . (2n + 2) 1 · 3 · 5 . . . (2n − 1) 2n + 2
Ratio test fails.
µ ¶k µ ¶k µ ¶k
an+1 2n + 1 −1 −1/2
= = 1+ = 1+ =
an 2n + 2 2n + 2 n+1
Xk µ ¶µ ¶j µ ¶ X k µ ¶µ ¶j
k −1/2 −1/2 k −1/2
= =1+k +
j=0
j n+1 n+1 j=2
j n+1
k µ ¶µ
X ¶j
k −1/2
Note that for k < ∞, <∞
n+1 j=2
j
¯ ¯
¯ k µ ¶µ ¶j ¯
¯X k −1/2 ¯
Let ¯ ¯≤M
¯ n + 1 ¯¯
¯ j=2 j
X X
k/2 = A > 1 or k > 2 means aj converges k/2 = A ≤ 1 or k ≤ 2 means aj diverges

10.20 Exercises - Alternating series, Conditional and absolute convergence, The convergence tests of Drichlet and
Abel. We will be using Leibniz's test alot, initially.

Theorem 32 (Leibniz's Rule). If aj is a monotonically decreasing sequence with limit 0,


P∞ j−1
j=1 (−1) aj converges.

P∞ Pn j−1
If S= j=1 aj , sn = j=1 (−1) aj ,
0 < (−1)j (S − sj ) < aj+1
P∞(−1)j+1 1
Exercise 1. √
j=1 . limj→∞ √ = 0 Converges conditionally.
j j
P∞ j

j

j
Exercise 2.
j=1 (−1) j+100 limj→∞ j+100 = 0. Converges by Leibniz's test.
√ 1 100 ≥ 1√
, so by comparison test, the series diverges absolutely. So the alternating series converges conditionally
j+ √j 101 j
by comparison test.
P∞ (−1)j−1 1
Exercise 3.
j=1 If s > 1, then the series absolutely converges. limj→∞ = 0 if s > 0. Converges conditionally
js js
for 0<s< 1. Otherwise, if s < 0 the series diverges absolutely.
182
P∞ ³ ´3
j 1·3·5...(2j−1)
Exercise 4.
j=1 (−1) 2·4·6...(2j) .

aj+1 1 · 3 · 5 · (2j + 1) 2 · 4 · 6 . . . (2j) 2j + 1


= =
aj 2 · 4 · 6 . . . (2j + 2) 1 · 3 · 5 . . . (2j − 1) 2j + 2
Absolutely converges.
P∞ (−1)j(j−1)/2 (−1)j(j−1)/2 P∞ 1 1/2
Exercise 5.
j=1 2j converges since limj→∞ 2j = 0; j=1 2j = 1−1/2 = 1 . Absolutely converges.

P∞ ³ ´j
j 2j+100
Exercise 6.
j=1 (−1) 3j+1 .
µ µ ¶¶ µ µ ¶¶ µ µ µ ¶ µ ¶¶¶
2j + 100 2 298 2 1 298
exp j ln = exp j ln + ≤ exp j ln + 2 =
3j + 1 3 9j + 3 3 3
9j + 3
µ ¶
2 146
= exp j ln +
3 3 + 1/j
µ µ ¶¶
2j + 100
0 ≤ lim exp j ln ≤ lim
j→∞ 3j + 1 j→∞
µ ¶
2 146
exp j ln + =0
3 3 + 1/j
µ µ ¶¶
2j + 100
=⇒ lim exp j ln =0
j→∞ 3j + 1
So the alternating series converges.
µ ¶
2j + 100 2j + 100 2.5j 5
< < = ( for j ≥ 200)
3j + 1 3j 3j 6
µ ¶j µ ¶j
2j + 100 5
=⇒ < for j ≥ 200
3j + 1 6
So the series absolutely converges by comparison with a geometric series.
P∞ (−1)j
Exercise 7. √ .
j=2 j+(−1)j
1
lim √ doesn't exist since ???
j→∞ j + (−1)j
To show divergence, we usually think of either taking the general term and nding the limit (and if it goes to a nonzero
constant, then it diverges), or we use ratio, root, comparison test on the general term. Since this is an alternating series, I've
observed that the general term is a sum of two adjacent terms, one even and one odd.

(−1)j

j + (−1)j
√ √
(−1)2j (−1)2j+1 1 −1 2j + 1 − 1 − ( 2j + 1)
√ + √ = √ + √ = √ √ =
2j + (−1)2j 2j + 1 + (−1)2j+1 2j + 1 2j + 1 + 1 ( 2j + 1)( 2j + 1 − 1)
√ √ √ q 1 √
2j + 1 − 2j − 2 2j 1 + 2j − 2j − 2
= √ √ = √ √ q =
( 2j + 1)( 2j + 1 − 1) ( 2j + 1)( 2j 1 + 2j 1
− 1)
√ ³ 1
´ √ Ã !
for j large
2j 1 + 4j − 2j − 2 −2 1 − 4√12j
−−−−−−→ ' √ √ ³ 1
´ =
j 2 − 2j1
+ 2√2j 1
( 2j + 1)( 2j 1 + 4j − 1) 3/2

Every term, since we considered any j , will contain −2. So we factor it out. Then
à ! à 1 !
1 1 − 4√12j 1 1 − 4√2j 1
1 1 > 1 =
j 2 − 2j + 2 2jj 3/2
√ j 4 − 2j√ 4j
1
By comparison test to
j the series diverges.
Exercise 8. Using the theorem

Theorem 33.
P
Assume |aj | converges
P P P
Then aj converges and | aj | ≤ |aj |.
183
So using the contrapositive,
P
If
Paj diverges,
|aj | diverges.

1 1 1 1
= 1 lim = ³ ´ =1
j 1/j e j ln j j→∞ j 1/j exp limj→∞ 1
ln j
j
Diverges absolutely.
P∞ j j
2
Exercise 9. j=1 (−1) 1+j 2 Diverges absolutely.
µ ¶
(2j)2 (2j − 1)2 4j 2 4j 2 − 4j + 2 (4j 2 − 4j + 1) (1 + 4j 2 )
2
− 2
= −
1 + (2j) 1 + (2j − 1) 1 + 4j 2 2
4j − 4j + 2 (4j 2 − 4j + 2) 1 + 4j 2
4j − 1
=
2(1 + 4j )(2j 2 − 2j + 1)
2

4j − 1 4 − 1/j 1
(j 3 ) = =
2(1 + 4j 2 )(2j 2 − 2j + 1) 2(4 + 1/j 2 )(2 − 2/j + 1/j 2 ) 4
P 1 P∞ j 2
By limit comparison test, with
j3 , j=1 1+j 2 converges.
P∞ (−1) n
Exercise 10.
n=1 log (en +e−n )
1 1
lim = lim =0
n→∞ log (en −n
+e ) n→∞ n
The series converges.

n n 1
lim = lim = lim ³ ´ =1
n→∞ log (en + e−n ) n→∞ log en + log (1 + e−2n ) n→∞ n+log (1+e−2n )
n
P 1
P 1
Since
n diverges, log (en +e−n ) diverges.
P∞ (−1)j
Exercise 11.
j=1 j log2 (j+1)
limj→∞ j log21(j+1) = 0 so by Leibniz's test, the alternating series.

1 1
<
n log2 (n + 1) n log2 (n)
Z Z µ ¶0
1 −1 −1 n→∞ 1
2 = = −−−−→
n log n log n log n log 2
1
Converges by comparison test to , which converges by integral test. So the series absolutely converges.
n log2 n
P∞ (−1)j
Exercise 12.
j=1 log (1+1/j) diverges absolutely.
³ ´ ³ ´
2j 2j+1
(−1) 1 −1 1 + log − log
2j−1 2j
³ ´+ ³ ´= ³ ´+ ³ ´= ³ ´ ³ ´ =
1 1 2j 2j+1 2j 2j+1
log 1 + 2j−1 log 1 + 2j log 2j−1 log 2j log 2j−1 log 2j
³ ´ ³ ´
(2j)2
log 4j 2 −1 log 1 + 4j 21−1
= ³ ´³ ³ ´´ = ³ ´ ³ ´=
2j 1 1 1
log 2j−1 log 1 + 2j log 1 + 2j−1 log 1 + 2j
³ ´
1 1
4j 2 −1 + o 4j 2 −1
=³ ³ ´´ ³ ³ ´´ ≈
1 1 1 1
2j−1 + o 2j−1 2j + o 2j
1
4j 2 − 2j 1 − 2j j→∞
≈ = −−−→ 1
4j 2 − 1 1 − 4j12
So the alternating series diverges.
P∞ (−1)j j 37
Exercise 13.
j=1 (j+1)! Use the ratio test.
µ ¶µ ¶37
aj+1 (j + 1)3 7 (j + 1)! 1 1
= = 1+ →0
aj (j + 2)! j 3 7 j+2 j
184
P P
Converges for |aj |. Then aj converges. The series absolutely converges.
P∞ R
n n+1 e−x
Exercise 14.
n=1 (−1) n x dx
Z n+1 Z n+1 ¯n+1 µ ¶µ ¶
e−x 1 e−2x ¯¯ −1 1 1
dx ≤ dx = = − 2n =
n x n e2x −2 ¯n 2 e2(n+1) e
e2 − 1
= <1
2e2n+2
Converges absolutely.
Pn
Exercise 15. sin (log j)
j=1
limj→∞ sin (log j) doesn't exist. So the series is divergent.
P∞ ³ ´
1
Exercise 16.
j=1 log j sin j Note that
µ ¶ µ ¶
1 sin 1/j
log j sin = log
j 1/j
µ ¶ µ ¶
sin 1/j sin 1/j
lim log = log lim = log 1 = 0
j→∞ 1/j j→∞ 1/j

³ ´2k+1
∞ 1
1 j X
= sin (−1)k
j (2k + 1)!
k=0
 ³ ´2k+1   ³ ´2k 
1 k 1 k


X j (−1)   −1 X∞
j (−1) 
log j  = log 1 + 2 + ≥
(2k + 1)! 6j (2k + 1)!
k=0 k=2
µ ¶
1 −1
≥ log 1 + − 2 ≥ 2
6j 6j
The series absolutely converges.
P∞ ³ ´
j 1
Exercise 17. j=1 (−1) 1 − j sin j
µ ¶0 µ ¶
1 1 1 −1
1 − x sin = − sin − x cos =
x x x x2
1 1 1 −x sin x1 + cos x1
= − sin + cos =
x x x x
∞ ¡ 1 ¢2j+1
1 X x (−1)j
sin =
x j=0 (2j + 1)!
−x sin x1 + cos x1
=
x
( x1 )
3
(+1) P∞ ( x1 )2j+1 (−1)j ¡ 1 ¢2 P∞ ( x1 )2j (−1)j
−1 + 3! + j=2 (2j+1)! +1− x /2 + j=2 (2j)!
= <0 for x large enough
x
P∞ ³ ´
j 1 1
j=1 (−1) 1 − j sin j converges since aj = 1 − j sin j is monotonically decreasing sequence with limit 0.
³ ´2k+1  ³ ´2k+1 
1 1
1

X j (−1)k 1

X j (−1)k 
1 − j sin =1−j =1−j + =
j (2k + 1)! j (2k + 1)!
k=0 k=1
 ³ ´2k  ³ ´2k
1 1
 X ∞
j (−1)k  X∞
j (−1)k+1 1
= 1 − 1 + = ≤
(2k + 1)! (2k + 1)! 6j 2
k=1 k=1

1
The series converges absolutely since the term itself is a series that is dominated by
6j 2 , so that by comparison test, the series
must converge.
185
P∞ ³ ´
Exercise 18. j=1 (−1)
j
1 − cos 1j .

µ µ ¶¶
1 1 −1 −1 1
(cos )0 = − sin = 2 sin < 0
x x x2 x x
P∞
j=1 (−1)
j
(1 − cos 1j ) converges since aj = (1 − cos 1j ) is monotonically decreasing to 0


X (1/j)2k (−1)k X (1/j)2k (−1)k+1 ∞
1 1
(1 − cos ) = 1 − = ≤ 2
j (2k)! (2k)! 2j
k=0 k=1

P 1
So the series converges absolutely, by comparison test with
j 2 which converges.
P∞ j 1
Exercise 19.
j=1 (−1) arctan 2j+1 .

µ ¶ µ ¶
1 1 −1 −2
(arctan )0 = ³ ´2 (2) = <0
2j + 1 1 (2j + 1)2 (2j + 1)2 + 1
1+ 2j+1

P∞ j 1 1
j=1 (−1) arctan 2j+1 converges, since aj = arctan 2j+1 is monotonically decreasing to 0

X X ∞ ∞
1
2
= (arctan x)0 = (−x2 )j = (−1)j x2j
1+x j=0 j=0

X x2j+1
=⇒ arctan x = (−1)j
j=0
2j + 1
³ ´2k+1 Ã ! ³ ´2k+1
∞ (−1)k 1 1 ∞ (−1)k 1
1 X 2j+1 1 (2j+1)3
X 2j+1
arctan = = + (−1) + >
2j + 1 (2k + 1) 2j + 1 3 2k + 1
k=0 k=2
1 1 3(4j 2 + 4j + 1) + (−1) 12j 2 + 12j + 2 2j 21
> + (−1) 3
= 3
= > > for j>2
2j + 1 3(2j + 1) 3(2j + 1) 3(2j + 1)3 (2j + 1)2 9j
P1 P 1
So by comparison test to arctan 2j+1
j, diverges absolutely. The series is conditionally convergent.

P∞ ¡
j π
¢
Exercise 20.
j=1 (−1) 2 − arctan log j

³π ´0 µ −1
¶µ ¶
1
− arctan log n =
2 1 + (log n)2 n

π π π
− arctan log n ≥ − arctan (n − 1) → − arctan n just change indices
2 2µ 2 ¶¯n
Z n³ ´
π π 1 ¯
− arctan x dx = x − x arctan x − ln (1 + x2 ) ¯¯
0 2 2 2 0
µ ¶ ³π ´ 1
π 1¡ ¡ ¢¢
= n − n arctan n − ln 1 + n2 =n − arctan n + ln (1 + n2 )
2 2 2 2
µ ³ ´ ¶
π 1
lim n − arctan n + ln (1 + n2 ) → ∞
n→∞ 2 2

Then by the integral test,
2 − arctan log n diverges absolutely. So the alternating series is conditionally convergent.

P∞ ³ ´
1
Exercise 21.
j=1 log 1 + | sin j|
³ ´ ³ ´
1 1
limj→∞ log 1 + | sin j| doesn't exist and log 1 + | sin j| >0 ∀j so the series diverges.

186
P∞ ³ ´
1
Exercise 22. j=2 sin jπ + log j
µ ¶ µ ¶
1 1
sin 2jπ + + sin (2j + 1)π + =
log 2j log (2j + 1)
µ ¶ µ ¶
1 1 1 1
= sin (2jπ) cos + sin cos (2πj) + sin (2j + 1)π cos + cos (2j + 1)π sin =
log 2j log 2j log (2j + 1) log 2j + 1
1 1
= sin − sin =
log 2j log 2j + 1
³ ´2k+1 ³ ´2k+1
1 1
X∞
log 2j (−1)k X ∞
log (2j+1) (−1)k
= −
(2k + 1)! (2k + 1)!
k=0 k=0
µ ¶ µ ¶
1 1
sin − sin =
log 2j log 2j + 1

õ ¶2k+1 µ ¶2k+1 !
X (−1)k 1 1
= −
(2k + 1)! log 2j 2j + 1
k=0
µ ¶2k+1 µ ¶2k+1
1 1
0 < log 2j < log 2j + 1 so − >0
log 2j log 2j + 1
µ ¶ µ ¶
1 1
and since for j > 1, and are < 1 and so we are adding smaller and smaller amounts
log 2j log 2j + 1
1 log 2j + 1
< −
log 2j =
³ ´
1
log (2j + 1) − log 2j log 1 + 2j 1
2j
= ≤ ≤
log 2j log 2j + 1 (log (2j))2 (log (2j))2
Z n ¯n
1 1 ¯¯ n→∞ 1
2
=− ¯ −−−−→
1 2j(log 2j) log 2j 1 log 2
P 1
So the series converges by using integral test, showing that
2j(log 2j)2 converges, so by comparison test, the series con-
verges.
P
Exercise 33.
n=1 nn z n
¯ ¯ ¯¯ ¯
¯
¯X ¯ ¯X ∞
¯ X ¡ ln j ¢j
¯ ¯
¯ nn z n ¯ = ¯¯ (jz)j ¯¯ = e z
¯ ¯ ¯ ¯
n=1 j=1
n
X
(eln j z)j =
j=1
ln j
e z − (eln j z)n
= −→ ∞
1 − eln j z
So z=0
P∞ (−1)j z 3j P∞ (−z 3 )j 3
Exercise 34.
j=1 j! = j=1 j! = e−z . C .
P∞z
P∞ ¡ 1 ¢j j
j
Exercise 35.
j=0 j = z
P j 3 Pn j=0 j 3
z be convergent or j=1 z bounded.
|z| < 3 and |z| = 3 if z 6= 3
P∞ zj
Exercise 36. j=1 j j {z} = C since

¯µ ¶¯
¯ z ¯
¯ ¯
¯ j ¯ < 1 for j ≥ N > |z|
P∞ (−1)j
Exercise 37.
j=1 z+j
1 j→∞
By Leibniz's Rule, since
z+j −−−→ 0, then the series converges. However, z cannot be equal to any negative integer since
one term in the series will then blow up.
187
P∞ ³ ´
zj 2j+1
Exercise 38. j=1

j
log j .
³ ´
µ ¶ log 2 + 1j
zj 1
√ log 2 + = zj √
j j j
log (2+ 1j ) log (2+ 1j )
Since limj→∞ √
j
= 0 so that √
j
is a monotonically convergent sequence.
Pn j
Then by Dirchlet's test, j=1 z must be bounded. |z| > 1, and |z| = 1 if z 6= 1.
P∞ ³ 1
´j 2
17j
P∞ ³ 1
´j 2
Exercise 39.
j=1 1 + 5j+1 |z| = j=1 1 + 5j+1 (|z|17 )j
õ ¶j !j
1 17
1+ |z|
5j + 1
µ ¶j µ ¶j
1 1/5
lim 1 + ≤ lim 1 + = e1/5
j→∞ 5j + 1 j→∞ j
e1/5 |z|17 < 1 =⇒ |z| < e−1/85

P∞ (z−1)j
Exercise 40.
j=0 (j+2)!
¯ ¯
¯∞ ¯ ∞ ∞
¯X (z − 1)j ¯ X |(z − 1)|j X |(z − 1)|j+2 |z − 1|
¯ ¯≤ ≤ = e|z−1| − 1 −
¯ ¯
¯ j=0 (j + 2)! ¯ j=0 (j + 2)! j=0
(j + 2)! 1!

The series converges ∀ z.


Exercise 41.

X ∞
X
(−1)j (z − 1)j (1 − z)j
= = log (1 − (1 − z)) = log z
j=1
j j=1
j

So the series converges ∀ z except for z = 0.


P∞ (2z+3)j
Exercise 42.
j=1 j log (j+1)

1 1
lim = 0 so is a monotonically convergent sequence
j→∞ j log (j + 1) j log (j + 1)
X |2z + 3| < 1 X
j
(2z + 3) 3 1 then (2z + 3)j converges
|z + | <
2 2
µ ¶0 µ ¶
1 −1 x
= log (x + 1) + < 0; for x > 0
x log (x + 1) (x log (x + 1))2 x+1

P∞ (2z+3)j 3 1
By Dirichlet's test, j=1 j log (j+1) converges for |z + | ≤ ; z 6= −1 .
2 2
P∞ ³ ´j P∞ ³ ´j
(−1)j 1−z (−1)j /2 1−z
Exercise 43.
j=1 (2j−1) 1+z = j=1 j− 12 1+z
P∞ ( z−1
z+1 )
j
1 1
=⇒ j=1 2j−1 = limj→∞ 2j−1 = 0 and 2j−1 is monotonically decreasing.
1
So is a monotonically decreasing convergent sequence of real terms.
2j−1¯ ¯
¯ z−1 ¯
For ¯
z+1 ¯ < 1,
¯ ¯
¯z − 1¯
¯ ¯
¯ z + 1 ¯ < 1 =⇒ |z − 1| < |z + 1|
(u − 1)2 + v 2 < (u + 1)2 + v 2
u2 − 2u + 1 < u2 + 2u + 1 <(z) > 0; z 6= 0
−u < u
188
Exercise 44.
∞ µ
X ¶j X ∞ µ ¶j X ∞ µ ¶j µ ¶j
z 1 −1/2 1 1
= + = 1−
j=1
2z + 1 j=1
2 2z + 1 j=1
2 2z + 1
µ ¶j
1
is a monotonically decreasing, convergent sequence
2
¯ ¯ ¯ ¯ ¯ ¯
¯ 1 ¯¯ ¯¯ 2z + 1 − 1 ¯¯ ¯¯ 2z ¯¯
¯
Now we want 1 − = = <1
¯ 2z + 1 ¯ ¯ 2z+ ¯ ¯ 2z + 1 ¯
|2z| < |2z + 1| −1
if <(z) =
¡ ¢ 4
|z| < |z + 1/2|
2 − 14 + iv −1
2 + 2iv
2 2 2 2 2
u + v < (u + 1/2) + v = u + u + 1/4 + v 2 ¡ −1
¢ = 1
2 4 + iv + 1 2 + 2iv
−1 1 −1
< u =⇒ <(z) > − =⇒ z 6=
4 4 4
P∞ ³ ´j
j z
Exercise 45.
j=1 j+1 .
2z+1
µ ¶0
x (x + 1) − x 1
= = >0
x+1 (x + 1)2 (x + 1)2
j
is a monotonically increasing and convergent sequence
j+1
¯ ¯
1 −1 ¯ z ¯
+ = ¯ ¯<1
2 2(2z + 1) ¯ 2z + 1 ¯
¯ ¯
¯ 2z + 1 ¯ 1
¯ ¯
¯ z ¯ > 1 =⇒ |2 + z | > 1

P∞ 1
P∞ ³ 1
´j
Exercise 46.
j=1 (1+|z|2 )j = j=1 1+|z|2

1
<1
1 + |z|2
1 < 1 + |z|2 ∀z except z=0

0 < |z|2
P∞ j 2j sin2j x
Exercise 47.
j=1 (−1) j
Use Dirichlet's Test.
1
j is a monotonically decreasing sequence converging to zero. Consider (−2)j sin2j x. The condition
for convergence is

|(−2 sin2 x)| < 1 for j ≥ N for some N


−π π
=⇒ x ∈ ( + nπ, + nπ), n∈Z
4 4
P
Exercise 49.
P aj converges.
aj a1j diverges.
P 1
Then since aj is a convergent series (by Abel's test),
aj is a divergent sequence.
P 1 1
Then
aj is divergent (since limj→∞ aj doesn't exist ).
P
Exercise 50.
P |aj | converges.
P
|aj | converges, then aj converges.
|aj |2 = a2j . |aj | converges, then
lim |aj | = 0
j→∞ |aj |2
lim =1
2
lim |aj | = 0 j→∞ a2j
j→∞
P 2
By limit comparison test, aj converges.
P ³ ´2 1
P1
Counterexample: converges, but
j j diverges.
189
P P
Exercise 51. Given aj , aj ≥ 0. aj converges.

lim aj = 0
X√ j→∞
1 µ ¶1/2
aj p √
(j) lim aj = lim aj =0
j→∞ j→∞
Z n−1
X n−1
X X xj n Z
j xj+1 1 − xn
x = = =
j=0 j=0
j + 1 j=1 j 1−x

aj
q
aj
A counterexample would be
jp = j .

Exercise 52.
P P P
(1) aj converges absolutely, then if |aj | converges, a2j converges.

a2j −1
=1+
1 + a2j 1 + a2j
a2j X X a2j
≤ a2j since a2j converges , converges
1 + a2j 1 + a2j
P
(2) aj converges absolutely, lim
´ j→∞ |aj | = 0
P ¯¯ aj ¯¯ P ³
1
¯ 1+aj ¯ = |aj | |1+aj | .
By Abel's test, since

1 1 1
lim = =1 shows that ≥ 0 is a monotonically convergent sequence
j→∞ |1 + aj | |1 + limj→∞ aj | |1 + aj |
P aj
By Abel's test,
1+aj is convergent.

10.22 Miscellaneous review exercises - Rearrangements of series. Exercise 1.

(1)
r µ µ ¶ µ ¶ ¶
p p p 1 p p 1 1 1
aj = j+1− j = j 1+ − j = j 1+ +o + −1 =
j 2 j j
µ µ ¶ µ ¶¶
p 1 1 1
= j +o
2 j j
lim aj = 0
j→∞

(2)
µµ ¶c ¶ µ µ ¶ µ ¶ ¶
1 1 1
aj = (j + 1)c − j c = j c 1+ − 1 = (j c ) 1 + c +o −1 =
j j j
µ µ ¶ µ ¶¶ µ µ ¶¶
1 1 1
= jc c +o = cj c−1 + j c o
j j j
if c > 1, aj diverges

if c = 1, lim aj = 1
j→∞
if c < 1, lim aj = 0
j→∞

Exercise 2.

(1)
 
µ ¶ X∞ n j j−1
1 1 1 (x ) (−1) n→∞
(1 + xn ) n = exp ln (1 + xn ) = exp  −−−−→ 1
n n j=1 j
³ ¡ ¢n ´1/n
(2) limn→∞ (an + bn )1/n = limn→∞ a 1 + nb = a if a > b.
an +an−1 an−1 +an−2 an−2 +an−3
Exercise 3. an+1 = 2 = 22 + 22
190
10.24 Exercises - Improper integrals.
R∞
Exercise 1. √ x dx
0 x4 +1
µ ¶µ ¶
x 1 x2
lim √ = lim p =1
x→∞ x4 + 1 1/x x→∞ x2 1 + 1/x4
R∞ 1
R∞
Since √ x dx
1 x diverges, so does 0 x4 +1
Exercise 2.
Z ∞ Z ∞ Z −∞ Z ∞ Z 0 Z ∞
2 2 2 2 2 2
e−x dx = e−x dx + e−x dx = e−x dx + − e−x dx = 2 e−x dx
−∞ 0 0 0 ∞ 0
Z ∞ Z ∞ ¯∞
−x2 −x −x2 ¯
e dx ≤ e dx = −e ¯ = 1
0 0 0

Converges by theorem.
R∞
Exercise 3. √ 1 dx
0 3
R ∞ 1x +1
Exercise 4. √ dx
0 ex
R ∞ e−√x
Exercise 5. √ dx
0+ x
R 1 log x
Exercise 6. √ dx
0+ x
R 1− log x
Exercise 7. + dx
R0∞ 1−x x
Exercise 8.
−∞ cosh x
dx
R 1− dx
Exercise 9. √
0+ x log x
R ∞ dx
Exercise 10.
x(log x)s

11.7 Exercises - Pointwise convergence of sequences of functions, Uniform convergence of sequences of functions,
Uniform convergence of sequences of functions, Uniform convergence and continuity, Uniform convergence and inte-
P∞ zj
gration, A sufcient condition for uniform convergence, Power series. Circle of convergence. Exercise 1. j=0 2j =
P∞ ¡ z ¢j
j=0 2

Using the comparison test,


¯ z ¯j ¯z¯
¯ ¯ ≤ tj ; ¯ ¯ < 1 =⇒ |z| < 2
2 2
P∞ ¡ z ¢j P∞ 2iθ j
Suppose |z| = 2, z 6= 2 j=0 2 = j=0 (e )
Pn 2iθ j 1
Now j=0 (e ) ≤ sin θ + 1, since
Xn
e2iθ − e2iθ(n+1) einθ − e−iθn eiθn+iθ sin nθeiθn+iθ 1
(e2iθ )j = 2iθ
= −iθ iθ
= <
j=1
1−e −e +e sin θ sin θ
P∞ ¡ z ¢j
So j=0 2 converges for |z| = 24, z 6= 2
P∞ z j
Exercise 2.
j=0 (j+1)2j

Use ratio test .


aj+1 z j+1 (j + 1)2j z (j + 1) z (1 + 1/j) j→∞ z
= j+1 j
= = −−−→
aj (j + 2)2 z 2 (j + 2) 2 (1 + 2/j) 2
P∞ P
If |z| < 2, a
j=0 j converges, if |z| > 2 , a j diverges.
P zj P 2iθ j ³ 1 ´
If |z| = 2, = (e )
(j+1)2j j+1
1
Now is a monotonically decreasing sequence of real terms.
Pj+12iθ j
(e ) is a bounded series.
P
By Dirichlet's test, aj converges if |z| = 2, z 6= 2
P∞ (z+3)j
Exercise 3.
j=0 (j+1)2j
Use ratio test:
µ ¶ µ ¶ µ ¶
aj+1 (z + 3)j+1 (j + 1)2j (z + 3) j+1 (z + 3) 1 + 1/j j→∞ z+3
= = = −−−→
aj (j + 2)2j+1 (z + 3)j 2 j+2 2 1 + 2/j 2
191
Using Theorem 11.7,

Theorem 34 (Existence of a circle of convergence).

X converges for at least z1 6= 0


Assume aj z j
diverges for at least one z2 6= 0
X absolutely converges for |z| < r
∃ r > 0, such that aj z j
diverges for |z| > r
|z+3|
P We can plug in real numbers to satisfy the condition 2 < 1 for convergence.
aj converges for |z + 3| < 2; diverges for |z + 3| >³ 2. ´
P P 1 1
Consider |z + 3| = 2; z 6= 1 aj = (e2iθ )j j+1 . Since
j+1 is a monotonically decreasing sequence of real
P 2iθ j
numbers and
P (e ) is a bounded series, by Dirichlet's test,
aj converges for |z + 3| = 2; z 6= −1.
P∞ (−1)j 22j z2j P∞ j−1 2j
Exercise 4.
j=1 2j = − j=1 (−1) (2j)(2z) . Look at what the terms look like.
Consider using Leibniz's Rule, Theorem 10.14.

Theorem 35 (Leibniz's rule). If


P∞ aj is a monotonic decreasing sequence and limj→∞ aj = 0,
j−1
then j=1 (−1) aj converges.

(−1)j 22j z 2j (−1)j (2z)2j


=
2j 2j
¯ ¯
¯ (2z)2j ¯ |2z|2j (2|z|)2j
Consider ¯ ¯
¯ 2j ¯ = 2j = 2j
Consider 2|z| = M 1/2 < ∞
(2|z|)2j Mj ej ln M
=⇒ = =
2j 2j 2j
1
converges for 0 < 2|z| = M ≤ 1 =⇒ |z| ≤
2
(we use Theorem 11.6 at this point, because real numbers are included in complex numbers).
P
Theorem 36. Assume aj z j converges for some z = z1 6= 0.
P j
(1)
P aj z j converges absolutely ∀z with |z| < |z1 |.
(2) aj z converges uniformly on every circular disk with center at 0 and R < |z1 |
We had rst used Leibniz's test to nd az1 on the real line.

24j z 4j (4j(1 − 4z12 ) − 2) 24j z 4j ((1 − 4z12 ) − 2


4j ) j→∞ 24j z 4j (1 − 4z12 )
= −−−→
4j(4j − 2) (4j − 2) (4j − 2)
1
If |z| > , the series diverges (by aj th general term test).
2
P∞ j j
Exercise 5.
j=1 (1 − (−2) )z .
Try limit comparison test .
The rst step is to test absolute convergence rst; it's easier.
¯ ¯µ ¯ ¯
¯ 1 ¶j
¯(1 − (−2)j )z j ¯ ¯
¯ ¯ j→∞
= ¯ − 1 ¯ −−−→ 1
j
|(−2z) | ¯ −2 ¯
P P
According to limit comparison test, for
P∞ (1 − (−2)j )z j to converge, (−2z)j must converge.
1 j j
So if |z| < , then j=1 (1 − (−2) )z absolutely converges.
2
1 −1
If |z| = , z 6= 2 ,
2 X X X
(1 − (−2)j )z j = zj − (−e2iθ )j =
X 1
=0− (e2iθ+πi )j <
sin (θ + π)
−1
If z =
2 ,
X µ 1 ¶j X
− − 1j → ∞
2
192
P∞j!z j
Exercise 6.
j=1 j j
A very big hint is to use Exercise 19 on pp. 399, in the section for Exercises 10.14.
Pn−1 Rn
Since j=1 f (j) ≤ 1 f (x)dx

n−1
X Z n n
X
ln j ≤ ln x = n ln n − n + 1 ≤ ln j
j=1 1 j=2

(n − 1)! ≤ nn e−n n ≤ n!
n!
≥ ne−n
nn
n!
lim n ≥ lim ne−n = 0
n→∞ n n→∞
n! n2
lim n ≤ lim n = 0
n→∞ n n→∞ e

n!
=⇒ lim n = 0
n→∞ n

n!
P
So then since
nPn is a monotonically decreasing convergent sequence of real terms; if z is a bounded series, then by
n!z n
Dirichlet's test,
nn is convergent.
|z| < 1; |z| = 1 if z 6= 1
Try the ratio test, because it's clear from the results of the ratio test where convergence and divergence begins and ends.
µ ¶n
(n + 1)!|z|n+1 nn n
= |z| =
(n + 1)n+1 n!|z|n n+1
µ ¶n
1 n→∞ 1
= |z| −−−−→ |z|
1 + 1/n e

Converges for |z| < e .

Now try plugging in a complexied e:


ej j!ej ei2θj j j e−j jej ei2θj i2θj
³ j´ = j
≥ e →∞
j (j) jj
j!

So the series diverges for |z| = e.


P∞ (−1)j (z+1)j
Exercise 7. j=0 j 2 +1

By the ratio test,

|z + 1|j+1 j 2 + 1 1 + 1/j 2 j→∞


= |z + 1| −−−→ |z + 1|
j 2 + 2j + 2 |z + 1|j 1 + 2/j + 2/j 2
The series absolutely converges for |z + 1| < 1.
P (−1)j 1 j→∞
For z = 0, j 2 +1 converges since j 2 +1 −−−→ 0

P (−1)j (−1)j P 1 1 1
P 1
For z = −2, j 2 +1 = j 2 +1 and j 2 +1 < j 2 , but j 2 is convergent (by integral test). So the series converges
for z = −2.
1
By Dirichlet's test, the series converges as well, if we treat aj = (−1)j (z + 1)j and bj = j 2 +1 to be the monotonically
decreasing sequence.

=⇒ |z + 1| ≤ 1 for convergence x
P∞ j 2 j
Exercise 8.
j=0 a z , 0 < a < 1

Use the root test .


2 j→∞
(aj z j )1/j = aj z −−−→ 0
So the series converges ∀z ∈ C
P∞ (j!) j2
Exercise 9.
j=1 (2j)! z
193
Use the ratio test

aj+1 ((j + 1)!)2 z j+1 (2j)! (j + 1)2 z j→∞ 1(z)


= 2 j
= −−−→
aj (2(j + 1))! (j!) z (2j + 2)(2j + 1) 4
X
for |z| < 4, aj absolutely converges
X
for |z| > 4, aj diverges
Let's test the boundary for convergence.

(j!)2 j (j!)2 j ln 4 (j!)2 ej ln 4 (j!)2 ej ln 4


4 = e = ≥ ≥
(2j)! (2j)! (2j − 2)! (2j)(2j − 1) (2j − 2)! (2j)2
(j!)2 j 2j j 2j
≥ 2
= → ∞
(2j − 2)! (j!) (2j − 2)!
where we had used
n!
(n − 1)! ≤ nn e−n n ≤ n! =⇒ ne−n ≤
nn
n n! nn en
≤ n =⇒ ≤
en n n! n
P∞ 3

j j
z
P∞ e

j ln 3 j
z
Exercise 10.
j=1 j = j=1 j

j+1 ln 3 j+1
µ ¶ √
z e j 1 √
√ = e( j+1− j) ln 3 z
j+1 e j ln 3 z j j+1
r µ ¶
p p 1 1 1 1
j+1− j = 1+ −1'1+ −1= (for large j )
j 2 j 2j
µ ¶
j 1
(for large j ) e 2j z → 0
j+1
=⇒ Converges ∀ z ∈ C
P∞ ³ 1·3·5...(2j−1) ´3 j
Exercise 11.
j=1 2·4·6...(2j) z
µ ¶3 µ ¶3 µ ¶3 µ ¶3
aj+1 1 · 3 · 5 . . . (2j + 1) j+1 2 · 4 · 6 . . . (2j) 1 2j + 1 −1/2
= z = z = 1+ z
aj 2 · 4 · 6 . . . (2j + 2) 1 · 3 · 5 . . . (2j − 1) zj 2j + 2 j+2
If |z| < 1, it converges by ratio test, if |z| = 1, then it converges by Gauss test
µ ¶3 X 3¶ µ −1/2 ¶k
3 µ
aj+1 −1/2
= 1+ z= |z| =
aj j+2 k j+2
k=0
à µ ¶ µ ¶2 µ ¶3 !
−1/2 −1/2 −1/2 j→∞
= |z| 1 + 3 +3 + −−−→ |z|
j+2 j+2 j+2

diverges for |z| > 1 (by ratio test )

P∞ ³ 1
´j 2
Exercise 12.
j=1 1+ j zj
õ ¶j 2 !1/j µ ¶j
1 j 1 j→∞
1+ z = 1+ z −−−→ e1 z
j j
1 X 1 2
|z| < , (1 + )j z j converges by root test
e j
1 X 1 j2 j
|z| > , (1 + ) z diverges by root test
e j
1
=r
e
P∞ j
Exercise 13.
j=0 (sin aj)z a>0
j j
| sin (aj)z | ≤ |z| P∞ P∞
j
By comparison test, j=0 (sin aj)z converges, since j=0 |z|j converges absolutely, for |z| < 1.
194
Note that if a = π , the series is zero.
P∞
j=0 (sin aj) → ∞ for a = 2π so r=1indeed.
P∞ j
P∞ ³ eaj −e−aj ´ j 1
³P
∞ a j
P∞ ¡ z ¢j ´
Exercise 14.
j=0 (sinh aj)z = j=0 2 z = 2 j=0 (e z) − j=0 e a ; a>0
1
P 1
If |z| <
ea , then sinh ajz j converges. So then the radius of convergence is r= ea
P∞ zj
Exercise 15.
j=1 aj +bj . Assume a < b

zj
¡ ¢j
bj (1 + ab )
 ³ ¡ a ¢j ´  Ã ¡ ¢j !
j ³z ´
z j+1 b 1 + b z 1 + ab j→∞
(ratio test) ¡ a ¢j+1   = ¡ ¢ −−−→
bj+1 (1 + b ) zj b 1 + a j+1 b
b

So then |z| ≶ b converges (diverges) by ratio test.


If a = b,
1 X ³ z ´j
X∞ ∞
zj
=
j=1
2aj 2 j=1 a

By comparison test with xj , if |z| ≶ |a|, the series converges (diverges).


P∞ ³ aj j
´
Exercise 16.
j=1 j + jb2 z j Use ratio test on each of the sums, separately.

(a|z|)j+1 j a|z| j→∞


= −−−→ a|z|
j + 1 (a|z|)j 1 + 1j
1
=⇒ |z| < then the series converges
a
à !2
(b|z|)j+1 j 2 1 j→∞
= b|z| 1 −−−→ b|z|
(j + 1)2 (b|z|)j 1+ j
1
=⇒ |z| <
b
So if a ≷ b, then r = a; (b)
R1 R1 ¯1
2
e−nx
2
¯ e−n −1 n→∞ 1
Exercise 17.
0
fn (x) = 0
nxe−nx = −2 ¯ = −2 −−−−→ 2
0
However,
2
limn→∞ nxe−nx = 0
This example shows that the operations of integration and limit cannot always be interchanged. We need uniform conver-
sin nx sin nx
gence. Exercise 18. fn (a) = n limn→∞ n =0
f (x) = limn→∞ fn (x)
fn0 (x) = n cos
n
nx
=⇒ limn→∞ fn0 (0) = 1
This example shows that differentiation and limit cannot always be interchanged.
P∞ sin jx
Exercise 19.
j=1 j2 = f (x)
sin jx 1
≤ 2 ∀x ∈ R
j2 j

X ∞
X
| sin jx| sin jx
by comparison test, converges, so converges
j=1
j2 j=1
j2
¯ ¯
¯ sin jx ¯ 1
¯ ¯
¯ j2 ¯ ≤ N 2 ∀ x ∈ R; ∀j ≥ N
P P sin jx
N N12 converges, so
j2 uniformly converges.

sin jx P sin jx
Then by Thm., since
j 2 are continuous, j2 is continuous.

195
P sin jx
Since uniformly converges.
j2
Z X sin jx XZ ¯π X
π π
sin jx X cos jx ¯¯ (−1)j − 1
= = ¯ = =
0 j2 0 j 2 3
−j 0 −j 3

X 1
= 2
j=1
(2j − 1)3

P∞ cos jx x2 πx π2
Exercise 20. It is known that j=1 j2 = 4 − 2 + 6 if 0 ≤ x ≤ 2π
(1) x = 2π

X∞
1 (2π)2 2π 2 π2 π2
2
= − + =
j=1
j 4 2 6 6
P cos jx
(2) As shown in Ex. 19,
j2 is uniformly convergent on R

X Z cos jx X µ sin jx ¶¯¯π/2 X (−1)j+1


= ¯ =
j2 j3 ¯ (2j − 1)3
0
Z 2 µ 3 ¶¯π/2
x πx π 2 1x πx2 π 2 ¯¯
− + = − + x ¯ =
4 2 6 3 4 4 6 0
µ ¶
3 1 1 1 1
= (π) − + = (π)3
12(8) 16 12 32

11.13 Exercises - Properties of functions represented by real power series, The Taylor's series generated by a function,
A sufcient condition for convergence of a Taylor's series, Power-series expansions for the exponential and trigono-
metric functions, Bernstein's Theorem. Sufcient Condition for convergence.

Theorem 37 (Bernstein's Theorem). Assume ∀x ∈ [0, r], f (x), f (j) (x) ≥ 0 ∀j ∈ N.


Then if 0≤x<r

X f (k) (0)
(25) xk converges
k!
Proof. If x = 0, we're done. Assume 0 < x < r.
n
X
f (k) (0) k
f (x) = x + En (x)
k!
k=0
Z
xn+1 1 n (n+1)
En (x) = u f (x − xu)du
n! 0
Z Since f (n+1) > 0, f (n+1) (x(1 − u)) ≤ f (n+1) (r(1 − u))
En (x) 1 1 n (n+1)
Fn (x) = n+1 = u f (x − xu)du En (x) En (r)
x n! 0 =⇒ Fn (x) ≤ Fn (r) =⇒ ≤ n+1
xn+1 r
Pn f (j) (0) j ¡ x ¢n+1
For f (x) = j=0 j! x + En (x) =⇒ En (x) ≤ r En (r)
n
X f (j) (0)
f (r) = rj + En (r) ≥ En (r) since f (j) (0) ≥ 0 ∀j
j=0
j!
¡ ¢n+1
0 ≤ En (x) ≤ xr
So then f (r)
n→∞
n → ∞ and f (t) will be some non-innite value, so En (x) −−−−→ 0. ¤

P∞ j 2j
Exercise 1. j=0 (−1) x
Consider absolute convergence. limj→∞ (x2 )j = 0 If x2 < 1

196
If |x| = 1, then consider
x2(2j) − x2(2j+1) = x4j (1 − x)

X ∞
X
(1 − x)x4j = (1 − x) (x4 )j
j=0 j=0

X
Indeed (−1)j x2j converges for |x| ≤ 1
j=0

P∞ xj 1
P∞ ¡ x ¢j
Exercise 2.
j=0 3j+1 = 3 j=0 3 The series converges for |x| < 3
P
Exercise 3. j = 0∞ jxj
Z ∞
xX ∞
X
jtj−1 = xj
0 j=0 j=0

So the series converges for |x| < 1. Note that we had used the integrability of power series.
P∞ j
Exercise 4.
j=0 (−1) jxj .

jxj jej ln x
(
j ln x ∞ if x>1
lim je =
j→∞ 0 if 0 < x < 1

If x = 1,
X
(2j)x2j − (2j + 1)x2j+1 = x2j (2j + −(2j + 1)x) = −1 (−1) = ∞
P
So (−1)j jxj converges only for 0 ≤ x < 1, |x| < 1.
P∞ P∞ ³ ´
j j+2 j j j+2 j
Exercise 5. j=0 (−2) j+1 x = j=0 (−1) j+1 (2x)

µ ¶
j+2
lim (2x)j = lim (2x)j if |2x| < 1
j→∞ j+1 j→∞

1
So when |x| < 2 , the series converges.

µ ¶ µ ¶
2j + 2 2j + 3 (2j + 2)(2j + 2) − (2j + 3)(2j + 1)
− =
2j + 1 2j + 2 (2j + 1)(2j + 2)
2 2
4j + 8j + 4 − (4j + 5j + 3) 3j + 1 (3j + 1)/2 (3j + 1)/2
= 2 = 2 =
4j 2 + 6j + 2 4j + 6j + 2 2j + 3j + 1 (2j + 1)(j + 1)
µ ¶
3j + 1 3j + 1 3(j + 1/3) 3 1 1
< < < +
4j 2 + 6j + 2 4j 2 + 43 4(j 2 + 1/3) 4 j 12j 2
1 1
Thus, it diverges, by comparison test with
j for x= 2.
P∞
Theorem 38. Let f be represented by f (x) = j=0 aj (x − a)j in the (a − r, a + r) interval of convergence
P∞
(1) jaj (x − a)j−1 also has radius of convergence r.
j=1
0
(2) f (x) exists ∀x ∈ (a − r, a + r) and

X
0
(26) f (x) = jaj (x − a)j−1
j=1

P∞ (2x)j P∞ ej ln 2x 1 ej ln 2x 1
Exercise 6.
j=1 j = j=1 j =⇒ |x| < it'll converge, since by comparison test,
j < j 2 if
2
1
0<x< 2.
197
P∞ (−1)j ¡ x ¢2j
Exercise 7.
j=0 (2j+1) .
2
µ ¶
1 X ³ x ´2j

X X x2j ∞ ∞ ∞
1 X e2j ln 2
x
x2j 1
< = =
j=0
2(j + 1/2)22j j=0
22j+1 j 2 j=0 2 j 2 j=0 j
x
e2j ln 2 1
< 2
since by comparison test, if 0 < x < 2
j j
1 1 1 1 2
− = − = ≤
2(2j) + 1 2(2j + 1) + 1 4j + 1 4j + 3 (4j + 1)(4j + 3)
If x = ±2 ,
1 1 X 1
≤ 2
(converges by comparison test to )
8j j2
P∞ (−1)j ¡ x ¢2j
For |x| < 2 , j=0 (2j+1) 2 converges.

P∞ (−1)j x3j P∞ (−x3 )j 3


Exercise 8.
j=0 j! = j=0 j! = e−x , which converges ∀ x ∈ R
P∞ xj 1
P∞ xj+3
P∞ j
Exercise 9. j=0 (j+3)! = x3 j=0 (j+3)! = x13 j=+3 xj! = x13 (ex − x2 /2 − x − 1) Thus, it converges for ∀ x ∈ R.

P∞ P∞ ³P ´ ¡ x−1 ¢ ex−1 − x
(x−1)j 1 (x−1)j+2 1 ∞ (x−1)j 1
Exercise 10.
j=0 (j+2)! = (x−1)2 j=0 (j+2)! = (x−1)2 j=2 j! = (x−1)2 e − (x − 1) − 1 =
(x − 1)2
P∞(log ax)j
Exercise 11. ax = ex log a = j=0 j!
(log ax)j+1 j! (log ax) j→∞ P∞ (log ax)j
(j+1)! (log ax)j = j+1 −−−→ 0 . By ratio test, j=0 j! converges for all x.
Exercise 12.
 
X∞ ∞
X ∞
x
e −e −x
1 xj
(−x)  X x2j+1
j
sinh x = =  − =
2 2 j=0 j! j=0 j! j=0
(2j + 1)!

x2j+3 (2j + 1)! x2 j→∞


2j+1
= −−−→ 0
(2j + 3)! x (2j + 3)(2j + 2)
Exercise 13.
P∞ (2x)2j j ∞
1 − cos 2x 1− j=0 (2j)! (−1) X 22j−1 x2j (−1)j+1
2
sin x = = =
2 2 j=1
(2j)!
22j+1 x2j+2 (2j)! 4x2 j→∞
= −−−→ 0
(2j + 2)! 22j−1 x2j (2j + 2)(2j + 1)
So the series converges ∀x
1
P∞ ¡ x ¢j 1
P∞ xj
Exercise 14.
1− x = j=0 2 2−x = j=0 2j+1
2

xj+1 2j+1 x
j+2 j
= <1 =⇒ (the series converges for |x| < 2, by ratio test)
2 x 2
If x = 2, the series would diverge

X ∞
1 (−2)j 1X 1 1
If x = −2 j
= (−1)j = 0 but = x
2 j=0
2 2 j=0 2 − (−2) 4

2 P∞ (−1)j x2j
Exercise 15. e−x = j=0 j!
x2j+2 j! x2 j→∞
(j+1)! x2j = j+1 −−−→ 0
Exercise 16. sin 3x = sin 2x cos x + sin x cos 2x = 3 sin x − 4 sin3 x.
 
X∞ 2j+1 j X∞ 2j+1 j
3 sin x − sin 3x 3 x (−1) (3x) (−1) 
sin3 x = =  − =
4 4 j=0 (2j + 1)! j=0
(2j + 1)!
   
X∞ j 2j+1 2j+1 ∞
X j+1 2j+1 2j+1
3 (−1) x (1 − 3 ) 3  (−1) (3 − 1)x
=  = 
4 j=0 (2j + 1)! 4 j=0 (2j + 1)!
198
q ³P P ´
1+x (+x)j (xj )(−1)j
Exercise 17. log 1−x = 12 (log (1 + x) − log (1 − x)) = 1
2 j=1 j (−1)
j−1
− j=1 j


X ∞
X X∞ ∞
X X∞
(−x)j+1 (−x)j xj+1 xj ((−1)j + 1)xj
ln (1 + x) = (−1) = (−1) − ln (1 − x) = = =⇒ −
j=0
j+1 j=1
j j=0
j + 1 j=1 j j=1
j

X x2j+1
=⇒
2j + 1
x2j+3 2j + 1 j→∞ 2
−−−→ x
2j + 3 x2j+1
|x2 | < 1, converges, with radius of convergence of 1

3x 1 1
P∞ xj
P∞ (−2x)j P∞ xj
Exercise 18.
1+x−2x2 = 1−x − 1+2x = j=0 j − j=0 j = j=0 j (1 − (−2)j )
¯³ ´¯
¯ 1
+ 2 ¯
x j+1
|(1 − (−2) j+1
)| j ¯ (−2)j ¯ j→∞
= x ¯³ ´ ¯ −−−→ 2x < 1
j+1 j j
|x (1 − (−2) )| ¯ 1 ¯
¯ (−2) j − 1 ¯

1
|x| <
2
1
For x=
2
x2j x2j+1 x2j ((2j + 1)(1 − 22j ) + x(1 + 22j+1 )(2j))
(1 − (−2)2j ) + (1 − (−2)2j+1 ) =
2j 2j + 1 (2j)(2j + 1)
¡ 1 ¢2j 2j 2j+2
2 (2j + 1 − (2j + 1)2 + 2jx + 2 jx) j→∞ −(2j + 1) + 4j 2j − 1
−−−→ = →0
(2j)(2j + 1) (2j)(2j + 1) (2j)(2j + 1)

3x
So
1+x−2x2 converges for |x| ≤ 12
12−5x
P∞ ³ (−1)j
´
Exercise 19.
6−5x−x2 = j=0 1 + 6j xj (|x| < 1).

X X ∞ ∞ µ ¶j ∞
X µ ¶j
12 − 5x 5x − 12 1 6 −x −1
2
= = + = xj + = xj (1 + )
6 − 5x − x (x + 6)(x − 1) 1 − x 6 + x j=0 j=0
6 j=0
6

¡ ¢j
|x| < 1 since for x = 1, limj→∞ (1 + −1 6 ) = 1.
1
P ∞ 2π(j+1) j
√2
x +x+1 =
Exercise 20. 2
3 j=0 sin 3 x (|x| < 1)

X ∞
1
= (x3 )j
x3 − 1 = (x − 1)(x2 + x + 1) 1−x3
j=0
1 x−1 1−x X¡ ∞
= 3 = 1−x ¢
x2 + x + 1 x −1 1 − x3 3
= (x3 )j − x3j+1
1−x j=0

By induction, it could be observed that x3j , −x3j+1 , 0 appear in sequences of 3 terms.

√2
3
sin 2π(j+1)
3 accounts for this.

P∞
=⇒ √2
3 j=0 sin 2π(j+1)
3 xj |x| < 1

P
sin 2π(j+1)
3 xj < xj (so by comparison test to xj , the radius of convergence is 1 )
199
Exercise 21.

X ∞ µ X¶0 X ∞ ∞
1 1 1
= xj ; 2
= jxj−1 =
= (j + 1)xj
1 − x j=0 (1 − x) 1−x j=1 j=0
 
X ∞ X ∞
1 1/4 1/2 1/4 1
= + + =  (xj + (−x)j ) + 2 jxj−1 
(1 − x)(1 − x2 ) 1 − x (1 − x)2 1+x 4 j=0 j=1
 
∞ ∞ ∞
x 1 X j+1 j
X
j 1X j
= x (1 + (−1) ) + 2 jx = x (1 + (−1)j−1 + 2j)
(1 − x)(1 − x2 ) 4 j=0 j=1
4 j=1

Exercise 22.
³ π ´ X (2x)2j+1
∞ ∞
X (2x)2j
sin 2x + = (−1)j ; cos 2x = (−1)j
4 j=0
(2j + 1)! j=0
(2j)!

X 2j+1
(2x)
sin 2x = (−1)j √
(2j + 1)! ∞
X
j=0 j 2

aj x = (sin 2x + cos 2x)
X (2x) 2j 2
j=0
cos 2x = (−1)j
j=0
(2j)!
Ã√ ! Ã√ !
298 (−1)49 2 −298 2
For j = 98, aj = =
98! 2 98! 2

Exercise 23.

f (x) = (2 + x2 )5/2
5
f 0 (x) = (2 + x2 )3/2 (2x) = 5x(2 + x2 )3/2
2
15
f 00 (x) = 5(2 + x2 )3/2 + x(2 + x2 )1/2 (2x) = 5(2 + x2 )3/2 + 15x2 (2 + x2 )1/2
2
15 15x2
f 000 (x) = (2 + x2 )1/2 (2x) + 30x(2 + x2 )1/2 + (2 + x2 )−1/2 (2x)
2 2
15 30x
f 0000 (x) = 15(2 + x2 )1/2 + (2 + x2 )−1/2 (2x)x + 30(2 + x2 )1/2 + (2 + x2 )1/2 (2x)+
2 µ ¶ 2
1
+ 45x2 (2 + x2 )−1/2 + 15x3 − (2 + x2 )−3/2 (2x)
2
5(23/2 )x2 0x3 45 √ 4
25/2 + 0x + + + 2x
2! 3! 4!
2
Exercise 24. f (x) = e−1/x if x 6= 0 and let f (0) = 0
(1)
∞ ¡ −1 ¢j ∞ ¡ −1 ¢j ∞
X −1 X X (−1)j x−2j
x2 x2
f (x) = =1+ 2 + =
j=0
j! x j=2
j! j=0
j!
X∞ X∞
(−2j)(−2j − 1) . . . (−2j − (k − 1)) −2j (−2j)!
f (k) = (−1)j x = (−1)j x−2j
j=0
j! j=0
(−2j − k)!j!
Use ratio test :

(−2(j + 1))! j!(−2j − k)! x−2(j+1) (−2j − k)(−2j − k − 1) −2 j→∞


−2j
= x −−−→ 0
(−2j − 2 − k)!(j + 1)! (−2j)! x (j + 1)(−2j)(−2j − 1)
Thus, by ratio test, every order of derivative exists for every x on the real line since the series representing the
derivative converges for every x.
P∞ −x−2j
(2) f (x) = j=0 j! . There are no nonzero terms of positive power, i.e. no xj ; j ≥ 1.

=⇒ f (j) (0) = 0 ∀j ≥ 1
200
11.16 Exercises - Power series and differential equations, binomial series.

Exercise 1. For (1 − x2 )y 00 − 2xy 0 + 6y = 0,



X
y= aj xj
j=0

X
y0 = jaj xj−1 f (0) = 1 =⇒ a0 = 1 f 0 (0) = 0 =⇒ a1 = 0
j=1

X ∞
X
y 00 = j(j − 1)aj xj−2 = (j + 2)(j + 1)aj+2 xj
j=2 j=0

X
2(1)a2 + 3(2)a3 x + −2(1)a1 x + 6a0 + 6a1 x + ((j + 2)(j + 1)aj+2 − j(j − 1)aj − 2jaj + 6aj )xj =
j=2

X
= 2a2 + 6 + 6a3 x + ((j + 2)(j + 1)aj+2 + aj (j + 3)(j − 2))xj
j=2

a2 = −3 (j + 3)(j − 2)
=⇒ aj+2 = aj
a3 = 0 (j + 2)(j + 1)

j = 2, a4 = 0, so then aj+2 = 0 for j = 2, 4, . . . .


For Likewise, since a3 = 0, then aj+2 = 0 for j = 3, 5, . . . .
=⇒ f (x) = 1 − 3x2
Exercise 2. Using the work from above, then for (1 − x2 )y 00 − 2xy 0 + 12y = 0

f (0) = 0 =⇒ a0 = 0 f 0 (0) = 2 =⇒ a1 = 2

X
2(1)a2 + 3(2)a3 x + −2(1)a1 x + 12a0 + 12a1 x + ((j + 2)(j + 1)aj+2 − j(j − 1)aj − 2jaj + 12aj )xj =
j=2

X
= 2a2 + 6a3 x + −4x + 0 + 24x + ((j + 2)(j + 1)aj+2 − aj (j + 4)(j − 3))xj
j=2

a2 = 0 (j + 4)(j − 3)
=⇒ aj+2 = aj
a3 = −10/3 (j + 2)(j + 1)

Forj = 3, a5 = 0, so then aj+2 = 0 for j = 3, 5, . . . . Likewise, since a2 = 0, then aj+2 = 0 for j = 2, 4, . . . .


=⇒ f (x) = −10/3x3 + 2
P∞ x4j d4 y
Exercise 3. f (x) =
j=0 (4j)! ; dx4 = y

µ ¶0
x4j x4j−1
=
(4j)! (4j − 1)!
(x4 )0 = 4x3 µ ¶00
x4j x4j−2
4 00 2 = X∞ X∞
(x ) = 12x (4j)! (4j − 2)! x4j−4 x4j
µ ¶000 = y 0000 = = f (x)
(x4 )000 = 24x x4j x4j−3 j=1
(4j − 4)! j=0
(4j)!
=
(x4 )0000 = 24 (4j)! (4j − 3)!
µ ¶0000
x4j x4j−4
=
(4j)! (4j − 4)!

To test convergence, use the ratio test


4j+4
x (4j)! x4 j→∞
4j
= −−−→ 0 ∀x ∈ R
(4j + 4)! x (4j + 4)(4j + 3)(4j + 2)(4j + 1)

So the series converges on R.


201
P∞ xj
Exercise 4. f (x) = j=0 (j!)2 xy 00 + y 0 − y = 0

X X∞
jxj−1 (j + 1)xj
y0 = =
j=1
(j!)2 j=0
((j + 1)!)2
X∞
(j + 1)jxj−1
y 00 =
j=1
((j + 1)!)2
X∞ µ ¶ X∞ µ ¶
(j + 1)j (j + 1) 1 j 1 1 1
+ − x + − 1 = − =0
j=1
((j + 1)!)2 ((j + 1)!)2 (j!)2 1! j=1
(j!)2 (j!)2
P∞ 1·4·7...(3j−2) 3j
Exercise 5. f (x) = 1 + j=1 (3j)! x ; y 00 = xa y + b (Find a and b )

X 1 · 4 · 7 . . . (3j − 2) 3j−1
f0 = x
j=1
(3j − 1)!

X ∞
X
1 · 4 · 7 . . . (3j − 2) 1 · 4 · 7 . . . (3j − 5)
f 00 = x3j−2 = x3j−2 =
j=1
(3j − 2)! j=1
(3j − 3)!

X ∞
X
1 · 4 · 7 . . . (3j − 5) 1 · 4 · 7 . . . (3j − 2)
=x+ x3j−2 = x + x3j+1
j=2
(3j − 3)! j=1
(3j)!

X 1 · 4 · 7 . . . (3j − 2)
xa f = −xa + x3j+a
j=1
(3j)!

So then a = 1; b=0 .
µ ¶
1 · 4 · 7 . . . (3j + 1) 3j+3 (3j)! 1 1(3j + 1) j→∞
x = x3 −−−→ 0
(3j + 3)! 1 · 4 · 7 . . . (3j − 2) x3j (3j − 2)(3j + 3)(3j + 2)(3j + 1)
So the series converges for all x.
P∞ x2j
Exercise 6. f (x) = j=0 j! ; y 0 = 2xy .

X X∞ X∞
2jx2j−1 x2j−1 x2j+1
f0 = =2 =2 = 2xf
j=1
j! j=1
(j − 1)! j=0
j!

x2j+2 j! x2 j→∞
2j
= −−−→ 0 ∀x
(j + 1)! x j+1
By ratio test, f converges ∀ x ∈ R.
P∞ xj
Exercise 7. f (x) = j=2 j! y0 = x + y
X∞ ∞
X
0 xj−1 xj
f = = =x+y
j=2
(j − 1)! j=1 j!

xj+1 j! x j→∞
= −−−→ 0
(j + 1)! xj j
So the series converges ∀ x ∈ R by ratio test.
Exercise 8.

X (−1)j (kx)2j
f (x) =
j=0
(2j)!
X∞ ∞
(−1)j (kx)2j−1 k X (−1)j+1 (kx)2j+1 k
f0 = = f 00 − k 2 f = 0
j=1
(2j − 1)! j=0
(2j + 1)!

X (−1)j+1 (kx)2j
f 00 = k2
j=1
(2j)!
2j+1
(kx) (2j)! kx j→∞
= −−−→ 0 by ratio test, f converges ∀x ∈ R.
(2j + 1)! (kx)2j 2k + 1
202
Exercise 9.

X∞ X∞
(3x)2j−1 9(3x)2j+1
f 00 = 9=
j=1
(2j − 1)! j=0
(2j + 1)!
X∞
(3x)2j+1
9(f − x) = 9(x + − x)
j=0
(2j + 1)!
(3x)2j+3 (2j + 1)! 9x2 j→∞
= −−−→ 0
(2j + 3)! (3x)2j+1 (2j + 3)(2j + 2)
(by ratio test, f converges ∀x ∈ R )

P∞ j x2j
P∞ j x2j+1
Exercise 10. J0 (x) = j=0 (−1) (j!)2 22j J1 (x) = j=0 (−1) j!(j+1)!22j+1 .

(1)

x2j+2 (j!)2 22j x2 j→∞


2 2j+2 2j
= −−−→ 0 by ratio test, f converges ∀x ∈ R
((j + 1)!) 2 x (j + 1)2 4
x2j+3 j!22j+1 x2 j→∞
2j+3 2j+1
= −−−→ 0 by ratio test, f converges ∀x ∈ R
(j + 2)!2 x (j + 2)(j + 1)4

(2)


X X ∞
x2j−1 x2j+1
J00 (x) = (−1)j = (−1)j+1
= −J1 (x)
j=1
(j − 1)!(j!)22j−1 j=0
j!(j + 1)!22j+1

(3)


X x2j+2
j1 (x) = xJ1 (x) = (−1)j

X j!(j + 1)!22j+1
x2j+1 j=0
j0 (x) = xJ0 (x) = (−1)j ∞
(j!)2 22j X (−1)j x2j+1
j=0
j10 =
j=0
(j!)2 22j
=⇒ j0 = j10

x2 y 00 + xy 0 + (x2 − n2 )y = 0.
Exercise 11.

n = 0 =⇒ x2 y 00 + xy 0 + (x2 )y = 0


X X ∞
x2j x2j−2
J0 = (−1)j = (−1)j−1
;
j=0
(j!)2 22j j=1
((j − 1)!)2 22j−2

X x2j−1
J00 = (−1)j ;
j=1
j!(j − 1)!22j−1
X∞
x2j−2 (2j − 1)
J000 = (−1)j
j=1
j!(j − 1)! 22j−1

X µ ¶
j (2j − 1) 1 −2j
(−1) + + =0
j=1
j!(j − 1)!22j−1 j!(j − 1)!22j−1 ((j − 1)!)j!22j−1
203
n = 1 =⇒ x2 y 00 + xy 0 + (x2 − 1)y = 0


X ∞ ∞
(−1)j x2j+1 x X (−1)j x2j+1 x X (−1)j−1 x2j−1
J1 (x) = 2j+1
= + 2j+1
= +
j=0
j!(j + 1)!2 2 j=1 j!(j + 1)!2 2 j=1 (j − 1)!(j)!22j−1

1 X (−1)j (2j + 1)x2j
J10 = +
2 j=1 j!(j + 1)!22j+1

X (−1)j (2j + 1)(2j)x2j−1
J100 =
j=1
(j!)(j + 1)!22j+1
x2 J100 + xJ10 + (x2 − 1)J1 =
X∞ µµ ¶ µ ¶µ ¶µ ¶µ 2¶ ¶
(−1)j (2j + 1) (−1)j (−1) j + 1 j 1 2 (−1)j
= x2j+1 ((2j) + (1)) + − +
j=1
(j!)(j + 1)!22j+1 (j − 1)!j! j + 1 j 22j−1 22 (j + 1)!(j!)22j+1
x x
+ − =
2 2
X∞ µ ¶
(−1)j x2j−1
= ((2j + 1)(2j + 1) + (−1)(2j)(2j + 2) − 1) = 0
j=1
(j!)(j + 1)!22j+1

Exercise 12. y 0 = x2 + y 2 ; y = 1 when x = 0.


X
y = a0 + a1 x + a2 x2 + aj xj
j=3
 2
X∞
y 2 = a20 + a21 x2 + a22 x4 +  aj xj  +
j=3
y 0 (0) = 0 + 12 = 1

X
+ 2a0 a1 x + 2a0 a2 x2 + 2a0 aj x j +
j=3

X ∞
X
+ 2a1 a2 x3 + 2a1 aj xj+1 + 2a2 aj xj+2
j=3 j=3

X
y 0 = a1 + 2a2 x + jaj xj−1
j=3

a1 = 1 since y 0 (0) = 1
Consider the rst few terms of x2 + y 2
a1 = 1 = a20 =⇒ a0 = 1
2a2 = 2a0 a1 =⇒ a2 = 1
a20 + 2a0 a1 x + a21 x2 + 2a0 a2 x2 + x2 = a1 + 2a2 x + 3a3 x2 =⇒
4
3a3 = a21 + 2a0 a2 + 1 = 4 =⇒ a3 =
3
204
Exercise 13. y 0 = 1 + xy 2 with y = 0 when x = 0 =⇒ a0 = 0

X
y = a1 x + a2 x2 + a3 x3 + aj x j
j=4
 2

X ∞
j X
y= aj x 2
y = a21 x2 + a22 x4 + a3 x + 
6
aj x j
+
j=1 j=4
X∞ ∞
X ∞
X
j−1
y= jaj x = (j + 1)aj+1 x6j + 2a1 a2 x3 + 2a1 a3 x4 + 2a1 aj xj+1 +
j=1 j=0
j=4

X ∞
X
+ 2a2 a3 x5 + 2a2 aj xj+2 + 2a3 aj xj+3
j=4 j=4

a1 = 1
x: 2a2 = 0 =⇒ a2 = 0 x2 : 3a3 = 0 =⇒ a3 = 0
1
x3 : 4a4 = 12 =⇒ a4 =
4
x4 : 5a5 = 0 =⇒ a5 = 0 x5 : 6a6 = 0 =⇒ a6 = 0
1
x6 : 7a7 = 2a1 a4 + 2a2 a3 =⇒ a7 =
14
x7 : 8a8 = 0 + 2a2 a4 = 0 =⇒ a8 = 0 x8 : 9a9 = 0 =⇒ a9 = 0
µ ¶2
1 1 23
x9 : 10a10 = + 2(1) =⇒ a10 =
4 14 1120

y0 = x + y2
Exercise 14. y = 0 when x = 0 =⇒ a0 = 0
0
y (0) = 0 + 0 = 0 =⇒ a1 = 0

X ∞
X ∞
X
y= aj xj y0 = jaj xj−1 = (j + 1)aj+1 xj
j=2 j=2 j=1
 2

X
y 2 = a2 x2 + a3 x3 + a4 x4 + aj xj  =
j=5
 2

X
= a2 x4 + a23 x6 + a24 x8 +  aj xj  +
j=5

X ∞
X ∞
X
+ 2a2 a3 x5 + 2a2 a4 x6 + 2a2 x2 aj xj + 2a3 a4 xy + 2a3 x3 aj xj + 2a4 x4 aj xj
j=5 j=5 j=5
0 2
y =x+y
1
x : 2a + 2 = 1 + 0 =⇒ a2 =
2
x2 : 3a3 = 0 =⇒ a3 = 0 x3 : 4a4 = 0 =⇒ a4 = 0
1
x4 : 5a5 = a22 =⇒ a5 = x5 : 6a6 = 0 =⇒ a6 = 0x6 : 7a7 = 0 =⇒ a7 = 0
20
1 1 1
x7 : 8a8 = 2( )( ) =⇒ a8 =
2 20 160
x8 : 9a9 = 0 =⇒ a9 = 0 x9 : 10a10 = 0 =⇒ a10 = 0
µ ¶2
1 1 1 7
x10 : 11a11 = 2( )( )+ =⇒ a11 =
2 160 20 8800
205
Exercise 15.
P∞ y 0 = αy P∞
j j
=⇒ j=0 (j + 1)aj+1 x = α j=0 aj x
αaj
aj+1 = (j+1)

αj
aj = a0 x (by induction)
j!
Exercise 16. y 00 = xy

X
y 00 = (j + 2)(j + 1)aj+2 xj =

X
j=0

= aj xj+1
X
j=0
= 2a2 + (j + 3)(j + 2)aj+3 xj+1
j=0
aj
=⇒ a2 = 0 and aj+3 =
(j + 3)(j + 2)
a0 a1
j=0 a+3= j=1 a4 =
3·2 4·3
a3 a1
j=3 a6 = j=4 a7 =
6·5 7·6·4·3
j−1 j−1
a0 Y a1 Y
a3j = (3k + 1) ; a3j+1 = (3k + 2)
(3j)! (3j + 1)!
k=0 k=0

Exercise 17. y 00 + xy 0 + y = 0

X
y= aj xj
j=0

X
y0 = jaj xj−1

X
j=1 −aj

y 00 + xy 0 + y = xj ((j + 2)(j + 1)aj+2 + jaj + aj ) = 0 =⇒ aj+2 =
X (j + 2)
j=1
y 00 = j(j − 1)aj xj−2
j=2

X
= (j + 2)(j + 1)aj+2 xj
j=0
−a0 −a1
a2 = a3 =
2 3
−a2 −a3 a1
a4 = a5 = =
4 5 15
(−1)j a0 a1
a2j = a2j+1 = (−1)j (could be shown by induction)
(2j)!! (2j + 1)!!

Exercise 18. Recall that



X
y= aj xj
j=0

X
y0 = jaj xj−1
j=1
X∞
= (j + 1)aj+1 xj
j=0
P∞
Knowing this, we could cleverly observe that e−2x = j=0 (2aj + (j + 1)aj+1 )xj is actually a differential equation!!!

=⇒ e−2x = y 0 + 2y
−A(x)
¡R x ¢ Rx
Solving this ODE using y(x) = e
a
Q(t)eA(t) dt + y(a) where A(x) = a P (t)dt,

y = e−2x (x + 1)
206
We had obtained the necessary initial conditions to solve this ODE from the information given, that a0 = 1, so that y(0) = 1.
By doing some simple computation and comparison of powers with e−2x , then a1 = 2, a2 = −2, a3 = 4/3
P∞ P∞
Exercise 19. cos x = j=0 aj (j + 2)xj for f (x) = j=0 aj xj .

P∞
(x)2j j
Using cos x =j=0 (2j)! (−1) representation, we can immediately conclude that for odd terms, a2j+1 = 0 and by
matching powers of x,
1
a2j (2j + 2) = (−1)j
(2j)!
a5 = 0
(−1)3 −7
=⇒ a6 =
a6 (6 + 2) =
6! 8!
P∞ j
P∞ j
P∞ j
Now notice that for cos x = j=0 aj (j + 2)x = j=1 jaj x + 2 j=0 aj x is actually a differential equation, cos x =
0
xy + 2y . We can solve this rst-order ODE using
¡R x ¢ Rx
y(x) = e−A(x) a Q(t)eA(t) dt + y(a) where A(x) = a P (t)dt. Then solving y 0 + 2y cos x
x = x ,
1
y=
(x sin x + cos x − (a sin a + cos a) + b)
x2
Plugging 0 as a good guess back into the ODE, cos 0 = 1 = y(0)(2) =⇒ y(0) = 12 With this initial condition, we get

sin x cos x − 1
f (x) = + if x 6= 0
x x2
1 −2
So f (0) = 2 and f (π) = π2
Exercise 20.

(1)
X∞ µ ¶
−1/2
(1 − x)−1/2 = (−x)j =
j=0
j
¡ −1 ¢ ¡ −3 ¢ ¡ −1 ¢ ¡ −3 ¢ ¡ −5 ¢ ¡ −1 ¢ ¡ −3 ¢ ¡ −5 ¢ ¡ −7 ¢
1 2 3
=1+ x+ 2 2
x + 2 2 2
x + 2 2 2 2
x4 +
2 ¢¡ ¢¡ 2 ¢¡ ¢¡ ¢
¡ −1 3! 4!
−3 −5 −7 −9
+ 2 2 2 2 2
x5 + · · · =
5!
1 3 5 35 5 63 5
= 1 + x + x2 + x3 + x + x + ...
2 8 8 128 256
P∞ ¡ ¢ P∞ P∞
(2) To make the notation clear, (1 − x)
−1/2
= j=0 −1/2
j (−x)j = j=0 bj xj = j=0 aj
Now ¡ ¢
α
α(α − 1) . . . (α − (j + 1) + 1) j! (α − j)
¡j+1
α
¢ = =
j
(j + 1)! α(α − 1) . . . (α − j + 1) (j + 1)
−1
So for α= 2 , µ ¶
aj+1 1/2 + j
=− (−x) < x
aj j+1
Using this, we further nd that
1
bj+1 < bj
50
µ ¶2
1 1
bj+2 < bj+1 < bj
50 50
1
¡ 1 ¢j
For x= 50 . So by induction, bn+j < bn 50

X ∞
X µ ¶j
1 1/50 an
rn = an+j < an = an =
j=1 j=1
50 1 − 1/50 49
an
rn <
49
207
−1/2 ¡ ¢
1 −1/2
¡ ¢−1/2 √
(3) Note that (1 − x) = 1 − 50 = 4950 = 572
µ ¶−1/2 µ ¶2 µ ¶3 µ ¶4 µ ¶5
7 1 1 3 1 5 1 35 1 63 1
1− =1+ + + + +
5 50 100 2 2(50) 2 2(50) 8 2(50) 8 2(50)

2 ' 1.4142135624

Exercise 21.
1732
¡ 176
¢−1/2 ¡ 3000000 ¢1/2
(1)
1000 1−3000000 = 1732
1000 √ 2999824
1/2 1/2
Obviously, (3000000) = 1000 3 so that we have 1732 (3/2999824) .
With long multiplication, we could show easily that 1732 ∗ 1732 = 2999824 (it's harder to divide). So then

µ ¶−1/2
1732 176 √
1− = 3
1000 3000000

(2)
R 1
Exercise 22.arcsin x = √1−x 2

P ∞ ¡ ¢ P∞ ¡ ¢
(1 − x2 )−1/2 = j=0 αj (−x2 )j = 1 + j=1 αj (−x2 )j
X∞ µ ¶
−1/2 (−1)j 2j+1
=⇒ arcsin x = x + x
j=1
j (2j + 1)
¡ −1 ¢ ¡ −3 ¢ ¡ ¢
. . . −12 −j +1 (1)(3) . . . (1 + 2j − 2) (2j − 1)!!
2 2
= (−1)j = (−1)j
j(j − 1) . . . (2)(1) (2j)!! (2j)!!
X∞
(2j − 1)!! x2j+1
=⇒ arcsin x = x +
j=1
(2j)!! 2j + 1

12.4 Exercises - Historical introduction, The vector space of n-tuples of real numbers, Geometric interpretation for
n ≤ 3.
Exercise 1.

A =(1, 3, 6)
B =(4, −3, 3)
C =(2, 1, 5)
(1) (5, 0, 9) = A + B
(2) (−3, 6, 3)
(3) (3, −1, 4)
(4) 7A − 2B − 3C = (7, 21, 42) − (8, −6, 6) − (6, 3, 15) = (−1, 26, 36) − (6, 3, 15) = (−7, 24, 21)
Exercise 2. See sketches.

Exercise 3. See sketches.

Exercise 4.

(1) See the sketch.


(2) I think it'll be a line.
(3) It'll ll a parallelogram with A and B as the sides.
(4) It'll ll the entire real, 2-dim. plane.

Exercise 5.
A = (2, 1)
B = (1, 3)
C = (c1 , c2 )
c2 − 3c1
x=
−5
xA + yB = (2x, x) + (y, 3y) = (2x + y, x + 3y) = (c1 , c2 )
c1 − 2c2
y=
−5
208
Exercise 6.
A =(1, 1, 1)
B =(0, 1, 1) D = xA + yB + zC
C =(1, 1, 0)
(1) D = (x + z, x + y + z, x + y)
(2) 0 = x + z = x + y + z =⇒ y = 0
x=0 z=0
(3) D = (1, 2, 3)
x+y+z =2=3+z z = −1
x−1=1 x=2 y =1
Exercise 7.
A =(1, 1, 1)
B =(0, 1, 1) D = xA + yB + zC
C =(2, 1, 1)
(1) D = (x + 2z, x + y + z, x + y + z)
(2) x = 2, z = −1, y = −1 and D = 0
(3) x + y + z = 2 but x + y + z = 3
Exercise 8.
A =(1, 1, 1, 0)
B =(0, 1, 1, 1) D = xA + yB + zC
C =(1, 1, 0, 0)
(1) D = (x + z, x + y + z, x + y, y)
(2) y = 0 =⇒ x = 0 =⇒ z = 0
(3) (1, 5, 3, 4) = D =⇒ y = 4, x = −1, z = 2
(4) (1, 2, 3, 4) = D =⇒ y = 4, x = −1, z = −1 but x + z = 1
Exercise 9.

A, B parallel =⇒ A = tB t 6= 0
C, B parallel =⇒ C = sB s 6= 0
A = tB = tC s and
t
s = k =
6 0 . So A, C are parallel.

Exercise 10. Given


C =A+B
A = sD
If C k D so that C = tD
C − A = (t − s)D = B . If t − s 6= 0 then B k C . Even if t = s, then that meant that C = A and B = 0 anyways.
If B k D , so that B = uD
c = (s + u)D =⇒ C k D
Exercise 11.

(1) Theorem 12.1 says that vector addition is commutative and associative.
Let A, B ∈ Vn ; A = (a1 , . . . , an ), B = (b1 , . . . , bn )
Use this denition,

Denition 1.
A = B iff a1 = b1 . . . an = bn
A + B = (a1 + b1 , . . . , an + bn )
cA = (ca1 , . . . , can )

A + B = (a1 + b1 , . . . , an + bn ) = (b1 + a1 , . . . , bn + an ) = B + A
A + (B + C) = (a1 + (b1 + c1 ), . . . , an + (bn + cn )) = ((a1 + b1 ) + c1 , . . . , (an + bn ) + cn ) =
= (A + B) + C
c(dA) = (c(da1 ), . . . , c(dan )) = ((cd)a1 , . . . , (cd)an ) = (cd)A
c(A + B) = (c(a1 + b1 ), . . . , c(an + bn )) = (ca1 + cb1 , . . . , can + cbn ) = cA + cB
(c + d)A = ((c + d)a1 , . . . , (c + d)an ) = (ca1 + da1 , . . . , can + dan ) = cA + dA
209
So commutativity and associativity follows from the associativity, commutativity, and distributivity of the one-
dim. reals (or even complex numbers).
(2) See sketch.

Exercise 12.
1 1
A+ C − A=
A =B − C 2 2
C =B − A 1 1 1
= A+ C = (A + C)
2 2 2
B =A + C 1
= B
2
It reminds me of how the diagonals of a parallelogram bisect each other.

12.8 Exercises - The dot product, Length or norm of a vector, Orthogonality of vectors.
Exercise 1.
A = (1, 2, 3, 4)
B = (−1, 2, −3, 0)
C = (0, 1, 0, 1)
(1) A · B = −6
(2) B · C = 2
(3) A · C = 6
(4) A · (B + C) = A · (−1, 3, −3, 1) = 0
(5) (A − B) · C = (2, 0, 6, 4) · C = 4

Exercise 2.
A = (2, 4, −7)
B = (2, 6, 3)
C = (3, 4, −5)
(1) (A · B)C = 7(3, 4, −5) = (21, 28, −35)
(2) A · (B + C) = A · (5, 10, −2) = 64
(3) (A + B) · C = (4, 10, −4) · C = 72
(4) A(B · C) = (30, 60, −105)
¡ 2 4 −7 ¢
(5) A/(B · C) =
15 , 15 , 15
Exercise 3. Given A · B = A · C , A 6= 0,
Then kAkkBk cos θAB = kAkkCk cos θAC
Let kBk = kCk but θAB = −θAC . Then B 6= C .

Exercise 4. Given A · B = 0 ∀ A,
kAkkBk cos θAB = 0, choose kAk 6= 0, cos θAB 6= 0
kBk = 0
Exercise 5.
A = (2, 1, −1)
; nd a nonzero C in V3 s.t. A · C = B · C = 0
B = (1, −1, 2)
C = (c1 , c2 , c3 ) without loss of generality, let c1 = 1
2 + c2 − c3 = 0 c2 = −5
=⇒
1 + −c2 + 2c3 = 0 c3 = −3
Exercise 6.
A = (1, −2, 3) B · B = 14
C = xA + yB
B = (3, 1, 2) C ·B =0

C · B = (xA + yB) · B = x(A · B) + yB · B = 0


x(7) + y(14) = 0 =⇒ x + 2y = 0
y = −1, x = 2 C = (−1, −5, 4)
Exercise 7.
A = (2, −1, 2) A=C +D
Given CkB B = tC
B = (1, 2, −2) B·D =0
tC · D = 0 t 6= 0
A · B = B · C = −4
210
B · B = tB · C = 9 =⇒ t = 9/ − 4
C = −49 (1, 2, −2) ¡ ¢ ¡ 22 −1 10 ¢
D = A − C = (2, −1, 2) − 49 , 89 , −8
9 = 9, 9 , 9
Exercise 8. Given that if
A = (1, 2, 3, 4, 5)
C, D ∈ V5
B = (1, 1/2, 1/3, 1/4, 1/5)
B = C + 2D D · A = 0, C k A; C = tA
=⇒ B = tA + 2D
A · B = 5 = tA · A + 2A · D = t(55) t = 1/11
1
C = 11 (1, 2, 3, 4, 5)
1
¡ A
¢ 1¡ ¡ 1 2 3 4 5 ¢¢
D = ¡2 B − 11 = 2 (1,
¢ 1/2, 1/3, 1/4, 1/5) − 11 , 11 , 11 , 11 , 11
5 7 1 −5 −7
D = 11 , 44 , 33 , 88 , 55
Exercise 9. √
kA + Bk = k(1, −3, 8)k = 74
A = (2, −1, 5) √
kA − Bk = k(3, 1, 2)k = 14
B = (−1, −2, 3) √
C = (1, −1, 1) kA + B − Ck = k(0, −2, 7)k = 53
kA − B + Ck = k(4, 0, 3)k = 5
Exercise 10. B · A = 0; kBk = kAk
(1) B = (1, −1)
(2) A = (1, −1), B = (1, 1)
(3) A = (2, −3), B = (3, 2)
(4) A = (a, b), B = (b, −a)
Exercise 11.
A = (1, −2, 3)
B = (3, 1, 2)
(4, −1, 5)
(1) A + B = (4, −1, 5) =⇒ C = √
42
(−2, −3, 1)
(2) A − B = (−2, −3, 1) C= √
14
√ √
(3) A + 2B = (7, 0, 7) =⇒ C = (1/ 2, 0, 1/ 2)
(−1, −5, 4)
(4) 2A − B = (−1, −5, 4) C= √
42
Exercise 12.
A = (4, 1, −3)
A·B =0
B = (1, 2, 2)
C ·D =0
C = (1, 2, −2)
C ·E =0
D = (2, 1, 2)
D·E =0
E = (2, −2, 1)
Exercise 13. The answer is easy to see geometrically. If you do the algebra, the condition of kAk = kBk xes the length of
the vector and A · B orthogonality xes the direction, up to π or 180◦ .
(1) A = (1, 2) =⇒ B = (−2, 1), (2, −1)
(2) A = (1, −2) =⇒ B = (2, 1), (−2, −1)
(3) A = (2, −1) =⇒ B = (1, 2), (−1, −2)
(4) A = (−2, 1) =⇒ B = (1, 2), (−1, −2)
Exercise 14.
A = (2, −1, 1)
A·B =6
B = (3, −4, −4)
A + B = (5, −5, −3)
B − A = (1, −3, −5)
211
We have the condition that we have one right angle for a right triangle: (A − C) · (C − B) = A · C − C 2 − A · B + B · C =
2
(A + B) · C − C − 6 = 0
(5, −5, −3) · C = 6 + C 2
5c1 − 5c2 + 0 = 6 + c21 + c22 =⇒ c2 = −5
c1 =2
−−−→ 10 − 5c2 = 10 + c22
We just need to nd one C. Let c3 = 0 for C = (c1 , c2 , c3 ). Then we get the above last two statements if we also choose
c1 = 2.
So we have C = (2, −5, 0)
So in summary, we get

B − A = (1, −3, −5) kA − Ck = 17

A − C = (0, 4, 1) and indeed kC − Bk = 18 and (A − C) · (C − B) = 0, as a right triangle should.

C − B = (−1, −1, 4) kB − Ak = 35
Exercise 15.
A = (1, −1, 2)
=⇒ C = (1, −5, −3)
B = (2, 1, −1)
Exercise 16. Given that
A=P +Q
A = (1, 2)
P k B =⇒ P = sB
B = (3, 4)
Q⊥B
µ ¶
11 −8 6
=⇒ P = (3, 4) , Q=A−P = ,
25 25 25
Exercise 17. Given A = (1, 2, 3, 4),
B = (1, 1, 1, 1), then for A = P + Q, P k B , Q · B = 0,
µ ¶
5 −3 −1 1 3
P = (1, 1, 1, 1) , Q = A − P = , , ,
2 2 2 2 2
Exercise 18. Given
A = (2, −1, 1) D = xB + yC
B = (1, 2, −1) A·D =0
C = (1, 1, −2) kDk = 1
then D = ± √12 B + ∓ √12 C
Exercise 19. kA + Bk2 − kA − Bk2 = A2 + 2A · B + B 2 − (A2 − 2A · B + B 2 ) = 4A · B
Exercise 20. kA + Bk2 + kA − Bk2 = A2 + 2A · B + B 2 + (A2 − 2A · B + B 2 ) = 2A2 + 2B 2
The sum of the square of the sides of a parallelogram is equal to the sum of the square of the diagonals of a paralellogram.

Exercise 21.

“The sum of the squares of the sides of any quadrilateral exceeds the sum of the squares of the diagonals by four times the
square of the length of the line segment which connects the midpoints of the diagonals.”

Sum of the squares of the sides = A2 + (C − A)2 + (C − B)2 + B 2 =


= A2 + C 2 − 2AC + A2 + C 2 − 2CB + B 2 + B 2
sum of the squares of the diagonals = C 2 + (A − B)2 =
= C 2 + A2 − 2AB + B 2
four times the square of the length of the line segment which connects the midpoints of the diagonals =
µ µ ¶¶2
C A+B
=4 − =
2 2
C 2 − 2C(A + B) + A2 + 2AB + B 2
So then if we take the sum of the squares of the sides - sum of the squares of the diagonals =
= four times the square of the length of the line segment which connects the midpoints of the diagonals, then we get the de-
sired equality.

kAk = 6 and for any pair of scalars, x, y ,


Exercise 22. Given that

(xA + yB) · (4yA − 9xB) = 4xyA2 − 9x2 A · B + 4y 2 A · B − 9xyB 2 = 0 =⇒ 144xy + (4y 2 − 9x2 )A · B = 9xyB 2
212
x = 2, y = 3 without loss of generality.
144xy = 9xyB 2 =⇒ 16 = B 2 , kBk = 4
This implies that A · B in order for the above statements to be true ∀ x, y
√ √
k(2A + 3B)k2 = 4A2 + 12A · B + 9B 2 = 144 + 144 = (12 2)2 =⇒ k2A + 3Bk = 12 2
Exercise 23. Given that
B =C +D
A = (1, 2, 3, 4, 5)
D⊥A
B = (1, 1/2, 1/3, 1/4, 1/5)
C k A or C = tA
B · A = 5 = C · A + D · A = tA2 = 55t
1
C= (1, 2, 3, 4, 5)
11
µ ¶
1 10 7 2 −5 −14
D = B − C = (1, 1/2, 1/3, 1/4, 1/5) − 11 (1, 2, 3, 4, 5) = , , , ,
11 22 33 44 55
Exercise 24. Given that
A · B 6= 0
CkA C = tA
since kAkneq0, if t 6= 0, then C exists.
D ⊥ A so that A · D = 0
B =C +D
A·B A·B
A · B = C · A + D · A = tA2 =⇒ t = A2 C= A
A2
A·B
D =B−C =B− A
A2
Exercise 25.

(1) Given that A · B = 0, kA + xBk2 = A2 + 2xA · B + x2 B 2 = A2 + x2 B 2 ≥ A2 since x2 B 2 is positive.


(2) Given that kA + xBk ≥ kAk, then A2 + 2xA · B + x2 B 2 ≥ A2 .
x(2A · B + xB 2 ) ≥ 0
If x>0 If x<0
2
2A · B + xB ≥ 0 2A · B + xB 2 ≤ 0
xB 2 ≥ −2A · B xB 2 ≤ −2A · B
then A · B ≥ 0 then A · B ≤ 0

=⇒ A · B = 0
We relied heavily upon the fact that x was any number, so we could make x as small or as big as we want.

12.11 Exercises - Projections. Angle between vectors in n-space, The unit coordinate vectors.

Exercise 1. Given
A = (1, 2, 3) A · B = 11
B = (1, 2, 2) B·B =9
A·B 11
So the projection of A on B is B= (1, 2, 2)
B·B 9
Exercise 2. Given
A = (4, 3, 2, 1) B·B =4
B = (1, 1, 1, 1) A · B = 10
5
So the projection of A on B is (1, 1, 1, 1)
2
Exercise 3.

6
cos a =
7
3
(1) A = (6, 3, −2). cos b =
7
−2
cos c =
7
213
1
(2)
7 (6, 3, −2)
Exercise 4. Given
A = (1, 2, 1) C = (1, 4, 1)
B = (2, 1, −1) D = (2, 5, 5)
√ √ √ A·B 1 π
= =⇒ θAB =
|A| = 6 |C| = 18 = 3 2 |A||B| 2 6
√ √ √ √
|B| = 6 |D| = 54 = 3 6 C ·D 3 π
= =⇒ θCD =
|C||B| 2 3
Exercise 5. If we let
√ √
A = (2, −1, 1) B − A = x1 = (−1, −2, −6) |x1 | = 41 x1 · x2 = −6 θ12 = −6/( 246)
√ √
B = (1, −3, −5) C − B = x2 = (2, −1, 1) |x2 | = 6 x1 · x3 = −35 θ13 = −35/( 1435)
√ x2 · x3 = 0
C = (3, −4, −4) A − C = x3 = (−1, 3, 5) |x3 | = 35 θ23 = 0
A·B
Exercise 6. Given that kAk = kCk = 5, kBk = 1, kA − B + Ck = kA + B + Ck, and |A||B| = cos π8 , then consider
(A − B + C) · (A − B + C) = (A + B + C) · (A + B + C)
((A + C) − B)2 = (A + C + B)2 =⇒ −2(A + C) · B = 2(A + C) · B
0 = A · B + B · C =⇒ −A · B = B · C
−5(1) cos π8 9π
=⇒ = cos θBC =⇒ θBC =
(1)5 8
A·C B·C A·C B·C
Exercise 7. Given that = cos θAC = |B||C|
|A||C| = cos θBC , then |A| = |B| or |B|A · C = |A|B · C .
So C · (|B|A − |A|B) = |B|(A · C) − |A|B · C = 0

Exercise 8. Given that


|A|2 = n
n
X
A = (1, 1, . . . , 1) |B|2 = j2
then j=1
B = (1, 2, . . . , n) n
X n(n + 1)
A·B = j=
j=1
2
Pn 2
To nd j=1 j , recall this trick:
n
X n−1
X n−1
X
S= j3 = (j + 1)3 = j 3 + 3j 2 + 3j + 1 =
j=1 j=0 j=0
n−1
X µ ¶
(n − 1)n
= S − n3 + 3 j2 + 3 +n
j=1
2
n
X (n + 1)(2n + 1)n
=⇒ j2 =
j=1
6

So now we could get a remarkable limit:

n(n+1) √ √
A·B 2 3 1 + 1/n n→∞ 3
cos θAB = =√ q = p −−−−→ or θAB = π/3
|A||B| (n+1)(2n+1)n 2 (1 + 1/n)(1 + 1/2n) 2
n 6

Exercise 9. Given that


n
X
|A|2 = (2j)2
j=1
Xn
A = (2, 4, 6, . . . , 2n)
then |B|2 = (2j − 1)2
B = (1, 3, 5, . . . , 2n − 1) j=1
Xn
A·B = 2j(2j − 1)
j=1
214
Doing the math,

X n
X 4(n + 1)(2n + 1)n
|A|2 = 4j 2 = 4 j2 =
j=1
6
n
X n
X (n + 1)(2n + 1)n n(n + 1)
|B|2 = (2j − 1)2 = (4j 2 − 4j + 1) = 4 −4 +n=
j=1 j=1
6 2
2
(4n + 6n + 2 − 6n − 6 + 3) n(4n2 − 1)
=n =
3 3
Xn Xn
(n + 1)(2n + 1)n n(n + 1)
A·B = 2j(2j − 1) = 4j 2 − 2j = 4 −2 =
j=1 j=1
6 2
(2(2n2 + 3n + 1) − 3n − 3)n (4n2 + 3n − 1)n
= =
3 3
A·B n(4n2 + 3n − 1)/3 4 + 3/n − 1/n2 n→∞ 4
=r ³ ´ =p −−−−→ p = 1
|A||B| 2(2n2 +3n+1)n n(4n2 −1)
2
2(2 + 3/n + 1/n)(4 − 1/n ) 2(2)4
3 3

n→∞
It seems to make sense that θAB −−−−→ 0, as the two vector getting closer and closer.

Exercise 10.

A = (cθ, −sθ) c2 + s2 = 1
(1)
B = (sθ, cθ) A · B = cs − cs = 0
(2) (x, y) ∈ V2 ,
x = xC + yS x(1 − C) = yS
(x, y) = xA+yB = x(C, −S)+y(S, C) = (xC+yS, −xS+yC) =⇒ =⇒
y = −xS + yC y(1 − C) = −xS
If θ 6= 2πj and y 6= 0, then
xy(1 − C) = y 2 S
x(−xS) = y 2 S
−x2 S = y 2 S
If θ 6= πj , then contradiction. If θ = (2j − 1)π , 2x = 0, y = 0. If θ = 2jπ , then ∀ x, y ∈ R

Exercise 11. For a rhombus, with adjacent sides A, B , |A| = |B|. The diagonals are given by A + B and A − B .
(A + B) · (A − B) = A2 − B 2 = 0

Exercise 12. (cos a, sin a) · (cos b, sin b) = cos a cos b + sin a sin b. But (cos a, sin a) · (cos b, sin b) = cos a − b/(1)(1)
because the angle between the two vectors is a − b.

Exercise 13.

kA − Bk2 = (A − B) · (A − B) = A2 + B 2 − 2kAkkBk cos theta

Pn
Exercise 14. Given A·B = k=1 |ak bk |,

n
X n
X
B·A= |bj aj | = |aj bj | = A · B
j=1 j=1
n
X n
X n
X
A · (cB) = |aj cbj | = |caj bj | = (cA) · B = |c| |aj bj | = |c|(A · B)
j=1 j=1 j=1
n
X
A·A= |aj |2 ≥ 0, A · A = 0 only if A = 0
j=1
215
To prove the Cauchy-Schwarz inequality for this metric, recall the proof for Thm. 1.41.

n
X n
X n
X n
X n
X
(|aj |x + |bj |)2 = |aj |2 x2 + 2|aj ||bj |x + |bj |2 = x2 |aj |2 + 2x |aj bj | + |bj |2 ≥ 0
j=1 j=1 j=1 j=1 j=1
n
X
A0 = |aj |2 = A · A
j=1
Xn µ ¶2
0 B0 A0 C 0 − B 02
x2 A0 + 2xB 0 + C 0 ≥ 0 where B = |aj bj | = A · B =⇒ A0 x + 0 + ≥0
j=1
A A0
n
X
C0 = |bj |2 = B · B
j=1
−B 0
Let x=
A0
n
X Xn n
X
=⇒ A0 C 0 ≥ B 02 or |aj |2 |b2j | ≥ ( |aj bj |)2
j=1 j=1 j=1

Exercise 15. Given A · B = 2a1 b1 + a2 b2 + a1 b2 + a2 b1


B · A = 2b1 a1 + b2 a2 + b1 a2 + b2 a1 = A · B = 2a1 b1 + a2 b2 + a1 b2 + a2 b1
A · B + A · C = 2a1 b1 + a2 b2 + a1 b2 + a2 b1 + 2a1 c1 + a2 c2 + a1 c2 + a2 c1 =
= 2a1 (b1 + c1 ) + a2 (b2 + c2 ) + a1 (b2 + c2 ) + a2 (b1 + c1 ) = A · (B + C)
(cA) · B = 2(ca1 )b1 + ca2 b2 + ca1 b2 + ca2 b1 = 2a1 (cb1 ) + a2 (cb2 ) + a1 (cb2 ) + a2 (cb1 ) =
= c(2a1 b1 + a2 b2 + a1 b2 + a2 b1 ) = A · (cB) = c(A · B)
a1 = 0
A · A = 2a21 + a22 + a1 a2 + a2 a1 = a21 + (a1 + a2 )2 ≥ 0, A · A = 0 only if =⇒ A = 0
a1 + a2 = 0
For the Cauchy-Schwarz inequality,

A2 B 2 = (a21 + (a1 + a2 )2 )(b21 + (b1 + b2 )2 )


(A · B)2 = (a1 b1 + a2 (b1 + b2 ) + a1 (b1 + b2 ))2 = (a1 b1 + (a1 + a2 )(b1 + b2 ))2 =
= (a1 b1 )2 + 2a1 b1 (a1 + a2 )(b1 + b2 ) + (a1 + a2 )2 (b1 + b2 )2

Now (a1 (b1 + b2 ) − b1 (a1 + a2 ))2 ≥ 0


a21 (b1 + b2 )2 − 2a1 b1 (b1 + b2 )(a1 + a2 ) + b21 (a1 + a2 )2 ≥ 0
This fact shows that A2 B 2 ≥ (A · B)2

Exercise 16. Given A · B = 2a1 b1 + a2 b2 + a3 b3 + a1 b3 + a3 b1


B · A = 2b1 a1 + b2 a2 + b3 a3 + b1 a3 + b3 a1 = A · B = 2a1 b1 + a2 b2 + a3 b3 + a3 b1 + a1 b3
A · B + A · C = 2a1 b1 + a2 b2 + a3 b3 + a1 b3 + a3 b1 + 2a1 c1 + a2 c2 + a3 c3 + a1 c3 + a3 c1 =
= 2a1 (b1 + c1 ) + a2 (b2 + c2 ) + a3 (b3 + c3 ) + a1 (b3 + c3 ) + a3 (b1 + c1 ) = A · (B + C)
(cA) · B = 2(ca1 )b1 + (ca2 )b2 + (ca3 )b3 + (ca1 )b3 + (ca3 )b1 = 2a1 (cb1 ) + a2 (cb2 ) + a3 (cb3 ) + a1 (cb3 ) + a3 (cb1 ) =
= c(2a1 b1 + a2 b2 + a3 b3 + a1 b3 + a3 b1 ) = A · (cB) = c(A · B)
a1 = 0
A·A= 2a21 + a22 + a23 + a1 a3 + a3 a1 = a21 + a22 + (a1 + a3 ) ≥ 0,2
A · A = 0 only if a2 = 0 =⇒ A = 0
a1 + a3 = 0
For the Cauchy-Schwarz inequality,

A2 B 2 = (a21 + a22 + (a1 + a3 )2 )(b21 + b22 + (b1 + b3 )2 )


(A · B)2 = (a1 b1 + a2 b2 + a3 (b1 + b3 ) + a1 (b1 + b3 ))2 = (a1 b1 + a2 b2 + (a1 + a3 )(b1 + b3 ))2 =
= (a1 b1 )2 + a22 b22 + 2a1 b1 a2 b2 + 2(a1 b1 + a2 b2 )(a1 + a3 )(b1 + b3 ) + (a1 + a3 )2 (b1 + b3 )2
216
As you could see, a direct proof has messy algebra. Instead, since this dot product satises all the usual dot product properties,
then we can just reuse the vector method proof of the Cauchy-Schwarz inequality, without reference to a specic coordinate
system, to conclude that the Cauchy-Schwarz inequality holds. That's the trick - think in terms of generality, of vectors in
general, and not specic coordinate systems.
Pn
Exercise 17. Given that kAk = j=1 |aj |
(1)
X X
|aj | > 0, |aj | = 0 if aj = 0 ∀ j
X X
kcAk = |caj | = |c| |aj | = |c|kAk
X X
kA + Bk = |aj + bj | < |aj | + |bj | = kAk + kBk
(2) See sketch. Looks
¯ like a¯diamond.
¯Pn ¯
(3) Given kAk = ¯ j=1 aj ¯

kAk > 0 if A 6= 0 but kAk = 0 for aj = (−1)j and n = 2m


¯ ¯ ¯ ¯
¯X ¯ ¯X ¯
¯ n ¯ ¯ n ¯
kcAk = ¯ ¯ ¯ ¯
caj ¯ = |c| ¯ aj ¯¯ = |c|kAk
¯ j=1 ¯ ¯ j=1 ¯
¯ ¯ ¯ ¯ ¯ ¯
¯ n ¯ ¯ n ¯ ¯ n ¯
¯X ¯ ¯X ¯ ¯X ¯
kA + Bk = ¯¯ (aj + bj )¯¯ ≤ ¯¯ aj ¯¯ + ¯¯ bj ¯¯ = kAk + kBk
¯ j=1 ¯ ¯ j=1 ¯ ¯ j=1 ¯

Exercise 18. Given that kAk = max1≤j≤n |aj |


(1) kAk ≥ 0 since |aj | ≥ 0 for any aj ∈ R
If A 6= 0 then ∃ at least one aj s.t. aj 6= 0
|aj | > 0, so kAk = 0 only if A = 0.

kcAk = max1≤j≤n |caj | = max1≤j≤n |c||aj |

Consider aJ = max1≤j≤n |aj |


|aJ | ≥ |aj | ∀ j 6= J
So |c||aJ | ≥ |c||aj | is valid. |c||aj | = |caj |. Then
max1≤j≤n |c||aj | = |c| max1≤j≤n |aj |

kA + Bk = max1≤j≤n |aj + bj | ≤ max1≤j≤n |aj | + |bj | ≤ max1≤j≤n |aj | + max1≤k≤n |bk | = kAk + kBk

(2) See sketch. Looks like a square.


Pn qP
n 2
Exercise 19. Given A = (a1 , . . . , an ), kAk1 = j=1 |aj |, kAk2 = max1≤j≤n |aj |, kAk = j=1 (aj ) .

kAk2 = max1≤j≤n |aj | = |aJ |.


Let
Pn
kAk = j=1 a2j ,
2
kAk2 sum obvious includes aJ term, and all a2j terms are positive.

=⇒ kAk2 ≥ kAk22 or kAk ≥ kAk2


³P ´2 Pn
n 2
j=1 |aj | ≥ j=1 (aj ) , because of the cross terms, which are positive from the absolute value function on each
individual term.

So then kAk1 ≥ kAk

Note that we had used the fact that if b2 > a2 and a, b > 0, then b > a. This is because
2 2
If a = b, then a = b . Contradiction.
2 2
If a < b, then a · a < a · b < b , from ab < b . Contradiction.

kAk2 ≤ kAk ≤ kAk1 means that 2-norm will “norm” or “expand” a given coordinate pair the least, assign to it the
smallest norm, so coordinates of bigger values are needed for 2-norm. In contrast, the 1-norm assigns bigger values for the
norm of a given pair of coordinates. The 1-norm requires smaller values for the coordinates to get the same norm value as the
2-norm and usual spherical norm.
217
Notice how going from 2 norm, kAk, and 1 norm, we go from a square, to a circle, and nally to a diamond.
Exercise 20. Given that d(A, B) = kA − Bk,

d(B, A) = kB − Ak = k(−1)(A − B)k = | − 1|kA − Bk = kA − Bk


d(A.B) = kA − Bk = 0 only if A − B = 0 =⇒ A = B
d(A, B) = kA − Bk = kA − C + C − Bk ≤ kA − Ck + kC − Bk = d(A, C) + d(C, B)

12.15 Exercises - The linear span of a nite set of vectors, Linear independence, Bases. Exercise 1. x(i − j) + y(i + j)
(1) i; x = 1/2, y = 1/2

(2) j ; x = −1/2, y = 1/2

(3) 3i − 5j ; x = 4, y = −1
(4) 7i + 5j ; x = 1, y = 6
Exercise 2.
A = (1, 2)
B = (2, −4) C = xA + yB =⇒ x = 1/4, y = 7/8
C = (2, −3)
Exercise 3.
A = (2, −1, 1)
B = (1, 2, −1) C = xA + yB
C = (2, −11, 7)
 ¯   ¯ 
2 1 ¯¯ 2 0 1 ¯¯−4
−1 2 ¯¯−11 =⇒ −1 0 ¯¯−3
1 −1 ¯ 7 0 1 ¯−4
x=3
=⇒
y = −4

Exercise 4. If C = (2, 11, 7),  ¯   ¯ 


2 1 ¯¯ 2 0 5 ¯¯24
−1 2 ¯¯11 = −1 2 ¯¯11
1 −1 ¯ 7 0 3 ¯29
Exercise 5.

(1) A k B =⇒ A = tB , t 6= 0
A − tB = 0 and t 6= 0. So A, B linearly dependent.
(2) A∦B
A − tB 6= 0 since @ t ∈ R s.t. A = tB . Then A, B linearly independent.
Exercise 6. Given (a, b), (c, d), the following statements must be true.
· ¸· ¸ · ¸
a c x 0
=
b d y 0
µ ¶· ¸· ¸· ¸ · ¸ · ¸
1 d −c a c x x 0
= =
ad − bc −b a b d y y 0
Ifad − bc 6= 0, then we could apply this inverse matrix to obtain x = y = 0.
¡ ad bc ¢
Suppose x = y = 0 and ad − bc = 0. Then ad = bc and we could rewrite (c, d) to be
b , a . Then
−d
b (a, b)
+ (c, d) = 0. But we had said that x = y = 0 were the only coefcients. Then ad − bc 6= 0
Exercise 7. Given (1 + t, 1 − t), (1 − t, 1 + t), and from above,

(1 + t)(1 + t) − (1 − t)(1 − t) = ad − bc = 4t 6= 0. So for t 6= 0, the two vectors are linearly independent.


Exercise 8. i, j, k are linearly independent. i + j + k = L({i, j, k}). By thm, {i, j, k, i + j + k} are linearly dependent

(Apostol's Thm. 12.8, any set of k + 1 vectors is dependent, S independent, |S| = k ).


i, j, k are linearly independent and pairwise linearly independent. i + j + k is not in the span of any pair from i, j, k
because i + j + k is a linear combination of one unit coordinate vector not included in the pair. Then the 3 vectors, including
i + j + k , are linearly independent (if i + j + k is not in the span of two other independent vectors, then there's no linear
combination of the three that would give a nontrivial representation of 0).
218
Exercise 9.

(1) c1 i + c2 (i + j) = 0 c1 = −c2 c2 = 0 Then c1 = 0


(2) i + j − i = j ∈ L(S)
(3) 3i − 4j = −4(i + j) + 5i
(4) i, i + j linearly independent |S| = 2 by Thm. 12.10, since we have 2 linearly independent vectors in V2 , S forms a
basis for V2 .

Exercise 10. Given A = i, B = i + j, C = i + j + 3k in V3


(1) xA + yB + zC = 0. Then z = 0. Then y = 0. x = 0. A, B, C linearly independent.
(2) j = B − A k = C−B 3
−14B + 5C
(3) 2i − 3j + 5k = + 5A
3
(4) {A, B, C} are linearly independent. Since |{A, B, C}| = 3 = n, for Vn = V3 , then by Thm. 12.10, {A, B, C} form
a basis for V3 .

A = (1, 2), B = (2, −4), C = (2, −3), D = (1, −2).


Exercise 11. Let

Note thatB = 2D (B, D are parallel).


A, B, C, D are all each linearly independent, by themselves.
{A, D}, {A, B}, {A, C}, {B, C}, {D, C}
Exercise 12.
A = (1, 1, 1, 0)
Let B = (0, 1, 1, 1)
C = (1, 1, 0, 0)
(1) A, B, C are linearly independent since for xA + yB + zC = 0, y = 0, x = 0 and so z = 0.
(2) −(0, 1, 2, 1) = D would make the set dependent.
(3) E = (1, 0, 0, 0)
(4) (1, 2, 3, 4) = 4B + −A − C + 3E

Exercise 13.

y = − 3z
√ √ √ √ −y
√ y
(1) x( 3, 1, 0) + y(1, 3, 1) + z(0, 1, 3) = 0. Then x 3 = −y =⇒ √
3
+ 3y + −√ 3
=0 So y = 0.

x + 3y + z = 0
Then
√ x = z =
√ 0 √ √
(2) ( 2, 1, 0) − 2(1, 2, 1) = (0, −1, − 2). Dependent.
(3) (t, 1, 0) + −t(1, t, 1) = (0, 1, t) =⇒ 1 − t2 = 1 so that t = 0 .
Exercise 14.
(1, 0, 1, 0)
(1) (1, 1, 1, 1)
(2, 0, −1, 0)
Note that we cannot get any bigger set, since (0, 1, 0, 1) is a linear combination of (1, 0, 1, 0) and (1, 1, 1, 1)

(2) (1, 1, 1, 1), (1, −1, 1, 1) are immediately linearly independent, since they're not parallel to each other.

x(1, 1, 1, 1) + y(1, −1, 1, 1) = (x + y, x − y, x + y, x + y), so the 3rd. and 4th. coordinate of any linear combi-
nation of these two vectors will be equal.

(1, 1, 1, 1)
Thus (1, −1, 1, 1) are linearly independent.

(1, −1, −1, 1)


x(1, 1, 1, 1)+
y(1, −1, 1, 1)+ = (x + y + z, x − y − z, x + y − z, x + y + z)
z(1, −1, −1, 1)
So any linear combination of the these three vectors, will always have the 1st. and 4th. coordinates equal. Then

(1, 1, 1, 1), (1, −1, 1, 1), (1, −1, −1, 1), (1, −1. − 1. − 1) are linearly independent.
219
(1, 1, 1, 1)
(0, 1, 1, 1)
(3) are linearly independent.
(0, 0, 1, 1)
(0, 0, 0, 1)

Exercise 15.

(1) x(A + B) + y(B + C) + z(A + C) = (x + y + z)A + (x + y)B + (y + z)C = 0


=⇒ x = −z = y = −x since A, B, C are linearly independent. Then x = y = z = 0.
x=y
(2) x(A − B) + y(B + C) + z(A + C) = (x + z)A + (−x + y)B + (y + z)C = 0 So we have y = −z
x = −z

If x, y 6= 0, z 6= 0 and condition for linear independence of A, B, C is still satised. Thus, A − B, B + C, A + C


lienarly dependent.

Exercise 16.

(1)
If S = BV3 ,
S = BV3 means ∀ x ∈ V3 , x ∈ L(S). Since i, j, k ∈ V3 , i, j, k ∈ L(S)

If i, j, k ∈ L(S), then L({i, j, k}) ⊆ L(S) (since every linear combination of i, j, k can be reexpressed as a linear
combination of vectors in S , since each i, j, k is a linear combination of vectors in S ).
Then since ∀ x ∈ V3 , x ∈ L({i, j, k}), x ∈ L(S)
So S forms a basis for V3 .
(2)
If S = BVn ,
S = BVn means ∀ x ∈ Vn , x ∈ L(S). Since e1 , . . . , en ∈ Vn , e1 , . . . , en ∈ L(S)

If e1 , . . . , en ∈ L(S), then L({e1 , . . . , en }) ⊆ L(S) (since every linear combination of e1 , . . . , en can be reex-
pressed as a linear combination of vectors in S , since each e1 , . . . , en is a linear combination of vectors in S ).
Then since ∀ x ∈ Vn , x ∈ L({e1 , . . . , en }), x ∈ L(S)
So S forms a basis for Vn .

Exercise 17. {(0, 1, 1), (1, 1, 1), (0, 0, 1)} and {(0, 1, 1), (1, 1, 1), (0, 0, −1)}
Exercise 18. {(0, 1, 1, 1), (1, 1, 1, 1), (0, 0, 0, 1), (0, 0, 1, 0)} and {(0, 1, 1, 1), (1, 1, 1, 1), (0, 0, 0, −1), (0, 0, −1, 0)}
Exercise 19. Given that
S = {(1, 1, 1), (0, 1, 2), (1, 0, −1)}
T = {(2, 1, 0), (2, 0, −2)}
U = {(1, 2, 3), (1, 3, 5)}
(1) Consider x(2, 1, 0) + y(2, 0, −2) = (2x + 2y, x, −2y)
    
1 0 1 a 2(x + y)
1 1 0   b  =  x 
1 2 −1 c −2y
 ¯   ¯ 
1 0 1 ¯¯2(x + y) 1 0 1 ¯¯ 2(x + y) a + c = 2(x + y)
1 1 0 ¯¯ x  =  1 −1 ¯ −x − 2y  =⇒
¯ b − c = −x − 2y
1 2 −1 ¯ −2y 2 −2 ¯−2x − 4y

2(x + y)(1, 1, 1) + (−x − 2y)(0, 1, 2)


220
(2)
 ¯     ¯ 
1 0 1 ¯¯a 0 1 0 1 ¯¯a a = −c
1 1 0 ¯¯ b  = 0 =⇒  1 −1 ¯¯ b  =⇒ so S is linearly dependent
b=c
1 2 −1 ¯ c 0 2 −2 ¯ c

3
2(2, 1, 0) − (2, 0, −2) = (1, 2, 3)
2 =⇒ L(U ) = L(T )
5
3(2, 1, 0) − (2, 0, −2) = (1, 3, 5)
2
(1, 1, 1) + (0, 1, 2) = (1, 2, 3)
=⇒ L(S) = L(U ) = L(T )
(1, 1, 1) + 2(0, 1, 2) = (1, 3, 5)

Exercise 20. A, B are nite subsets.


P
(1) Consider xA ∈ L(A)
T , xA = cj AJ . Since ∀ Aj ∈ B , xA ∈ L(B). Then L(A) ⊆ L(B)
(2) Consider cj ∈ A B so that cj ∈ A and cj ∈ B
P P
cjP
Cj ∈ L(A) sinceT ∀ Cj ∈ A and Tcj Cj ∈ L(B)Tsince ∀ Cj ∈ B
=⇒ cj C Tj ∈ L(A) L(B) T =⇒ L(A B) ⊆ L(A) L(B)
(3) Now L(A B) ⊆ L(A) L(B).
A = {(1, 1, 0), (1, 0, 1)}
If
B = {(1, 0, 0), (0, 1, 1)}
T T
then we could see that A B = ∅, but L(A) L(B) is nonempty; for instance, it'll include (2, 1, 1) = (1, 1, 0) +
(1, 0, 1) = 2(1, 0, 0) + (0, 1, 1).

12.16 Exercises - The vector space Vn (C) of n-tuples of complex numbers.


A =(1, i)
Exercise 1. Given B =(i, −i)
C =(2i, 1)

(1) A · B = −i − 1
(2) B · A = A · B = −1 + i
(3) (iA) · B = i(A · B) = 1 − i
(4) A · (iB) = −i(A · B) = −1 + i
(5) (iA) · (iB) = i(−i)A · B = −i − 1
(6) (i, −1) · (−1, 1) = B · C = 2 + −i
(7) A · C = −2i + i = −i
(8) (B + C) · A = (3i, 1 − i) · A = 3i − i − 1 = 2i − 1
(9) (A − C) · B = (1 − 2i, i − 1) · B = (−i − 2, −1 − i) = −3 − 2i
(10) (A − iB) · (A + iB) = (A − (−1, 1)) · ((1, i) + (−1, 1)) = (2, −1 + i) · (0, 1 + i) = 2i

Exercise 2. Given
A = (2, 1, −i)
and V3 (C)
B = (i, −1, 2i)
A · C = (2, 1, −i) · (c1 , c2 , c3 ) = 2c1 + c2 − ic3 = 0 c2 = c1 (−4 + i)
=⇒
B · C = (i, −1, 2i) · (c1 , c2 , c3 ) = ic1 + −c2 + 2ic3 = 0 c3 = c1 (−2i − 1)
C = (1, i − 4, −1 − 2i)
Exercise 3.

kA + Bk2 = (A + B) · (A + B) = (A + B) · A + (A + B) · B = A · (A + B) + B · (A + B) =
= A · A + A · B + B · A + B · B = kAk2 + kBk2 + A · B + B · A =

= kAk2 + kBk2 + A · B + A · B
221
Exercise 4.

kA + Bk2 − kA − Bk2 = (A + B) · (A + B) − (A − B) · (A − B) =
= (A + B) · A + (A + B) · B − (A − B) · A + (A − B) · B =
= A · (A + B) + B · (A + B) − A · (A − B) + B · (A − B) =
=A·A+A·B+B·A+B·B−A·A+A·B+B·A−B·B =

= 2(A · B + A · B)
Exercise 5.

kA + Bk2 + kA − Bk2 = (A + B) · (A + B) + (A − B) · (A − B) =
= (A + B) · A + (A + B) · B + (A − B) · A − (A − B) · B =
= A · (A + B) + B · (A + B) + A · (A − B) − B · (A − B) =
=A·A+A·B+B·A+B·B+A·A−A·B−B·A+B·B =

= 2kAk2 + 2kBk2

Exercise 6. A, B ∈ Vn (C)
(1) (A · B + A · B) = A · B + A · B = A · B + A · B = A · B + A · B So then A · B + A · B must be real.
(2)

Now kA + Bk2 = kAk2 + kBk2 + A · B + A · B, so then

A B 2 A·B+A·B
k + k =1+1+ ≥0
kAk kBk kAkkBk
A·B+A·B
=⇒ ≥ −2
kAkkBk

Now kA − Bk2 = (A − B) · (A − B) = (A − B) · A − (A − B) · B = kAk2 − B · A − A · B + kBk2 ≥ 0

A B 2 (A · B + A · B)
so then k − k =2− ≥0
kAk kBk kAkkBk
A·B+A·B
=⇒ 2 ≥
kAkkBk
1
2 (A·B+A·B)
Exercise 7. With this denition, that θ = arccos kAkkBk ,

We had already showed that


kA − Bk2 = (A − B) · (A − B) = (A − B) · A − (A − B) · B = kAk2 − B · A − A · B + kBk2 ≥ 0
2 2 2 2 2
Thus kA − Bk = kAk + kBk − (A · B + A · B) = kAk + kBk − 2kAkkBk cos θ

Exercise 8. Vn (C)
A = (1, 0, i, i, i) 1
(A·B+A·B)
θ = arccos 2 kAkkBk
B = (i, i, i, 0, i)
A · B = −i + 1 + 1 = 2 − i A · B = 2 + i =⇒ A · B + A · B = 2 − i + 2 + i = 4
kAk2 = (1, 0, i, i, i) · (1, 0, i, i, i) = 1 + 1 + 1 + 1 = 4
kBk2 = (i, i, i, 0, i) · (i, i, i, 0, i) = (1 + 1 + 1 + 1) = 4
1
24
θ = arccos 2·2 = arccos 1/2 =⇒ cos θ = 1/2 or θ = π/3
Exercise 9.
A = (1, 0, 0)
Given that B = (0, i, 0)
C = (1, 1, i)
(1) aA + bB + cC = (x, y, z)
c = z/i = −iz Then b = −i(y + iz) and a = x + iz .

If x = y = z = 0, then c = 0 = b = a. So A, B, C does form a basis.


(2) (5, 2 − i, 2i) = 2C + 3A − B
222
Exercise 10.
n
X
cj ej = (x1 , . . . , xn ) = X
j=1
then the cj 's simply equal the complex coordinates of X cj = xj

If X = 0, then each cj = 0. So the ej 's are linearly independent . =⇒ ej 's form a basis for Vn (C)

13.5 Exercises - Introduction, Lines in n-space, Some simple properties of straight lines, Lines and vector-valued
functions.
X0 = P + tA = P + t(Q − P )
Exercise 1. Recall that if X0 ∈ L(P, A), and P, Q ∈ L(P, A), then .
X0 − P = t(Q − P )
Q − P = (4, 0), P = (−3, 1), Q = (1, 1)
(1) (0, 0) no.
(2) (0, 1) yes.
(3) (1, 2) no
(4) (2, 1) yes
(5) (−2, 1) yes
Exercise 2. P = (2, −1); Q = (−4, 2), Q − P = (−6, 3)
(1) (0, 0) yes
(2) (0, 1) no
(3) (1, 2) no
(4) (2, 1) no
(5) (−2, 1) yes.
Exercise 3. P = (−3, 1, 1) A = (1, −2, 3)
(1) (0, 0, 0) no
(2) (2, −1, 4) no
(3) (−2, −1, 4) yes
(4) (−4, 3, −2) yes
(5) (2, −9, 16) yes
P = (−3, 1, 1)
Exercise 4. and Q − P = (4, 1, 6)
Q = (1, 2, 7)
(1) (−7, 0, 5) no
(2) (−7, 0, −5) yes
(3) (−11, 1, 11) no
(4) (−11, −1, 11) no.
(5) (−1, 3/2, 4) yes.
(6) (−5/3, 4/3, 3) yes.
(7) (−1, 3/2, −4) no.
Exercise 5. P = (2, 1, 1), Q = (4, 1, −1), R = (3, −1, 1)
(1) Q − P = (2, 0, −2) R − P = (1, −2, 0). No.
(2) P = (2, 2, 3), Q = (−2, 3, 1), R = (−6, 4, 1)
Q − P = (−4, 1, −2), R − P = (−8, 2, −2). No.
(3) P = (2, 1, 1); Q = (−2, 3, 1), R = (5, −1, 1)
Q − P = (−4, 2, 0) R − P = (3, −2, 0). No.
Exercise 6.
E = (1, 1, 1)
A = (2, 1, 1) D = (−2, 3, 1)
B − A = (4, −2, 0) F = (−4, 4, 1)
B = (6, −1, 1) Since D − A = (−4, 2, 0)
C − A = (−8, 4, 0) G = (−13, 9, 1)
C = (−6, 5, 1) F − A = (−6, 3, 0)
H = (14, −6, 1)
{A, B, C, D, F }
Exercise 7. P = (1, 1, 1) A = (1, 2, 3)
223
P + tA = Q + uB 1 + t = 2 + 3u
=⇒ u = 1, t=4
(1, 1, 1) + t(1, 2, 3) = (2, 1, 0) + u(3, 8, 13) 1 + 2t = 1 + 8u
(5, 9, 13) is an intersection point.

Exercise 8.

(1) X = P + tA = Q + uB =⇒ P − Q = uB − tA
P + (−a)A ∈ L(P ; A)
P − Q = aA + bB =⇒ P + (−a)A = Q + bB and
Q + bB ∈ L(Q; B)
0
(2) L = {(1, 1, −1) + t(−2, 1, 3)}L = {(3, −4, 1) + t(−1, 5, 2)}
(1, 1, −1) + t(−2, 1, 3) = (3, −4, 1) + u(−1, 5, 2)

u = 8/9
1 + −2t = 3 − u 1 + t = −4 + 5u 3 = −5 + 9u
t = −5/9
But t = −5/9, u = 8/9, doesn't work for the 3rd. coordinate.

Exercise 9. X(t) = P + tA P = (1, 2, 3) A = (1, −2, 2), Q = (3, 3, 1)


(1)

kQ − X(t)k2 = (Q − X) · (Q − X) = Q2 − 2Q · X + X 2 = 19 − 2Q · (P + tA) + P 2 + 2tP · A + t2 A2 =


= 19 − 24 + −2t(−1) + 14 + 2t(3) + t2 9 = 9 + 8t + 9t2
d
(2) X(t0 ) kQ − X(t)k min. dt kQ − X(t)k2 = 18t + 8 = 0 t = −4/9
q q q √
13·5
kQ − X(t)k = 9 + −32 9 + 16
9 = 81−16
9 = 65
9 = 3
(3)
(Q − X(t0 )) · A = Q · A − X · A = Q · A − (P + tA) · A = Q · A − P · A − tA · A =
µ ¶
−4
= −1 − 3 − (9) = 0
9
Exercise 10. Q∈
/ L(P ; A)
(1) f (t) = kQ − X(t)k2 X(t) = P + tA
(Q − X) · (Q − X) = Q2 − 2Q · X + X 2 = Q2 − 2Q · (P + tA) + P 2 + 2tP · A + t2 A2 =
= Q2 − 2Q · P − 2tQ · A + P 2 + 2tP · A + t2 A2
d
dt (kQ− X(t)k2 ) = 2tA2 + 2(P · A − Q · A) = 0 t = (Q−P
A2
)·A

(Q−P )·A 2
(2) (Q − X) · A = Q · A − (P + tA) · A = Q · A − P · A − tA2 = (Q − P ) · A − A2 A =0
Exercise 11. L(P ; A), L(Q, A) in Vn
Consider x1 = P + tA.
X1 = Q + (P − Q) + t1 A
if P − Q = u1 A, then X1 ∈ L(Q, A)

X2 = Q + tA
Likewise, if
X2 = P + (Q − P ) + t2 A
If P − Q = −u2 A, then X2 ∈ L(P ; A)

If P − Q k A, then X1 ∈ L(Q, A) and X2 ∈ L(P, A), so that L(Q, A) = L(P, A)


If P − Q ∦ A, then X1 ∈/ L(Q, A) and X2 ∈
/ L(P ; A), ∀ X1 ∈ L(P, A), ∀ X2 ∈ L(Q, A)
X1 = Q + (P − Q) + t1 A 6= Q + (u1 + t1 )A =⇒ P − Q 6= u1 A so that X1 notinL(Q, A)
Exercise 12. L(P ; A), L(Q; B), A ∦ B

X1 = P + tA X1 − X2 = 0 only if P − Q = u1 B − t1 A
Consider X2 = Q + uB If P − Q = u2 B − t2 A, then (u1 − u2 )B = (t1 − t2 )A
X1 − X2 = P − Q + tA − uB but A∦B so t1 = t2 , u1 = u2
(only one intersection point). Otherwise, if @ u 1 , t1 s.t. P − Q = u1 B − t1 A, then there's no intersection.
224
13.8 Exercises - Planes in Euclidean n-space, Planes and vector-valued functions.
X = P + sA + tB
Exercise 1. Let M = {P + sA + tB}, P = (1, 2, −3), A = (3, 2, 1), B = (1, 0, 4). Remember, if
X − P = sA + tB

(1) (1, 2, 0) X − P = (0, 0, 3); no


(2) (1, 2, 1) X − P = (0, 0, 4); no
(3) (6, 4, 6) X − P = (5, 2, 9) = A + 2B , yes
(4) (6, 6, 6) X − P = (5, 4, 9) no
(5) (6, 6, −5) X − P = (5, 4, −2) = 2A + −B yes.

Exercise 2.
P = (1, 1, −1)
Q − R = (0, 4, 4)
Q = (3, 3, 2) so that
P − R = (−2, 2, 1)
R = (3, −1, −2)
(1) (2, 2, 1/2) X − R = (−1, 3, 5/2) yes
(2) (4, 0, −1/2) X − R = (1, 1, 3/2) yes
(3) (−3, 1, −3) X − R = (−6, 2, −1) yes
(4) (3, 1, 3) X − R = (0, 2, 5) no
(5) (0, 0, 0) X − R = (−3, 1, 2) no

Exercise 3.

x=1+t
(1) (1, 2, 1) + s(0, 1, 0) + t(1, 1, 4) = (x, y, z) =⇒ y = 2 + s + t
z = 1 + 4t
(2) (0, 1, 0) + s(1, 1, 1) + t(1, 0, 4)

Exercise 4. (1, 2, 0) + s(1, 1, 2) + t(−2, 4, 1) or N = (−7, −5, 6) (by nding the cross product of the two spanning vectors),
so that −7x − 5y + 6z = −17.
(1) (0, 0, 0) no.
(2) (1, 2, 0) yes.
(3) (2, −3, −3) yes.

Exercise 5. M = {P + s(Q − P ) + t(R − P )}


(1) P + s(Q − P ) + t(R − P ) = (1 − s − t)P + sQ + tR
1 − s − t = p, for pP , 1 = p + s + t
Since
X = P + s(Q − P ) + t(R − P ) =
(2)
= (1 − s − t)P + sQ + tR
p = 1 − s − t, p + s + t = 1

Exercise 6. ax + by + cz = d
(1) (2, 3, 1) spanned by (3, 2, 1) and (−1, −2, −3),
X = P + sA + tB
        x = 2 + 3s − t
x 2 3 −1
y  = 3 + s 2 + t −2 =⇒ y = 3 + 2s − 2t
z 1 1 −3 z = 1 + s − 3t

=⇒ x − 2y + z = −3

P = (−2, −1, −3)


(2) If Q − P = (4, 4, 4) (x, y, z) = (−2, −1, −3) + s(1, 1, 1) + t(3, 2, 2)
R − P = (6, 4, 4)

x = −2 + s + 3t
y = −1 + s + 2t =⇒ y + 1 = z + 3 or y − z = 2
z = −3 + s + 2t
225
(3) (2, 3, 1) parallel to the plane through the origin, spanned by (2, 0, −2) and (1, 1, 1)
P+ sA+tB
       
x 2 2 1
y  = 3 + s  0  +t 1
z 1 −2 1
x = 2 + 2s + t
=⇒ y = 3 + t =⇒ x + 2y + z = −3
z = 1 − 2s + t
Exercise 7. M 3x − 5y + z = 9
s.t.
(0, −2, −1)
(0, −2, −1)
(1) Out of (−1, −2, 2),
(−1, −2, 2)
(3, 1, −5)
(0, −2, −1) − (2, 1, 8) = (−2, −3, −9)
(2) =⇒ (2, 1, 8) + s(2, 3, 9) + t(1, 1, 2)
(−1, −2, 2) − (2, 1, 8) = (−3, −3, −6)
Exercise 8. Given two planes, M = {P + sA + tB}, M 0 = {Q + sC + tD}, and the specic coordinates,
P = (1, 1, 1) Q = (2, 3, 1)
A = (2, −1, 3) C = (1, 2, 3)
B = (−1, 0, 2) D = (3, 2, 1)
It's easiest to nd the Cartesian coordinate equations for each of the planes, to nd the intersection, and the easiest way to
nd the Cartesian coordinate equations is to nd the normal to each of the planes, using the cross product on A, B and on
C, D. A × B = (−2, −7, −1) and C × D = (−4, 8, −4). Then

M :2x + 7y + z = 10
M 0 : − 4x + 8y + −4z = 12
It's easy to check that these are the correct equations because we could plug in P, Q respectively to check the equation and
plug in A, B and C, D, respectively to check that we get 0 (because adding arbitrary amounts of A, B or C, D, respectively,
should not affect each respective equation).

Thus, solving for both M and M 0 , we get x + 9y = 13. Then two distinct points include (4, 1, −5), (−5, 2, 6)
Exercise 9.

(1) Given a plane that


A = (1, 2, 3)
P = (2, 3, 1), and x − 2y + z = 0, so that (with a Cartesian coordinate equation, we could easily
B = (3, 2, 1)
plug in numbers to nd 3 points that this plane contains)
Q − P = (1, 1, 1)
P = (2, 1, 0) R − P = (−3, −2, −1)
Q = (3, 2, 1)       So since these two planes have L(A, B) = L(Q − P, R −
2 1 −3
R = (−1, −1, −1) X = 1 + s 1 + t −2
0 1 −1
P ) (since (1, 2, 3) + (3, 2, 1) = 4(1, 1, 1) and (3, 2, 1) = −(−3, −2, −1), then the two planes are parallel.)

(2) Given that


M 0 : x − 2y + z =0
then to nd the intersection, solve for the two equations to get 2x + 2z = 0 =⇒ x = −z .
M : x + 2y + z =0
0
T
Then two points in the intersection, M M 00 are (1, 0, −1), (−1, 0, 1)
Exercise 10.    
1 2
L = 1 + s −1
1 3
     
1 2 0
M =  1  + t 1 + u 1
−2 3 1
226
Condition for intersection:
         
1 2 1 2 0
1 + s −1 =  1  + t 1 + u 1
1 3 −2 3 1
Then

         
0 2 2 0 2 −2 0 s
 0  = s −1 − t 1 − u 1 = −1 −1 −1  t 
−3 3 3 1 3 −3 −1 u
−3
 ¯   ¯  s=  
2 −2 0 ¯¯ 0 2 0 0¯¯ −3 2 −2
−1 −1 −1¯ 0  = 0 −2 0¯ 3  =⇒ 3 =⇒ X =  5/2 
¯ ¯ t=
3 −3 −1¯ −3 0 0 1¯ 3 −2 −7/2
u=3

Exercise 11. L = {(1, 1, 1) + t(2, −1, 3)}


(1) M = {(1, 1, −2) + s(2, 1, 3) + t(3/4, 1, 1)}
2 = 2s + 3/4t
Suppose (2, −1, 3) = s(2, 1, 3) + t(3/4, 1, 1). Then . Doing the algebra, we get t = −16/5, s =
−1=s+t
11/5. But the third coordinate doesn't work, 33/5 − 16/5 = 17/5. So M, L are not parallel since (2, −1, 3) ∈
/
L({(2, 1, 3), (3/4, 1, 1)})

C = (2, 4, 4)
(2) (1, 1 − 2), (3, 5, 2), (2, 4, −1). sC + tD = (2, −1, 3). Doing the algebra, we get t = −5, s = 7/2
D = (1, 3, 1)
from the rst two coordinates, but it doesn't agree with the third coordinates, so that 7/2C + −5D 6= (2, −1, 3). No,
not parallel.
(3) From, x + 2y + 3z = −3, we get the spanning vectors easily by plugging in numbers that'll make the LHS equal to
zero: (−3, 0, 1), (1, 1, −1). Again, we nd that (2, −1, 3) 6= (−1)(1, 1, −1) + (−1)(−3, 0, 1). No, not parallel.

Exercise 12.P, Q ∈ M .
Consider M = {P + SA + tB}. Q ∈ M , so that P + sA + tB = Q. =⇒ Q − P = sA + tB .
Now c1 (sA + tB) + c2 A = (c1 s + c2 )A + c1 tB = 0
A, B linearly independent, so c1 = 0, then c2 = 0.
So sA + tB = Q − P , A are linearly independent, and Q − P = sA + tB, A span the same space as A, B , so by theorem,
M = {P + s(Q − P ) + tA}

For X ∈ L, X = P + s(Q − P ) = P + s(Q − P ) + 0A so L ⊆ M .


Exercise 13. For L containing (1, 2, 3) and k to (1, 1, 1), L = {P + sA}.
Consider M = {P + sA + tB}, containing Q. Then Q = (2, 3, 5) = P + sA + tB . If we let s = t = 1, then

B = (2, 3, 5) − (1, 2, 3) − (1, 1, 1) = (0, 0, 1)

(0, 0, 1) (1, 1, 1) and so a possible M is M = (1, 2, 3) + s(1, 1, 1) + t(0, 0, 1)


is linearly independent of or −x + y = 1.
Exercise 14. L = {Q + sA} We have a plane that we want: M = {Q + sA + t(P − Q)}.
Consider M 0 s.t. L ⊆ M 0 and P ∈ M 0 , M 0 = {R + sB + tC}.
Note that P =
6 Q + sA ∀ s ∈ R

In general, X = Q + sA = (R + b0 B + c0 C) + s(b1 B + c1 C) so that


Q = R + b0 B + c0 C, b0 , c0 6= 0
P = R + b2 B + c2 C , b2 , c2 6= 0, since P ∈ M 0 .
A = (b1 B + c1 C), b1 , c1 6= 0
So then Q − P = (b0 − b2 )B + (c0 − c2 )C = b3 B + c3 C and

b3 c1
b3 B + c3 C = b3 (A − c1 C)/b1 + c3 C = A + (c3 − )C = Q − P
b1 b1
Q − P − bb31 A
C= c1
(c3 − b1 )
227
X = R + sB + tC
= Q − b0 B − c0 C + sB + tC =
= Q + (s − b0 )B + (t − c0 )C =
(s − b0 ) −(s − b0 )c3 + b3 (t − c0 )
= Q + (s − b0 )((Q − P ) − c3 C)/b3 + (t − c0 )C = Q + (Q − P ) + C=
b3 b3
b
(s − b0 ) −(s − b0 )c3 + b3 (t − c0 ) Q − P − b31 A
=Q+ (Q − P ) +
b3 b3 (c3 − cb11 )
=⇒ M 0 ⊆ M
Suppose if c1 = 0 (without loss of generality)
A = b1 B(B =⇒ A)(without loss of generality)
P − Q = (b2 − b0 )A + (c2 − c0 )C since (P 6= Q + sA, so (c2 − c0 ) 6= 0)

R + sB + tC = Q − b0 A − c0 C + sA + tC =
µ ¶
P − Q − (b2 − b0 )A
= Q + (s − b0 )A + (t − c0 ) =
c2 − c0
µ ¶ µ ¶
(b0 − b2 )(t − c0 ) t − c0
= Q + (s − b0 ) + A+ (P − Q) ∈ M
c2 − c0 c2 − c0
=⇒ M 0 ⊆ M
Q + sA + t(P − Q).
Q = R + b0 B + c0 C
If P = R + b2 B + c2 C and
A = (b1 B + c1 C)

Q + sA + t(P − Q) = R + b0 B + c0 C + s(b1 B + c1 C) + t(b2 B + c2 C − b0 B − c0 C) =


= R + (b0 + sb1 + t(b2 − b0 ))B + (c0 + sc1 + t(c2 − c0 ))C
=⇒ M ⊆ M 0
So then M = M 0.

13.11 Exercises - The cross product, The cross product expressed as a determinant.
A = (−1, 0, 2)
Exercise 1. Given B = (2, 1, −1)
C = (1, 2, 2)
¯ ¯
¯ e1 e2 e3 ¯
¯ ¯
(1) A × B = ¯¯−1 0 2 ¯¯ = (−2, 3, −1)
¯2 1 −1¯
(2) B × C = (4, −5, 3)
(3) C × A = (4, −4, 2)
(4) A × (C × A) = (8, 10, 4)
(5) (A × B) × C = (8, 3, −7)
(6) A × (B × C) = (10, 11, 5)
(7) (A × C) × B = (−2, −8, −12)
(8) (A+B)×(A−C) = (A+B)×A−(A+B)×C = B ×A−A×C −B ×C = (2, −3, 1)+(4, −4, 2)−(4, −5, 3) =
(2, −2, 0)
(9) (A × B) × (A × C) = (−2, 0, 4)
Exercise 2.

A = (1, 1, 1) 1
(1) A × B = (−4, 3, 1) =⇒ √ (−4, 3, 1)
B = (2, 3, −1) 26
A = (2, −3, 4) ±1
(2) A × B = (−41, −18, 7) =⇒ √ (−41, −18, 7)
B = (−1, 5, 7) 2054
228
A = (1, −2, 3) ±(1, 2, 1)
(3) A × B = (−4, −8, −4) =⇒ √
B = (−3, 2, −1) 6
Exercise 3.
A = (0, 2, 2)
(1) B = (2, 0, −1) C − A × B − A = (3, 2, −2) × (2, −2, −3) = (−10, 5, −10)
C = (3, 4, 0)
kC − A × B − Ak = 15/2
A = (−2, 3, 1)
(2) B = (1, −3, 4) C − A × B − a = (3, −1, 0) × (3, −6, 3) = (−3, −9, −15)
C = (1, 2, 1)
√ √
C −A×B−A= 315 = 3 35/2
A = (0, 0, 0) √
3
(3) B = (0, 1, 1) kC − A × B − Ak =
2
C = (1, 0, 1)
A = (2, 5, 3)
Exercise 4. B = (2, 7, 4) (A − C) × (B − A) = (−1, 2, −3) × (0, 2, 1) = (8, 1, −2)
C = (3, 3, 6)
Exercise 5. kA × Bk2 = kAk2 kBk2 − (A · B)2
If kA × Bk = kAkkBk, A · B = 0 so cos θ = 0 or θ = π/2.

If A · B = 0, then kA × Bk = kAkkBk
Exercise 6. Given A, B ∈ V3
C = (B × A) − B
(1) A · (B + C) = A · B + A · (B × A) − A · B = 0
(2)
B · C = B · (B × A) − B · B = −B 2
C · C = ((B × A) − B) · ((B × A) − B) = (B × A)2 + B 2
B·C −B 2 −B
= cos θBC = =p =
kBkkCk B((B × A)2 + B 2 )1/2 (BA sin θAB )2 + B 2
−1
=p
A sin2 θAB + 1
2

Socos θBC , since A2 sin2 θAB ≥ 0, ranges from −1 (if sin θAB = 0), to 0 (if kAk → ∞). So then θBC ranges from
π/2 to π . √
(3) kBk = 1, kB × Ak = 2. C 2 = 4 + 1 = 5 =⇒ C = 5
Exercise 7.

(1) (A × B)2 = A2 B 2 − A · B = 1. A, B, A × B normal.


A · (A × B) = B · (A × B) = 0 orthonormal.
c1 A + c2 B + c3 (A × B) = 0
A
−→ c1 A · A = 0 c1 = 0
B
→ c2 B 2 = 0
− c2 = 0 A, B, A × B linearly independent. By Thm., A, B, A × B form a basis

(A×B)
−−−−→ c3 (A × B)2 = 0 c3 = 0
for V3 .
(2) C = (A × B) × A
p
k((A × B) × A)k = kA × Bk2 kAk2 − ((A × B) · A)2 = 1
A · B = Aj BJ = 0 and
(3) Remember that
A · A = Aj AJ = B · B = Bj BJ = 1
229
(A × (A × B))j = ²jkl Ak (A × B)l = ²jkl Ak ²lmn Am Bn = −²jkl ²nml Ak Am Bn =
= (notice how the l index is repeated) = −(−Ak Aj Bk ) + −(Ak Ak Bj ) = 0 − Bj
=⇒ (A × (A × B)) = −B
Now A, B are interchangeable labels (the choice of name or label is not special or unique at all), so that we could
reuse what we had just proven

(A × B) × A = B =⇒ (B × A) × B = A
=⇒ −(A × B) × B = A or (A × B) × B = −A
Exercise 8.
A · B = kAkkBk cos θAB
(1) We know that Certainly, if A×B is 0, then its magnitude is zero. Suppose kAk, kBk 6=
kA × Bk = kAkkBk sin θAB
0 and so sin θAB = 0. But then A · B = 0 and so cos θAB = 0. But cos and sin must obey 1 = C 2 + S 2 . So then at
least one of A or B must be zero.
(2) Given that A 6= 0
A×B =A×C A × (B − C) = 0
and , then so then B − C = 0 or B = C
A·B =A·C A · (B − C) = 0
A = (2, −1, 2)
Exercise 9. Given that ,
C = (3, 4, −1)
(1) what I did was this: A × C = (−7, 8, 11) = Bp
B must have some of this A × C part to twist A into C , because C is orthogonal to both A, B : (B must not have any
part of C ).
If B has any part of A, A × cA = 0 anyways.
µ ¶
Bp Bp 7 −8 −11
A× =C = , ,
−9 −9 9 9 9
Yes, there's more than one solution, since we could attach scalar multiples of A to Bp / − 9.
Bp 14+8−22
(2) A· −9 = 9 =0
1 Bp
A · cA = 1 =⇒ c = 9 So then the desired vector is −9 + 19 A. Yes, there's only one vector.

A 6== 0 and C · A = 0. Suppose that for B , there's A × B = C and A · B = 1.


Exercise 10. Given that

∃ B1 s.t. A × B1 = C and A · B1 = 1. Then


Suppose
A × B1 = C = A × B and A · B1 = A · B
. Then B1 − B = 0 or B1 = B , from Exercise 8.
=⇒ A × (B1 − B) = 0 and A · (B1 − B) = 0
Exercise 11. Given
A = (1, 0, 1)
B = (−1, 1, 1)
C = (2, −1, 2)
(1)

A = (1, 0, 1) A = (1, 0, 1) B = (−1, 1, 1)


B − A = (−2, 1, 0) C − A = (1, −1, 1) A − B = −(−2, 1, 0)
C − B = (3, −2, 1) B − C = −(3, −2, 1) C − A = (1, −1, 1)
D − C = D − (2, −1, 2) D − B = D − (−1, 1, 1) D − C = D − (2, −1, 2)
A − D = (1, 0, 1) − D A − D = (1, 0, 1) − D B − D = (−1, 1, 1) − D
D − C = −(B − A) D − B = −(C − A) D − C = −(A − B) = (−2, 1, 0)
A − D = −(C − B) A − D = −(B − C) B − D = −(C − A) = −(1, −1, 1)
D = (4, −2, 2) D = (−2, 2, 0) D = (0, 0, 2)
3 is the total number of possible D's, fourth vertex (imagine permutating a 4 vertex ring).
(2) B − A × C − A = (−2, 1, 0) × (1, −1, 1) = (1, 2, 1)

kB − A × C − Ak 6
=
2 2
230
Exercise 12. Given that A · B = 2; kAk = 1, kBk = 4, and C = 2(A × B) − 3B

A · (B + C) = A · B + A · C = A · B − 3A · B = −4
8
kCk2 = 4(A × B)2 + 9B 2 = 4((1)2 42 ) + 9(16) = 16(3 + 9) = 16 · 12
4

kCk = 8 3

B·C −3B 2 − 3
= =
kBkkCk BC 2

Exercise 13.

(1) c1 (A + B) + c2 (A − B) = (c1 + c2 )A + (c1 − c2 )B = 0


c1 + c2 = 0 c1 − c2 = 0, so then c1 = c2 = 0.
=⇒ A + B, A − B linearly independent.

So then A + B, A − B, A × B must be linearly independent (since A × B ⊥ A + B, A − B ).


c1 (A + B) + c2 (A + (A × B)) + c3 (B + (A × B)) =
(2)
= (c1 + c2 )A + (c1 + c3 )B + (c2 + c3 )(A × B) = 0
c1 + c2 = 0
c1 + c3 = 0 =⇒ c3 = 0 = c2 = c1 .
c2 + c3 = 0
(3) (A + B) × (A − B) = B × A − A × B = 2(B × A). Linearly independent since we already know that A × B is
orthogonal to A and B , and A, B are linearly independent.

Exercise 14.

(1) If A, B, C lie on a line, then C = A + t(B − A).


(B − A) × (C − A) = (B − A) × (t(B − A)) = 0
If(B − A) × (C − A) = 0, then t(B − A) = C − A (otherwise, at least one is zero, and two points have a line
C = A + t(B − A) and so C would be part of a line joining A and B .
through them, and so we'd be done). Then
(2) A 6= B ,
L = {A + t(B − A)}
X = A + t(B − A) = B + s(A − B)
X − A = t(B − A)
=⇒ (X − A) × (X − B) = 0
X − B = s(A − B)

Exercise 15. Given that A ⊥ B , kAk = kBk = 1 and P × B = A − P ,


(1) P · B = (A − P × B) · B = A · B = 0

P 2 = A2 − 2A · (P × B) + (P × B)2 = A2 − 2A · (P × B) + P 2 B 2 = 1 − 2A · (P × B) + P 2
=⇒ A · (P × B) = 1/2
A · (P × B) = A · (A − P ) = A2 − P · A = 1/2 =⇒ P · A = 1/2 = P cos θP A

A × (P × B) = (A · B)P − (P · A)B = −(P · A)B = −A × P =⇒ (P · A)B = (A × P )


1
(P · A)2 B 2 = A2 P 2 sin2 θP A = = P 2 sin2 θP A
4
1 1 1
P 2 S 2 θP A + P 2 C 2 θP A = + = = P 2
4 4 2
1
=⇒ P = √
2
Note that we had used the cab-bac rule above, which we'll prove with tensors in the next set of exercises.
231
(2) P, B, P × B .
P · B = 0, so P, B orthogonal. P, B orthogonal to P × B . By thm., P, B, P × B form a basis (3 orthogonal vectors
in V3 form a basis).
(3)
(B × (B × P ))j = ²jkl Bk (B × P )l = ²jkl Bk ²lmn Bm Pn = ²jkl ²mnl Bk Bm Pn =
= Bk Bj Pk − Bk Bk Pj = (B · P )Bj − (B · B)Pj = −Pj
=⇒ (B × (B × P )) = ((P × B) × B) = P
(4)
A × B = (P × B) × B + P × B = −P + P × B
(P × B) + P = A
A − (A × B)
P =
2
13.14 Exercises - The scalar triple product, Cramer's rule for solving a system of three linear equations.
Exercise 1.

A = (3, 0, 0)
(1) B = (0, 4, 0) =⇒ 96
C = (0, 0, 8)
A = (2, 3, −1)
(2) B = (3, −7, 5) 22 − 3 + 8 = 27
C = (1, −5, 2)
A = (2, 1, 3)
(3) B = (−3, 0, 6) −60 + 21 + 45 = 6
C = (4, 5, −1)
Exercise 2. Given
(0, 1, t)

(1, t, 1) =⇒ t + t(1 − t2 ) = 0 or t = 0, ± 2 . Then, by theorem, since the scalar product is zero, then the vectors are

(t, 1, 0)
linearly dependent.

Exercise 3. Given
(1, 1, 0)
(0, 1, 1) =⇒ 1 + 1 = 2
(1, 0, 1)
Exercise 4. Want A × B = A · (B × e1 )e1 + A · (B × e2 )e2 + A · (B × e3 )e3
Now (A × B)j = ²jkl Ak Bl
A · (B × ej ) = Al (B × ej )l = Al ²lmn Bm (ej )n = Al ²lmj Bm =
= ²jlm Al Bm = ²jkl Ak Bl
Exercise 5. Prove that i × (A × i) + j × (A × j) + k × (A × k) = 2A.
We'll show this in two ways to be instructive.

i × (A × i) =? = −i × (i × A)
¯ ¯ ¯ ¯
¯ e1 e2 ¯
e3 ¯ ¯e 1 e 2 e3 ¯¯
¯ ¯
¯1 0 0 ¯¯ = (0, −a3 , a2 ) ¯1 0 0 ¯¯ = (0, −a2 , −a3 )
¯ ¯
¯a1 a2 a3 ¯ ¯ 0 −a3 a2 ¯
−i × (i × A) = −i × (0, −a3 , a2 ) = −(−a3 e3 − a2 e2 ) = a3 e3 + a2 e2
We could interchange the labels to get the results forj × (A × j) and k × (A × k).
We could also use the tensor notation:
(ej × (A × ej ))l = ²lmn ((ej )m (A × ej )n ) = ²lmn (ej )m ²noj Ao ej =
= ²ljn ²noj ej ej Ao = ²jkl ²jko ej ej Ao = ²jkl ²jkl ej ej Al =
= Al for l 6= j
So for each j , we run through l 6= j so that we have the Ak ek and Al e l components. Then we get 2A.
232
Exercise 6.

(1) (a, b, c) · ((0, 0, 1) × (6, 3, 4)) = 3 = (a, b, c) · (3, 6, 0) = −3a + 6b = 3 =⇒ −a + 2b = 1 or a = 2b − 1.


(2b − 1, b, c) = b(2, 1, 0) + (−1, 0, c)
(2) We need to minimize a2 + b2 + c2 . Since each term in this sum is positive and c is arbitrary, let c = 0.
2 2 2 2 2 2 2 2
a + b + c = a + b = (2b − 1) + b = 5b − 4b + 1
4
=⇒ b = 2/5 =⇒ a = 2b − 1 = − 1 = −1/5
5
=⇒ (−1/5, 2/5, 0)

Exercise 7.

(1) (A + B) · (A + B) × C = D · (D × C) = 0
(2) Want: A · (B × C) = −B · (A × C)
(A + B) · (A × C + B × C) = 0 =⇒ B · (A × C) = −A · (B × C)
(3) Want: A · (B × C) = −A · (C × B)
A · (B × C) = A · (−C × B) = −A · (C × B)
(4) A · B × C = −B · A × C = B · C × A = −C · B × A
Exercise 9. Tensor notation way:

(A × (B × C))j = ²jkl Ak (B × C)l = ²jkl Ak ²lmn Bm Cn = ²jkl ²lmn Ak Bm Cn =


= ²jkl ²mnl Ak Bm Cn = Ak Bj Ck − Ak Bk Cj = (C · A)Bj − (B · A)Cj
or

(i × (B × C))j = ²jkl (e1 )k (B × C)l = ²j1l e1 ²lmn Bm Cn =


= ²j1l ²mnl Bm Cn = Bj C1 − B1 Cj
i × (B × C) = c1 B − b1 C
more generally

(et × (B × C))j = ²jkl (et )k (B × C)l = ²jtl ²lmn Bm Cn =


= ²jtl ²mnl Bm Cn = Bj Ct − cj Bt = ct Bj − bt Cj
=⇒ et × (B × C) = ct B − bt C
a1 i × (B × C) + a2 j × (B × C) + a3 k × (B × C) = (C · A)B − (B · A)C
Exercise 10.

(1) (A × B) × (C × D) = (D · A × B)C − (A × B · C)D


(2)
A × (B × C) + B × (C × A) + C × (A × B) =
= (C · A)B − (B · A)C + (B · A)C − (B · C)A + (C · B)A − (C · A)B = 0
(3) Want: A × (B × C) = (A × B) × C iff B × (C × A) = 0
A × (B × C) + B × (C × A) + C × (A × B) = 0 (in general).
If B × (C × A) = 0, A × (B × C) = −C × (A × B) = (A × B) × C .
If A × (B × C) = (A × B) × C ,
A × (B × C) + B × (C × A) + −(A × B) × C = (A × B) × C − (A × B) × C + B × (C × A) =
= 0 + B × (C × A) = 0 =⇒ B × (C × A) = 0
(4) Remember, we can cyclically permute the scalar triple:

(A × B) · (C × D) = C · (D × (A × B)) = C · ((D · B)A − (D · A)B) =


= (B · D)(A · C) − (B · C)(A · D)
A×C ·B =5
A×D·B =3
Exercise 11. Given that A, B, C, D ∈ V3
C + D = (1, 2, 1)
C − D = (1, 0, −1)
(A × B) × (C × D) = (D · (A × B))C − ((A × B) · C)D = (B · D × A)C + (A × C · B)D =
= (−B · A × D)C + (A × C · B)D = (−3)(1, 1, 0) + 5(0, 1, 1) = (−3, −3, 0) + (0, 5, 5) = (−3, 2, 5)
233
Exercise 12.
Consider (A × B) · (B × C) × (C × A)
(B × C) × (C × A) = (A · (B × C))C − (C · (B × C))A = (A · (B × C))C
=⇒ (A × B) · (B × C) × (C × A) = (A · (B × C))C · (A × B) = (A · (B × C))2
In the last step, we cyclically permutated the scalar triple.

Exercise 13. Prove or disprove A × (A × (A × B)) · C = −A2 A · B × C


A × ((A · B)A − A2 B) · C = −A2 (A × B) · C = −A2 C · (A × B) = −A2 A · (B × C)
Exercise 14. A, B, C, D ∈ V3 .
(1) If we consider A to be the origin, then consider the parallelogram with sides B − A, C − A, D − A → B, C, D . The
total volume of the parallelogram is given by
|(B − A) · (C − A) × (D − A) = (D − A) · (B − A) × (C − A)|.

Note that we have a parallelogram from B, C, D vectors. B, C, D themselves are (3 + 1) (1 vertex from A, the
“origin”) vertices of this parallelogram, and a parallelogram has 8 vertices.

We can translate D vector from the A origin to 3 other unique positions - on B , on C , and on B + C (diagonal
across from A, containing A). This will generate 3 new tetrahedrons that are congruent to the the BCD tetrahedron,
because of either the parallelogram has congruent opposite parallel sides or uses or shares the same vertices and
vectors as the BCD tetrahedron.
Be careful because the parallelogram is not “lled up”, its total volume is not accounted for, by these 4 congruent
tetrahedrons, even though each tetrahedron shares 3 of its sides each with each of the other 3 tetrahedron.
Indeed, consider a cube of sides
√ a,a,a. Consider a tetrahedron made up of its sides. The diagonal connecting B
and C , B − C , is of length a 2.
Volume of parallelogram = 6 × ( Volume of a tetrahedron )
1
=⇒ Volume of a tetrahedron = |(B − A) · (C − A) × (D − A)|
6
(2)

A = (1, 1, 1) 1 1
|(B − A) · (C − A) × (D − A)| = |(−1, −1, 1) · (−1, 2, −1) × (3, −1, −1)| =
B = (0, 0, 2) 4 4
For
C = (0, 3, 0) 1 1
= |(−1, −1, 1) · (−3, −4, −5)| =
D = (4, 0, 0) 4 2

Exercise 15.

(1)
C −B
k × (A − B)k = kA − Bk sin θ1
kC − Bk
θ1 is the angle between C − B/kC − Bk and A − B . From the geometry, it's clear that kA − Bk sin θ1 is the
perpendicular distance of A from the line L = {B + t(C − B)}.
(2) Given that A = (1, −2, −5), B = (−1, 1, 1), C = (4, 5, 1)
√ p
k(2, −3, −6) × (5, 4, 0)k/k(−5, −4, 0)k = k(24, −30, 23)k/ 41 = 2005/41

Exercise 16.

(1) Recall that (A × B)j = ²jkl Ak Bl so that

(A × B) = ²jkl Ak Bl ²jmn Am Bn = ²jkl ²jmn Ak Am Bl Bn = A2 B 2 − (A · B)2


2

and −2A · B = kA − Bk2 − kAk2 − kBk2 is immediately apparent by doing the algebra. Then

1
4S 2 = 4( area of the triangle )2 = 4( kA × Bk)2 = kA × Bk2 = a2 b2 − (A · B)2 =
2
µ ¶2
−1 1¡ 2 ¢2
= a2 b2 − (c2 − a2 − b2 ) = a2 b2 − c − a2 − b2 =
2 4
1
= (2ab − c2 + a2 + b2 )(2ab + c2 − a2 − b2 ) (just complete the square)
4
234
(2)

16S 2 = ((a + b)2 − c2 )(−(a − b)2 + c2 ) = ((a + b) + c)((a + b) − c)(c − (a − b))(c + (a − b))
1
=⇒ S 2 = (a + b + c)(a + b − c)(c − a + b)(c + a − b)
16
Exercise 17.        
x + 2y + 3z = 5 1 2 3 5
2 x + −1 y + 4 z = 11
2x − y + 4z = 11 =⇒ 0 −1 1 3
0 + −y + z = 3 Ax + By + Cz = D
Recall that
DBC DCA DAB
x= y= z=
ABC ABC ABC
A · (B × C) = (1, 2, 0) · (3, −5, 11) = −7
D · (B × C) = (5, 11, 3) · (3, −5, 11) = −7
D · (C × A) = −D · (2, −1, −2) = −(5, 11, 3) · (2, −1, −2) = 7
D · (A × B) = D · (−2, 1, −5) = (5, 11, 3) · (−2, 1, −5) = −14
=⇒ x = 1, y = −2, z = 2
Exercise 18.
x + y + 2z = 4        
1 1 2 4
3x − y − z = 2 3 x + −1 y + −1 z = 2
2x + 5y + 3z3 2 5 3 3
A · (B × C) = (1, 3, 2) · (2, 7, 1) = 25
D · (B × C) = (4, 2, 3) · (2, 7, 1) = 25
D · (C × A) = −D · (11, 1, −7) = −(4, 2, 3) · (11, 1, −7) = −25
=⇒ x = 1, y = −1, z = 2
Exercise 19.
x+y =5        
1 1 0 5
x+z =2 1 x + 0 y + 1 z = 2
y+z =5 0 1 1 5
=⇒ x = 1 = z, y = 4
     
P = (1, 1, 1) x 1 2
Exercise 20. X = P + tA =⇒ y  = 1 + t  1 .
A = (2, 1, −1) z 1 −1
Note that X · N = P · N + tA · N = P · N .

x−y+z =1 (1, −1, 1) · X = 1


x + y + 3z = 5 =⇒ (1, 1, 3) · X = 5
3x + y + 7z = 11 (3, 1, 7) · X = 11
We could immediately then see that for all X = P + tA, X satises this system of linear equations because X ·N =P ·N
and P ·N is equal to the left hand side (LHS) of each of the equations in this system. Also, A · N = 0 for each of the normal
vectors, as expected.

13.17 Exercises - Normal vectors to planes, Linear Cartesian equations for planes.
A = (2, 3, −4)
Exercise 1. Given
B = (0, 1, 1)
(1) A × B = (7, −2, 2)
(2) =⇒ 7x − 2y + 2z = 0
(3) 7x − 2y + 2z = 9
Exercise 2. x + 2y − 2z + 7 = 0 or x + 2y − 2z = −7.
1
(1)
3 (1, 2, −2)
235
(−7, 0, 0)
x 2y 2z
(2)
−7 − 7 + 7 = 1 so the intercepts are (0, −7/2, 0)
(0, 0, 7/2)
N
(3)
|N | ·P =d =⇒ 13 (1, 2, −2) · (0, 0, 7/2) = 7/3
(4) −7/9(1, 2, −2)
Exercise 3. (1, 2, −3) is in 3x − y + 2z = −5 and (2, 0, −1) is in 3x − y + 2z = 4.
N
|N | = (3,−1,2)

14
N
|N | · P1 = (3,−1,2)

14
· (1, 2, −3) = −5

14
N
|N | · P2 = (3,−1,2)

14
· (2, 0, −1) = √4
14

9
So the distance between the two planes is √
14
Exercise 4.

(1)
x + 2y − 2z =5 (1, 2, −2)
3x − 6y + 3z =2 3(1, −2, 1)
2x + y + 2z = −1 (2, 1, 2)
x − 2y + z =7 (1, −2, 1)
So by denition, the second and fourth are parallel to each other and the rst and the third are perpendicular to each
(1, 2, −2) · (2, 1, 2) = 0
other, since
(1,−2,1)
(2) N = √
6
(0, −1/3, 0) is on
. the second plane, 3x − 6y + 3z = 2, and (3, 0, 4) is on the fourth plane,
x − 2y + z = 7.

2/3
N · (0, −1/3, 0) = √ √
6 2/3 19/3
=⇒ 7/ 6 − √ = √
7 6 6
N · (3, 0, 4) = √
6
Exercise 5. Given (1, 1, −1), (3, 3, 2), (3, −1, −2), the spanning vectors for a plane through all three points is

(2, 2, 3), (2, −2, −1)

(1) The normal vector to this plane is (4, 8, −8)


(2) x + 2y − 2z = 5
(1,2,−2)
(3)
3 · (1, 1, −1) = 5/3
Exercise 6. Given (1, 2, 3), (2, 3, 4), (−1, 7, −2), the spanning vectors for a plane through all three points are
(1, 1, 1), (−2, 5, −5)
The normal vector to this plane is (−10, 3, 7).
Then the Cartesian equation could easily be obtained: −10x + 3y + 7z = 17

Exercise 7.
x+y =1 (1, 1, 0)
y+z =2 (0, 1, 1)
1 1
√ √ = = cos θ =⇒ θ = π/3 or 2π/3
2 2 2
Exercise 8. (1, 2, 9) =⇒ x + 2y + 9z = −55
Exercise 9. 4x − 3y + z = 5 → (4, −3, 1). (4, −3, 1) is perpendicular to this plane.
     
x 2 4
y  =  1  + s −3
z −3 1
Exercise 10. X(t) = (1 − t, 2 − 3t, 2t − 1)
   
1 −1
(1) X(t) =  2  + t −3
−1 2
236
(2) (−1, 1, 1)
(3) Plug in (1, 2, 1) and t(−1, −3, 2) into 2x + 3y + 2z to see which t will give −1 on the right hand side. t = 1
(4) X(3) = (−2, −7, 5). For a plane to be parallel to another plane, then they each have the same normal vector.
=⇒ (2, 3, 2). So the Cartesian equation is 2x + 3y + 2z = −15
(5) X(2) = (−1, −4, 3). So then a plane perpendicular to L and through X(2) would be
−x − 3y + 2z = 19
Exercise 11. Try N = (n1 , n2 , n3 ).
N · e1 n1 1
=p 2 =
|N | 2 2
n1 + n2 + n3 2
=⇒ n1 = n3
n3 1
p =
2 2 2
n1 + n2 + n3 2

n2 1 n1 1
p = √ =⇒ =√
2n21
+ n22
2 n 2 2

(1, 2, 1)
N=
2
√ √
=⇒ x + 2y + z = 2 + 2

Exercise 12.
x + 2y + 3z = 6
x y z
+ + =1
6 3 2

(6, 0, 0)
36
(0, 3, 0) =⇒ = 6
6
(0, 0, 2)

Exercise 13. x − y + 5z = 1 Then two linearly independent spanning vectors for this could be
(1, 1, 0)
.
(0, 5, 1)
We want this vector to be perpendicular to (1, 2, −3):
(a(1, 1, 0) + b(0, 5, 1)) · (1, 2, −3) = (a + 2(a + 5b) − 3b) = 3a + 7b = 0 =⇒ b = −3a/7
−3 −8 −3 1
so a((1, 1, 0) + (0, 5, 1)) = a(1, , ) =⇒ √ (7, −8, −3)
7 7 7 122

(1, 1, 0)
Exercise 14. Given a plane that needs to span with since it needs to be parallel to these two vectors, and intercept the
(0, 1, 1)
intercept (2, 0, 0)

(1, 1, 0) × (0, 1, 1) = N = (1, −1, 1) =⇒ x − y + z = 2


Exercise 15.
3x + y + z = 5        
3 1 1 5
3x + y + 5z = 7 =⇒ 3 x +  1  y + 5 z = 7
x − y + 3z = 3 1 −1 3 3
(1, 1, −1) × (1, 5, 3) = (8, −4, 4) =⇒ A · B × C = 16
D·B×C 24 3
x= = =
A·B×C 18 2

(1, 5, 3) × (3, 3, 1) = (−4, 8, −12) =⇒ y = 0 then z = 1/2


x = 3/2, y = 0, z = 1/2
237
Exercise 16. We could try to solve for a system of 3 linear equations that represents the 3 planes:

a1 x + b1 y + c1 z = d1
a2 x + b2 y + c2 z = d2
a3 x + b3 y + c3 z = d3
a1 , a2 , a3 6= 0
If

x + b1 y + c1 z = d1  ¯ 
1 b1 c1 ¯d 1
¯
=⇒ x + b2 y + c2 z = d2 =⇒ 1 b2 c2 ¯d 2 
¯
x + b3 y + c3 z = d3 1 b3 c3 ¯d 3

Since the normals of the planes are linearly independent, this means that
 ¯ 
1 1 1 ¯0
¯
b1 b2 b3 ¯ 0
¯
c1 c2 c3 ¯0

has a unique solution and we'll be left with the identity matrix. Then for our problem above, we can do the same operations,
since the equations are linear, and leave the LHS to be an identity matrix. Then we've solved for a unique point in R3
Exercise 17. Given a line through (1, 2, 3) and parallel to the two planes given by x + 2y + 3z = 4 and 2x + 3y + 4z = 5, to
nd a direction vector for the line that is parallel to both planes, determine a vector that would make each of the two previous
equations equal to 0 on the LHS. (1, −2, 1), plugged into the LHS of each of the 2 equations, would make it equal 0. The line

is through (1, 2, 3) so then X = (1, 2, 3) + t(1, −2, 1)


L, s.t. X = P + uA and plane M , s.t. X = Q + sB + tC , and that
Exercise 18. So given a line

L ∦ M , so that sB + tC ∦ uA ∀ s, t, u ∈ R,
T T
Consider L M. Consider X1 ∈ L M
X1 = P + t1 A
X1 − P = t1 A =⇒ X = P + tA = P + (X1 − P ) + (t − t1 )A = X1 + tA (since t∈R)
Since X1 ∈ M as well,

X1 = Q + a1 B + b1 C
X = Q + sB + tC = Q + a1 B + b1 C + (s − a1 )B + (t − b1 )C = X1 + sB + tC
so we have

XL = X1 + tA
XM = X1 + sB + tC
S
Suppose X2 ∈ L M
X2 = X1 + u2 A
=⇒ u2 A = s2 B + t2 C
X2 = X1 + s2 B + t2 C
but sB + tC 6= uA ∀ s, t, u ∈ R
Contradiction, so X2 = X1
Exercise 19.
N (a,b,c)
(1) Given ax + by + cz = −d, then the normal vector for this plane, that's normalized, is |N | = √
a2 +b2 +c2
.

Note that the Cartesian equation for the plane is simply X · N = P · N.


Then the perpendicular distance from the point X0 to a point on the plane, P , is given by (X0 −P )·N = X0 ·N −P ·N .
|ax0 + by0 + cz0 + d|
dist. = √
a2 + b2 + c2
N (5,−14,2)
(2) 5x − 14y + 2z = −9. The normal to this plane is
|N | = en = 15 .

(Q−P )·N
The perpendicular distance from Q to plane is dist.
|N | = −234+9
= 15 = −15
So to get this point on the plane, we simply have to go from Q and go +15 distance along the normalized normal
vector en :
(−2, 15, −7) + 15en = (3, 1, −5)
238
N (2,−1,2)
Exercise 20. Given 2x − y + 2z + 4 = 0, a plane parallel to this plane will have the same normal vector, |N | = en = 3 .

The distance from Q to this plane is


(Q − P ) · N (3, 2, −1) · (2, −1, 2) − (−4)
= = (2 + 4)/3 = 2
|N | 3
To nd the other plane, simply go a distance 2 along en from Q, to get a point on that other plane.
µ ¶
(2, −1, 2) (9, 6, −3) + (4, −2, 4) (13, 4, 1)
(3, 2, −1) + 2 = = = P2
3 3 3
26 4 2
2x − y + 2z = − + = P2 · N = 8 =⇒ 2x − y + 2z = 8
3 3 3
Exercise 21.

(1) Given 3 points A, B, C , they determine a plane =⇒ M = {A + s(B − A) + t(C − A)}


N (B−A)×(C−A)
The normal vector, normalized, to this plane is
|N | = en = |(B−A)×(C−A)|

(Q − P ) · N P =A (Q − A) · (B − A) × (C − A)
−−−→
|N | |(B − A) × (C − A)|
(2) Given
Q = (1, 0, 0)
Q − A = (1, −1, −1)
A = (0, 1, 1)
B − A = (1, −2, 0)
B = (1, −1, 1)
C − A = (2, 2, 3)
C = (2, 3, 4)
(Q − A) · (B − A) × (C − A) (1, −1, −1) · (−6, −3, 6)
= = −1
|(B − A) × (C − A)| 9
Exercise 22. M, M 0 not parallel. eNm 6= eNm0 .
x∈M
x + b1 y + c1 z = d1 x + b2 y + c2 z = d2 =⇒ (1, b1 , c1 ) 6= (1, b2 , c2 ) so that either
x ∈ M0
b1 6= b2 or c1 6= c2
x + b1 y + c1 z = 0
Let's solve for
x + b2 y + c2 z = 0
If b2 − b1 6= 0, · ¸ · ¸
c1 b2 −b1 c2
1 b1c1 1 0 b2 −b1
=⇒
1 b2c2 b2 − b1 c2 − c1
µ ¶
b2 c1 − c2 b1 −(c2 − c1 )
=⇒ (x, y, z) = z , ,1
b1 − b2 b2 − b1
Otherwise, if c2 − c1 6= 0
· ¸ " ³ ´ #
1 b1 c1 1 b1c(c2 2−c
−c1 )
− b2 −b1
c2 −c1 c1 0
=⇒ 1
0 b2 − b1c2 − c1 0 b2 −b1
1
c2 −c1
µ ¶
b2 c1 − b1 c2 −(b2 − b1 )
=⇒ (x, y, z) = y , 1,
c2 − c1 c2 − c1
So these two equations with RHS being 0 result in a solution that's completely determined, give a multiplicative factor.
We can now nd at least one point in the intersection by solving the system of equations

x + b1 y + c1 z = d1
If b2 − b1 6= 0,
x + b2 y + c2 z = d2
· ¯ ¸ · ¯ #
1 c2 ¯ b2 d1 −b1 d2
1 c1 ¯¯d1
b1 1 0 c1 bb22 −b
−b ¯ b2 −b1
=⇒ 1
¯ d2 −d1
1 c2 ¯d2
b2 1 c2 −c1
b2 −b1 ¯ b2 −b1
µ ¶
b2 d1 − b1 d2 − (b2 c1 − c2 b1 ) d2 − d1 − (c2 − c1 )
=⇒ If z = 1, (x, y, z) = , ,1
b2 − b1 b2 − b1
239
Otherwise, if c2 − c1 6= 0
· ¯ ¸ " ¯ #
¯ d1 c2 −c1 d2
1 b1 c1 ¯¯ d1 1 b1 cc22 −c
−c1 b2
0 ¯ c −c
=⇒ 1
¯ 2 11
0 c2 − c1 ¯d2 − d1
b2 − b1 0 b2 −b1
c2 −c1 1 ¯ dc22 −d
−c1
µ ¶
d1 c2 − c1 d2 − (b1 c2 − c1 b2 ) d2 − d1 − (b2 − b1 )
=⇒ If y = 1, (x, y, z) = , 1,
c2 − c1 c2 − c1
T 0
Thus, it's always possible to nd a point in M M . This T intersection is spanned by only one vector, give a multiplicative
factor, which we have found above. Then the intersection M M 0 of 2 planes is a line.
x + 2y + 3z = 4
Exercise 23. Given
2x + y + z = 2
· ¸ · ¸
1 2 3 1 0 −1/3
=⇒
2 1 1 0 1 5/3
µ ¶
1 −5
=⇒ , ,1
3 3
Let's nd a point on the intersection, which is a line, for the two planes:
· ¯ ¸ · ¯ ¸
1 2 3 ¯¯4 1 0 −1/3 ¯¯0
=⇒
2 1 1 ¯2 0 1 5/3 ¯2
=⇒ k(1, −3, 3)
The normal to this plane containing the intersection line and is parallel to e2 = j is

(0, 1, 0) × (1/3, −5/3, 1) = (1, 0, 1/3)

1
Then x+ z =2
3
Exercise 24. Given two equations for the two planes,
x+y =3
, we can immediately get the spanning vector for the line that is the intersection of these planes: (−1, 1, −2/3).
2y + 3z = 4

Also, by inspection, we can nd a point on the intersection of these 2 planes: (1, 2, 0).
We want our plane to be parallel to (3, −1, 2). We can now determine the normal to this plane:
µ ¶
−4
(3, −1, 2) × (−1, 1, −2/3) = , 0, 2
3
−4 −4
=⇒ x + 2z =
3 3
13.21 Exercises - The conic sections, Eccentricity of conic sections, Polar equations for conic sections.

Exercise 1. F is in the positive half-plane determined by N.


kX − F k = ed(X, L)
kX − F k = e|(X − F ) · N + d|
Exercise 2.

(1)
kX − F k = ed(X, L)
kX − F k = e|(X − F ) · N + d|
F = 0 =⇒ kXk = e|(X · N ) + d|; r = e|r cos θ + d|
ed
=⇒ r = e(r cos θ + d) =⇒ r =
1 − e cos θ
ed
(2) The right branch for the hyperbola is given by r= 1−e cos θ because X · N > 0. The left branch for e > 1,
kX − F k = ed(X, L) = e|(X − F ) · N + d| =
= e|X · N + d| = −e(d + r cos θ) = r
−ed
r=
(1 + e cos θ)
240
Exercise 3. For points below the horizontal directrix,

kX − F k = ed(X, L)
F = 0 =⇒ kXk = ed(X, L) = e(|(X − F ) · N − d|) = e|X · N − d| = e|r sin θ − d|
ed
Now Thm. 13.18 says r= if 0<e≤1
e cos θ + 1
ed
=⇒ r = e(d − r sin θ) =⇒ r =
1 + e sin θ
For the “right” or upper-half branch of a hyperbola.

kX − F k = e(|(X − F ) · N − d|) = e(r sin θ − d) = r


−ed ed
r= =
1 − e sin θ e sin θ − 1
ed
Exercise 4. kX − F k = ed(X, L); kXk = e|(X − F ) · N − d| = e|r cos θ − d| = e(d − r cos θ) =⇒ r = 1+e cos θ
e = 1, d = 2 .

3 6( 12 )
Exercise 5. r= 1+ 12 cos θ
= 1+ 12 cos θ
. e = 21 ; d = 6
6 2
Exercise 6. r= 3+cos θ = 1+ 13 cos θ
. e = 31 ; d = 6.
1
Exercise 7. r= −1 .
2 +cos θ

ed(X, L) = kX − F k = e|(X − F ) · N − d| = e|r cos θ − d| = er cos θ − ed = r


−ed ed
=r=
1 − e cos θ e cos θ − 1
2
So for r= 2 cos θ−1 , e = 2, d = 1.
4
Exercise 8. r = e = 2, d = 2.
1+2 cos θ
4
Exercise 9. r = e = 1 d = 4.
1+cos θ
¡3 ¢
Exercise 10. 3x + 4y = 25 =⇒ 53 x + 45 y = 5. N = 4
5, 5 .

L = {x = P + tA}, N · X = N · P.
To nd the distance from the focus, at the origin, to the directrix,

dN = P + tA; dN · N = d = N · P
So for this problem, d = 5.
¯ ¯
¯3 4 ¯
r = kX − F k = ed(X, L) = e|(X − F ) · N − d| = e|X · N − d| = ¯ r cos θ + r sin θ − 5¯¯
¯
5 5
µ ¶
1 3 4
r= 5 − r cos θ − r sin θ
2 5 5
5/2
r= 3 4
1 + 10 cos θ + 10 sin θ
¡ ¢
Exercise 11. e = 1, 4x + 3y = 25 45 x + 35 y = 5; N = 45 , 35 .
d = 5. ¯ ¯
¯4 3 ¯
kX − F k = ed(X, L) = e|(X − F ) · N − d| = e ¯ r cos θ + r sin θ − 5¯¯
¯
5 5
µ ¶
4 3
r =5−r cos θ + sin θ
5 5
5
r=
1 + 45 cos θ + 53 sin θ
Exercise 12. e = 2, hyperbola, so there's 2 branches.
1 1 1
√ x+ √ y = √ L = {x = P + tA} X ·N =N ·P
2 2 2
1
dN = P + tA; dN · N = d = N · P = √
2
241
Note that the sign of d here tells you what side the focus, at the origin, lies on.
1 1
kX − F k = ed(X, L) = kXk = e|(X − F ) · N − d| = e(d − √ r cos θ − √ r sin θ)
2 2

2/ 2
r=
1 + √22 cos θ + √22 sin θ
But for the right side branch,

1 1
kX − F k = ed(X, L) = kXk = e|(X − F ) · N − d| = −e(d − √ r cos θ − √ r sin θ)
2 2

−2/ 2
r=
1 − √22 cos θ − √22 sin θ
Exercise 13. e = 1 parabola.
(1)
π
kX − F k = kXk = ed(X, L) = 1|(X − F ) · N − d| = d − X · N = d − r cos
µ ¶ 3
3 3
d=r = × 108 mi
2 2
3
× 108 mi 3
r= 2
θ = 0, r= × 106 mi
1 + cos θ 4
(2) Focus is in the positive half-plane determined by N.
kX − F k = kXk = ed(X, L) = |(X − F ) · N + d| = r cos θ + d
θ 1
d = r(1 − cos θ) = 108 mi(1 − cos ) = × 108 mi
3 2
1
d × 108 mi 1
r= = 2 r(θ = π) = × 108 mi
1 − cos θ 1 − cos θ 4
13.24 Exercises - Conic sections symmetric about the origin, Cartesian equations for the conic sections.
Quick Review.

Consider symmetry about the origin.

kX − F k = ed(X, L) = e|(X − F ) · N − d| = e|X · N − F · N − d| = |eX · N − e(F · N + d)|


2 2 2
kX − F k = kXk − 2X · F + kF k = e2 (X · N )2 − 2aeX · N + a2
X → −X; X · F = aeX · N
ed e2 d
X = (F − aeN ) = 0 =⇒ F = aeN ; F · N = ae; a = F = N
1 − e2 1 − e2
2
=⇒ kXk + (ae) = e (X · N ) + a2
2 2 2

2
if X = ±aN ; kXk + (ae)2 = e2 (X · N )2 + a2 is satised
0 2 2 2 2
if X = ±bN ; b + (ae) = e (0) + a b = a (1 − e2 )
2 2

x2 y2
Exercise 1. b2 = a2 (1 − e2 ) 100 + 36 =1
q
b2 4
1− a2 = e =⇒ e = .
¡ 4 ¢5
|F | = |aeN | = 10 5 = 8. f = (±8, 0). (0, 0) center. Vertices (±0, 6).
y2 x2
Exercise 2.
100 + 36 = 1. 45 = e; f = (0, ±8).
(0, 0) center; vertices (±6, 0), (0, ±10).
(x−2)2 (y−3)2

7

Exercise 3.
16 + 9 = 1. Center (2, −3). |F | = ae = 4 4 = 7.
q q √ √ √
b2
1− a2 = 1 − 16 9
= 47 = e; (2 + 7, −3), (2 − 7, −3) foci.
Vertices (6, −3), (−2, −3), (2, 6), (2, −12).
x2
¡ ¢¡ ¢
Exercise 4. + y 2 = 1. Center x = (0, 0). |F | = ae = 53 45 = 43 .
( 25
9 )
242
q q
b2 9 4
e= 1− a2 = 1− 25 = 5 . Foci: (± 43 , 0). Vertices (± 53 , 0), (0, ±1).
y2 x 2
Exercise 5. + (1/3)
(1/4) =1
1
¡ 1
¢ 1
|F | = ae = √3 2 = 2√3 .
q ³ ´
1 − 1/4
1/3 = 1
2 = e . Foci:
±1
√ , 0 . Center (0, 0).
2 3

Vertices (±1/ 3, 0), (0, ±1/2).

Exercise 6. Center (−1, −2).


q q
2
1 − ab 2 = 1 − 16 3
25 = 5 = e; |F | = ae = 5 35 = 3.
Foci: (−1, −1), (−1, −5).
Vertices: (−1, 3), (−1, −7), (3, −2), (−5, −2).
¡1¢
Exercise 7. F = ae = 43 . a = 1, e = 43 . b2 = a2 (1 − e2 ); b2 = 1 4 .

x2 + 4y 2 = 1 .

(x+3)2 (y−4)2
Exercise 8. 2a = 4. a2 = 4. 2b = 3. b2 = 9/4. =⇒ 4 + 4 =1
(x+3)2 (y−4)2
Exercise 9.
9/4 + 4 = 1.
(x+4)2 (y−2)2
Exercise 10. 2a = 6, a = 3. 9 + 1 = 1.
¡ 16
¢
Exercise 11. 2a = 10, a = 5. |F | = ae = 5e = 4 e = 4/5. b2 = a2 (1 − e2 ) = 25 1 − 25 = 9.
(x−2)2 (y−1)2
Exercise 12.
a2 + b2 = 1;
(x−2)2 (y−1)2
a = 4 from (6, 1). b = 2 from (2, 3). =⇒ 42 + 4 =1
2 2 2
Exercise 13. b = a (1 − e ).

x2 y2 64
100 − = 1; b2 = 100(1 − e2 ) = −64. 1 + 100
64 = e2 .
√ √
2 41 41
Center (0, 0). e = 10 = 5 .
√ √
Vertices; (±10, 0). F = ae = 2 41. Foci: (±2 41, 0).

x2 y2 x,y→∞ ±4
100 = 64 + 1 −−−−−→; y= 5 x
y2 x2
Exercise 14.
100 − 64 = 1; Center (0, 0), a2 = 100; b2 = −64.
√ √
b2 = a2 (1 − e2 ). e = 541 . Vertices (0, ±10). F = ae = (0, ±2 41).
x2 y 2 x,y→∞ ±5
64 + 1 = 100 − −−−−→ 4 x = y .
(x+3)2
Exercise 15.
4 − (y − 3)2 = 1.
q q √
2 5
Center (−3, 3). e = 1 − ab 2 = 1 − −1
4 = 2 .
√ √ √ √
Foci: ae = 2 25 = 5. (−3 + 5, 3), (−3 − 5, 3).
Vertices: (−3, 4), (−3, 2);
(1, 3), (−7, 3).
(x+3) 2 ±(x + 3)
2 x,y→∞
4 = 1 + (y − 3) −−−−−→ =y−3
2
x2 y2 x2 y2
Exercise 16.
144/9 − 144/16 =1= 16 − 9 .
q
−9 5
e= 1− 16 = 4 . Center (0, 0). |F | = ae = 5. (5, 0), (−5, 0). Vertices (±4, 0).
Foci:

¡ ¢ q
y2 x2
Exercise 17. 20 = 5y 2 − 4x2 . Center (0, 0). |F | = ae = 2 23 = 3. Foci: (0, ±3). 1 = 4 − 5 . e= 1− −5
4 = 3
2.
Vertices: (0, ±2)
(x−1)2 2
Exercise 18. − (y+2)
4 9 = 1.
q √ √ √ √ √
13 13
Center (1, −2). e = 1 − −9
4 = 2 ; |F | = 2 2 = 13. Foci: (1 + 13, −2), (1 − 13, −2).
Vertices: (5, −2), (−3, −2).
Exercise 19. F = ae = 2(2) = 4.
243
x2 y2 y2 x2 x,y→∞

4 + −12 = 1. 12 +1= 4 −−−−−→ y = ± 3x.
b2 = a2 (1 − e2 ) = 4(1 − 4) = −12.

Exercise 20. F = ae = 2 = (1)e. b2 = a2 (1 − e2 ) = 1(1 − 2) = −1. =⇒ y 2 − x2 = 1.
x2 y2
Exercise 21.
4 − 16 =1
(x+1)2
Exercise 22. (y − 4)2 − −3 =1 where

F = ae = | − 2| = ae. b = a (1 − e2 ) = 1(1 − 4) = −3
2 2
2
(y+3)2
Exercise 23. ± (x−2)
a2 ∓ b2 =1
1 4
(3, −1) =⇒ ± ∓ 2 =1 (y + 3)2 (x − 2)2
a2 b =⇒ − =1
9 9 27/8 (27/5)
(−1, 0) =⇒ ± 2 ∓ 2 = 1
a b
x2 −1
Exercise 24.
3 = y 2 . 2x 0
3 = 2yy . yy 0 = x3 .
3x − 2y = C. m = 2 =⇒ y0 2 = x0 . 81
3 9 2 2
4 y0 − 1 = 3y0 =⇒ y0 =
±2

69
.

x2 −1
The asymptotes of y2 = 3 are y = ± √x3 .
µ ¶ µ ¶
±9 2 ±23
3 √ − 2 ±√ = √ =C
69 69 69
r
23
3x ± = 2y
3
2 2
±x2 ±x2
Exercise 25. + ∓ yb2 = 1
a2 a2
y
∓ 4a 2 = 1.

(3, −5) → ±9 ∓ 25
4 =a ;
2 2 11
a = 4.

x2 y2
=⇒ − =1 .
11/4 11
Quick Review of Parabolas.

F on positive half plane to N.


kX − F k = e|(X − F ) · N + d|
Let N = ~
ex ; d = 2c; F = (c, 0); e = 1.
(x − c)2 + y 2 = e2 ((x − c) + 2c)2 = (x − c)2 + 4c(x − c) + 4c2
y 2 = 4cx
Thus, for ellipses, the vertex is equidistant to the focus and directrix (conrming the other denition).
Let N = ~ey , d = 2c; F = (0, c), e = 1.
x2 + (y − c)2 = ((y − c) + 2c)2 = (y − c)2 + 4c(y − c) + 4c2
x2 = 4cy

Exercise 26. 4c = −8 (0, 0) vertex. y = 0 symmetry axis. x = 5 directrix.


Exercise 27. 4c = 3. Vertex: (0, 0). Symmetry axis: y = 0. Directrix: x = −3/4.
¡ ¢
Exercise 28. (y − 1)2 = 12(x − 21 ). 4c = 12, c = 3. Symmetry axis: y = 1. Directrix: −5
2 ,1 .
1 1 1
Exercise 29. x2 /6 = y . 4c = 6 c= 24 . Vertex: (0, 0). Directrix: y = − 24 . Symmetry axis: x = 0.
−1
Exercise 30.
2
x + 8y = 0. 4c = 8 ; c = −1 1
32 . y = 32 directrix; x = 0 axis.
Exercise 31. (x + 2) = 4(y + 94 ). 4c = 4;
2
c = 1. Center (−2, −9/4). Directrix: y = −13/4. Axis: x = −2.
2
Exercise 32. y = −x .
Exercise 33. x2 = 8y .
Exercise 34. (y − 3) = −8(x + 4)2 .
Exercise 35. c = 45 5(x − 74 ) = (y + 1)2
244
Exercise 36. y = ax2 + bx + c
1 1 2 3
(0, 1) → c = 1 (1, 0) → 0 = a + b + 1(2, 0) → 0 = 4a + 2b + 1 a= =⇒ y = x − x+1
2 2 2
Exercise 37. 4c(x − 1) = (y − 3)2 . 4c(−2) = (−4)2 = 16. c = −2. −8(x − 1) = (y − 3)2 .
Exercise 38. kX − F k = ed(X, L) = |(X − F ) · N − d|
L = {(x, y)|2x + y = 10; √25 x + √y = 10
√ }.
5 5
10
d = N = xL dN · N = d = xL · N = √
5
.

µ ¶2
−2 y 10
F = 0 =⇒ kXk2 = |X · N − d|2 = √ x + −√ + √ = x2 + y 2
5 5 5
5x2 + 5y 2 = (−2x − y + 10)2 = 4x2 + y 2 + 100 + 4xy − 40x − 20y
=⇒ x2 + 4y 2 − 4xy + 40x + 20y − 100 = 0
13.25 Miscellaneous exercises on conic sections.

Exercise 1.
µ ¶2 µ ³ x ´2 ¶
y2 x2 2 2 bx 2
=1− 2 y =b − =b 1−
b2 a a a
Z a r ³ x ´2 Z 1 p
y=2 b 1− dx = 2 ab 1 − x2 dx = (ab) area of a circle of radius 1
−a a −1
Exercise 2.
q ¡ ¢2
2a in the x-axis. y = b 1 − xa
(1) Without loss of generality, let the major axis be
Z a µ ¶ Z 1
x2 4
V = πb2 1 − 2 dx = πb2 a (1 − x2 )dx = π(1)3 b2 a
−a a −1 3
x2 b2
(2) If rotated about the minor axis, suppose, without loss of generality, 2a is the minor axis (just note that a2 + a2 =1
have x, y, a, b as dummy labels).
=⇒ V = 43 π(1)3 b2 a, where 2a is the minor axis, 2b is the major axis.
2
q
x2 y 2 2
Exercise 3.
(3/A) + (3/B) =1 By 2 = 3 − Ax2 =⇒ y 2 = B3 − Ax B ; y = B3 − Ax B . So the area inside this ellipse is
r Z √3/A r Z √3/A s
1 p 3 x2
2 √
2
3 − Ax dx = 2 √ 1− ¡3¢
B − 3/A B − 3/A A
2 2
³ ´³ 2
´ q q
x y 3 x 3 x2
For the other ellipse equation,
3/(A+B) + 3/(A−B) = 1. y 2 = A−B 1 − (3/(A+B)) ; y = A−B 1 − (3/(A+B)) .

Thus, the area inside this ellipse is


v  2
r Z √ A+B u
3 u
3 u x
2 √ t1 −  q 
A−B − 3
A+B
3
A+B
Equating the two areas after making an appropriate scale change,
r r Z r r Z
3 3 1 p 3 3 1 p
2 1 − x2 dx = 2 1 − x2 dx
B A −1 A−B A+B −1

Thus A2 − B 2 = AB =⇒ A2 − BA − B 2 . Simply try treating B as a number and solve the quadratic equation in terms
of A. p √ √
B ± B 2 − 4(1)(−B 2 ) B±B 5 B(1 + 5)
A= = =
2(1) 2 2
Exercise 4. y = − 4h 2
b2 x .
Z b/2 µ ¶ ¯b/2 µ ¶
4h 2 4h 3 ¯¯ b b 2hb
x +h = x + h + =
−b/2 −b2 −3b2 ¯−b/2 2 2 3
2
R2
Exercise 5. y = 8x. 0
π8tdt = 4π(2)2 = 16pi
Exercise 6. y 2 = 2(x − 1). y 2 = 4(x − 2).
245
(1)
Z 2 p Z 3 p √
A=2 2(x − 1) + 2 2(x − 1) − 2 x − 2 =
1 2
¯2 ¯3 ¯3
√ 2 ¯
3/2 ¯
√ 2 ¯ 2 ¯
= 2 2 (x − 1) ¯ + 2 2 (x − 1)3/2 ¯ − 4 (x − 2)3/2 ¯
3 1 3 2 3 2
√ 2 √ 4 3/2 √ 2 2
= 2 2 + 2 (2) − 2 2 − 4 = 8/3
3 3 3 3
(2)
Z 2 ¶¯2 µ µ ¶
1 2 ¯ 1
2(x − 1) = 2x − x ¯¯ = 2 (4 − 1) − (2 − 1) = 1
1 2 1 2
Z 3 Z 3
¯3
(−2x + 6)dx = −x2 ¯2 + 6x|2 = (−9 + 4 + 6(3 − 2)) = 1
3
(2(x − 1) − 4(x − 2)) dx =
2 2
Z 2 Z 3
=⇒ V = π 2(x − 1) + π (2(x − 1) − 4(x − 2)) = 2π
1 2
y2 2
(3)
2 + 1 = x, y4 + 2 = x
Z 2 õ 2 ¶2 µ 2 ¶2 ! Z 2µ ¶ µ ¶¯2
y y −3y 4 −3 5 ¯
2π +2 − +1 = 2π + 3 dy = 2π y + 3y ¯¯ =
0 4 2 0 16 80 0
µ ¶ µ ¶ µ ¶
−3(32) −96 + 480 384 48
= 2π + 6 = 2π = 2π =π
80 80 80 5
1
Exercise 7. By Apostol's denition of conic sections, we are basically given the conic section denition with e= 2 . So just
plug in the pt. (0, 4).
à µ ¶2 !
x2 y2 (0,4) 2 2 2 2 1
2
+ 2 = 1 −−−→ b = 4 b = a (1 − e ) = 16 = a 1−
a b 2
x2 y2
+ =1
64/3 16
Exercise 8. F =0 kX − F k = kXk = ed(X, F ) = |X · N + d| = x
√ + √y + √1 because for the directrix
2 2 2
y + x = −1
¶ µ
1 1 1 x 1
N = √ ,√ √ y + √ = −√
2 2 3 2 2
XL = P + tA √
√ dN = XL XL · N = d = −1/ 2
XL · N N · P = −1/ 2
So by squaring both sides of the vector equation,

x2 y2 1
x2 + y 2 = + xy + + +x+y
2 2 2
x2 y2 1
+ − xy − x − y =
2 2 2
x2 + y 2 − 2xy − 2x − 2y = 1
Exercise 9. Center (1/2, 2) because we equate the asymptotes to see where they intersect: y = 2x + 1 = −2x + 3.

(y − 2)2 (x − 1/2)2 (0,0) 4 1 3


2
− = 1 −−−→ 2 − 2 = 2 = 1
a a2 /4 a a a
(y − 2)2 (x − 1/2)2
− =1
3 3/4
x2 y2
Exercise 10. px2 + (p + 2)y 2 = p2 + 2p. p+2 + p = 1.
q
2
(1) Since p + 2 > p, the foci must lie on the x axis. a2 = p + 2; b2 = a2 (1 − e2 ) = p = (p + 2)(1 − e2 ). e = p+2
√ √
F = ae = 2. (± 2, 0).
246
√ √ q
(2) F = ae = 2 = a( 3) =⇒ a = 23 ; b2 = 32 (1 − 3) = −4
3 .

x2 y2
− =1
2/3 4/3

Exercise 11. e = 1 for an ellipse.

kX − F k = |X · N − a| = a − X · N
k−X − F k = kX + F k = | − X · N − a| = a + X · N
kX − F k + kX + F k = 2a

Exercise 12.

kX − F k = e|(X − F ) · N − d| = e(d − (X − F ) · N )
kX + F k = ed(X, L) = e|(X − F ) · N + d| = e(−d − (X − F ) · N )
kX − F k − kX + F k = 2ed
X → −X so for the other branch, kX + F k − kX − F k = 2ed

Exercise 13.

(tx)2 2 ¡ b ¢2 2 2 ¡ ¢2
(1)
a2 + (by)b2 = 1 t = a (1−e
t2
)
= at (1 − e2 )
2
(2) b1 = a21 (1 − e2 ) b22 = a22 (1 − e2 ).

b21 b22 b21 b22


1− = 1 − ; =
a21 a22 a21 a22
³³ ´ ´2 ³ ´2
b2 b2
x12
y 2 b1 x b1 y
+ 2 =1= +
a21 b1 a22 (b2 y)2
2
(ty)2 ¡ b ¢2 −a2 (e2 −1) ¡ a ¢2
(3) ± (tx)
a2 ∓ b2 =1 t = t2 =− t (e2 − 1).

b21 = a21 (e2 − 1) b22 = a22 (e2 − 1)


b21 b22 b21 b2
+ 1 = e2 +1 = e2 = 22
a21 a22 a1 2 a2
³ ´ 2 Ã b !2
µ ¶2 µ ¶2 b2
b1 x b1 y
2
x y
± ∓ = ±  ∓ =1
a1 b1 a2 b2

x2 y2 x y 0
Exercise 14.
a2 + b2 = 1. =⇒ a2 + b2 y =0

−b2 x −a2 (1−e2 )x (e2 −1)x


=⇒ y 0 = ya2 = a2 y = y
Exercise 15.

(1) y = ax2 + bx + c ty = a(tx)2 + btx + c → y = atx2 + bx + c/t = y = Ax2 + b + C


(2) y = tx2 , t 6= 0

Exercise 16. x − y + 4 = 0 y = 4 x (y 2 = 16x); y 0 = 2x−1/2 .

y 0 (x = 4) = 1 (x, y) = (4, 8).


Exercise 17.

(1) If we treat the two given parabolas, y 2 = 4p(x − a) and x2 = 4qy , as two vector objects free from any specic
coordinate system then we observe that we can disregard the sign of q and p and simply state that they are both
positive. What matters is that we observe that p and q are the distance of the foci to the vertex for each of the
respective parabolas.
Second, observe that a is not given. By diagram, if p, q are given, a must be moved along the x-axis to t the
tangency condition. Thus, in terms of doing the algebra, just eliminate p and q from the relations.
247
If (h, k) is the point of contact,
√ √
x2 = 4qy y 2 = 4p(x − a) y = 2 p x − a
x √ 1
= y0 y0 = p √
2q x−a
h √ 1
y 0 (h) = y 0 (h) = p √
2q h−a
µ ¶2
h p
(Tangent condition) = =⇒ (h2 )(h − a) = (2q)2 p
2q h−a
h2 k2
(one point of contact condition) with q = ,p=
4k 4(h − a)
µ 2 ¶2 2
h k h2
=⇒ h2 (h − a) = =⇒ (h − a)2 =
2k 4(h − a) 16
2a ± a/2
=⇒ h = = 4a/3
15/8
(2)

h p
=√
2q h−a
√ √
2a p 3p √ p
=p = √ ; 2a a = 3 3pq
3q a/3 a
=⇒ 4a3 = 27pq 2

Exercise 18. First hint: Vector methods triumph over algebraic manipulations of Cartesian coordinates. Think of the locus
2
in terms of vector objects that are coordinate-free and the conic section will emerge. I mean, try evaluating kP − Ak =
2 2 2
(x − 2) + (x − 3) = (x + y)
A = (2, 3), N = √12 (1, 1), X = (x, y).
√ √
kX − Ak = x + y = 2(X · N ) = 2(X · N − (F · N − d))
where F ·N =d=A·N = √5
2
d= distance from focus to the directrix .

y = x + 1 (axis of the hyperbola)


p √
d = √52 = (2 − x)2 + (3 − y)2 = 2(2 − x) x = − 12 , y = 12
¡ −1 1 ¢ 1
¡ 1
¢
2 , 2 must also be the center. y − 2 = − x + 2 is the directrix.
¡ ¢ ¡ ¢
y − 21 = α x + 12 is the general form of the asymptote.
Consider asymptotes in general. kX − F k = ed(X, L).

kX − F k kX − F k kX − F k
=e= =
d(X, L) |X · N − (F · N − d)| (X − F ) · N + d|
For kX − F k → ∞, kX − F k > d. To keep ratio of e, X − F must be ultimately directed by N by a ratio of e.
kX − F k 1
=⇒ e = =
kX − F k cos φ cos φ
x2 y2

e.g. Consider N = ~ex . a2 − b2 = 1 =⇒ y = ab x = e2 − 1x.
From the vector equation,
p
(X − F ) · N = (x − c, y) · N = (x − c)2 + y 2 cos φ = x − c
p
(x − c)2 + y 2 1 (x − c)2 + y 2
= = e; = e2 ;
x−c cos φ (x − c)2
y2 p
2
= e − 1 =⇒ y = e2 − 1x
(x − c)2
248
For our problem, consider the conic section approaching the asymptote. Then the conic section will look more like those
linear asymptotes.

p
(x − 2)2 + (y − 3)2 = x + y
sµµ¶ ¶2 µ µ ¶ ¶2 µ ¶
y− 12 =α(x+ 12 ) 1 5 1 5 1 1
−−−−−−−−−→ x+ − + α x+ − =x+α x+ +
2 2 2 2 2 2
s µ ¶2 µ ¶ √
1 1 25 x→∞ 1 + α2
=⇒ (1 + α2 ) x + − 5(1 + α) x + + −−−−→
2 2 2 (1 + α)
=⇒ α = 0

1 −1
The asymptotes are y= 2 and x= 2 .

In the second part, each quadrant must be checked. So far, I only have that quadrant II is lled: points in quadrant III and
quadrant IV cannot satisfy the given condition. To see this, consider quadrant II.

µ ¶
√ −1 1
kx − Ak = −x + y = 2(x, y) √ ,√
2 2
³ ´
−1 √1
For quadrant II, N= √
2
, 2 . By diagram, (X − F ) · N > 0 and X · N > 0.

1 −1 1
A·N = √ −A·N = √ d= √
2 2 2
|(X − F ) · N + d| = (X − F ) · N + d

The equation for the axis of the conic section is y = −(x − 5).
By taking the asymptotic limit like above, we can show that α =0 again. We only sketch the part of the hyperbola in
quadrant II.
By similar procedure, I found that quadrant III, IV cannot satisfy the condition.

Exercise 19.

kX − F k = d(X, L) = |(X − F ) · N + d|
x2 + y 2 = (X · N + d1 )2 = y 2 + 2yd1 + d21
F =0 x
−−−→ x2 = 2yd1 + d21 y10 =
d1

kX − F k = |(X − F ) · N − d2 | = d2 − (X − F ) · N
F =0
−−−→ kXk = d2 − y
x + y = d22 − 2d2 y + y 2
2 2

−x
=⇒ x2 = d2 − 2d2 y y20 =
d2

Point of intersection x20 = 2y0 d1 + d21 = d22 − 2d2 y0


d2 − d1
2(d1 + d2 )y0 = d22 − d21 =⇒ y0 =
2
x20 = d2 (d2 − 2y0 ) = d2 d1
√ r
0 ± d2 d1 d2
y1 = =±
d1 d1
=⇒ √
∓ d2 d1 1
y20 = = ∓q
d2 d2
d1

Exercise 20.

249
(1) Use X → −X symmetry.

kX − F k = ed(X, L) = e|(X − F ) · N + d| = e|X · N − F · N + d| = |eX · N − a|


2 2
kXk − 2X · F + kF k = e2 (X · N )2 − 2ea(X · N ) + a2
2 2
X → −X =⇒ kXk + 2X · F + kF k = e2 (X · N )2 + 2ea(X · N ) + a2
=⇒ kX 2 k + kF k2 = e2 (X · N )2 + a2
x2 + y 2 + c2 = e2 x2 + a2 |F | = c = ae
µ 2 2

a −c 2 2 2 2 x2 y2
x + y = a − c =⇒ + =1
a2 a2 a2 − c2
x yy 0 −(a2 −c2 )x
(2)
a2 + a2 −c2 = 0 =⇒ y 0 = ya2
µ 2 ¶2 3
a − c2 x
xy(y 0 )2 =
a2 y
µ 2 2
¶µ ¶
a −c 2 −(a2 − c2 )x
(x2 − y 2 − c2 )y 0 = x2 + x − a2
=
a2 ya2
(a2 − c2 ) x
= (a4 + −(a2 − c2 )x2 − a2 x2 )
a4 y
³ 2 2´
a −c
−xy 2 x(a2 − c2 − a2 x2 )
−xy = =− =
y y
= (a2 − c2 )(−a4 + a2 x2 )x/(a4 y)
=⇒ xyy 02 + (x2 − y 2 − c2 )y 0 − xy = 0
(3) For y 0 , consider − y10 at every (x, y).
µ ¶2
−1 −1 −xy 1
xy 0
+ (x2 − y 2 − c2 ) 0 − xy = 0 = 0 + (x2 − y 2 − c2 ) 0 + xy
y y y y
if y 0 6=0
−−−−→ −xy + (x2 − y 2 − c2 )y 0 + xy(y 0 )2 = 0
Thus S→S since the dening differential equation is invariant under the transformation of the slope.

Exercise 21. For a circle centered at C , then kX − Ck = r02 for all points X on that circle.

For the condition of being tangent to a given line, L = P + tA, then (XC − C) · A = 0 and the point lies on the circle so
kXC − Ck = r02 .
Call the point that all the circles pass through F . Then kC − F k = kC − XC k. kC − XC k is by denition d(X, L), the
distance from the circle center to the line. kC − F k = kC − X0 k is by denition a parabola.
Exercise 22. Consider a circle that's part of the mentioned family that has its center directly below the given circle with radius

r0 , and center Q.
It's given that the center is equidistant from the point of tangency and the line. This hints at a parabola because the parabola's
vertex is equidistant from the focus and the directrix. Thus, we need to show that d(X, L) is equal to the distance from the
circle center C to the bottom point of Q.
LetN be a unit normal vector pointing from the line towards the focus, placing the focus in the positive half-plane.
Let C be the center of an arbitrary circle in the family and r1 its radius.
Let X1 be the point of tangency between circle Q and circle C .
We want k(Q + r0 N ) − Ck = kX2 − Ck.
The tangency condition between circle Q and C means that
r1
(X1 − C) = −α(X1 − Q); α > 0 α=
r0

Q − r0 N − C = Q − X1 − r0 N − C + X1
take the magnitude 2 2
−−−−−−−−−−→ kQ − X1 k + kX1 − Ck + r02 + 2(Q − X1 )(X1 − C) + 2(X1 − Q)r0 N + 2(C − X)r0 N
r02 + r12 + r02 + 2αr02 + 2r0 (1 + α)(x1 − Q) · N
2r02 + r12 + 2r1 r0 + 2(r1 + r0 )(X1 − Q) · N
250
1
I had thought the key is to use the law of cosines to evaluate (X1 − Q) · N = α (C − X1 ) · N .

Length l = d(X, L) = d(C, L).

But that just gets us back to the same place.


I had found the solution by a clever construction. But to come to that conclusion it required me to be “unstuck” - if
something doesn't work, move onto the next - don't try to make something work and go in circles. And persistence is key
because there can be many false eurekas.
Again, consider a particular circle with its centerC2 right below the given Q circle that just makes C2 tangent with
L2 . The directrix is not going to be L2 but L1 , a line translated below L2 , line of tangency, by r0 , so that
the given line
kQ − C2 k = r2 + r0 = d(C2 , L1 ). It is a clever articial construction.
Let's show this for any circle C of radius r1 in the family.

Tangent to the circle Q condition: X1 − C = α(Q − X1 ).

So then kQ − Ck = r1 + r0

Tangent to the line L2 = B2 + tA2 : (X2 − C) · A2 = 0

kX2 − Ck = r1
Consider L1 , a line translated by r0 from L2 away from Q.

If L2 = B2 + tA2 , L1 = B2 − r0 N + tA2 .

Since X2 − C = r1 (−N ) then X2 − r0 N − C = (r1 + r0 )(−N ) will point from C to L1 , because (X2 − r0 N ) =
((B2 + tA2 ) − r0 N ) ∈ L1 .
=⇒ kQ − Ck = r1 + r0 = kX2 − r0 N − Ck = d(C, L1 ).
Exercise 23. Without loss of generality, use y 2 = 4cx.

The latus rectum intersect the parabola at (c, +2c), (c, −2c).

Thus 4c = length of latus rectum = 2d = 2( distance from focus to directrix ).


√ √ √
y = 2 cx y 0 = c/ x y 0 (c) = ±1

Tangent lines: y = ±(x + c).


intersection
−−−−−−→ +(x + c) = −(x + c) x = −c (at the directrix)

Exercise 24. Center of circle is given to be 0.

Collinear with center and center not between them: P = αQ; α>0
2
kP k kQk = r02 = α kQk

For the line dened in Cartesian coordinates as x + 2y − 5 = 0, the vector form of this line is given by
µ ¶
1 2 √
XL = B + tA XL · N = (x, y) · √ , √ =N ·B+0= 5
5 5
µ ¶
−2 1
A= √ ,√ is a vector that's perpendicular to N ;
5 5
B = (1, 2) since we can simply plug it in to satisfy the equation
µ ¶
−2 1
XL = (1, 2) + t √ , √ t∈R
5 5
2
Q = B + tA =⇒ kQk = B 2 + 2tB · A + t2 A2 = 5 + t(0) + t2 = 5 + t2
4
(5 + t2 )(α) = r02 = 4 α =
5 + t2
µ µ ¶¶
4 −2 1
P = αQ = (1, 2) + t √ , √
5 + t2 5 5
251
14.4 Exercises - Vector-valued functions of a real variable, Algebraic operations. Components; Limits, derivatives,
and integrals.
Exercise 1. F 0 = (1, 2t, 3t2 , 4t3 ).
Exercise 2. F 0 = (− sin t, 2 sin t cos t, 2 cos 2t, sec2 t)
³ ´
1 √−1
Exercise 3. F 0 = √1−t 2
, 1−t2
Exercise 4. F 0 = (2et , 3et ).
¡ ¢
Exercise 5. F 0 = sinh t, 2 cosh 2t, −3e−3t
³ ´
t 1 −2t
Exercise 6.
1+t 2 , 1+t 2 , 2
(1+t )2
³ ´
0 2 −4t2 4t
Exercise 7. F = 1+t2 + (1+t 2 )2 , (1+t2 )2 , 0 .

4t(1 + t2 ) −4t2 (2t) 4t − 4t3


F0 · F = + + =0
(1 + t2 )3 (1 + t2 )3 (1 + t2 )3
¡1 2 1
¢
Exercise 8.
2, 3, e −1
Exercise 9. Ã √ √ √ !
− 3 π/4 2 2
+ 1,
(− cos t, sin t, − ln | cos t|)|0
= , − ln
2 2 2
1 ¡ ¡ 1+e
¢ ¡ ¢¢
t t
Exercise 10. (ln (1 + e ), t − ln (1 + e ))| = ln
0 2 , 1 − ln 1+e
2
¡ t ¢¯ 1
t 2 t t t
Exercise 11. te − e , t e − 2te + 2e , −te
−t
− e−t ¯ = (1, e − 2, −2e−1 + 1)
0
Exercise 12. (2, −4, 1) = A.
R1 ³ ³ ´´¯1
e−2t ¯
0
(te2t , t cosh 2t, 2te−2t )dt = 12 te2t + −1 2t t sinh 2t
4 e , 2 − cosh 2t
4 ,2 −1 −2t
2 te − 4 ¯ =
0
¡ ¡ ¢¢
= 41 e2 + 14 , sinh
2
2
− cosh
4
2
+ 14 , 2 −3 4 e
−2
+ 14 .
A · B = 12 e2 + −2 sinh 2 + cosh 2 + −3 2 e
−2
.
0 0
Exercise 13. F (t) = B = 1 = kF (t)k |B| cos θ(t).

Given θ(t) = θ0 constant, kF 0 (t)k must be a constant.


2 2
kF 0 (t)k = F 0 (t) · F 0 (t) = g g 0 = 2F 00 (t) · F 0 (t) = 0 since kF 0 k constant .
00 0
=⇒ F (t) · F (t) = 0
Exercise 14.
F 0 = 2e2t A + −2e−2t B
F 00 = 4e2t A + 4e−2t B = 4(F )
Exercise 15. G0 = F 0 × F 0 + F × F 00 = F × F 00
Exercise 16.
G = F · (F 0 × F 00 )
G0 = F 0 · (F 0 × F 00 ) + F · (F 00 × F 00 + F 0 × F 000 ) = F · (F 0 × F 000 )
Exercise 17. If limt→p F (t) = A, ∀j th component,
r r
² ²
∀ > 0, ∃ δj > 0 such that |Fj (t) − Aj | < if |t − p| < δj
n n
Consider min δj = δ0
j=1,...n
n
X n µr ¶2
X ²
|Fj (t) − Aj |2 < =² whenever |t − p| < δ0
j=1 j=1
n
=⇒ lim kF (t) − Ak = 0
t→p
qP
n 2
limt→p kF (t) − Ak = 0,
If ∀² > 0, ∃ δ > 0 such that j=1 (Fj (t) − Aj ) < ² if |t − p| < δ .
Pn
=⇒ j=1 (Fj (t) − Aj )2 < ²
Pn
² > j=1 (Fj (t) − Aj )2 > (Fk (t) − Ak )2 > 0
=⇒ ² > |Fk (t) − Ak | if |t − p| < δ .
252
Exercise 18. If F is differentiable on I , then
n
X 1
F0 = fj0 ~ej fj0 = lim (fj (t + h) − fj (t))
j=1
h→0 h
n
X n
1 1X 1
F0 = lim
(fj (t + h) − fj (t)) = lim (fj (t + h) − fj (t))ej = lim h → 0 (F (t + h) − F (t))
h→0 h h→0 h h
j=1 j=1
Pn
F 0 (t) = limhto0 h1 (F (t + h) − F (t)) = limh→0 h1 j=1 (fj (t + h) − fj (t))ej =
If
Pn P n
= j=1 limh→0 h1 (fj (t + h) − fj (t))ej = j=1 fj0 (t)ej

So F 0 is differentiable.
Exercise 19. F 0 (t) = 0, ∀j = 1 . . . n, fj0 (t) = 0. By one-dimensional zero-derivative theorem, fj (t) = cj constant. Thus
Pn
F (t) = j=1 cj ~ej = C on an open interval I .
1 3
Exercise 20.
6t A + 12 t2 B + Ct + D
0
Exercise 21. Y (x) + p(x)Y (x) = Q(x). Then ∀j = 1, . . . , n
yj0 (x) + p(x)yj (x) = Qj (x)
Since p, Q are continuous on I , and given this initial value condition yk (a) = bk ,
Rx
µ Z x Rt

yj (x) = e− a p(t)dt bj + Qj (t)e a p(u)du dt
a
n
X Rx
µ Z x Rt

− p p
=⇒ jj (x) = Y (x) = e a B+ Qe a dt
j=1 a

Exercise 22.

tF 0 = F + tA =⇒ F 0 + tF 00 = F 0 + A
=⇒ tF 00 = A tF 0 = F + tA =⇒ At ln t + Bt = A(t ln t − t) + Bt + C + tA
F 00 (t) = A/t C = 0, B = 3A
=⇒ F 0 (t) = A ln t + B F (t) = A(t ln t − t) + 3At
=⇒ F (t) = A(t ln t − t) + Bt + C F (3) = A(3 ln 3 − 3) + 9A = 3A ln 3 + 6A
F (1) = A(−1) + B + C = 2A

Exercise 23.
Z x
0 x x1 1
F (x) = e A + xe A + − 2 F (x) = F (t)dt +
x 1 x
F (x) F (x)
= ex A + xex A + ex A − + = 2ex A + xex A = (2 + x)ex A
x x
F 0 (x) = (2 + x)ex A; F (x) = 2ex A + A(xex − ex ) + C = Axex + ex A + C
Z x
¡ ¢¯x
(Atet + et A + C)dt = A(tet − et ) + et A + Ct ¯1 =
1
= A(xex − ex ) + ex A + C(x − 1) − eA = Axex + C(x − 1) − eA
C(x − 1) eA
xex A + Aex + − =⇒ C = eA
x x
Exercise 24. F 0 (t) = α(t)F (t)
µ ¶ Z t
fk (t)
=⇒ fk0 (t) = α(t)fk (t); ln = α(x)dx
fk (a) a
Rt
α
fk = fk (a)e a

n
X Rt Rt n
X
α α
F (t) = fj (a)e a ej = e a fj (a)ej = u(t)A
j=1 j=1

253
14.7 Exercises - Applications to curves. Tangency. Applications to curvilinear motion. Velocity, speed, and accelera-
tion.
Exercise 1.

~r(t) = ((3t − t3 ), 3t2 , (3t + t3 ))


v = (3 − 3t2 , 6t, 3 + 3t2 ) = 3(1 − t2 , 2t, 1 + t2 )
a = 3(−2t, 2, 2t) = 6(−t, 1, t)
p √
s = 3 (1 − 2t2 + t4 + 4t2 + 1 + 2t2 + t4 ) = 3 2(1 + t2 )
Exercise 2.
r = (c, s, et )
v = (−s, c, et )
a = (−c, −s, et )
p
s = 1 + e2t
Exercise 3.
r = (3tc, 3ts, 4t)
v = (3(c + −ts), 3(s + tc), 4) = 3(c, s, 0) + (−3ts, 3tc, 4)
a = 6(−s, c, 0) + (−3tc, −3ts, 0)
p p
s = 9(c2 − 2tsc + t2 s2 ) + 9(s2 + 2tsc + t2 c2 ) + 16 = 9 + 9t2 + 16
Exercise 4.
r(t) = ((t − sin t), (1 − cos t), 4 sin (t/2))
v = (1 − c, s, 2 cos (t/2))
a = (s, c, − sin (t/2))
s µ ¶
p 1+c
s= (1 − 2c + c2 + s2 + 4c2 (t/2)) = 2 − 2c + 4 = 2
2
Exercise 5.
r = (3t2 , 2t3 , 3t)
v = (6t, 6t2 , 3)
a = (6, 12t, 0)
p p
s = 36t2 + 36t4 + 9 = 3 4t2 + 4t4 + 1
Exercise 6.
r = (t, sin t, (1 − cos t))
v = (1, c, s)
a = (0, −s, c)

s= 2
Exercise 7.
r = (ac(ωt), as(ωt), bωt)
r0 = (−ωas(ωt), ωac(ωt), bω) = ω(−as, ac, b)
p
|r0 | = ω a2 + b2
r0 bω b
0
· ez = √ = √
|r | 2
ω a +b2 a + b2
2


Exercise 8. |r0 | = ω a2 + b2
a = ω 2 (−ac, −as, 0),|a| = ω 2 a

kv × ak 1 3 2 ω 3 b2 a2 + a4 a
= √ |(ω (bas, −bac, a ))| = √ = 2
kvk3 (ω a2 + b2 )3 ω 3 ( a2 + b2 )3 a + b2
254
Exercise 9. u = (sin (ωt), − cos (ωt), 0)
v × a = ω 3 (bas, −bac, a2 ) = ω 3 bau + ω 3 a2 ez
A = ω 3 ba
B = ω 3 a2
Exercise 10.
d
(v · v) = a · v + v · a = 2v · a
dt
Exercise 11.

(1)

v v·c
· c = cos θ; = |v|
|v| cos θ
(r · c)0 = r0 · c = 2e2t
2e2t
=⇒ = |v|
cos θ
(2)
4e4t
= |v|2
cos2 θ
1 d 2 8e4t
from the previous exercise, v = =a·v
2 dt cos2 θ
Exercise 12. r = (a cosh θ, b sinh θ). a = b = 1.
1
Draw a diagram. You'll immediately see that to get sector OAP, we need to take the area of a right triangle,
2 cosh θ sin theta =
1
2( x-coordinate )( y -coordinate) and subtract away from it the curved piece below the graph of the hyperbola. Thus
Z cosh θ p x2 − y 2 = 1
1
cosh θ sinh θ − x2 − 1dx = A(θ) with p
2 1 x2 − 1 = y
Then
1 p 1
A0 (θ) = (cosh2 θ + sinh2 θ) − cosh2 θ − 1 sinh θ =
2 2
θ
A(θ) =
2
Exercise 13.
r = (a cosh ωt, b sinh ωt)
v = ω(a sinh ωt, b cosh ωt)

a = ω 2 (a cosh ωt, b sinh ωt) = ω 2 r


So a is centrifugal, since a = ω 2 r.
Exercise 14. Parabola.

kX − F k = d(X, L) = |(X − (−2a)ex ) · ex | = X · ex + 2a


Let F = 0, kX − F k = kXk
Let X = d1 u1
X 0 = d01 u1 + d1 u01 kXk = d1 = X · ex + 2a
X 0 · u1 = d01 = X 0 · ex d01 = X 0 · ex
X0
kX 0 k
=T T · u1 = T · ex
−−−−−→
cos θ1 = cos θ2
Exercise 15.

(1) |a| = 4|r|. =⇒ a = −4r.


r(0)=4ex ; v(0)=6ey x = 4 cos (2t)
−−−−−−−−−−−−→
y = 3 sin (2t)
³ x ´2 ³ y ´2
(2) + =1 . Motion is counterclockwise along an ellipse.
4 3
255
x2
Exercise 16. Given that x2 + c(y − x) = 0, then y = x − c

x2
r = (x, y) = (x, x − )
c
2xx0
r0 = (x0 , x0 − )
c
2
r00 = (x00 , x00 − (x02 + xx00 ))
c
2
x = x − (x02 + xx00 )
00 00
c
=⇒ xx00 + x02 = 0 or
(xx0 )0 = 0 so then xx0 = constant =k

dx
x = k =⇒ xdx = kdt
dt
1 2 1 −c2
(xf − x2i ) = k(tf − ti ) =⇒ (−c2 ) = k(T ) so that k =
2 µ ¶ µ2 ¶ 2T
1 ³ c ´2 1 −3c2
−c2
− c2 = = (tf − 0)
2 2 2 4 2T
3T
= tf
4

Exercise 17. Given that Y (t) = X(u(t)),


Y 0 (t) = X 0 (u)u0 (t)

So Y0 is a scalar multiple of X 0; Y 0 k X 0

Y 00 = X 00 (u)(u0 (t))2 + X 0 (u)u00 (t)

So Y 00 k X 00 only if X 0 (u) k X 00 (u) as well. This is not necessarily so.

14.9 Exercises - The unit tangent, the principal normal, and the osculating plane of a curve.
Exercise 1. t=2

(1)

v (1 − t2 , 2t, 1 + t2 )
T = = √
s 2(1 + t2 )
(−3, 4, 5)
T (t = 2) = √
2(5)

v0 s0 v (−2t, 2, 2t) 2(2t)(1 − t2 , 2t, 1 + t2 )
T0 = − 2 = √ −
s s 2(1 + t2 ) 2(1 + t2 )2
√ √ √ √
0 2(−2, 1, 2) 2(2)(−3, 4, 5) 2 2
T (t = 2) = − = ((−10, 5, 10) − (−6, 8, 10)) = (−4, −3, 0)
5 25 25 25
(−4, −3, 0)
N (t = 2) =
5

(2)


a(t = 2) = 6(−2, 1, 2) = 12 2T (t = 2) + 6(N )

Exercise 2. t=π
256
(1)
v (−s, c, et )
T = = √
s 1 + e2t
(0, −1, eπ )
T (t = π) = √
1 + e2π
v0 s0 v (−c, −s, et ) e2t (−s, c, et )
T0 = − 2 = √ −√
s s 1 + e2t 1 + e2t (1 + e2t )
π 2π
(1, 0, e ) e (0, −1, eπ
T 0 (t = π) = √ −√ =
1 + e2π 1 + e2π (1 + e2π )
(1 + e2π , e2π , eπ )
=
(1 + e2π )3/2
(1 + e2π , e2π , eπ )
N (t = π) = √
1 + 3e2π + 2e4π
(2)
√ √
e2π 1 + e2π T (t = π) + 1 + 3e2π + 2e4π N (t = π)
a(t = π) =
1 + e2π
Exercise 3.

(1)

v 3(1, 0, 0) + (0, 0, 4) (3, 0, 4)


T = = =
s 5 5
³ v ´0 v 0
sv0
T0 = = − 2 =
s s s
9t
6(−s, c, 0) + (−3tc, 3ts, 0) √
2
(3(c, s, 0) + (−3ts, 3tc, 4))
= √ − 9+9t +16
9 + 9t2 + 16 9 + 9t2 + 16
6 6
T 0 (t = 0) = (0, 1, 0) − 0 = (0, 1, 0)
5 5
0
N (t = 0) = (0, 1, 0)
(2)

a(t = 0) = 6(0, 1, 0) = 0T + 6N 0 (t = 0)
Exercise 4.

(1)
1p
|T 0 | =(1 + s2 (t/2))
2
v 1 T0 (s, c, − sin (t/2))
T = = (1 − c, s, 2 cos (t/2)) N= 0 = p
s 2 |T | (1 + s2 (t/2))
1
T 0 = (s, c, − sin (t/2)) T (t = π) = (1, 0, 0)
2
√ √
N (t = π) = (0, −1/ 2, −1/ 2)
(2)
a(t = π) = (0, −1, −1)
a(t = π) = 0T + 2N (t = π)
Exercise 5. t=1
(1)

v0 s0 (2, 4t, 0) 4t + 8t3


2 T0 = − 2v = √ − (2t, 2t2 , 1)
(2t, 2t , 1) s s 4t2 + 4t4 + 1 (4t + 4t4 + 1)3/2
2
T =√
4t2 + 4t4 + 1 (2, 4, 0) 12
T 0 (t = 1) = − (2, 2, 1) = 1/9(−2, 4, −4)
T (t = 1) = 1/3(2, 2, 1) 3 27
N (t = 1) = 1/3(−1, 2, −2)
257
(2)

a = (6, 12, 0) = 4(3T ) + 2(3N ) = 12T + 6N

Exercise 6.

(1)

v (1, c, s)
T == √ v0 s0 v (0, −s, c)
s 2 T0 = − 2 = √
µ ¶ s s 2
1 1 1 √ √
T (t = π/4) = √ , , N (t = π/4) = (0, −1/ 2, 1/ 2)
2 2 2
(2)
√ √
a(t = π/4) = (0, −1/ 2, 1/ 2) = N

Exercise 7.
a=0
v = v0 = const.
r = v0 t + r0

Exercise 8.

kv × ak v
=k × ak = kT × ak
kvk kvk
a = s0 T + sT 0 , so the normal component of a is s|T 0 | since sT 0 = s|T 0 |N .
T × a = sT × T 0
p
kT × ak = ksT × T 0 k = |sT |2 |T 0 |2 − (sT · T 0 )2 =
T ·T 0 =0
−−−−−→ |sT ||T 0 | = s|T 0 |

Exercise 9.

(1)

v = v0
r = v0 t + r0 r ∈ line

(2) Counterexample: imagine the particle moving at constant speed, but in an S-shape in one plane and then an S shape
in another plane, out of the rst plane.
(3)

a = a0
v = a0 t + v0
a0
r = t2 + v0 t + r0
2
r = a20 t2 + v0 t + r0 denes a plane.
(4) v·a=0
a = s0 T + sT 0
a · T = s0 = 0 =⇒ s constant
a = sT 0
=⇒
v = kT
So s is constant and a = sT 0 . We already showed in part (b) of this exercise that there's a counterexample.

Exercise 10.

(1)

d
(r × v) = v × v + r × a = r × a = 0
dt
=⇒ r × v = const.
258
(2) Use what you've learned in these sections: use what you've learned about a being completely in the osculating
plane . The osculating plane idea is very useful.

r × v = c =⇒ r × a = 0
a = s T + sT 0 =⇒ r × (s0 T + sT 0 ) = 0
0

Possibilities:

r = c(s T + sT ) (r is a scalar multiple of s0 T + sT 0 ) =⇒ r ∈


0 0
osculating plane

a = 0, v = v0 , r = v0 t + r0 , r is on a line
r = 0, r stationary

(3) If a = |a(t)|r(t)/|r(t)| then r × a = 0 since a is a scalar multiple of r. Then from above, r × a = 0 imples that r is
on a plane.
(4) r × v not necessarily constant if r lies in a plane.
Counterexample:

r = (tc, t2 s, 0)
v = (c − ts, 2ts + t2 c, 0)
r × v = (0, 0, 2t2 cs + t3 c2 − t2 sc − t3 s2 ) = (0, 0, t2 sc + t3 (c2 − s2 ))

Exercise 11. Given that r ∈ plane, the v is also in the plane, otherwise, for some time t, r(t + ∆t) = r + v∆t out of the plane.
(1) We want T × a = v × a = 0.
Given that T · c = cos θ0 = k0 , (T · c)0 = 0 = T 0 · c + T · c0 = T 0 · c.
c 6= 0, so if T · c 6= 1, T 0 · c = 0 implies T 0 = 0. Then T × a = 0.

If T · c = 1, v = sc a = v 0 = s0 c
v × a = sc × s0 c = ss0 (c × c) = 0
(2) Example:
r = (c(t), s(t), t3 )
v = (−s, c, 3t2 )a = (−c, −s, 6t)
Exercise 12.

(1) If y = g(t) ≷ 0, vx ≶ 0. counterclockwise


(2)

3x2 + y 2 = 1
p
r(t) = (f (t), g(t)) = (x, y) = (x, ± 1 − 3x2 )
µ ¶
3x
r0 = x0 , ∓ √ x0
1 − 3x2
0
x = vx = −g(t) = y
∓3x0 x ∓3y
√ = x = ∓3x
1 − 3x 2 y
so vy = 3x (since we want counterclockwise motion)

(3) x0 = 1 − 3x2 =⇒
Z xf Z tf √ ¯x f
dx arcsin ( 3x) ¯¯ 1 √
√ = dt = T = √ ¯ = √ (π/2 − −π/2) = π/ 3
x0 1 − 3x 2
ti 3 ¯ 3
x0
¯ √
Z xf √ 1/ 3
arccos ( 3x) ¯¯ 0 − (−π) √
=T = √ ¯ = √ = π/ 3 (we need T to be positive)
x0 3 ¯ √ 3
−1/ 3

=⇒ 2π/ 3
259
Exercise 13. p
r = (x, y) = (x(t), y(t)) r · ex = x = x2 + y 2 cos θ = r cos θ
0 0 0
r = (x , y ) r0 · ex = x0 = |r0 | cos φ

4 y0 4x
tan φ = cot θ =⇒ ± 0 =
3 x 3y
y0 dy −4x x dy
0
= = (since > 0 for C in rst quadrant, and < 0 in rst quadrant)
x dx 3y r dx
−4 2
=⇒ y 2 = x +C
3
(3/2,1) x2 y2
−−−−→ + =1
3 4
Exercise 14.
(x0 , y 0 )
T =p
x02 + y 02
(y 0 , −x0 )
N=p
x02 + y 02

(±d)(y 0 , −x0 )
(x, y) + (±d)N = (x, y) + p = (x2 , 0)
x02 + y 02

((±)d)y 0
x+ p = x2
x02 + y 02
((±)d)(−x0 )
y+ p =0
x02 + y 02
¯ ¯
¯ ±dy 0 ¯ |d||y 0 |
¯ ¯
|x2 − x| = 2 = ¯x + p − x¯ = p
¯ x02 + y 02 ¯ x02 + y 02
|d||y 0 | 2
Divide the above 2 by the expression for y =⇒ =
±dx0 y
dy 2
± = =⇒ ±ydy = 2dx
dx y
1 2
=⇒ ± (yf − yi2 ) = 2(xf − xi )
2
(1,2) y 2 = 4x
−−−→ ±(y 2 − 4) = 4(x − 1) =⇒
y 2 = −4x + 8

Exercise 15.r0 = A × r
00
(1) r =a=A×v =⇒ a · A = A · A × v = 0
(2)
v =A×r
a=A×v
a = s0 T + sT 0 , a · T = s0
a · T = (A × v) · T = 0 = s0 =⇒ s constant
Now note that
√ 1 1 v·r
|r|0 = ( r · r)0 = √ (v · r + r · v) = √
2 r·r r·r
however v · r = (A × r) · r = 0

=⇒ |r|0 = 0
So the length of the position vector r is held constant. So if r(0) = B , then |r| = kBk for all time t > 0.

We have
|r0 (t)| = s = |A × r| = kAkkrk sin θAr
260
Now s is constant, kAk, kBk are constant. Then sin θAr must be constant as well. r(0) = B so θAr = θAB .

|r0 (t)| = s = |A × r| = kAkkBk sin θAB

(3) See sketch. r starts off as B and r rotates around A like a top.

Exercise 16.

(1)

Y (t) = X[u(t)]
dY (t) d dX du
Y 0 (t) = = (X(u(t))) = (u)
dt dt du dt
0 d dX(u) du du
Y (t) dt (X(u(t))) du dt ¯ dt ¯
= TY (t) = = ¯ ¯ = T X (u(t))
kY 0 (t)k d
k dt (X(u(t)))k k dX(u) k ¯ du ¯ ¯ du ¯
du dt dt
dX(u)
dX X0
since for X(u(t)), (u) = X 0 =⇒ du
= TX (u) = dX(u)
du kX 0 k k du k
¯ ¯
¯ du ¯
¯ ¯ dt ¯ ¯
Now ¯ ¯ du ¯ ¯ = 1 always, so for
¯ dt
¯
¯ ¯
du du ¯¯ du ¯¯
strictly increasing u , >0 =⇒ / =1
dt dt ¯ dt ¯
¯ ¯
du du ¯¯ du ¯¯
strictly decreasing u , <0 =⇒ / = −1
dt dt ¯ dt ¯
strictly increasing u, TY (t) = TX (u)
strictly decreasing u, TY (t) = −TX (u)

(2)

µ ¶0
0 Y0 Y 00 Y0
(TY ) = = − |Y 0 |0
|Y 0 | |Y 0 | |Y 0 |2
Y 0 = X 0 u0
µ ¶
00 d dX(u) 0 d2 X(u) 02 dX(u) 00
Y = u (t) = u + u = X 00 u02 + X 0 u00
dt du du2 du
Y 00 X 00 u02 + X 0 u00
= √
|Y 0 | X 0 u0 · X 0 u0
√ Y 0 · Y 00 X 0 u0 · (X 00 u02 + X 0 u00 )
|Y 0 |0 = ( Y 0 · Y 0 )0 = √ = √
Y0·Y0 X 0 u0 · X 0 u0
0
−Y 0 0 −(X )(X · (X 00 u02 + X 0 u00 ))
0 0
|Y | = √
|Y 0 |2 (X 0 )2 X 0 u0 · X 0 u0
Y 00 Y0 X 00 u02 + X 0 u00 (X 0 )(X 0 · X 00 u02 + X 02 u00 ) X 00 u02 X 0 (X 0 · X 00 )u02
0
− 0 2 |Y 0 |0 = √ − √ =√ − √ =
|Y | |Y | 0 0
X u ·X u 0 0 02 0
X ( X u ·X u) 0 0 0 0 0
X u ·X u 0 0 X 02 X 0 u0 · X 0 u0

u02 = |u0 |2
√ µ ¶
u02 = |u0 | X 00 X 0 |X 0 |0
−−−−−−−−−−→ − |u0 |
|X 0 | |X 0 |2
261
µ ¶ µ 00 ¶
Y 00 Y0 Y Y0 Y 002 2Y 00 · Y 0 Y 0 · Y 00 Y 02 (Y 0 · Y 00 )2
kTY0 k2 = 0
− 0 2
|Y 0 0
| · 0
− 0 2
|Y 0 0
| = 02 − 0 3 0
+ 04 =
|Y | |Y | |Y | |Y | Y |Y | |Y | |Y | Y 02
Y 002 (Y 0 · Y 00 )2
= 02 −
Y (Y 0 )4
(X 00 u02 + X 0 u00 )2 (X 0 u0 · (X 00 u02 + X 0 u00 ))2
= 0 0 2
− =
(X u ) (X 0 u0 )4
X 002 u04 + 2X 00 · X 0 u02 u00 + X 02 u002 (X 0 · X 00 )2 u06 + 2X 0 · X 00 u04 X 02 u00 + (X 02 )2 u02 u002
= 0 0 2
− =
(X u ) (X 0 u0 )4
µ 002 ¶
X (X 0 · X 00 )2
= − u02
X 02 X 04
TY0 T 0 |u0 | T0
0 = X 0 0
= X 0 k =⇒ NY = NX
kTY k kTX k|u | kTX
Thus, the osculating plane is invariant under a parameter change, since the osculating plane can still be made up
of NY = NX and TY = ±TX .

14.13 Exercises - The denition of arc length, Additivity of arc length, The arc-length function.
Exercise 1.

r(t) = a(1 − cos t, t − sin t) 0 ≤ t ≤ 2π, a > 0


0 v 2 = a2 (sin2 t + 1 − 2 cos t + cos2 t) = 2a2 (1 − cos t)
r (t) = a(sin t, 1 − cos t)

√ √
v= 2a 1 − cos t = 2a sin t/2
Z Z

vdt = 2a sin t/2 = −(4a) cos t/2|0 = 8a

Exercise 2.

r(t) = (et cos t, et sin t); 0≤t≤2


0 t t
r (t) = (e (c − s), e (s + c))

v 2 = e2t (c2 + s2 + s2 + c2 ) = 2e2t =⇒ v = 2et
Z 2√ √
2et dt = 2(e2 − 1)
0

Exercise 3.

r(t) = (a(cos t + t sin t), a(sin t − t cos t)) = a(cos t + t sin t, sin t − t cos t)
r0 = a(−s + s + tc, c − c + ts) = a(tc, ts)
v = at
Z 2π
1
at = a 4π 2 = 2π 2 a
0 2

Exercise 4.
µ ¶ µ 3 3¶ 0 ≤ t ≤ 2π
c2 3 c2 3 2 c s
r(t) = cos t, sin t = C ,
a b A B c2 = a2 − b2 , 0 < b < a
µ 2 ¶
−c s s2 c
r0 = 3C ,
A B
µ 4 2 ¶ µ 2 ¶
2 2 c s s4 c2 4 2 2 c s2
v = 9C + 2 = 9C c s + 2
A2 B A2 B
r √
c2 s2 B 2 + C 2 s2
v = 3C 2 |cs| 2
+ 2 = 3C 2 |cs|
A B AB
262
Z p 1
cs B 2 + C 2 s2 = (B 2 + C 2 s2 )3/2
3C 2
ÃZ Z Z 3π/2 p Z !
3C 2 π/2 p π p 2π p
s= cs B 2 + C 2 s2 − cs B 2 + C 2 s2 + cs B 2 + C 2 s2 − cs B 2 + C 2 s2 =
AB 0 π/2 π 3π/2

3C 2 4
= ((B 2 + C 2 )3/2 − B 3 ) = 4(A3 − B 3 )/AB
AB 3C 2
Exercise 5.
r(t) = (a(sinh t − t), a(cosh t − 1))
r0 (t) = a(cosh t − 1, sinh t)
v 2 = a2 (cosh2 t − 2 cosh t + 1 + sinh2 t) = a2 (2)(cosh t)(cosh t − 1)

Knowing that cosh (2t) = cosh2 t + sinh2 t = 2 cosh2 t − 1 = 1 + 2 sinh2 t


Z Z T √ p √ Z Tq √
s= |v|dt = a 2 cosh t(cosh t − 1)dt = 2a 2 cosh2 t/2 − 1 2 sinh t/2dt =
0 0

u = cosh t/2 2u = y
1 dy
du = sinh t/2dt Z p du = √ Z p
2 2 2 4a
−−−−−−−−−−−−−−→ 2a 2u − 12du −−−−−−−−−→ √ y 2 − 1dy
2
Z p p Z p Z 2 p Z p Z
2 2
y2 2
y −1+1 2 2
−1
y − 1dy = y y − 1 − p =y y −1− p =y y −1− y − 1dy + p
2
y −1 2
y −1 y2 − 1
à !
p 1 y 1
since (ln (y ± y 2 − 1))0 = p 1± p = ±p
y± y −1 2 2
y −1 2
y −1
Z p p p
1
=⇒ y 2 − 1dy = (y y 2 − 1 − ln (y + y 2 − 1))
2
µ q q ¶¯T
4a 1 √ √ ¯
s= √ 2 cosh t/2 2 cosh2 t/2 − 1 − ln ( 2 cosh t/2 + 2 cosh2 t/2 − 1) ¯¯ =
22 0
Ã√ √ !
√ √ 2 cosh T /2 + cosh T
= 2a(cosh T /2 cosh T − 1) + − 2a ln √
2+1

Exercise 6.
r(t) = (sin t, t, (1 − cos t)); (0 ≤ t ≤ 2π)
0
r = (c, 1, s)
Z 2π √ √
2 2 2
v =c +1+s =2 =⇒ 2dt = 2(2π)
0

Exercise 7.

r(t) = (t, 3t2 , 6t3 ), (0 ≤ t ≤ 2)


0 2
r = (1, 6t, 18t )
v 2 = 1 + 36t2 + 182 t4
Z 2p Z 2
¯2
s= 2 2 2
(18t ) + 2(18)t + 1dy = (18t2 + 1)dt = (6t3 + t)¯0 = 50
0 0

Exercise 8.
π
r = (t, log (sec t), log (sec t + tan t)), (0 ≤ t ≤ )
4
r0 = (1, tan t, sec t)
v 2 = 1 + tan2 t + sec2 t = 2 sec2 t
Z π/4 √ √ √ √
π/4
s= 2 sec tdt = 2 (ln | tan t + sec t|)|0 = 2 ln (1 + 2)
0
263
Exercise 9.

r(t) = (a cos ωt, a sin ωt, bωt), (t0 ≤ t ≤ t1 )


0
r = (ω)(−as(ωt), ac(ωt), b)
2 2 2 2
v = ω (a + b )
Z t1 p p
s= |ω| a2 + b2 = |ω| a2 + b2 (t1 − t0 )
t0

Exercise 10.

r = (x, y) = (g(y), y)
µ ¶
dg(y(t)) dy dy
r0 = ,
dy dt dt
õ ¶ !µ ¶
2 2
02 dg dy
r = +1
dy dt
s µ ¶2 s µ ¶2
Z tf Z y(tf )
dg(y(t)) dy dg
s= 1+ dt = 1+ dy
t0 dy dt y(t0 ) dy

Exercise 11. Given y 2 = x3 ,

r = (x, ±x3/2 )
dy 3
= ± x1/2
dx 2
Z r ¯1 Ã √ !
1
9 8 9 3/2 ¯¯ 8 26 13
1+ x= (1 + x) ¯ = −2
0 4 27 4 0 27 8

r = (y 2/3 , y)
dg 2
= y −1/3
dy 3
s q ¡ ¢2
Z 1 µ ¶2 Z 1 ¯1 ¯0
2 y 2/3 + 23 2/3 2 3/2 ¯ 2/3 2 3/2 ¯
1+ y −2/3 dy = dy = (y + (2/3) ) ¯ − (y + (2/3) ) ¯ =
−1 3 −1 |y|1/3 0 −1

µ ¶3/2 µ ¶3 õ ¶3 µ ¶3/2 ! à √ !
13 2 2 13 13 13 8
= − − − = 2 −
9 3 3 9 27 27

Exercise 12.

r = (cos θ, sin θ)
r0 = (− sin θ, cos θ)
v2 = 1
Z θ1 µ ¶
θ1 − θ0
s= 1dθ = (θ1 − θ0 ) = 2
θ0 2

So the arc AB has a length equal to twice the area of the sector.
Exercise 13.

(1)

y = ex , 0 ≤ x ≤ 1
Z 1p
1 + e2x dx
0
264
(2)
(x, y) = (t + log t, t − log t); 1 ≤ t ≤ e
1 1
r0 = (1 + , 1 − )
t t s
µ ¶2 µ ¶2
02 1 √ 1
r = 2(1 + ); =⇒ 2 1 +
t t
t = ex
Z e √ p dt = ex dx Z 1 √ p √ Z 1 p
2 1 + (1/t)2 dt −−−−−−−−→ 2 1 + e−2x ex dx = 2 1 + e2x dx
1 0 0
Exercise 14.
y = c cosh (x/c)
dy
= sinh (x/c)
Z aq dx Z
a
x ¯¯a a
1 + sinh2 (x/c)dx = cosh x/adx = c sinh ¯ = c sinh
0 0 c 0 c
Ra
So indeed,
0
c cosh (x/a) dx gives the area underneath a curve cosh x/a from x = 0 to x = a.
Exercise 15. Given y = cosh x, (0, 1) and (x, cosh x) is sinh x if x > 0
Z xp
x
1 + sinh2 tdt = sinh t|0 = sinh x
0

Exercise 16.  s 
s µ ¶2 µ ¶2
Z b Z b Z b
dy k dy
A= f (x)dx = k 1+ dx =⇒ 1+ − f (x) dx = 0
a a dx a dx
r³ ´ Z y
y 2 dy dy
± −1= ; ±(x − x0 ) = q¡ ¢
k dx y0 y 2
k −1
Ry
q dy could be evaluated by cosh u = y/k substitution, and then solve for u, (inverse of cosh u), by cosh u =
y0 2
( ky ) −1
eu +e−u
2 = y/k and then use the quadratic equation trick to get eu .
µ q ¶
y y2 r
k + k2 − 1 y0 y02
±(x − x0 ) = k ln µ q ¶ Let C0 = ± −1
y y2 k k2
k + k2 − 1
µ µ ¶ ¶2
±(x − x0 ) y y2
C0 e − = 2 −1
k k k
µ ¶ µ ¶
2 ±2(x − x0 ) ±(x − x0 ) y
C0 e + 1 = 2C0 e
k k k
³ ³ ´ ³ ´´
k C0 e ±(x−x k
0)
+ e ∓(x−xk
0)

y=
2
µ ¶
x − x0
If we choose y0 = k , so that C0 = 1, then y = k cosh . Also note that y = k works as well.
k
Exercise 17.
r = (a sin t, b cos t) 0<b<a
0
r = (ac, −bs)
µ µ 2 ¶ ¶
02 2 2 2 2 2 2 a − b22 2 2
r = a c + b s = a (1 − s ) + b s = a 1 − s2
a2
Z 2π p Z π/2 p
L= |a| 1 − e2 sin2 tdt = 4a 1 − e2 sin2 tdt
0 0
265
Exercise 18.

b
r(t) = (a(t − sin t), a(1 − cos t), b sin t/2) = a(t − sin t, (1 − cos t), sin t/2)
a
b
r0 = a(1 − cos t, sin t, cos t/2)
2a
µ ¶2 µ ¶2
02 2 b 2 2 b
r = a (1 − 2 cos t + 1 + cos t/2) = 2a (1 − cos t + (1 + cos t)) =
2a 4a
µ ¶2 Ã µ ¶2 !
b b
= 2a2 (2 sin2 t/2 + (2 + −2 sin2 t/2)) = 4a2 + k 2 sin2 t/2 = 4a2 (1 − k 2 + k 2 sin2 t/2) =
4a 4a
= 4a2 (1 − k 2 cos2 t/2)
p
|r0 | = 2a 1 − k 2 cos2 t/2
Z 2π p Z π p
2a 1 − k 2 cos2 t/2 dt = 4a 1 − k 2 cos2 t dt
0 0
Z −π/2 p Z π/2 p
t=π/2−θ
−−−−−−→ = (−4a) 1 − k 2 sin2 t dt = 8a 1 − k 2 sin2 t dt
π/2 0

Exercise 19.
A · B = cos π/3 = 1/2
(A × B)2 = A2 B 2 − (A · B)2 = 1 − 1/4 = 3/4
µ ¶3/2
2 2
r = tA + t B + 2 t A×B
3
µ ¶1/2
2
r0 = A + 2tB + 2 t A×B
3
µ ¶ µ ¶
1 2t 2t 3
r02 = 1 + 2t + + (2t)2 + 4 = 1 + 4t + 4t2 = (2t + 1)2
2 2 3 4
Z T
L= (2t + 1)dt = T 2 + T = 12 =⇒ (T + 4)(T − 3) = 0 T = 3
0
Exercise 20.

(1)
The circle rotates about its center and the circles
rolls (moves along x-axis) without slipping, translating its origin by how far its circumference moved
through.

θ0 = −π/2
a(cos θ, − sin θ) (clockwise rotation of pt. on circumference)
a(cos (θ − θ0 ), − sin (θ − θ0 )) = a(− sin θ, − cos θ)
(θa, a) is the position of the circle center.

(θa, a) + a(− sin θ, − cos θ) = a(θ − sin θ, 1 − cos θ)


dy sin θ 2 sin θ/2 cos θ/2
(2)
dx = 1−cos θ = 2 sin2 θ/2
= cot θ/2
(1, cot θ/2)
T =p = (sin θ/2, cos θ/2)
1 + cot2 θ/2
µ ¶
π θ
T · ex = sin θ/2 = cos φ = cos −
2 2
π−θ
φ=
2
We'll show that the tangent line passes through the highest point at each place the marked point rotates through.

Marked point on the circle rotated through by θ: P = a(θ − sin θ, 1 − cos θ)


Tangent line: P + sT = a(θ − sin θ, 1 − cos θ) + s(sin θ/2, cos θ/2)
266
Highest pt. on the circle: (aθ, 2a)

equate the y -coordinates: 2a = a(1 − cos θ) + s cos θ/2 =⇒ s = 2a| cos θ/2|

a(θ − sin θ) + s sin θ/2 = aθ − a sin θ + s sin θ/2 =


equate the x-coordinates: = aθ − a sin θ + a2 cos θ/2 sin θ/2 =
= aθ
So indeed, the tangent line will always intersect the highest point of the circle.

Exercise 21.
dY dX du g(u) = t
=
dt du dt u = u(t)
Z d Z d ¯ ¯
¯ du(t) ¯
kY 0 (t)kdt = kX 0 (u(t))k ¯¯ ¯ dt du dg
dt ¯ = 1/ (u(t))
c c dt du
g(u) = t is strictly increasing as it traces the arc, since c < t < d.
du
Then
dt > 0
Z d µ ¶ Z u(d)
du
=⇒ kX 0 (u)k dt = kX 0 (u)kdu
c dt u(c)
1
Exercise 22. For t= k, k = 2, . . . , 2n
2n
X
kπ(P )k = kr(tk ) − r(tk−1 )k + kr(t = 0) − r(t2n )k + (term to close up this polygon)
k=2
µ ¶
1 √
kr(t = 0) − r(t2n )k = 2
2n
³π´
r = (t, t cos )
µ 2t µ ¶ µ ¶¶
−1 1 kπ 1 kπ
r(tk ) − r(tk−1 ) = , cos − sin
k(k − 1) k 2 k−1 2
µ ¶2 µ ¶ ¡ ¢ ¡ kπ ¢ µ ¶
2 1 1 2 kπ −2 cos kπ 2 sin 2 1 2 kπ
kr(tk ) − r(tk−1 )k = + 2 cos + + sin =
k(k − 1) k 2 k(k − 1) (k − 1)2 2
µ ¶2 µ ¶ µ ¶
1 1 2 kπ 1 2 kπ
= + 2 cos + sin since k ∈ Z
k(k − 1) k 2 (k − 1)2 2
k = 2, . . . , 2n
k = 2j j = 1, . . . , n
µ ¶2
1 1
kr(tk ) − r(tk−1 )k2 = + 2
k(k − 1) k
s
1 1 1
kr(tk ) − r(tk−1 )k = 1+ 2
>
k (k − 1) k
k = 2j + 1, j = 1, . . . , n − 1
µ ¶2 µ ¶2 µ ¶2 µ ¶
1 1 1 1
kr(tk ) − r(tk−1 )k2 = + = 1+ 2
k(k − 1) k−1 k−1 k
r
1 1 1 1
kr(tk ) − r(tk−1 )k = 1+ 2 > >
k−1 k k−1 k
These don't seem like very “tight” inequalities, but it'll work for the purposes of this problem.
2n
X
kπ(P )k = kr(tk ) − r(tk−1 )k + kr(t = 0) − r(t2n )k + (term to close up this polygon) >
k=2
2n √ 2n √ 2n
X 1 2 X 1 2 X 1
> + + kr(t = 0) − r(t = 1)k = + +1> >
k 2n k 2n k
k=2 k=2 k=1
267
Now, by denition, Λ, the arclength, is the least upper bound on all possible polgonal approximations to the length of a curve.
P2n 1
k=2 k is a series that is unbounded as n → ∞. So Λ does not exist.

14.15 Exercises - Curvature of a curve.


Exercise 1.

(1)

r = ((3t − t3 ), 3t2 , (3t + t3 )) a(t = 2) = 6(−2, 1, 2); |a| = 18


2 2 2 2

v = (3 − 3t , 6t, 3 + 3t ) = 3(1 − t , 2t, 1 + t ) v(t = 2) = 3(−3, 4, 5); |v| = 15 2
a = 3(−2t, 2, 2t) = 6(−t, 1, t) a · v = 18(6 + 4 + 10) = 360
p
182 152 2 − (18 · 20)2 1
κ= √ =
3
15 2 · 2 75

(2)
p
r = (c, s, et ) a(t = π) = (1, 0, eπ ) |a| = 1 + e2π
p
v = (−s, c, et ) v(t = π) = (0, −1, eπ ) |v| = 1 + e2π
a = (−c, −s, et ) a · v = e2π
√ √
1 + 2e2π + e4π − e4π 1 + 2e2π
κ= 2π 3/2
=
(1 + e ) (1 + e2π )3/2

(3)

r = (3tc, 3ts, 4t) a(t = 0) = 6(0, 1, 0) |a| = 6


v = (3(c − ts), 3(s + tc), 4) v(t = 0) = (3, 0, 4) |v| = 5
a = 6(−s, c, 0) + (−3tc, 3ts, 0) a·v =0
6
κ=
25
(4)

r00 · r0 = s − cs + sc − 2 cos t/2 sin t/2 = 0


r0 = (1 − c, s, 2 cos t/2)
v = k(2, 0, 0)k = 2
r00 = (s, c, − sin t/2) √
a = k(0, −1, −1)k = 2

2
κ=
4
(5)

t=1
r00 · r0 = 9(4t + 8t3 ) −−→ 9(12)
r = (3t2 , 2t3 , 3t) p
a = 3 (4 + 16t2 )
r0 = (6t, 6t2 , 3) = 3(2t, 2t2 , 1) √
a(t = 1) = 6 5
r00 = 3(2, 4t, 0) p
v = 3 4t2 + 4t4 + 1; v(t = 1) = 9
q √
ka × vk (6 5)2 92 − (9(12))2 2
3
= 2
=
v 27 27
(6)

r00 · r0 = 0
r(t) = (t, sin t, (1 − cos t))
a=1
r0 (t) = (1, cos t, sin t) √
ka × vk 2 1
r00 = (0, −s, c) κ= = 3/2 =
v3 2 2
268
Exercise 2.

r = (ac(ωt), as(ωt), bωt) r0 · r00 = 0


p
r0 = ω(−as(ωt), ac(ωt), b) |r0 | = ω a2 + b2
r00 = ω 2 (−ac(ωt), −as(ωt), 0) |r00 | = ω 2 a

kr00 × r0 k a a2 + b2 a
= = 2
|r0 |3 (a2 + b2 )3/2 a + b2

Exercise 3. Given A, B , A · B = cos θ. A, B xed. |A| = |B| = 1.


0
r =A×r
=⇒ r(0) = B, kr(t)k = 1
r00 = A × r0
r00 × r0 = (A × r0 ) × (A × r) = (A × (A × R)) × (A × r) =
= (A × r) × ((A × r) × A) = (A · (A × r))(A × r) − (A × r)2 A = −(A × r)2 A
kr00 × r0 k 1 1
= =
kA × rk3 kA × rk kBk sin θ

Exercise 4.

(1) r(t) lies on a plane since a plane M = {P + sA + tB}


r(t) = 4(c, s, c) = 4c(1, 0, 1) + 4s(0, 1, 0) = 4cA + 4sB
r(t) ∈ M1 where M1 = {0 + sA + tB} = {sA + tB}
(1,0,1)
Normalize A, B → A = √
2
; B = (0, 1, 0)

X = 4 2cA + 4sB

Recall an ellipse: kX − F k = ed(X, L) = e|(X − (F + dN )) · N | = e|(X − F ) · N − d|


Suppose kF k = f and supposeF = fA

X − F = (4 2c − f )A + 4sB
√ √
kX − F k2 = 32c2 − 8 2cf + f 2 + 16s2 = 16 + 16c2 − 8 2cf + f 2
e2 ((X − F ) · N − d)2 = e2 (((X − F ) · N )2 − 2((X − F ) · N )d + d2 )
√ e2 =1/2 √ 1
((X − F ) · N )2 = 32c2 − 8 2cf + f 2 −−−−−→ 16c2 − 4 2cf + f 2
√ 2
−2e2 ((X − F ) · N )d = −d(4 2c − f ) =⇒ d = f
1
e2 d2 = f 2 =⇒ 16 + f 2 = 2f 2 or f = 4
2
q
√ √
kX − F1 k = k(4 2c − 4)A + 4sBk = 32c2 − 32 2c + 16 + 16s2 =
q
Indeed for
√ √
= 16c2 − 32 2c + 32 = 4( 2 − c)
√ √
kX − F2 k = k(4 2c + 4)A + 4sBk = 4( 2 + c)

=⇒ kX − F1 k + kX − F2 k = 8 2
A Cartesian equation for the plane containing this ellipse would be −x + z = 0 .
(2)

r = 4(c, s, c) r02 = 16(1 + s2 )


r0 = 4(−s, c, −s) r002 = 16(1 + c2 )
r00 = 4(−c, −s, −c) = −4(c, s, c) r0 · r00 = −16(−sc) = 16sc
p √
kr00 × r0 k 162 (1 + c2 )(1 + s2 ) − 162 (sc)2 2
0 3
= 0 3
=
|r | |r | 4(1 + s2 )3/2
1 √
= 2 2(1 + s2 )3/2
κ
269
Exercise 5. √ √ √ √ √
r = (et , e−t ,
2t) r00 × r0 = ( 2e−t , 2et , −1 − 1) = ( 2e−t , 2et , −2)
√ p √
0 t −t
r = (e , −e , 2) kr00 × r0 k = 2e−2t + 2e2t + 4 = 2(et + e−t )
r00 = (et , e−t , 0) r02 = e2t + e−2t + 2 = (et + e−t )2

kr00 × r0 k 2
=
|r0 |3 et + e−t
Exercise 6.

(1)
p
kr00 × r0 k (x002 + y 002 )(x02 + y 02 ) − (x00 x0 + y 00 y 0 )2
= =
|r0 |3 (x02 + y 02 )3/2
r(t) = (x(t), y(t)) p
x002 x02 + x002 y 02 + y 002 x02 + y 002 y 02 − (x002 x02 + 2x00 x0 y 00 y 0 + y 002 y 02 )
r0 = (x0 , y 0 ) = =
(x02 + y 02 )3/2
r00 = (x00 , y 00 ) p
(x00 y 0 − y 00 x0 )2 |x00 y 0 − y 00 x0 |
= =
(x02 + y 02 )3/2 (x02 + y 02 )3/2
(2) For (x, y(x)), then we could treat the x as the parameter. Then x0 = 1 and x00 = 0. We can reuse our derived
expression above:

|x00 y 0 − y 00 x0 | |y 00 |
κ= =
(x02 + y 02 )3/2 (1 + y 02 )3/2
Exercise 7. A point moves so its velocity and acceleration vectors always have constant lengths.

X X 0 = v0 T
X0 |X 0 | = v0 X 00 = v0 T 0 = v02 κN
X 00 |X 00 | = a0 kX 00 k = v0 κ = a0
kT 0 k a0
since κ= , so then κ=
v0 v02

Exercise 8. If 2 plane curves with y = f (x), y = g(x) have the same tangent at a point (a, b) and the same curvature, prove
|f 00 (a)| = |g 00 (a)|.
X1 = (x, f (x)) X2 = (x, g(x))
X10 0
= (1, f (x)) X20 = (1, g 0 (x))
X100 = (0, f 00 (x)) X200 = (0, g 00 (x))
p
kX100 × X10 k |f 00 |2 (1 + f 02 ) − (f 00 f 0 )2
= =
(1 + f 02 )3/2 (1 + f 02 )3/2 kX200 × X20 k |g 00 |
, likewise =
|f 00 | (1 + g 02 )3/2 (1 + g 02 )3/2
=
(1 + f 02 )3/2
For (a, b), the 2 plane curves have the same tangent, so then f 0 (a) = g 0 (a). So if the curvature is the same at (a, b), then
00 00
|f (a)| |g (a)|
= =⇒ |g 00 (a)| = |f 00 (a)|
(1 + f 02 (a))3/2 (1 + g 02 (a))3/2
Exercise 9.

(1) This problem could be done without reference to the same curvature or the same tangent line. We only need to use
the fact that they intersect at the same point.

Given f = ax(b − x), (x + 2)g = x,


(b + 2)g = b
If b 6= 2
If x = b, f = 0
b
g=
b+2

Same as x = 0, so that f = 0, g = 0
270
So one point of intersection for any a, b is (0, 0). Otherwise,

x
ax(b − x) = x 6== −2( this is okay, plug x = −2 into (x + 2)g = x and you'll get a contradiction )
x+2
p
−(2 − b) 4 + 4b + b2 − 4/a
=⇒ x = ±
2 2
So we must have, for only one pt. of intersection, (0, 0), b = 2 and
p p
4 + 4b + b2 − 4/a = 4 + 8 + 4 − 4/a = 0 =⇒ a = 1/4

(2) See sketch. The parabola and (x+2)g = x coincide at (0, 0), have the same tangent line, and have the same direction
at this intersection point.

Exercise 10. Remember that the radius of curvature is the reciprocal of the curvature.

(1) Without loss of generality, y = 4cx2 . Then


0
y = 8cx 00
|f |
=⇒ κ = . Since 1 > 0, 64 + c2 x2 > 0, and |8c| constant, for κ maximized, then x = 0 .
y 00 = 8c (1+f 02 )3/2

(2) Given 2 xed unit vectors A, B , A · B = cos θ,


r = tA + t2 B r00 × r0 = 2B × (A + 2tB) = 2B × A
r0 = A + 2tB |r00 × r0 | = 2 sin θ
r00 = 2B r02 = 1 + 4t cos θ + 4t2
(r02 )0 = 8t + 4 cos θ = 0t = − cos θ/2 (r02 )00 = 8 > 0

cos θ cos2 θ
So t = − cos θ/2 minimizes r02 and so maximizes κ. Then r=− A+ B
2 4
Exercise 11.

(1) We have for this plane curve,


r = (x, y)
p (x(0), y(0)) = (0, 0)
r0 = (x0 , y 0 ) Given x02 + y 02 = 5, and given
(x0 (0), y 0 (0)) = (0, 5)
r00 = (x00 , y 00 )

a × v = ez (x00 y 0 − x0 y 00 )
|a × v| |x00 y 0 − x0 y 00 |
3
κ== = 2t
v 125
0 0 0 0 0
We want (x , y ) · ex = 5 cos θ = x . Then y = 5 sin θ . Also, note that x (0) = 0 =⇒ θ0 π/2

=⇒ 250t = | − 5 sin θθ0 (5 sin θ) − 5 cos θ5 cos θθ0 |

10t = |θ0 | θ = 5t2 + π/2


(2) v = 5(cos (5t2 + π/2), sin (5t2 + π/2))
Exercise 12.
p x0 = 2 cos θ
Given that x02 + y 02 = 2, then θ0 = 0 since v0 = 2i
y 0 = 2 sin θ
|x00 y 0 − x0 y 00 | | − 2 sin θθ0 2 sin θ − 2 cos θ2 cos θθ0 | |θ0 | θ0 = 8t
κ(t) = = = = 4t
v3 23 2 θ = 4t2

v = 2(1, 1)

14.19 Exercises - Velocity and acceleration in polar coordinates, Plane motion with radial acceleration, Cylindrical
coordinates.

Exercise 1.

271
dr dr dθ
v= = ~er + r ~eθ = ~er + r~eθ (θ = t)
dt
à dt dt
2
µ ¶2 ! µ ¶
d r dθ 1 d 2 dθ
a= −r ~er + r ~eθ = −r~er + 2~eθ
dt2 dt r dt dt
v = ~er + t~eθ = cos t~ex + sin t~ey + −t sin t~ex + t cos t~ey =
= (cos t − t sin t)~ex + (sin t + t cos t)~ey
a = (−t cos t − 2 sin t)~ex + (−t sin t + 2 cos t)~ey
Exercise 2.
v = ~er + r~eθ + ~ez = (cos t − t sin t)~ex + (sin t + t cos t)~ey + ~ez
a = (−t cos t − 2 sin t)~ex + (t sin t + 2 cos t)~ey
Exercise 3. (a).

π
r = sin t; θ = t; z = log sec t; θ ≤ t <
2
2 1 2 2 1 1
(r cos θ) + (r sin θ − ) = r − r sin θ + =
2 4 4
(b).

dt dθ tan θ sec θ
v= ~er + r ~eθ + log sec t~ez = cos t~er + r~eθ + ~ez
dt dt sec θ
vz = tan θ; v 2 = cos2 t + r2 + tan2 θ = sec2 θ
tan θ
cos φ = = sin θ = r = sin t
sec θ
φ = arccos (sin θ)
Exercise 4. Given r = f (θ); a ≤ θ ≤ b ≤ a + 2π
X = rur X 02 = r02 + r2 θ02
p
X 0 = r0 ur + ruθ θ0 |X 0 | = r2 + r02
s µ ¶2
Z p Z b
2 02 02 t=θ 2
dr
s= r + r θ dt −−→ s = r + dθ
a dθ
Exercise 5. Given
r = a(1 + cos θ)
so then r2 + r02 = a2 (1 + 2 cos θ + cos2 θ + sin2 θ) = 4a2 cos2 θ/2
r0 = −a sin θ
Z p Z 2π µZ π Z 2π ¶
s= r2 + r02 dθ = 2a
cos θ/2dθ = 2a cos θ/2dθ + − cos θ/2dθ =
0 0 π
³ ´
π 2π
= 4a sin θ/2|0 − sin θ/2|π = 4a(1 + 1) = 8a

Exercise 6.
Z Z 2π ¯2π
2 1 2cθ e2cθ ¯¯ e4πc − 1
A= R (θ)dθ = e dθ = =
0 2 4c ¯0 4c
Exercise 7.
Z π Z π Z π µµ ¶ µ ¶¶
1 1 1 1 − cos 2θ 1 − cos 4θ
sin4 θdθ = sin2 θ(1 − cos2 θ)dθ = − dθ = 3π/16
0 2 2 0 2 0 2 2(4)

See sketches for exercises 8-11. Exercise 8. Given r = θ; 0≤θ≤π


Z π p1 ³ p 2 p ´¯π
¯
s= θ2 + 1dθ =
θ θ + 1 + ln (θ + θ2 + 1) ¯ =
0 2 0

1 p p
= (π π 2 + 1 + ln (π + π 2 + 1))
2
272
Exercise 9. Given r = eθ ; 0≤θ≤π
dr
= eθ
dθ Z
πp √ Z π √
s= e2θ + e2θ dθ = 2 eθ dθ = 2(eπ − 1)
0 0

dr
Exercise 10. r = 1 + cos θ ; dθ = − sin θ ; 0≤θ≤π
s µ ¶2 Z π
Z π √ √ √ Z π√
2
dr π
r + = 2 1 + cos θ = 2 2 cos θ/2 = 4 (sin θ/2)|0 = 4
0 dθ 0 0

dr
Exercise 11. Given r = 1 − cos θ; 0 ≤ θ ≤ 2π , then dθ = sin θ
s µ ¶2 Z 2π
Z 2π √ √ √ Z 2π √
dr 2π
r2 + = 2 1 − cos θ = 2 2| sin θ/2| = 4 − cos θ/2|0 = 8
0 dθ 0 0

1
Exercise 12. Given r = f (θ), ρ = radius of curvature = κ, r0 = f 0 (θ), r00 = f 00 (θ)
X = rur
X 0 = r0 ur + ruθ θ0 ur = (cos θ, sin θ)
X 00 = r00 ur + 2r0 uθ θ0 + −rθ0 ur + rθ00 uθ = uθ = (− sin θ, cos θ)
1 ur × uθ = ez
= (r00 − rθ0 )ur + (r2 θ0 )0 utheta
r
1 2 0 0 1
X × X = ((r − rθ )ur + (r θ ) uθ ) × (r0 ur + ruθ θ0 ) = (r2 θ0 )0 r0 (−ez ) + (r00 − rθ0 )rθ0 ez
00 0 00 0
r r
kX 00 × X 0 k = r00 rθ0 − r2 θ02 − 2r02 θ0 − rr0 θ00
|X 0 |2 = r02 + r2 θ02

If we let θ = t, since the curve is described completely by r = f (θ), then


|r2 − r00 r + 2r02 | (r02 + r2 )3/2
κ= =⇒ ρ =
(r02 + r2 )3/2 |r2 − r00 r + 2r02 |

(r 2 +r 02 )3/2
Exercise 13. Using ρ= |r 2 −rr 00 +2r 02 |
(1)
r=θ
(θ2 + 1)3/2
r0 = 1 ρ=
θ2 + 2
r00 = 0
(2)

r = eθ
(e2θ + e2θ )3/2 √
r 0 = eθ ρ= 2θ
= 2eθ
2e
r00 = eθ
(3) θ = π/4
r = 1 + cos θ p √
0 (1 + 2c + c2 + s2 )3/2 (2(1 + c))3/2 2 2+ 2
r = − sin θ ρ= = =
|1 + 2c + c2 + (c + c2 ) + 2s2 | 3(1 + c) 3
r00 = − cos θ

(4) θ = π/2
r = 1 − cos θ √ √
(2(1 − c))3/2 2 2(1 − c)1/2 π/2 2 2
r0 = sin θ ρ= = −−→
(3 + 2c)(1 − c) 3 + 2c 3
r00 = cos θ
273
Exercise 14.
X = rur
X 0 = r0 ur + ruθ θ0 =⇒ X · X 0 = rr0 = rv cos φ or r0 = v cos φ
X 0 · X 0 = r02 + r2 θ02 = v 2

v 2 = v 2 cos2 φ + r2 θ02
Use θ as the parameter for t, since the curve is invariant. =⇒ r = v sin φ
Exercise 15. Place target at the center (without loss of generality). The strategy is to break up v into the polar coordinate unit
vectors.
r = r~er
dr dθ
v= ~er + r ~eθ
dt dt
dr
= vr = v cos (π − α) = −v cos α
dt

r = v sin α
dt
v sin α r dθ
dt dθ 1 dr
= − tan α = dr =r ; = − tan α
−v cos α dt
dr r dθ
r = e− tan αθ
Exercise 16. Admittedly, I had looked in the back of the book.

The big trick is that the choice of origin is important!. The clever choice of origin will cut out the algebra in
dealing with Cartesian coordinates. The clever choice of origin will illuminate the problem entirely. So try ipping around,
reecting, and turning around the problem until the right choice of origin is selected.

Let the positive axis be from where the missle, the target missile, m, is sighted to the ground crew. So the origin is at the
target missile, not the ground crew from which the anti-missile missile, a, is launched! It'll make the algebra much easier.
Let a go 3 miles in the negative x direction, to allow for the possibility that the missile is returning to the ground crew,
that is4 mi. in the positive x direction from the origin, at the time of sighting. Let t = 0 be the time that missile a nishes
going these 3 miles and is 1 mile in the positive x direction from the origin.
Let

(xa , ya ) = Xa = ra ua
(x0a , ya0 ) = Xa0 = ra0 ua + ra θ0 uθ
(xm , ym ) = 1(t + 1)(cos θm0 , sin θm0 ) = (t + 1)um0 = Xm p
|Xa0 | = ra02 + (ra θ0 )2 = 3
(x0m , ym
0
) = um0
Xa (t = 0) = Xa0 = (1, 0) = ra (0)(cos θ(0), sin θ(0))
θ(0) = 0, ra (0) = 1
At some point t2 , (xm , ym ) = (xa , ya ), i.e. Xm (t2 ) = Xa (t2 ). Note that we only need one t2 ; θm0 constant, but unknown.

(t2 + 1) cos θm0 = ra cos θ


(t2 + 1) sin θm0 = ra sin θ =
2
(t2 + 1) = ra2 ; (t2 + 1) = ra (t2 ); ra0 (t2 ) =? well


(ra0 (t2 ))2 + ((t2 + 1) (t2 ))2 = 9
dt
Try 1 = ra0 (t2 ).
1 + (t2 + 1)2 θ02 = 9
µ ¶ √
dθ √ 2 2
(t2 + 1) = 2 2 =⇒ dθ = dt
dt t2 + 1
µ ¶
√ θ
=⇒ θ = 2 2 ln (t2 + 1) t2 = e √ −1
2 2
µ ¶
θ
r=e √
2 2
274
Exercise 17.

A rst order differential equation of the form y 0 = f (x, y) is homogeneous if f (tx, ty) = f (x, y). Then

f (r cos θ, r sin θ) = f (cos θ, sin θ) = f (θ)

We nd that
dy dr
= sin θ + r cos θ
dθ dθ
dx dr
= cos θ − r sin θ
dθ dθ
Thus
dr
dy dθ sin θ + r cos θ
= dr
= f (θ)
dx dθ cos θ − r sin θ

Exercise 18.

v = ωk × r
dr dθ
v= ~er + r ~eθ
dt dt
dt dθ dθ
v · ~er = 0, so = 0; ωk × r = r ~eθ = ωr~eθ = r
dt dt dt
µ ¶2
2 dθ
|ωk × r| = ω 2 r2 = r2
dt
¯ ¯
¯ dθ ¯
ω = ¯¯ ¯¯ , ω > 0
dt

Exercise 19. (a)

dr dθ dθ dr
v= ~er + r ~eθ = r ~eθ ; = 0; ~eθ = ~ez × ~er
dt dt dt dt
dθ dθ
v = r ~ez × ~er = ~ez × r~er = ω × r
dt dt
(b).

a = v 0 = ω 0 × r + ω × r0 = α × r + ω × r0
ω × r0 = ω × (ω × r) = (ω · r)ω − ω 2 r
(c).

Now ω · r = 0 =⇒ a = −ω 2 r

Exercise 20.
d
The distance |rp (t) − rq (t)| is independent of t, so dt |rp (t) − rq (t)| = 0, which implies

d 2 drp drp drq drq


(rp − 2rp · rq + rq2 ) = 2rp · −2 · rq − 2rp · +2 · rq = 0
dt dt dt dt dt
drp drq
· (rq − rp ) = − · (rp − rq )
dt dt
Suppose

vp = ωp × rp
vq = ωq × rq
Then

vp · (rq − rp ) = −vq · (rp − rq )


ωp × rp · rq = −ωq × rq · rp = = ωq × rp · rq
((ωp − ωq ) × rp ) · rq = 0
Thus, ωp = ωq .
275
14.21 Miscellaneous review Exercises - Applications to planetary motion.
Exercise 1. Use polar coordinates if Cartesian coordinates doesn't help!

y2 = x r2 sin2 θ = r cos θ
X = (r cos θ, r sin θ) = rur cos θ
r=
sin2 θ
X 0 = r0 ur + rθ0 uθ x = r cos θ
p =⇒
|X 0 | = r02 + (rθ0 )2 y = r sin θ
r0 s2 + 2r2 scθ0 = −sθ0
X 0 0
r ur + rθ uθ 0 µ ¶
T = =p 0 1 + c2
|X 0 | r02 + (rθ0 )2 r =− θ0
s3

rr0 r0
r·T = p = r cos α =⇒ p = cos α
r02 + (rθ0 )2 r02 + (rθ0 )2

X · er = r cos θ
µ ¶
1 + 2c2 + c4 c2 02 1 + 3c2
r02 + (rθ0 )2 = θ 02
+ θ =
s6 s4 s4
(1 + c2 )/s3 1 + c2
cos α = 2 6 1/2
=√
((1 + 3c )/s ) 1 + 3c2
1 + 3c2 − (1 + 2c2 + c4 ) c2 s2
sin2 α = 1 − cos2 α = 2
=
(1 + 3c ) (1 + 3c2 )

sin α cs/ 1 + 3c2 tan θ
= tan α = √ =
cos α 2
(1 + c )/ 1 + 3c 2 1 + sec2 θ

Exercise 2.
µ ¶ ¡y ¢
y2 X0 ,1 1
X= ,y = p 2c =p T
4c 0
|X | 1 + (y/2c)2 1 + (y/2c)2
³y ´
0 0
X =y ,1 N = (2c, −y) =⇒ T · N = 0
2c

Exercise 3.
µ ¶
y2
(x, y) = ,y
4c ¡y ¢
³y ´ X0 ,1 2c
X0 = , 1 y0 0
= p 2c ; m = 1/(y/2c) =
2c
r |X | 1 + (y/2c)2 y
³ y ´2
0 0
|X | = y (1 + )
2c
2c
m = m(x0 ) =
y0

2c y02 y0 y0
y = mx + b =⇒ y0 = +b= + b so that b = = c/m
y0 4c 2 2
2c
y0 =
m
=⇒ y = mx + c/m
4c2 1 c
x0 = 2 = 2
m 4c m

Exercise 4.

276
(1)

µ ¶
(y − y0 )2
X= + x0 , y
4c
³ ´
(y − y0 )2 = 4c(x − x0 ) X0 y 0 (y−y 0)
2c , 1
=⇒ T = |X 0 | = r
(y − y0 )2 ³ ´2
+ x0 = x |y | 1 + (y−y
0 0)
4c 2c

2c
m=
y − y0

µ ¶µ ¶
2c (y1 − y0 )2
2c
m(y1 ) = y1 = + b + y0
y1 − y0 =⇒ y1 − y04c
y = m(y1 )(x − x0 ) + b + y0 y1 − y0
y1 − y0 = +b
2
c
(y − y0 ) = m(x − x0 ) +
m

(2)

(x − x0 )2 = 4c(y − y0 )
(x − x0 )2
X = (x, + y0 )
¡ x−x4c ¢
y − y0 = m(x − x0 ) + b
1, 2c 0 µ ¶
T =q (x1 − x0 )2 (x1 − x0 )2
2
1 + (x−x
4c2
0) y1 − y0 =
4c
=
2c
+b
x − x0
m= = m(x)
2c

−(x1 − x0 )2
b=
4c
=⇒ y − y0 = m(x − x0 ) + −m2 c

x2 = 4cy
X = (x, x2 /4c)
¡ x¢ 0 y = mx + b
1, x
T = p 2c x21 x2 x2
|x | 1 + (x/2c)2
0 y1 = +b= 1 = 1 +b
2c 4c 2c
m = x/2c
−x21
b=
4c
=⇒ y = mx + cm2

µ ¶
y2
X= ,y
Exercise 5. Given
4c =⇒ m = 2c
³y ´ y
X 0 = y0 ,1
2c

2c 2c y12 y1 y1
y1 = x1 + b = = + b =⇒ b =
y1 y1 4c 2 2
2c y1
y = x + ; =⇒ y1 y = 2cx + 2x1 c = 2c(x + x1 )
y1 2
277
2c
Exercise 6. (y − y0 )2 = 4c(x − x0 ) =⇒ m(y) = y−y0 .
µ¶
2c
y − y0 = (x − x0 ) + b
y1 − y0
2c(y1 − y0 ) c
b = (y1 − y0 ) − =
µ ¶ 4c m
2c (y1 − y0 )
y − y0 = (x − x0 ) +
y1 − y0 2
(y1 − y0 )2
(y1 − y0 )(y − y0 ) = 2c(x − x0 ) + = 2c((x − x0 ) + (x1 − x0 ))
2
2
For X = (x, x4c ) =⇒ m(x) = x
2c

x1 x21
y= x+b =⇒ b = = −cm2
2c −4c
x1 x
y= − y1 =⇒ 2c(y + y1 ) = x1 x
2c
³ 2
´
For X = x, (x−x
4c
0)
+ y0 =⇒ m = x1 −x0
2c
µ ¶ µ ¶
x1 − x0 x1 − x0 (x1 − x0 )2
y − y0 = (x − x0 ) + b y − y0 = (x − x0 ) + −
2c 2c 4c
(x1 − x0 )2 (x1 − x0 )2 −(x1 − x0 )2 (x1 − x0 )(x − x0 )
− = =b y − y0 = − (y1 − y0 )
4c 2c 4c 2c
(x1 − x0 )(x − x0 )
(y − y0 ) + (y1 − y0 ) =
2c
Exercise 7.

(1) Given y = x2 , then


tN + (x, y)
X = (x, x2 ) (−2x, 1)x0
(x, x2 ) + t √ = (0, yq ) = Q
X0 (1, 2x)x0 |x0 | 1 + 4x2
0
= √ √
kX k |x0 | 1 + 4x2 2xt t 1 + 4x2
=⇒ x = √ y = √ + x2
= √ + x2
(−2x, 1)x0 1 + 4x 2
1 + 4x2 2 1 + 4x 2
N= √ r
|x0 | 1 + 4x2 1 1
t= + x2 = + x2
4 2
(2)

Exercise 8. Given
√ y = x2
± c = x when y = c p

Circle condition: kC − Rk = k(0, y0 ) − (± c, c)k = c + (y0 − c)2 = y0
1+c
=⇒ = y0
2

c → 0, then y0 = 1/2 , radius of circle.

x2 y2
Exercise 9. Consider the condition,
a2 + b2 = 1, pts. on an ellipse.

b2 x2 + a2 y 2 T a2 b2
x2 y2
+ 2 T 1 ⇐⇒ r2 (b2 cos2 θ + a2 (1 − cos2 θ)) = r2 (a2 + (b2 − a2 ) cos2 θ) =
a2 b
= r2 a2 (1 − e2 cos2 θ)
⇐⇒ r2 (1 − e2 cos2 θ) T b2

Suppose for each xed θ1 , we consider all pts. along a line ray from the origin (which is inside the ellipse) to innity, radially
out. Then r2 (1 − e2 cos2 θ1 ) T b2 ∀ r T r1
278
x2 y2
Exercise 10. For X = (x, y, ) and given that a2 + b2 = 1, or b2 x2 + a2 y 2 = a2 b2 , so then

−b2 xx0
2b2 xx0 + 2a2 yy 0 = 0 =⇒ y 0 =
a2 y
2
µ ¶
0 0 0 0 −b x −x0 b2 −y x
X = (x, y) =⇒ X = (x , y ) = x (1, 2 ) = ,
a y y b2 a2
µ ¶ ³x y´
−y x 0
T = , k X N = , ⊥ X 0 since N · X 0 = 0
b2 a2 a2 b2
x0 /a2 b2 x0
m= =
−y0 /b2 −a2 y0
2
b x0 b2 x20 a2 y0 b2 x 0 b2
(y − y0 ) = m(x − x0 ) =⇒ y = x + + y0 = x +
−a2 y0 a2 y0 a2 y0 −a2 y0 y0
x0 x yy0 x cos θ0 y sin θ0
+ 2 = 1 =⇒ + =1
a2 b a b
Exercise 11. Using the work of the previous exercise, Exercise 10, then

x cos θ0 y sin θ0 xa cos θ0 yb sin θ0


+ =1= + = xx0 /a2 + yy0 /b2
a b a2 b2
¡ −y x ¢ ¡ ¢
Exercise 12. Use T =
b 2 , a2 ; N = ax2 , by2 from the previous problems.
Without loss of generality, center the ellipse on the origin and the major axis on the x axis. Then F = aeex and P = (x, y).
¯ ¯ ex ¯ ¯ xe
¯ (F − P ) · N ¯ ¯ (−F − P )· ¯
¯ ¯ = q1 − a ¯ ¯ = q1 + a
¯ |N | ¯ x2 y2
¯ |N | ¯ x2 y2
a4 + b 4 a4 + b4
Multiply the above distances together to get
µ ¶
2 −1/2 ³
µ 2 ¶−1/2 µ 2 ¶
x2 y xe ´ ³ ex ´ x y2 x y2 e2 2
+ 4 1+ 1− = + 4 + 2 − 2 x = b2
a4 b a a a4 b a2 b a
2 2
Exercise 13. x2 + 4y 2 = 8 or x8 + y2 = 1. We want to nd the tangent lines parallel to x + 2y = 7 or y = 7
− x
2 . So the
1
¡ −y x ¢ ¡ −y x ¢ 2
slope of this line is − . Using T =
2 b2 , a2 = 2 , 8 , then
x/8 x
= = −1/2 or x = 2y
−y/2 −4y
plugging back into the ellipse equation , (2, 1), (−2, −1)

Exercise 14. By equidistant property of the circle, the focus length from origin, ae, must equal the minor axis length, b.
√ √
ae = b = a 1 − e2 =⇒ e = 1/ 2
Exercise 15.

x2 y2 1 2 y2
(1)
2
− 2 =1 . Recall that b2 = a2 (1 − e2 ) = −3a2 , so that (x − ) = 1 .
a b a2 3
(2)
sµ ¶ r³ ´
Z x 2
t 1 x 2
A= b − 1dt − (x − a)b −1
a a 2 a
r is the length of V P :
q¡ ¢
x2 y2 x 2
Exercise 16. Given
a2 − b2 = 1, then y = b a −1
 ¡ x ¢  Ã 2 ¡ x ¢!
b b a2 b2 ³ y x ´
X 0 = 1, q¡ ¢a  = 1,
2
= ,
x 2 y y b2 a2
a − 1
So X 0 k T ; and X 0 · N = 0
b2 x0
Exercise 17. m(x0 ) = a2 y0 from the tangent vector above and so, using the ellipse equation,

b2 x0 x0 x y0 y
(y − y0 ) = 2
(x − x0 ) =⇒ 2 − 2 = 1
a y0 a b
279
Exercise 18. Given X = (x, y),

X 0 = (x0 , y 0 ); N k (y 0 , −x0 )
y = sx0
(x, y) + s(y 0 , −x0 ) = (x1 , 0) =⇒ =⇒ x1 = x + yy 0 /x0
x + sy 0 = x1
r³ ´2
p yy 0
Isosceles condition: x2 + y 2 = k(x1 , 0) − (x, y)k = x0 + y2

x dy/dt dy y2 x2
= = =⇒ − =1
y dx/dt dx 2C 2C
Note there's some ambiguity with the signs.

Exercise 19. N k (y 0 , −x0 )

(x, y) + t(y 0 , −x0 ) = (0, y1 )


(x, y) + s(y 0 , −x0 ) = (x1 , 0)
x = −ty 0
y = sx0
y − tx0 = y1
yy 0
x + sy 0 = x1 = x + 0 xx0
x y+ = y1
y0
k(x, y) − (x1 , 0)k = k(x, y) − (0, y1 )k
µ 0 ¶2 µ 0 ¶2 µ ¶2
2 2 2 2 yy 2 2 xx dy x2
=⇒ (x − x1 ) + y = x + (y − y1 ) =⇒ + y = x + =⇒ =
x0 y0 dx y2
(4,5)
−−−→ y 2 − x2 = 9

x2 y2 y
Exercise 20. Without loss of generality, consider a hyperbola given by
a2 − b2 = 1 with asymptotes of
x = ± ab . The
distances from a point on the hyperbola to the asymptotes are the following:

(X − P ) · N1 (x, y) · (−b, a) −bx + ay


= √ = √
|N1 | a2 + b2 a2 + b2
(X − P ) · N2 (x, y) · (b, a) bx + ay
= √ =√
|N2 | 2
a +b 2 a2 + b2
µ ¶µ ¶
| − bx + ay| bx + ay b2 x2 − a2 y 2 a2 b2
√ √ = 2 2
= 2
2
a +b 2 2
a +b 2 a +b a + b2

Exercise 21. Recall that for X = rur (useful trick to change into polar coordinates),

X 0 = r0 ur + ruθ θ0 Z p
p =⇒ r02 + (rθ0 )2 dt
|X 0 | = r02 + (rθ0 )2
Rθ √
(1)
θ0
r02 + r2 dθ = k(θ − θ0 )
µ
¶2 µ ¶2
p r0 r
r02 + r2 −C =k √ + √ =1
=⇒ k k
r = k − r2
02 √ √
r = k sin θ or r = k
R θ q¡ dr ¢2
(2)
θ0 dθ + r2 dθ = k(r(θ) − r(θ0 ))

r θ − θ0
r02 + r2 = (kr0 )2 ln =√
r0 k2 − 1
=⇒ r2 µ ¶
r02 = θ
k2 − 1 r = Ce √
k2 − 1
280
Rθ √ Rθ 1 2
(3)
θ0
r02 + r2 dθ = k θ0 2
r dθ
kr Z 2
sec t = ktan t sec t
2 √ = θ − θ0
k2 4 k dr
2
k | sec t|sec2 t − 1
r02 + r2 = r tan t sec t = µ ¶
4 kr
r0 −−−−−−−−−−−2−− dt→ arcsec =θ−C
=⇒ q¡ ¢ =1 2
kr 2
|r| 2 − 1 2 2
r= sec (θ − θ0 ) or r=
k k
r(b)−r(a)
Exercise 22. Given r0 (t) = (r(a) × r(b)), r0 (t) = b−a for some t ∈ [a, b] (by mean-value thm. on each of the
components).
r0 (t) · (r(a) × r(b)) = 0
0
Geometrically it means that there must be some component of r (t) that belongs in the plane containing r(b) − r(a) because
0
the velocity vector r (t) must “take” r(a) to r(b).

281

Das könnte Ihnen auch gefallen